MCQ Masterset KROM

Ace your homework & exams now with Quizwiz!

3

?

A

ANSWER D

Absolute Contraindication to ECT A. Cochlear implants B. Epilepsy C. Pregnancy D. Raised intracranial pressure E. Myocardial infarction

ANSWER D raised ICP recent MI is also a fairly definite contraindication (in prev 6/52)

Defibrillator Current

Alternating current

The ratio of MAC to MAC Awake for sevoflurane is: A. 0.22 B. 0.33

B

Y

B

What is NOT useful in the treatment of Torsades? A. Isoprenaline B. Procainamide C. DCCV D. Electrical pacing (Amiodarone was not an option)

B Procainamide prolongs the QT interval

P

B -

W

C

How quickly does the CO2 rise in the apnoeic patient ? A. 1 mmHg per min B. 2 mmHg per min C. 3 mmHg per min D. 4 mmHg per min E. 5 or ?8 mmHg per min

C ournal of clinical Anaesthesia 1989 328 3.4 mmHg every minute

Compared to lignocaine, bupivacaine is A. Twice as potent B. Three times as potent C. Four times as potent D. Five times as potent E. Same potency

C - four times

Neuromuscular block + carotid bodies

Carotid bdies have nicotinc Ach receptors. These are blocked by vet/atracurium and will impair the hypoxic ventilatory response

Best single predictor of difficult intubation in obese patient? A. Mallampati score B. Interincisor distance C. Severe OSA D. pretracheal soft tissue volume

D - soft tissue (neck circumference) (A is next best)

C

E

Incarcerated inguinal hernia in a child with a mild URTI. Most appropriate course of action? a. postpone for two weeks b. continue but with a spinal anaesthetic only c. continue but without using an endotracheal tube d. he requires antibiotics e. continue, but with careful monitoring

E Can't be postponed.

PPH

Maternal blood volume 100mL/kg 500mL > 1000mL severe Tone Trauma Tissue Thrombin Theatre

Metabolites of volatiles

Sevoflurane - Compounds A- E Inorganic and organic fluorides; carbon monoxide Desflurane - Trifluoroacetic acif

A

not entered

P

(

122. Repeat- How to estimate weight in child-

(Age+4) x2 Age

a

-

Atrial septal defect, where is the murmur heard the loadest? A. PV B. MV C. ASD D. AV E. TV

A "ASD with moderate-to-large left-to-right shunts result in increased right ventricular stroke volume across the pulmonary outflow tract creating a crescendo-decrescendo systolic ejection murmur. This murmur is heard in the second intercostal space at the upper left sternal border"

Central sensitization occurs due to a. Primary events mediated by the NMDA receptor b. Alterations in gene expression c. Increased magnesium

A "Activation of NMDAR is an essential step in both initiating and maintaining activity-dependent central sensitization as its blockade by noncompetitive (MK801) or competitive (D-CPP) NMDAR antagonists prevent and reverse the hyperexcitability of nociceptive neurons induced by nociceptor conditioning inputs"

What is the half life of clopidogrel? a. 6 hours b. 14 hours c. 24 hours d. 7 days

A - 6 hours

Which of the following are feature of Conn's syndrome? A. Normoglycaemia, hypernatremia , hypokalemia B. Hypoglycaemia, hypernatremia, hypokalemia C. Hyperglycaemia, hyponatremia, hyperkalemia D. Normoglycaemia, hyponatremia, hyperkalemia E. Hypoglycaemia, hyponatremia, hyperkalemia

A - normal BSL, high Na, Low K Conn syndrome is an aldosterone-producing adenoma.

Major cause of death following difficult intubation with perforated oesophagus? A. Sepsis B. Failure to intubate C. Failure to ventilate

A - sepsis

Blood flow across which of the following is used to estimate pulmonary artery pressures during echocardiography? A. Tricuspid valve B. Pulmonary valve C. Mitral Valve

A - some TR is required, the velocity of the jet gives a pressure differential across the TR via modified bernouilli equation. The RV and PA pressures then are this pressure plus your estimate of RAP.

The nerve supplying area of skin between greater trochanter and iliac crest: A. subcostal nerve B. ilioinguinal nerve C. genitofemoral nerve D. femoral nerve E. lat cutaneous femoral nerve

A - subcostal nerve

Red Blood Cells

1 cell 270 million Hb; Hb takes up 2/3 of cell. Store 2.5g of ferritin, 65% of total body. Lipid bilayer, higher Cl- in RBC due to hamburger effect. Carbonic anhydrase in RBC CO <=> H2CO3 <=> H+ + HCO3- H+ is formed but trapped; HCO3- gets out in exchange for Cl- coming in.

Dabigatran Hlaf Life

8 hrs after 1 dose, 14 after multiple- doubles in severe renal impairment; steady state reached at 3 days. Dose dep in crease in aPTT & PT, thrombin time but wide variation. Normal thrombin clotting time exclude clinical effect.

36. Repeat- Nerve supply to the upper eyelid- A. ophthalmic branch of trigeminal and sympathetic from superior collicus ganglion

? NIL A

116. ? Post op pneumonectomy short of breath- investigation

??

FOB - can see a trifurcation. Where are you? A. RUL B. RLL C. RML D. LLL

A

Patient with mastocytosis. Intraop would probably be: A. Severe hypotension

A

Post CEA on ward, patient seizes. BP has been hard to control. What to do to prevent further seizures? A: Add another antihypertensive B: Start antiplatelet drugs C: Start anticonvulsants D: Do angio and stent E: Nimodipine

A

Post Carotid Endarterectomy on ward, patient seizes. BP has been hard to control. What to do to prevent further seizures? A: Add another antihypertensive B: Start antiplatelet drugs C: Start anticonvulsants D: Do angio and stent E: Nimodipine

A

Post epidural and LSCS, the next day patient have persistent paraesthesia anterior thigh. What other injuries would indicate more of nerve roots instead of peripheral nerve injuries A. Weakness on hip flexion and thigh adduction B. Weakness on knee flexion and plantar flexion C. Urinary incontinence D. Foot drop

A

Risk factors for postoperative ulnar nerve injury? A. is more common in diabetics B. is more common in women C. is often associated with contralateral clinical neuropathy D. is usually found to be the result of excessive external pressure E. usually presents within 24 hours

A

The commonest symptomatic cardiac condition in pregnancy a) MS b) AS c) Eisenmengers

A

The most common clinically significant valvular lesion in pregnancy is: A. MS B. MR C. AS D. AR E. TR

A

Torsades, what's not useful? A. Amiodarone B. Isoprenaline C. ? D. E.

A

Troponin can be detected for how long: A. 5-14 days B. ?

A

When does effect of diclofenac on platelets wear off? A. 12 hours B. 1-2 days C. 4 days D. 7 days E. 10 days

A

Which drugs below does not need dose adjustment in renal failure patient A. Buprenorphine B. Morphine C. Tramadol D. ? E. ?

A

Which of the following are feature of Conn's syndrome? A. Normoglycaemia, hypernatremia , hypokalemia B. Hypoglycaemia, hypernatremia, hypokalemia C. Hyperglycaemia, hyponatremia, hyperkalemia D. Normoglycaemia, hyponatremia, hyperkalemia E. Hypoglycaemia, hyponatremia, hyperkalemia

A

\A female with type 1 von Willebrand disease presents for a dilation and curettage. She is a Jehovah's Witness. In regards to desmopression to prevent haemorrhage in this patient all of the following are true EXCEPT: A. It is a synthetic substance and therefore acceptable to Jehovah's Witnesses B. It is likely to reduce haemorrhage in this patient C. It should be given as an infusion 30 minutes prior to surgery D. The effect will last 5 days E. The dose is 0.3μg/kg

ANSWER D

Timing of peak respiratory depression post intrathecal 300 mcg morphine: A. < 3.5 hours (think it was one hour) B. 3.5 - 7.5 hours (then three hours) C. 7 - 12.5 hours (then 7.5 - 12.5 hrs) D. 12.5 -18 hours E. > 18 hours

Ans B Peak at 6 hrs CEACCP Morphine-induced late onset respiratory depression occurs between 3.5 and 12 h after injection with a peak at 6 h *

Patient has undergone a bilateral lung transplant. All of following are impaired EXCEPT A. Mucociliary clearance B. Cough reflex distal to anastomosis C. Hypoxic pulmonary vasoconstriction D. Response to CO2 E. Lymphatic drainage

ANSWER C

Using the American Heart Association specification, the colours of the electrodes in a 3-lead electrocardiographic (yes the typo is what they used) is A. Right arm = Black; Left arm = White; Left leg = Red B. Right arm = White; Left arm = Black; Left leg = Green C. Right arm = Black; Left arm = Green; Left leg = Red D. Right arm = White; Left arm = Black; Left leg = Red E. Right arm = Red; Left arm = White; Left leg = Green

ANSWER D

What organism most commonly causes meningitis post spinal anaesthesia A. Staphylococcus epidermidis B. Staphylococcus aureus C. Streptococcus pneumonia D. Streptococcus salivarius E. Escherichia coli

ANSWER D

what do these terms mean Decontamination Disinfection Sterilisation Asepsis

Asepsis - Prevention of microbial contamination of living tissue or material. Decontamination - removal of microorganisms and unwanted matter from material or living tissues. Describes whole process of cleaning, disinfection or sterilisation. Disinfection - Inactivation of micro-organisms using thermal or chemical means Sterilisation - Complete destruction of micro-organism incl spores. Includes radiation & autoclave.

Witnessed VF arrest. First response a) Defibrillation b) CPR c) Secure airway

B

Young child with WPW undergoes general anaesthesia. Intra operatively developed tachycardia. HR 220, BP 80/40. Best drug to cardiovert A. Adenosine B. Amiodarone C.

B

hy is tachycardia bad in mitral stenosis? A. B. Increases transvalvular pressure gradient C. D. E.

B

Young asthmatic male in emergency department. RR 26, pCO2 27, SAO2 92%, struggling talking in sentences. Given nebulised salbutamol, and ipratropium, 200mg IV hydrocortisone. After 30 minutes - no improvement. Further management: A. IV salbutamol B. IV aminophylline C. IV magnesium D. Intubate and ventilate E. ?

C

What is the best predictor of severe bleeding in cirrhosis? A. Thrombocytopaenia B. Hypofibrinogenaemia C. Prolonged PT D. Hypoalbuminaemia E. Pulmonary hypertension

C ?? A OHA says that bleeding is more likely due to thrombocytopaenia than clotting factor deficiency A lot of other sites just say it is a multi-factorial process

Pain from the uterus during labour is transmitted via a. From the anterior roots of T10-L1 b. Parasympathetic fibres c. The inferior hypogastric plexus d. Via grey rami communicantes

C ?? A and C OpenAnaesthesia "Pain travels via sympathetic nerve fibers (going through the inferior hypogastric plexus on the way to the sympathetic chain) that originate from the T10-L1 segments of the spinal cord

The safe maximal pressure for endotracheal cuff at the lateral side of the trachea A. 0-10 cm water B. 10-20 cm water C. 20-30 cm water D. 30-40 cm water E. 40-50 cm water

C ?? ANSWER B

p

C ?? B

Patient undergoing partial hepatic resection develops Venous Air Embolism. Best position should be A. Head down left side up B. Head down right side up C. Head up right side up D. Head up left side up

C ?? B Open Anaesthesia: "place patient in left lateral decubitus and Trendelenburg"

What colour is the label for subcutaneously administered drugs a. Pink b. Yellow c. Beige d. Red e. Blue

C National Recommendations for User-applied Labelling of Injectable Medicines, Fluids and Lines Intra-arterial = Red Intravenous = Blue Epidural / Intrathecal / Regional = Yellow Subcutaneous = Beige 723 Other routes = Pink

Complex regional pain syndrome. What proportion of patients have motor involvement ? A. 0 % B. 25 % C. 50 % D. 75 % E. 95 %

E ?? D CRPS type I (reflex sympathetic dystrophy) is diagnosed where there is no evidence of a precipitating nerve injury CRPS Type II (causalgia) where a nerve injury is present. Motor / trophic changes - motor dysfunction 57-98% - weakness 75-95% - limited range of movement 80-88% - incoordination 47% - tremor 48% - spasm 13% - dystonia 14% - myoclonus 4-20%

Lymphatics

Lymph - interstitial fluid that enters lymphatics Lymphatics present in most tissue; absent in brain, bone. Function - 1. Return of protein to systemic circulation 2. Immune cells lymphocytes, macrophages in LN 3. Transport system of fat from intestine (chyle) One way valves, lymph moves by contraction of skeletal muscle. Hepatic sinusoids very permeable to proteins.

Airways resistance; compliance & time constant

Main airways resistance is in the medium bronchi not the small airways . This is due to the fact that the small airways have such a large surface area. Airways resistance ~ 2cmH2O/L/sec Compliance - 100mL/ cmH2O (when ventilating to get TV of 500mL may need 10cm pressure for positive pressure ventilation) Time constant = compliance x resistance -> to get a result of seconds. 2cmH2O/L/sec x o.1L/cmH2O = 0.2 seconds

104. New- Neonate desaturate faster than adult at induction because A. FRC decrease more B. Faster onset of induction agents C. More difficult to pre-oxygenation

NIL ??? Reasons for neonatal desat 1. High metabolic rate 2-3 x adult (required mainly for temperature control, high BSA to mass) 2. Small absolute FRC (same as adult 30ml/kg) therefore less oxygen 3. CC >FRC, neonate generates autoPEEP by partial closure of glottis, this is lost at induction, causing airway closure and V/Q mismatch

Nitric Oxide

NO - highly soluble gas which is produced by endothelial cells by enzyme nitric oxide synthase (eNOS). 1. Vasodilation 2. Decreased vascular tone in muscular arteries (e.g. coronaries) & veins 3. Inhibits platelet aggregation 4. Inhibits proliferation of vascular muscle

A child with 10% dehydration is likely to have a. Bradycardia b. Rapid deep breathing

NONE Just deep breathing according to Dr Sally Wharton's notes ? B RCH CPG Dehydration Moderate dehydration (4-6%) • Delayed CRT (> 2 secs) • Increased respiratory rate • Mild decreased tissue turgor Severe dehydration (>/= 7%) • Very delayed CRT > 3 secs, mottled skin • Other signs of shock (tachycardia, irritable or reduced conscious level, hypotension) • Deep, acidotic breathing • Decreased tissue turgor

Phases of clinical research

Preclinical - testing on non human subjects Phase 0 - PK studies & PD e.g. oral bioavailablity Phase I -testing on healthy volunteers for dose ranging Phase II - testing on patient grp to evaluate safety & see if it has an effect Phase III - testing on a patient group to see if there is therapeutic effect & look at safety Phase IV - post marketing surveillance

Nociceptor Pain afferents

Receptor preferentially sensitive to noxious stimuli or stimulus that would become noxious if prolonged. Triggers 1. Thermal 2. Chemical - K+/bradykinin/substance P (binds to NK1) 3. Mechanical - shear/distension A delta - fast medium myelinated. Sharp localised pain. Conduction is 15 m.sec (Motor is 10oms) Synapse in lamella I of dorsal horn C - slow as small & unmyelinated fibres, stimulation is experienced as dull, diffuse, throbbing. 1ms. Synapse level II

Highly Protein Bound Drugs

Warfarin Phenytoin Propranaolol Propofol Diazepam With these drugs alterations in protein binding will actually have an effect on free drug. Neonates have less alpha 1 glycoprotein so binding of drugs such as diazepam , morphine will be decreased.

How far to insert PICC line in a kid beyond the carina A: At the carina B: 1cm below C: 1cm above

We have C (JK&PN) ANSWER A The ideal position is above the pericardial reflection in the SVC which in most patients would be at the level of the tracheal bifurcation. http://www.anzca.edu.au/news/e-newsletter/e-news-articles/Coroners%20report%20PICC%20line%20AR%20summary%20Feb%2010.doc/view?searchterm=picc%20coroners

alfentanil

potenet mu agonist faster onset than fentanyl due to 90% unionised at physiological pH. Extensively protein bound so actually relatively low lipid solubility

More functions of LOH

reabsorb Mg/Ca paracellulary Water resorption NaCl resorption est osmotic gradietn macula densa for GFR control Thin LOH has aquaporin 1 ; epithelial cells Thick does not have any; cuboidal cells.

What is excitation-contraction coupling

series of physiological steps which links depolarisation of muscle membrane with muscle fibre contraction T- tubules conduct action potential into muscle & then depolarise, Ca released from SR, this stimulates mores Ca++ release. Ca binds to tropomyosin on troponin C which initiate a conformational change in protein & exposes binding sites on actin. Myosin binds to actin, sarcomere shortens ATP required to release bond of actin/myosin & to pump Ca back in.

During elective major vascular surgery the best way to reduce the risk of acute renal failure is to maintain a normal A. cardiac output B. central venous pressure C. mean arterial blood pressure D. pulmonary capillary wedge pressure E. renal blood flow During elective major vascular surgery the best way to reduce the risk of acute renal failure is to maintain a normal A. central venous pressure B. mean arterial blood pressure C. renal blood flow D. systemic vascular resistance E. urine output

wiki says - Maintain renal blood flow Limit AoX time Avoid supra renal AoX

The diagram below is of a transgastric short axis view obtained during a transoesophageal echocardiography examination. Which letter corresponds to the area supplied by the right coronary artery? A. A B. B C. C D. D E. E

//d3cgb598vs7bfg.cloudfront.net/images/upload-flashcards/554968/540512_m.jpg\d\dA\d\d//d3cgb598vs7bfg.cloudfront.net/images/upload-flashcards/554968/540513_m.jpg

When does PaCo2 peak in laparoscopic surgery?

15 minutes "CO2 absorption in the peritoneal cavity or retroperitoneal space will result in hypercapnia and respiratory acidosis. In otherwise, healthy individuals without having pulmonary disease, CO2 is eliminated efficiently through respiration by increases in minute ventilation (increases in respiratory rate and/or tidal volume). Similar to the phasic cardiac response, CO2 excretion rises acutely, from the typical baseline degree of 125 mL/min to 200 mL/min, within approximately Quarter-hour of pneumoperitoneum then plateaus thereafter" www.laparoscopyhospital.com *

What means are there to keep neonate warm?

1. Minimise heat loss Convection - blankets,hats, warm IV fluids Conduction - warm surface blankets, hat Evaporation - Dry at birth, humidify gases Radiation - Warm room/theatre, warm resusciteur

What causes allergic reactions in anaesthesia

1. Neuromuscular blocking drugs - six highest due to shape & size. High cross reactivity on skin tests but clinical response rate is less. 80% of Neuromuscular Blocking Drug reactions occur without prior exposure. Atopy a risk factor for sensitivity. 2. Antibiotics 15% - typically penicillins Latex Colloids - Haemaccel > gelo. Typically a non allergic rxn Chlorhexidine Patent blue (use methylene blue as alternative as less problems) Opioids - most reactions are histamine releasing and not IgE mediated. Propofol - uncommon, no evidence to support soya or egg cross reactivity.

Calcium Channel Blockers These are class iV antiarrythmics Taret is L type calcium channels

1. Phenylalkamines: Cardioselective Verapamil 2. Dihydropyridines: Blood vessels arteries, cause vasodilation Amlodipine, felodipine, nifedipine 3. Benzothazepines - work on both Diltiazem

What factors make neonate more prone to heat loss or heat gain?

1. Radiation - larger body surface area to volume 2. Evaporation - wet at birth 3. Conduction - being placed on cool surface. 4. Higher metabolic rate 5. High minute ventilation 6. Thin subcutaneous tissue 7. Ineffective shivering (low muscle mass) 8. Higher thermoregulatory zone 7. Inability to control personal environment 8. Impaired sweating

How can we reduce electrical risk to patients

1. Type of equipment 2. Maintenance of equipment 3. Design of environment Decrease static by maintaining humidity > 50% Keep fluid away from electrical equipment 4. Safety features - Isolated circuits by using transformers. A transformer uses coils of wire around iron core to create magnetic field and then induces a current. This means that the patient is not directly connected to mains power so protected from surges/faults.

CSF pressure

10-18cmH2O lying 20-30cmH2O sitting

Fentanyl Patch - What is time to reach peak concentration?

24 hrs Transdermal drug delivery system, duration 72 hrs. Rectangular patch, rounded corners with backing layer and fentanyl containing silicone layer. Strengths dep on area. 12.5mcg/25/50/75/100 per hour. After initial patch application serum levels increase until 12-24hrs then remain stable until 72 hrs. (Model suggests adding a new patch after 24 hrs will increase levels still) Mean half life after 72hr patch is 24 hrs (IV doses 3-12 hrs; slower transdermally due to ongoing slow absorption from skin)) Medsafe datasheet *

Trauma patient. CXR (not given): air fluid levels adjacent to heart/diaphragm/ribs. A. Ruptured diaphragm B. Hiatus hernia

A B is also possible

Neonatal Resuscitation Breaths

3:1

Central anticholinergic syndrome, which is NOT true: A. Will improve with neostigmine B. Peripheral anticholinergic symptoms C. Caused by Anti-Parkinson drugs D. CNS depression E. Associated with agitation, delirium, and ???

A - won't improve with neostigmine as will not cross BBB

Long-standing T6 paraplegia. Which is INCORRECT? A. Flaccid paralysis B. Poikilothermia C. Labile BP

A get spastic paralysis

Carbon dioxide is the most common gas used for insufflation for laparoscopy because it A. is cheap and readily available B. is slow to be absorbed from the peritoneum and thus safer C. is not as dangerous as some other gases if inadvertently given intravenously D. provides the best surgical conditions for vision and diathermy E. will not produce any problems with gas emboli as it dissolves rapidly in blood

ANSWER E better than C for me "CO2 may be the gas of choice for laparoscopic surgeries because it is noncombustible, extremely soluble, and readily eliminated with the lungs. Other gases for example nitrous oxide have fallen out of favor. Nitrous oxide can result in bowel distension and decreased working space and is also combustible. Helium and argon, although inert and nonflammable, are a lesser amount of soluble than CO2 and therefore are hard to eliminate through ventilation. Pneumothorax caused by helium and argon resolves much slower than CO2 pneumothorax, and gas embolisms of those insoluble gases aren't well tolerated and can rapidly lead to death" www.laparoscopyhospital.com *

Severe asthmatic- tachycapnia, HR120, speaking in words, pH 7.45, pCO2 46, pO2 96, HCO3 24. Then given nebulised salbutamol continuously, nebulised ipratropium bromide, and hydrocortisone- The next step:

ANSWER Magnesium

Anaesthetic gas scavenging

Aim is to precent build up of gas in theatre. Flows for scavenging typically 80L/min Need exhaust port to receive waste gas then transfer hose -> receiving system + disposal system.

Body water composition in neonate & kidneys

At birth neonate "wet" with water 75% cf 60% adult Pre-term even more. Newborn in negative balance first few days The GFR of neoate is low only 30% adult due to - immature LOH - poor response to ADH - Poor concentrating ability - Decreased ability to excrete acid load - High K+ in first 48 hrs well toleratedCardio

Mydriasis can be caused by these drugs

Atropine (bella donna) Phenylephrine (avoid using 10% sol prior to GA as sensitizes myocardium) Glycopyrolate Tropicaminde

What is the ratio of MAC awake:MAC of sevoflurance a. 0.2 b. 0.34 c. 0.5

B MAC = 2.1 Mac awake = ~0.7 0.7/2.1 = 0.3

Renal vascular resistance

Balance of vascular tone between afferent + efferent arterioles. Afferent Dilatation with Efferent Constriction will result in increased GFR. Afferent vasoconstriction with efferent dilatation will result in decreased GFR. Changes in arteriolar resistance produces same effect on renal blood flow regardless of it being afferent or efferent a the vessels are in series. What does change however if GFR. Thus kidneys can regulate GFR at local level independent of blood flow. Afferent vasodilation - prostaglandins - nitric oxide Efferent constriction - Sympathetics Afferent vasoconstriction - Angiotensin II - Sympathetics - Vasopressin - Adenosine Efferent vasodilation - Prostaglandins

75 year old with non-valvular AF usually on warfarin has their warfarin stopped for one week. What is their daily risk of stroke? A: 1% B: 0.1% C: 0.01% D: 4% E: 10%

C

Characteristic Resp Pattern in C5 Spinal Cord Injury

C3-C5 - diaphragm involvement, vital capacity < 30% predicted. Partial phrenic nerve involvement, weak/ineffective cough. As it's above T8 loss of intercostals. Chest wall immobility won't be immediate Arterial hypoxaemia; rapid resp rate - unsure which is correct prob both. I will go with rapid resp rate.

Symapthetic innervation of heart

Cardiac plexus formed from stellate ganglion and thoracic sympathetic chain. Rostral ventrolateral medulla (RVLM) in brainstem Right sided chain looks after SA/AV node, L) ventricle from L) paravertebral ganglia.

Name some derivatives of phenothiazine

Chlorpromazine Amitriptylline Promethazine Pethidine

Creatine clearance & GFR

Clearance = volume of plasma cleared by plasma per unit time (mL/min). It can be measured with inulin in research as this is 100% filtered + not secreted, absorbed, metabolised or stored. More practically measure with creatinine. Creatinine is released from muscle at a relatively constant rate. It is freely filtered (small amount secreted) by kidney it can be used to estimate renal plasma flow. Collect it over 24 hrs or use formula to estimate its clearance. Use Cockroft Gault Formula CrCl = (140 - age) x weight/plasma creat x 72 Adjustment for females x 0.85

How can depth of anaesthesia be measured

Clinical - HR/BP/reaction to voice, stimulation/pupil size - Isolated forearm technique (tourniquet before NMB) - Guedal stages - 1 analgesia, 2 excitation, 3 anaesthesia with planes 1 - iv i - loss of swallow ii - loss of laryngeal reflex iii - loss of light reflex, pupils dilate iv - diaphragm paralysis with apnoea Monitoring - Depth of anesthesia monitoring (spectral array (hill/valley; entropy; BIS) - Evoked auditory potentials - click in ear, record response amplitude & latency - End tidal monitoring - Heart beat variation - loss of sinus arrythmia with increasing depth

Muscle NOT supplied by sciatic nerve: A. biceps femoris B. semitendinosus C. semimembranosus D. gluteus maximus E. adductor magnus

D Gluteus maximus: innervated by inferior gluteal nerve L5,S1,2

Which of the following is most effective way to reduce renal failure in AAA surgery A. Fluid bolus prior to aortic clamping B. Fluid bolus after aortic clamp release C. Frusemide D. Minimize cross-clamp time E. Mannitol

D - Cross clamp time

Neonate intubation- at lips

DEPTH Mouth Nose Term 9 11 6 mo 11 13 1yo 12 14 Age/2 +12 +15

Cephalothin doesn't cover: a. Proteus b. E coli c. Staph d. Strep e. Pseudomonas

E

Cephalothin doesn't cover: a. Proteus b. E coli c. Staph d. Strep e. Pseudomonas

E - Pseudomonas ? B - E Coli?? 1st gen cephalosporin. Good for many gram +ve, and a few gram -ve

Predictors of intraoperative hypoxaemia

Factors predictive of hypoxaemia in OLV - Ventilation of Left rather then Right Lung - Low PaO2 on 2 lungs - Absence of reduction of perfusion to areas of lung pathology - Supine rather than lateral decubitus CEACCP

GABA A receptors

Gamma amino butyric acid receptors are the main inhibitory receptors in the CNS. The natural agonist is GABA. The receptors are ligand gate chloride channels with 5 subunits. They are post synaptic. Binding of agonists causes an increase in permeability of chloride through the channel leading to hyperpolarisation of cell and inhibition. Drugs the act on GABA receptors include many GA agents - Barbiturates - Propofol - Etomidate also - Alcohol - Benzodiazepines which bind to benzo modulatory site on alpha receptors, I - causes anxiolysis; II causes sedation. Potentiate the effect of GABA. Other GABA receptors are GABA B - These are GPCR; inhibited by baclofen which crosses BBB unlike GABA. GABA C Retina

Plasma Proteins Albumin GLOBULINS Fibrinogen other

Globulins - alpha/beta/gamma Alpha inhibit antithrombin III Alpha 1 - antitrypsin, acid glycoproteins, lipoproteins Alpha 2 - macroglobulins, caerucoplasmin (copper) Beta - Transfrerrin Gamma - IgG, IgA, IgM IgE (from most to least in plasma)

Complement

Heat labile proteins found in plasma which are produced by liver & macrophages. Have a role as inflammatory mediators e.g. opsonisation.

An 85y.o for open AAA repair. Refuses blood because of risk of vCJD. You tell him you won't anaesthetise him as the risk is too high. This is an example of: A: Autonomy B: Beneficence C: Malevolence D: Coercion E: Paternalism

I have E - kate

Diastolic dysfunction

Impaired relaxation of R or L heart reduces ventricular compliance. Elevated filling pressures are required to fill ventricles. Causes incl ischaemia Relaxation (luisotrophy) is an active process where energy is required for calcium reuptake Will shift pressure volume loops of heart upwards as pressure raised.

Autologous transfusion results in less a. Cost b. Blood waste c. Incompatible transfusion d. Unrequired transfusion

Increased risk of overall transfusion but decreased exposure to incompatible blood.

Pain Pathways in Labour

Labour is expulsion of products of conception, it is associated with sever pain. Modulated by progesterone. Comes in waves of contractions + evokes neuroendocrine stress response. Experience will affected by - Fatigue - Cultural background - Expectations - Support - Fear - Coping mechanisms First stage : Cervical dilatation & uterine contractions - visceral pain intermittent, poorly localised, dull, aching, associated pressure in bladder/rectum - T1o -L1 dermatomes receive referred pain through Adelta + C fires. - Cervical dilatation in parasympathetic nerve S2-S3 Second stage - Perineal streching. - Somatic pain - Localised to pudendal nerves to S2-S3 nerve roots Third stage - Expulsion of placenta Not as severe, as above.

Coarctation of the aorta

Lesion of aorta usually distal to subclavian Causes outflow obstruction BP raised with higher in upper limbs than lower limbs Weak/delayed pulses in feet. The pressure below lesion is still raised as hypo perfusion of kidneys results in activation of RAA so BP raised. BaroR become desensitised to high pressure so no added value.

Iodine

Less useful against viruses/fungi

Parasympathetic innervation of heart

Medulla (nucleus ambiguens) -> vagal motor nucleus -> right vagus -> SA node -> left vagus - > AV node Dominant system at rest,

46. New- After transfusion of 5 unit of FFP what is least likely to occur A. Haemolytic reaction B. Hypocalcaemia C. Infection D. Hyperkalaemia

My answer A ?? ANSWER D A. FALSE: can occur if not matched B. FALSE: likely to occur if tranfused >1ml/kg/min C. FALSE: viral transmission possible D. TRUE: FFP does not contain potassium

Codeine

Naturally occurring alkaloid, a methylated morphine derivatine. A - Little first pass metabolism so good OBA D - M- Liver via glucuronidation, methylation & cytochrome P450 system to morphine. CYP2D6 has a lot of genetic variability so people may be fast or slow metabolisers of prodrug which will affect analgesia. Excretion - Kidney, decrease dose in renal failure MOA - Low affinity for mu R; 10% converted to morphine. Used as analgesic for moderate pain, anti-tussive. Causes constipation. Not for IV as causes hypotension. Side effects incl N/V, dizziness constipation.

Absolute contraindication to sitting position for posterior fossa craniotomy for meningioma A: Prescence of patent ventriculo-atrial drain/shunt B: PFO C: Oesophageal stricture so transoesophageal echo placement is out D: ? E: ?

Now agreed on A!! ANSWER A or B ABSOLUTE CONTRAINDICATIONS * Patent ventriculo-atrial shunt * Severe cardiovascular disease * Large patent foramen ovale or other pulmonary-systemic shunt * Cerebral ischaemia when upright and awake * Anaesthesia or surgical team not familiar with the position

What is a phase II block

Occurs after a repeated dose of depolarising neuromuscular blocking drug

Alveolar Gas Equation

PAO2 = FiO2 (Patmosphere - PH2O) - PaCO2/R = 0.21 (768-47) - PaCO2/0.8 Used to est PAO2 in a "perfect alveoli"

Variability in drug response

PK - Bioavailabiltiy - cardiac output - Renal/hepatic fxn - Age Pharmacodynamic - Enzyme activity - Genetic variation Drug interactions

Dalton's Law of Partial Pressure

Pressure exerted by a mixed amount of gas mixture equals the pressure it would exert if alone. This means that the pressure exerted by mixture of gases is equal to sum of partial pressures of each gas

ACE inhibitors - Captopril - Enalapril -Lisinopril -Quinapril

Produce vasodilation by inhibiting conversion of angiotensin to angiotensin II in lungs. Angiotensin II has half life 2 mins. It binds to angiotensin receptors on smooth muscle and via GPCR causes relaxation. Angiotensin II also promotes release of NA + prevents the repute of NA so ACE inhibitors will block this sympathetic pathway. ACE-i 1. Dilate arteries & veins (arteriolar > venous) causing decrease in afterload & preload. 2. Block NA release 3. Block angiotensin II effects on kidneys (thus Na is excreted with H2O & aldosterone is not released so K+ retained) 4. Inhibits cardiovascular remodeling that occurs in disease stares - chronic HTn, CHF, MI. SEs ACE i also prevent breakdown of bradykinin so stimulate cough reflex. Loss of taste, angioedema, agranualocytpsis, thrombocytopenia.

Anaemia

Reduction in total red blood cell mass below the normal range Affected by sex, age, gender, environment.

Commonest organism causing meningitis post spinal:

Strep salivarius Baer, Post Dural Puncture Bacterial meningitis, Anaesth 2006 Most of the organisms that cause PDPM are commensals of the mouth and upper airway. These observations support the droplet mechanism for the pathogenesis of most cases of PDPM. That is, the aerosolized organism that enters the CSF during dural puncture originates in the upper airway of medical personnel. Kate - NAP3 - Post epidural Staph. Post spinal Streptococcus *

Glycine Receptors

These are inhibitory receptors, imp in spinal cord neurotransmission. Produce hyper polarisation like GABA receptors. Tetanus toxin binds to these and inactivate them resulting in spasms.

TAVI (TAVR) vs optimal medical therapy

UP to date: Comparison to medical therapy in inoperable patients — Evidence of a benefit of balloon-expandable TAVR compared to standard medical care was provided by the Placement of Aortic Transcatheter Valves (PARTNER) multicenter trial (cohort B) [8,9]. The investigators randomly assigned 358 patients with severe aortic stenosis who were not considered surgical candidates to either standard therapy including balloon aortic valvotomy (see "Percutaneous balloon aortic valvotomy") or TAVR with an Edwards SAPIEN valve via transfemoral approach. The two treatment groups were similar, although the logistic EuroSCORE was slightly lower in the TAVR group (mean 26.4 versus 30.4). (See "Estimating the mortality risk of valvular surgery".) The following findings were noted: ●At one & 2 yrs, the mortality rate was reduced with TAVR compared to medical therapy including balloon aortic valvotomy. ●Among survivors at one and two years, functional class was better with TAVR versus medical therapy ●The stroke rate was significantly higher in the TAVR group than in the medical therapy group at 30 days *

Wat is the pieso electric effect?

Using an electrical current to generate mechanical energy in quartz crystal. A current is passed through crystal causing it to contract. When placed nan AC filed it oscillates.

Tissue Factor (FIII)

Widespread constitutive protein involved in coagulation. Present in most interstitial tissue. May be expressed in hypoxia tissue damage + inflammation.

A 60 year-old man with anterior mediastinal mass is having a mediaastinoscopy. During induction they lose cardiac output, desaturate and drop their ETCO2. What is the best management strategy A. Adrenaline B. CPR C. CPB D. Place prone E. ?,

it is during induction so i think prone is what wiki says....i will accept

CSF composition compared to plasma

pH 7.32 lower than plasma Glucose 50% lower Protein 0.3g/L; plasma 60 g/L WCC < 1 Potassium ~50% Bicarbonate 90% of plasma Osmolarity same 289 Higher chloride

pH

pH = -log10 [H+] Normal H+ conc 40 NANOmoles per litre Gives pH of 7.4 10 x change in H+ conc per each unit on pH scale. pH of 6 1000 nano moles pH of 7 100 nano moles pH of 8 10 nanomoles

Allergic question, which is true A. Collect tryptase 8hours B. RAST test most sensitive/ specific C. Absent of trytase exclude anaphylactic D. Skin and intradermal test- sensitivity, specificity

? D ANSWER ??? Skin prick is easy to perform, safe and provides the best sensitivity/specificy combo. However it requires a skilled proceduralist Interdermal has higher sensitivity but higher anaphylaxis rate. RAST is 100% specific but only 75% senstive. Used to determine if pt has IgE antibodies to particular agents. It does not diagnoses anaphylaxis (as this requires 2 IgE to crosslink)

154. When treating serotonin syndrome, which treatment is incorrect? a. bromocriptine b. chlorpromazine c. diazepam d. cycloheptadine e. non-depolarising neuromuscular blockers

A

155. Regarding dexmedetomidine, which is correct? a. avoid in patients with 2nd degree heart block b. caution in pulmonary hypertension

A

157. You wish to compare a new method of blood pressure measurement with the current gold-standard. Which is the best statistical test to perform? a. Bland Altman plot b. linear correlation

A

32 y/o male. Weakness distal and prox muscles, infection 10 days ago, no sensory involvement, temp 37.8, facial weakness. Cause: A. Guillian Barre B. Myasthenia Gravis D. Poliomyelitis E. ?Acute encephalitis F. ?Polymyositis

A

A

A

A 12 year old child with hip dislocation at 4pm. Ate something 1 hour after injury. Now 11 pm. Best anaesthetic: A. RSI with ETT B. delay until next day then treat elective C. inhalational induction and continue with face mask D. Reduce immediately with iv sedation E. inhalational induction and continue with face mask

A

A patient in recovery post op total hip replacement develops crushing central chest pain, ECG shows ST segment elevation (NB- no BP etc given, beta blockade was not an option). The most appropriate action is to give a. Aspirin b. IV GTN c. IV heparin d. Calcium channel blocker e. T/L

A

Best position for tip of IABP is 1-2 cm: A Distal to Left subclavian artery B Proximal to Left subclavian artery

A

Cause of hypoxia in 1 lung ventilation? A Blood to non-ventilated lung B V/Q mismatch in ventilated lung C ?hypoxic pulm vasoconstriction

A

Clinically the most significant murmur in pregnancy is? A. MS B. AS C. MR D. AR E. VSD

A

Diastolic dysfunction is NOT caused by: A Adrenaline B Aortic stenosis C Hypertension D myocardial fibrosis E ?

A

During scoliosis surgery with monitoring of somatosensory evoked potentials, which tract are they mainly monitoring? A: Dorsal column B: Spinothalamic tract C: Lateral Corticospinal tract D: Cerebrospinal tract E: Anterior horn cells

A

Elderly COAD patient. On home oxygen. 24 hours of worsening condition. Various blood gases given: A. paO2 > 50, paCO2 70 HCO3 30 etc... B. C. D. E.

A

Half life or tirofiban? A. 2 hours

A

How do you calculate the inspiratory time constant for lungs A. resistance multiplied by compliance B. resistance divided by compliance C. compliance divided by resistance D. resistance minus compliance E. resistance plus compliance

A

In congenital diaphragmatic hernia A. there is hyperplasia of pulmonary arterioles in the hypoplastic lung B. right-sided lesions are more pathologically significant C. vasodilator drugs are contraindicated D. right-sided lesions through the foramen of Bochdalek are the most common E. intrapulmonary shunts are the major cause of cyanosis

A

Maximum dose of local infiltration of 0.5% bupivacaine in an x kg child? A. dose corresponding to 2.5 mg/kg; there was no option corresponding to 2 mg/kg

A

Negative predictive value is best described as A. the proportion of subjects with a negative test result who are correctly diagnosed B. Chance of a negative test in people without a disease. C. D. E.

A

Obesity dosing. For which drug is it most important to use lean body weight? a. Remifentanil b. Sux c. Alfentanil d. ? Vec

A

Patient with diastolic dysfunction. Is it caused by: A. Restrictive cardiomyopathy B. Dilated cardiomyopathy C. D. E.

A

Transfusion related acute lung injury (TRALI) A. can be caused by all homologous blood components, but particularly FFP (fresh frozen plasma) B. is associated with significantly elevated pulmonary artery pressure C. is the commonest cause of morbidity associated with blood transfusion D. should be treated with high dose steroids E. typically presents 24 hours following transfusion

A

What is a contraindication to an IABP? A. Aortic regurgitation B. Aortic stenosis

A

What is the ratio of breaths to compressions in neonatal resuscitation a. 1:3 b. 1:5 c. 2:15 d. 2:30

A

What is the ratio of compressions to breaths in neonatal resus? A. 3:1 B. 15:2 C. 30:2 D. 5:1 E. 15:1

A

When analyising a study containing a control and two test groups, the best statistical method to use is.... A. Analysis of variance B. Chi squared with Bonnferoni correction C. ? D. E.

A

cTnI remains elevated for up to? A. 5-14 days B. 1-2 days C. 1 month D. 1 year

A

The average expected depth of insertion of an oral endotracheal tube, from the lip, in a normal newborn infant is A. 7.5 cm B. 8.5 cm C. 9.5 cm D. 10.5 cm E. 11.5 cm

C

The commonest initial presenting feature in anaphylaxis is A. coughing B. desaturation C. hypotension D. rash E. wheeze

C

Troponin is elevated post-infarct A. 1-2 days B. 2-5 days C. 5-14 days D. 7-21 days E.

C

What is the major cause of death in a patient with perforation of the pharynx, oesophagus or trachea? A: failure to intubate B: failure to ventilation C: sepsis

C

What will not increase A-a gradient a. decreased cardiac output b. Increased FiO2 c. Decreased FiO2 d. increased shunt

C

You see a man in his 60s in clinic 1 week prior to laparoscopic cholecystectomy. He has dilated cardiomyopathy with an ejection fraction of 30%, but does not get dyspnoeic with normal activities of daily living. What is the most appropriate management of his heart failure? a. Frusemide b. Amiodarone c. ACE d. Digoxin e. Biventricular pacemaker

C

What statistical test would be best to evaluate the effects of ? 2 drugs in patients at ? 3 different points in time a. ANOVA b. Mantel Hantzel c. Crusckall Wallis d. Students t test

C ? A Apparently ANOVA is used to test for significant differences between the means of two or more groups. Kruskall-Wallis is the non-parametric equivalent

Severe pre-eclampsia. WORST treatment option: A. Magnesium B. Nifedipine C. Metoprolol D. SNP E.

C ?? Ans D Magnesium if questions say about hypertension Beta blockers could tip into LVF and APO however SIG says some B blockers (metoprolol, pindolol, propranolol) are ok. SNP is rarely used and not recommended due to hypotension, paradoxical bradycardia with severe PET and unknown risk of fetal cyanide toxicity

Patient indicated for prophylaxis of infective endocardititis A. amoxicillin orally 2 hours prior B. amoxicillin IV 1 hourly prior C. amoxicillin IV just before incision D. cefazolin IV 1 hour prior

C ??? ANSWER D ORAL Amoxillin 2g orally or 50mg/kg Allergic to penicillin: Clindamycin 600mg orally or 20mg/kg OR Cephazolin 2g orally or 50mg/kg OR Azithromycin or Clarithromycin 500mg orally or 15mg/kg INTRAVENOUS Ampicillin 2g IV or 50mg/kg Allergic to penicillin: Clindamycin 600mg IV or 20mg/kg IV OR Cefazolin or Ceftriaxone 1g IV/IM or 50mg/kg IV/IM

A term primip with meconium-stained liquor has a caesarian section. On initial assessment the neonate if pale and floppy with a heart rate of 90 bpm. Initial treatment should be: A. Positive pressure ventilation B. Dry and stimulate C. Suction the trachea D. Start CPR

C ARC Guidelines - Management of the Airway in the Presence of Meconium Stained Liquor • Suctioning before delivery makes no difference • Routine suctioning of babies who are vigorous no longer advocated (doesn't improve outcomes) • No evidence to support or refute endotracheal suctioning in the non-vigorous neonate • If tracheal suctioning is performed it must be accomplished before spontaneous or assisted respirations have commenced... stimulation to breath should not be provided beforehand

An infant is born with meconium stained liquor and is apnoeic and floppy... your first step should be a. Stimulate and dry b. Positive pressure ventilation c. Suction the trachea

C ARC Guidelines - Management of the Airway in the Presence of Meconium Stained Liquor • Suctioning before delivery makes no difference • Routine suctioning of babies who are vigorous no longer advocated (doesn't improve outcomes) • No evidence to support or refute endotracheal suctioning in the non-vigorous neonate • If tracheal suctioning is performed it must be accomplished before spontaneous or assisted respirations have commenced... stimulation to breath should not be provided beforehand

What is the best way to measure neonatal heart rate during resus? A. Palpate a femoral artery B. Palpate a carotid artery C. Auscultate the precordium D. Palpate the umbilical stump

C ARC Neonatal Resuscitation • "Heart rate can be determined by listening to the heart with a stethoscope (most reliable) or in the first few minutes after birth, by feeling for pulsations at the base of the umbilical cord" • Pulse oximetry can provide and accurate and continuous display of the heart rate within about a minute of birth" ?more accurate than auscultation

Ibuprofen

Not for < 1 yr

What changes occur in neuropathic pain

"Pain caused by lesion or disease of somatosensory system" (2011) Has characteristics such as burning, shooting, electrical pain, allodynia & hyperalgesia. It has a separate mechanism from nociceptive pain. Often many disease states will have components of nocicpetive and neuropathic pain i.e. a mixed picture. Neuropathic pain is poorly responsive to opioids and identifying leads to better treatment. Mechanisms are multiple May be central or peripheral There are risk factors for it's development How does it occur? 1. Activation threshold in afferents is lowered 2. Local releaed nerve growth factors cause sprouting 3. Increased peripheral inputs form pathological synapses with 2nd order nocicepticve neutrons 4. Touch/pressure fibres grow from lamellae grow into superficial dorsal horn to cause allodynia 5. Wind up occurs -increase in glutamate activity in doral horn leading to gene transcriptiom.

Parasympathetic Outflow in Spinal Tract

1. Cranium - CN III - VII & IX, X - Vagus transmits 75% of signals - heart, bronchial tree, liver, spleen, kidneys, GI tract (except distal colon) - 2. Sacrum Leave cord & synapse in ganglia in pelvis Sympathetic fibres also travel to ganglia Loss of sacral outflow can be compensated by cranial parasympathetic nervous system.

Details of immune reactions with blood product transfusion

1. Acute transfusion reaction (haemolytic) ABO incompatibility most common cause; other may be antibody incompatibility Kidd,duffy which are on RBC. Plasma antibodies react to donor blood RBC Severity - depends on degree of incompatibility, rate, amount of blood Anxiety dysphoria, dyspnoea, fever, lumbar pain, shock, DIC, renal failure, hypotension, death 2. Delayed Transfusion Reaction Occures 4-6 weeks with mild symptoms, Hct decrease, increase in bilirubin. Pt may have "-ve antibodies" but really just have very low level so not picked up on testing 3. Febrile non-haemolytic Antibodies against WCC in donor blood. This is less common if blood leucodepleted Temp >1, rigours, headache, back pain. Slow rate & give paracetamol. 4. Allergic reaction (to perhaps unknown component) Common, mild fever urticaria, headache, Pre-treat with anti-histamine. Rarely this can be an anaphylactic reaction. 5. Graft vs host disease Donor blood attacks immuncompromised recepient. Can also occur if immunocompetent individual receives blood from close relative! 4-30 days. high mortality 90%. Fever, rash, bullae, pancytopaenia. Reduce by irradiating blood to damage DNA in lymphocytes. 6. TRALI Antibodies in donor serum attack WBC. Decrease by using male donors to decrease antibody load as child birth increases antibodies. Causes resp distress.

Peripheral Chemoreceptors

1. Carotid Bodies (glossopharyngeal nerve) Respond to changes in pH/CO2/O2 Responds to oxygen tension rather than oxygen carried by haemoglobin (so anaemia won't stimulate) High blood flow to area much in excess of metabolic demand/weight 2. Aortic Bodies (vagus) Respond to Co2/O2/temp MOA Acetylcholine nicotinic receptors (blocked by neuromuscular drugs) Rapid action Cause increase in RR. eg hypotension will decrease perfusion so stimulate ventilation. Hypercapnia will increase responsiveness.

What cutaneour responses are there to regulate temp?

1. Ciculatory system of dermis has aterio-venous shunts which can redirect blood flow if required. Present in hands, feet, ears, nose, lips. If temp too high then these shunts will open & redirect blood superficially with resultant heat loss to environment. Blood flow can increase 30x. 2. Counter current mechanism, deep veins take heat from arteries travelling to peripheries.

Renal Blood Flow

125omL/min Calculate by working out plasma flow then relate this to haematocrit. Renal Plasma Flow Calculation based on clearance of para-amino hippuric acid (PAH) which is entirely excreted. Renal Plasma Flow = Clearance of PAH = Urine PAH x urine flow/plasma PAH Renal blood flow = Renal perfusion pressure/renal vascular resistance or = Renal plasma flow/1-haematocrit renal blood flow 2 arterioles in series + 2 capillary beds. Cortical radial arteries -> afferent -> glomerulus - > efferent arterioles -> peritubular capillaries or medulla

Time to normal platelet aggregation after abciximab?

48 hrs Monoclonal antibody against GP IIa/IIIb R used to prevent platelet aggregation; used in ACS + angio. Need 48 hrs before regional. Advers effects of MABs - Immunosuppression - infection (strict asepsis) - Wound healing - stop 1 week pre-op -Immune reactions - Cytokine release syndrome, serum sickness, anaphlyaxis, tumour lysis syndrome -Tb reactivation -Hep B - Cardiotoxicity - Lung injury - Renal effects - mild proteinuuria & HTN -Neurological - weakness Uses of MABS - Widespread - RA, cancer, B cell lymphoma, Crohn's, anti-platelet, MS, psoriasis, severe allergic asthma, osteoporosis,organ transplant, ank spond, nocturnal haematuria, pain (bony)

Coronary Blood Flow

5% of cardiac output 250mL/min; most of blood flow occurs in diastole Even in systole blood flow is not distributed equally and subendocardial flow ceases CBF = CPP/coronary resistance

Endocarditis prophylaxis in patient with MVR appropriate for? A. Dental procedure B. Rigid bronchoscopy C. Upper endoscopy with biopsy D. D&C E. Lithotripsy

A 2007 AHA Endocarditis Prophylaxis • All dental procedures that involve manipulation of gingival tissues or periapical region of teeth or perforation of oral mucosa • Procedures on respiratory tract • Procedures on infected skin, skin structures, or musculoskeletal tissue • Antibiotic prophylaxis solely to prevent IE is not recommended for GU or GI tract procedures including vaginal delivery and hysterectomy • Note: AN prophylaxis is not recommended for bronchoscopy unless the procedure entails incision of the respiratory mucosa

An infant with failure to thrive is noted to have an apical systolic murmur, weak pulses, with the femoral felt most easily. They most likely have a. Patent ductus arteriosis b. Ventriculoseptal defect

A ? Neither seem quite right. Definitely not PDA though Medscape - PDA "In the low birth weight premature infant, the classic signs of a patent ductus arteriosus (PDA) are usually absent. The classic continuous murmur is rarely heard. A rough systolic murmur may be present along the left sternal border, but a small baby with a large patent ductus arteriosus (PDA) and significant pulmonary overcirculation may have no murmur. In that case, typically, precordial activity is increased and peripheral pulses are bounding. The increased precordial activity is caused by the large left ventricular stroke volume. Bounding pulses are caused by the relatively low systemic arterial blood pressure due to the continuous runoff of blood from the aorta into the pulmonary artery." Medscape - VSD "The characteristic harsh, holosystolic murmur is loudest along the lower left sternal border (LSB), and it is well localized" Can't find anything about weak pulses

What is the oxygen concentration in a standard bottle of heliox? A. 21% B. 25% C. 30% D. 33%

A ?? Seems 28%, though this was apparently not an option A Review of the use of Heliox in the Critically Ill. Critical Care and Resuscitation 2006; 8: 64-72 "Mixtures with oxygen are colour coded brown with white shoulders while pure helium cylinders are brown alone. Heliox28 is a recently released dedicated delivery system with a fixed concentration of 28% O2 and 72% helium" TGA also lists 28% on product information page https://www.ebs.tga.gov.au/servlet/xmlmillr6?dbid=ebs/PublicHTML/pdfStore.nsf&docid=EEA07D98531013F5CA2577DD0001BE19&agid=(PrintDetailsPublic)&actionid=1

103 MC30b ANZCA version [2004-Aug] Q128, [2005-Apr] Q55 A patient with pulmonary hypertension secondary to lung disease presents for a laparotomy. Regarding this patient's anaesthetic management A. an alpha-agonist is the inotrope of choice B. hypothermia is protective against rises in pulmonary artery pressure C. isoflurane will tend to decrease pulmonary artery pressure D. ketamine is an appropriate anaesthetic agent E. right heart failure is not a concern

A A patient with pulmonary hypertension secondary to lung disease presents for a laparotomy. Regarding this patient's anaesthetic management * A. an alpha-agonist is the inotrope of choice - probably true and best answer: o there are No α-1 adrenergic receptors are present in the pulmonary circulation (Blaise, Anaesthesiology, 2003, 99(6):1421) so α-1 agonists are fine and may assist RV function by increasing coronary perfusion pressure (although some prefer dobutamine initially becuase it increases contractility and may pulmonary vasodilate) o the wording is confusing and might subequently change now. Both the Blaise article and Stoelting 5th ed. suggest that causes of hypotension are multifactorial and should be treated accordingly. Specifically pulm HTN crisis requiring inotropy, the 'inotrope' of choice might be milrione (or possibly dobutamine), however R heart ischaemia and low SVR (with fixed PVR) are important causes of hypotension specifically treated with noradrenaline * B. hypothermia is protective against rises in pulmonary artery pressure - false o Hypothermia increases PVR (A & A ,Volume 96(6), June 2003, pp 1603-1616) * C. isoflurane will tend to decrease pulmonary artery pressure - false o PVR does not change with volatiles except N2O which does increase PVR (Stoelting Pharmacology p47) o Isoflurane has no effect on baseline pulmonary vessel tone. (Blaise, Anaesthesiology, 2003, 99(6):1421) * D. ketamine is an appropriate anaesthetic agent - false o 'In patients who have pulmonary artery pressure, ketamine seems to cause a more pronounced increase in pulmonary than systemic vascular resistance' (Miller, p348) o 'The sympathomimetic properties of ketamine may preclude use in the setting of pulmonary hypertension (Yao, p96) o In-vitro ketamine increases PVR in rat lung...(and)...ketamine attenuates endothelium-dependent pulmonary vasorelaxation in response to acetylcholine and bradykinin ...(and)...sympathetic innervation of the pulmonary circulation does exist (Blaise, Anaesthesiology, 2003, 99(6):1421) * E. right heart failure is not a concern - false

Day 4 epidural. On 40 mg SC enoxeparin daily postoperatively (8 pm). When is the most appropriate time to remove the epidural? A. Day 5 at 12 midday B. Day 5 at 6 am C. Day 5 at 6 pm D. Day 6 at ?

A American Society of Regional Anaesthesia (ASRA) guidelines of neuraxial anaesthesia and anticoagulation "An indwelling epidural catheter should be removed 10-12 hours after the last dose of LMWH"

What is the commonest symptomatic cardiac condition in pregnancy a. Mitral stenosis b. Aortic stenosis c. Eisenmengers d. Tetralogy of fallot

A Anaesthesia for Caesarean Section in Patients with Cardiac Disease. Journal of The Pakistan Medical Association "Rheumatic heart disease at present is the most common cardiac disorder in pregnancy, with mitral stenosis (MS) as a single most prevalent lesion"

The median nerve A. can be blocked at the elbow immediately medial to the brachial artery B. can be blocked at the wrist between palmaris longus and flexor carpi ulnaris C. can be blocked at the wrist medial to flexor carpi ulnaris D. is formed from the lateral, medial, and posterior cords of the brachial plexus E. provides sensation to the ulnar half of the palm

A Ans B

Patient with Marfan's and 2 hours of severe chest pain, mild hypertension and ECG showing ischaemia. The next best step is urgent: A: CT B: TOE C: ? D: Angiography and PCI E: Thrombolysis

A Aortic dissection prompt diagnosis and emergency treatment are critical Cleveland Clinic Journal of Medicine 2011 Diagnosis and management of aortic dissection CEACCP 2009 Ans: A is probably preferred in common institution. TOE requires sedation, generally in haemodynamically unstable patient perioperatively.

43. New: You see a patient in your clinic for a total knee replacement. He is 65 and has atrial fibrillation for which he takes dabigatran. He is otherwise well. A spinal anaesthetic is planned. What is the correct advice regarding his medication? a. he should stop his dabigatran 7 days prior b. he should stop his dabigatran 3 days prior c. he should stop his dabigatran 3 days prior and have bridging enoxaparan d. he should stop his dabigatran the day before and have an INR on the day of surgery e. he should continue to take his dabigatran until the morning of surgery

A As per Horlocker in ASRA 2012 conf and BJA

Which radiological finding is most consistent with atlantoaxial instability in a patient with rheumatoid arthritis a. A 9mm gap between the anterior arch of C1 and the odontoid peg

A Atlantoaxial subluxation occurs in 25% of patients with severe RA Maxiumum gap between odontoid and arch of the atlas is 3mm - more than this is significant

Bleeding in trauma has been shown to be reduced by a. Tranexamic acid b. Recombinant factor VIIa c. DDAVP d. Prothrombinex

A CRASH-2 - Reanalysis published 2012 in Lancet Findings 10 096 patients were allocated to tranexamic acid and 10 115 to placebo, of whom 10 060 and 10 067, respectively, were analysed. 1063 deaths (35%) were due to bleeding. We recorded strong evidence that the effect of tranexamic acid on death due to bleeding varied according to the time from injury to treatment (test for interaction p<0•0001). Early treatment (≤1 h from injury) significantly reduced the risk of death due to bleeding (198/3747 [5•3%] events in tranexamic acid group vs 286/3704 [7•7%] in placebo group; relative risk [RR] 0•68, 95% CI 0•57-0•82; p<0•0001). Treatment given between 1 and 3 h also reduced the risk of death due to bleeding (147/3037 [4•8%] vs 184/2996 [6•1%]; RR 0•79, 0•64-0•97; p=0•03). Treatment given after 3 h seemed to increase the risk of death due to bleeding (144/3272 [4•4%] vs 103/3362 [3•1%]; RR 1•44, 1•12-1•84; p=0•004). We recorded no evidence that the effect of tranexamic acid on death due to bleeding varied by systolic blood pressure, Glasgow coma score, or type of injury. Interpretation Tranexamic acid should be given as early as possible to bleeding trauma patients. For trauma patients admitted late after injury, tranexamic acid is less effective and could be harmful.

Marfan syndrome. All EXCEPT- A. Aortic stenosis

A Cardiac Manifestations MVP AR Aortic dissection Conduction abnormalities

The plasma half-life of clopidogrel is: A. 6 hrs B. 14 hrs C. 24 hrs D. 7 days E. 14 days

A Clopidogrel product information "After a single, oral dose of 75 mg, clopidogrel has a half-life of approximately 6 hours"

Which of the following is NOT a side effect of cyclosporine a. Alopecia b. Hypertension c. Renal impairment d. Gum hyperplasia

A Cyclosporin product information • No mention of alopecia • Hypertension in up to 50% • Impaired renal function very common • Gum hyperplasia very common Cyclosporin -nephrotoxic, neurotoxic, causes hypertension (due to renal vasoconstriction and increased sodium reabsorption) increases risk of SCC & infection. Gingival hypertrophy, hirsutism which is not seen with tacrolimus (another calcineurin inhibitor). MOA - inhibits growth activity of T cells

Essential diagnostic criteria on ECG for LBBB A. Loss of septal Q's in V5 and V6 B. RSR in V1 C. Large slurred S in V6 D. T-waves opposite to direction of QRS E. QRS duration minimum 0.2 s

A Diagnostic criteria for LBBB: • 1) Total QRS duration >0.12 s. • 2) No secondary R wave in V1 to indicate RBBB. • 3) No septal q wave in V5, V6 or in leads further to the left (lead I and aVL in horizontal hearts).

When hearing loss occurs following spinal anaesthesia it is usually in which of the following frequency ranges: A. 125 - 1000 Hz B. 1500 - 3000 Hz C. 3500 - 5500 Hz D. 6000 - 10000Hz E. > 11000Hz

A Does spinal anesthesia cause hearing loss in the obstetric population? Anesth analg 2002 No it does not Does hearing loss after spinal anesthesia differ between young and elderly patients Anesth analg 2002 Hearing loss in low frequencies 125-500 Hz in young patient after spinal

Child with murmur- what would make it more likely for you to investigate if you heard the murmur A. persist in supine position B. louder or softer with various manouveres

A EXCLUDE ON HISTORY • Good exercise tol/playing • No sig fam hx: HOCM, CCD, sudden unexplained death • No sig perinatal hx: prematurity, maternal diabetes, foetal distress, birth asphyxia • No syndromes or congenital defects • Child >1 yr EXCLUDE ON EXAM • No dysmorphic features, VACTERAL • No thrill • Apex not displaces • Soft, grade <3 • Short duration • No radio-fem delay or rad-rad delay • Femoral pulse not bounding (L-R shunt) or decrease LVOTO • No cyanosis • Diastolic, pansystolic, continuous or very load need investigation

A diagnostic test has a sensitivity of 90% and a specificity of 99% in detecting a certain disease. From this we can conclude that A. the false positive rate of this test is 1% B. the false negative rate of this test is 1% C. the positive predictive value of this test is 90% D. the negative predictive value of this test is 90% E. this test would be a useful screening test for this disease

A False Positive Rate = 1 - specificity False Negative Rate = 1 - sensitivity And as a further reminder: PPV = TP / (TP + FP) And NPV = TN / (TN + FN)

The nerve supplying area of skin between greater trochanter and iliac crest: A. subcostal nerve B. ilioinguinal nerve C. genitofemoral nerve D. femoral nerve E. lat cutaneous femoral nerve.

A Fundamentals of Regional Anaesthesia A. Subcostal: sends fibre to the first lumbar nerve and its lateral cutaneous branch runs over the iliac crest to innervate the skin of the lateral aspect of the buttock as far as the greater trochanter B. Ilioinguinal: enters the inguinal canal accompanies the spermatic cord and supplies the skin of the rrot of the penis and anterior part of the scrotum, mons pubis and labium majorum. C. Genitofemoral: two branches. a. Genital branch enters the inguinal canal and supply the spermatic cord and innervate the same cutaneous area as the ilioinguinal nerve. b. Femoral branch: skin over the femoral triangle. D. Femoral: supplies the muscles and the skin of the anterior compartment of the thigh E. Lateral cutaneous nerve: a. Anterior branch: supplies the skin over the antero-lateral aspect of the thigh down to the knee b. Posterior branch: the skin of the lateral aspect of the leg from the greater trochanter to the mid-thigh Iliohypogastric nerve innervate skin overlying the lateral aspect of the buttock and runs medially and superficial to the inguinal canal to innervate the skin over the pubis.

Most common cause of awareness? A. Failure to check apparatus

A Human error on many other versions of this question

How do you calculate the inspiratory time constant for lungs A. resistance multiplied by compliance B. resistance divided by compliance C. compliance divided by resistance D. resistance minus compliance E. resistance plus compliance

A Lung Mechanics & Mechanical Ventilation - Lexington Pulmonary and Critical Care Mathematically, the time constant is defined as compliance multiplied by the airway resistance and the resulting value has units of seconds of time

Patient with severe Rheumatoid arthritis. Has C1/C2 instability. Most likely C-spine Xr finding would be A. Anterior Atlantoodental interval >9 B. Increased sagittal diameter C. Posterior atlantodental interval >14 D. Midpart of C1 over C2 E. Tear drop sign of C2

A Most common type of atlanto-axial instability is anterior AAS (80%), where C1 moves forward on C2 from destruction of transverse lig

Long-standing T6 paraplegia. All present EXCEPT ? A. Flaccid paralysis B. Poikilothermia C. Autonomic hypereflexia D. Labile BP E. Hyperkalaemia with suxamethonium

A OHA p240 Return of muscle tone and reflexes after perhaps 3 days to 8 weeks Rest are longer term (i.e. 9 months)

Malignant hyperthermia. The number of people in the community at any given time with a predisposition is called the: A. Prevalence B. Incidence C. D. E.

A Prevalence = measures how much of some disease or condition there is in a population at a particular point in time. Incidence = the rate of occurrence of new cases of a disease or condition. Incidence of MH (per wiki) is 1:15000 anaesthetics for children and adolescents and 1:50-150000 anaesthetics for adults. Prevalence for susceptabiity may be as high as 1:2000

Where should the tip of an IABP lie a. 2cm distal to the left subclavian b. 2 cm proximal to the left subclavian c. 2cm proximal to the renal artery d. 2 cm distal to the renal artery

A The Carina as a Useful Radiographic Landmark for Positioning the Intraaortic Balloon Pump. Anaesthesia & Analgesia. Vol. 105, No. 3, September 2007 "Ideally, the tip of the balloon should be positioned 2-3 cm distal to the origin of the left subclavian artery (LSCA)"

After an infusion of normal saline causing isovolumetric haemodilution what occurs? a. Increased cardiac output b. Increase oxygen extraction c. Capillary vasodilatation

A Though B and C possibly too? Isovolaemic haemodilution leads to a decrease in [Hb]. Without a significant increase in either SaO2 or paO2 (not possible in the standard state at sea level and room air), the only way to return oxygen flux to normal is through an increase in CO. CO is a product of heart rate (HR) and stroke volume (SV), and is dependent upon preload, afterload and myocardial contractility. Increased CO occurs through several factors: 1. Decreased blood viscosity an increased tendency for venous return (VR) SV and thus CO. 2. Decreased blood viscosity SVR and afterload CO. Poiseuille's law shows resistance in a vessel is directly related to viscosity (and length, and inversely related to radius to the fourth power). 3. Metabolic Autoregulation - Local tissue factors lead to vasodilation in order to increase regional blood flow and restore oxygen delivery to normal. Increased regional blood flow tendency for VR CO. Any small decrease in systemic blood pressure (BP) due to the subsequent fall in SVR as regional circulations vasodilate, is quickly detected by the carotid and aortic baroreceptors, leading to an increased sympathetic outflow and thus increased HR, SV and subsequently CO. Increased tissue O2 extraction: Local factors independent of the increase in CO also assist in maintaining tissue oxygen supply despite a fall in oxygen flux. These adaptions occur to increase the oxygen extraction by tissues. An important mediator of this is a right-ward shift in the oxyhaemoglobin dissociation curve, increasing the p50 (normal 26.6 mmHg) and thus assisting the offloading of oxygen from Hb. Also, the lower pO2 places the tissues on a steeper position of the oxyHb dissociation curve, further facilitating oxygen unloading as greater amounts of O2 are off-loaded per unit drop in pO2

Hypocalcaemia - earliest sign: A. Tingling of face and hands B. Chvostek's sign C. Carpopedal spasm D. E.

A Wiki CATS go numb (Convulsions, Arrythmias, Tetany and numbness/parasthesias in hands, feet, around mouth and lips) Oral, perioral and acral paresthesias, tingling or 'pins and needles' sensation in and around the mouth and lips, and in the extremities of the hands and feet. This is often the earliest symptom of hypocalcaemia.

The features of Pierre Robin sequence include cleft palate, micrognathia and: A. Glossoptosis B. Craniosynostosis C. Macroglossia D. Microstomia

A Wiki PRS is characterized by micrognathia, Glossoptosis (posterior displacement or retraction of the tongue), and cleft palate

Blood flow across which of the following is used to estimate pulmonary artery pressures during echocardiography? A. Tricuspid valve B. Pulmonary valve C. Mitral Valve

A for Systolic PAP, Tricuspid valve, for Mean / Diastolic PAP, Pulmonary valve

What is the innervation of the hard palate? A. Greater palatine and nasopalatine B. Superior labial nerve and greater palatine nerve C. Interior orbital nerve and nasopalatine nerve D. Glopharyngeal nerve and ... E. Anterior ethmoidal nerve and

A http://ozradonc.wikidot.com/anatomy:focused-hard-palate "The hard palate is innervated by branches of the maxillary nerve, both of which initially pass through the pterygopalatine ganglion. The greater palatine nerve descends through the greater palatine foramen with its companion artery, and runs anteromedially to supply the mucosa of the posterior hard palate. The nasopalatine nerve descends through the incisive foramen to supply the most anterior parts of the hard palate"

Features of ventricular tachycardia DO NOT include a. Absence of p waves b. Monophasic waves c. Prominent R wave in V1 d. A-V dissociation

A • False i.e AV dissociation is a sign of VT - P-waves occurring at different rate • Monoasic R-wave in V1 and V6 consistent with RBBB pattern VT • Prominent R-wave in V1 = RBBB pattern VT • AV dissociation consistent with VT

Residual current devices: A. Compare current between active and neutral lines B. C. D. Must be fitted in cardiac protected areas E. Must be fitted in operating theatres

A All operating theatres and ICU are cardiac protected and must have either line isolation monitor or residual current device, and have equipotential earthing junctions.

Contraindication to IABP? A. Aortic regurgitation B. Aortic Stenosis

A Also: Peripheral vascular disease, aortic stent, aneurysm

Regarding a Thallium scan: A. High NPV B. Less useful in comparison to a DSE

A Binds specifically to myocardium and show's movement of muscle.

Amniotic fluid embolism. Cause of death in first half hour ? A. Pulmonary hypertension B. Malignant arrhythmia C. Pulmonary oedema D. Hypovolaemic shock E.

A CEACCP 2007

Patient with Hx COAD and suspected pneumonia - clinical findings supporting R pneumonia on examination: A. R Dull percussion note & increased vocal resonance B. R Dull percussion note & decreased vocal resonance C. R Decreased air entry D. Tracheal deviation to left E. Tracheal deviation to right

A Pneumonia = dull percussion note (consolidation doesn't transmit sound as well as air), decreased breath sounds, may be bronchial breath sounds, increased vocal resonance (through consolidated lung)

Penetrating injury to chest. What part of the heart most likely injured? A. RV B. LV C. RA D. LA E. Right coronary

A right ventricle 43% left ventricle 34% right atrium 16% left atrium 7%

Regarding post craniotomy pain: A. Local infiltration proven to reduce long-term pain B. Local more painful than discrete nerve blocks C. Local infiltration more efficacious than discrete nerve blocks D. Local infiltration more efficacious than opioid analgesia E. Local infiltration more efficacious with clonidine included

A Ans A Acute pain management - scientific evidence - third edition 2010 p247 Local anaesthetic scalp block A comparison between scalp nerve block and morphine showed no relevant differences in any analgesic parameters (Ayoub et al, 2006 Level II). Scalp infiltration was also no more effective than IV fentanyl (Biswas & Bithal, 2003 Level II) However, comparisons of scalp blocks with bupivacaine or ropivacaine and placebo showed better analgesia with the local anaesthetic blocks in a number of trials. Scalp infiltration with ropivacaine also reduced the incidence of persistent pain 2 months after craniotomy, from 56% to 8 % A comparison between SC local anaesthetic infiltration and occipital/supraorbital nerve block showed no difference between groups in the postoperative period, but nerve blocks were less painful than infiltration analgesia

A patient undergoing liver surgery has a venous air embolism, what is the most appropriate position to place them in: a. Reverse trendelenburg, right side up b. Reverse trendelenburg left side up c. Reverse trendelenburg, neutral d. Trendelenburg right side up e. Trendeleburg left side up

A (consensus AMS & kate)

The incidence of fat embolism syndrome following a unilateral closed femoral fracture is A. 0 -3% B. 4 - 7% C. 8 -11% D. 12 - 15% E. 16 - 19%

A - 0-3% Fat Embolism - CEACCP 2007 "Any single long bone 1-3%" "It has been reported in up to 33% of patients with bilateral femoral fractures"

What is the half life of clopidogrel? a. 6 hours b. 14 hours c. 24 hours d. 7 days

A - 6 hrs (google)

You see a patient in your clinic for a total knee replacement. He is 65 and has atrial fibrillation for which he takes dabigatran. He is otherwise well. A spinal anaesthetic is planned. What is the correct advice regarding his medication? a. he should stop his dabigatran 7 days prior b. he should stop his dabigatran 3 days prior c. he should stop his dabigatran 3 days prior and have bridging enoxaparan d. he should stop his dabigatran the day before and have an INR on the day of surgery e. he should continue to take his dabigatran until the morning of surgery

A - CHADSVASC65 - Direct thrombin inhibitor. Pro drug, 5% bioavailability, 2 huyr peak levels, 8hr half life in normal renal function after single dose, 80% renally excreted.

Performing a caudal block in a child and add clonidine to prolong duration of block. What significant complication is increased? A. Sedation B. Urinary retention C. Bradycardia D. PONV E. Respiratory depression

A - Clonidine produces post-op sedation OCH; morphine increases urinary retention. *

64. The desflurane vaporizer is heated because of its A. High SVP B. High boiling point C. Low SVP D. High MAC E. Low MAC

A - High SVP Critical temp - temp above which substance exists as gas no matter how much pressure is applied. At or below this it may exist as liquid or gaseous form - later called vapor. SVP - In any liquid some molecules near the surface will have enough energy to leave liquid, becoming vapor (this is evaporation). Requires heat. Increasing temp will increase evaporation. In a closed contained an equilibrium will be reached with molecules leaving liquid + re-entering. At this point the vapor is saturated and the pressure it exerts is called SVP. Boiling - as liquid is heated more energy is added to system and liquid can enter into vapour phase within the liquid (which forms bubbles). Below this temp the bubble would be crushed by greater atmospheric pressure). Thus boiling point = temp at which SVP equals atmospheric pressure. Vaporisers need to deliver an accurate adjustable concentration of vapor. Desflurane 1) has a very high SVP (88.5 at 20C). A conventional vaporizer would require high fresh gas flows to dilute the agent within clinically useful concentrations making it uneconomical. 2) Low boiling point of 23.5 degrees which means it will intermittently boil at room temp causing large fluctuations in agent delivery. Heated vaporizer heats liquid desflurane to a constant temperature (39C) so the SVP is constant (nb the SVP may change, the BP is when the SVP = atmospheric pressure and that won't change at constant altitude). In addition the SVP at 39 degrees is almost 2 atmospheres (194kPa) so the carrier gas does not need to be pressurized to overcome resistance in the chamber Causes 26 times global warming effect of servo per 1 hr of anesthetic. Desflurane injected into circuit rather than being taken up by gas flow as it would require a high fresh gas flow to dilute it sufficiently.

Performed a brachial plexus block. Normal sensation still remains in medial forearm. Which part of brachial plexus is most likely to have been missed A. Inferior trunk B. Ulnar nerve C. Median brachial cutaneous nerve D. Anterior division E. Posterior cord

A - Inferior Trunk Medial cutaneous nerve of forearm /medial antebrachial cutaneous which comes from medial cord, division, inferior trunk.

Iron deficiency anaemia: A. Low ferritin, low serum iron B. Low ferritin, low TIBC C. Elevated ferritin, low marrow iron D. Elevated ferritin, ? E. Elevated ferritin, ?

A - Low ferritin, low serum iron

What drug should NOT be used for tocolysis in 32/40 female? A. Indomethacin B. Magnesium C. Nifedipine D. Salbutamol

A - Magnesium ineffective in preterm threatened labour (chang)

Severe asthma attack. Given continuous nebs & IV hydrocortisone but not responding. PaCO2 low. SpO2 low. Next appropriate treatment? A. IV Magnesium B. IV Aminophylline C. Heliox D. IV salbutamol infusion E. Intubate/ventilate

A - MgSO4

Best renal protection for endoluminal AAA repair? A. NaCl B. NAC

A - NaCl Endoluminal EVAR repair. Liberal use of contrast media for deployment & placement of graft. No evidence for diuretics.

Systemic Vascular resistance index (SVRI) is equal to: A. SVR x BSA B. SVR / BSA

A - SVR x BSA Cardiac INDEX = CO/BSA SVR = 80 x (MAP - RAP)/CO (800 - 1200 dynes - sec.cm-5) Systemic Vascular Resistance Index (SVRI) 80 x (MAP - RAP)/CI (1970 - 2390 dynes.sec.cm-5/m2)

Most common cause of mortality post transfusion? A. TRALI B. Contamination/infection C. Mismatched blood D. GvHD E. Anaphylaxis

A - TRALI

A 29 year old female undergoes craniotomy for posterior fossa tumour. Which of the following is an absolute contraindication to the sitting position A. Patent ventriculo-atrial shunt B. Previous back surgery C. Pacemaker D. Small patent foramen ovale E. Oesophageal stricture contraindicated for transoesophageal echocardiogram

A - VA shunt ABSOLUTE CONTRAINDICATIONS • Patent ventriculo-atrial shunt • Severe cardiovascular disease • Large patent foramen ovale or other pulmonary-systemic shunt • Cerebral ischaemia when upright and awake • Anaesthesia or surgical team not familiar with the position

18/12 old undergoing routine SV GA under LMA. Sudden onset SVT with HR 220 BP 84/60 ETCO2 32 SpO2 98.Management: A. Adenosine 100mcg/kg B. DCR 2J/kg C. DCR 4J/kg D. Amiodarone 5mg/kg E. CPR

A - adenosine

Photograph of an Arndt endobronchial blocker. Orifice labelled 'X'. What goes in 'X'? A. Bronchoscope B. the circuit/Users/kateromeril/Desktop/9957885_orig.jpg C. the guidewire D. Nylon... E. Blocker

A - arndt endobronchial blocker (not aintree!)

Young infant with Failure to Thrive. Born on the 20th percentile now is on the 5th percentile. Found to have a systolic murmur, tachynpnoea with weak femoral pulse. The most likely diagnosis is a. Coarctation b. HOCM c. PDA d. AS

A Coarctation of Aorta • Cardinal Features: resting systolic hypertension, absent or diminished femoral pulses, wide pulse pressure • Signs in 1st year of life: tachypnoea, pallor, tachycardia, failure to thrive, cardiomegaly, pulmonary oedema, hepatomegaly

A 30 year-old pregnant patient develops contractions at 30/40 weeks gestation which of the following cannot be used for tocolysis A. Clonidine B. Indomethacin C. Magnesium D. Salbutamol E. Nifedipine

A - clonidine has no tocolytic effect Tocolysis in Pregnancy Aim to decrease time to delivery of foetus to allow time for antenatal steroids to be given. Corticpsteroids 24 - 34+6 weeks associated with decreased ARDS. Best if >24 hrs but still can give if delivery sooner as decreased risk of metal death. No maternal side effects. Cautiion in sepsis, Tb. Betamethasone or dexamethasone used but no evidence of superirority. - Indomethacin (non selective COX inhibitor) is an effective agent for tocolysis, up until about 34 weeks of age, when it becomes contraindicated due to premature PDA closure. -Magneisum - evidence for effect Salbutamol - Beta mimetics used. - Nifedipine - Evidence for benefit of placebo. Insufficient evidence for nitric oxide donors eg nitrates as TOCOLYTIC. *

Best position for IABP is 1-2 cm: A. Distal to Left SCA B. Proximal to Left SCA C. Distal to artery of Adamkiewicz D. Distal to renal artery E. Proximal to renal artery

A - distal to left SCA

During scoliosis surgery with monitoring of somatosensory evoked potentials, which tract are they mainly monitoring? A: Dorsal column B: Spinothalamic tract C: Lateral Corticospinal tract D: Cerebrospinal tract E: Anterior horn cells

A - dorsal column

Indicator in sodalime? A. Ethyl violet B. Potassium permangenate C. Blue ? D. ? E. ?

A - ethyl violet

The features of Pierre Robin sequence include cleft palate, micrognathia and: A. Glossoptosis B. Craniosynostosis C. Macroglossia D. Microstomia

A - glossoptosis Pierre robin features - glossoptosis, micrognathia and cleft palate

What is the immediate compensation for the dilutional anaemia when 3 litres of normal saline is given at the start of a case? A. Increased CO B. Capillary dilatation C. Increased oxygen delivery D. Right shift in the oxygen dissociation curve

A - increased Cardiac output

Advantages of bronchial blockers over double lumen tubes: A. Able to achieve lobar isolation B. Lower cuff pressure C. Quicker deflation of isolated lung D. Pneumonectomy E. Lower incidence of malposition

A - lobar isolation Bronchial blockers are a better choice for patients with difficult airways, for selective lobar ventilation, or wherever postoperative mechanical ventilation is contemplated

Risk factor for PPH? A. Prolonged labour B. Age <20 yrs old C. Primiparity D. FV Leiden Deficiency (yes it said deficiency!) E. Oligohydramnios

A - long labour

A 78 year old man with past difficult intubation for arm surgery. Supraclavicular block with 25 mls 0.5% bupivacaine. Shortly after begins convulsing. INITIAL management? A. Midazolam 5mg B. Intralipid 20% 1.5 ml/kg C. Thiopentone 150mg D. Suxamethonium 50mg E. Propofol 50mg

A - midazolam *

Central anticholinergic syndrome, which is NOT true: A. Will improve with neostigmine B. Peripheral anticholinergic symptoms C. Caused by anti-Parkinson drugs D. CNS depression E. Associated with agitation, delirium, and ???

A - need physostigmine

Amiodarone PK

A - poor absorption D - High protein binding > 95%, takes up binding sites for warfarin, phenytoin, beta blockers. High Vd M- Liver to produce some active metabolites E - Biliary but also skin, glands. Elimination half life long ~ 50 days. CI in porphyria.

Dilated CM (LVEF 30%). No dyspnoea with ADLs. Best management? A. Start ACEI B. Stop beta-blocker

A - start ACEi

Endocarditis prophylaxis is appropriate in? A. Unrepaired CHD B. Cyanotic heart disease repaired 12 months ago

A - unrepaired cyanotic heart disease 2008 Australian Guidelines (still current) Prophylaxis only required for the following conditions which are considered HIGHEST RISK 1. Prosthetic cardiac valve or presence of material in valve repair 2. Prev infective endocarditis 3. Cardiac transplantation with subsequent valvulopathy 4. Congenital heart disease BUT only if - unprepared cyanotic defects incl shunts/conduits - repaired defects with prosthetic material in first 6 months since repair - repaired defects with residual defects close by (these don't get endothelialised) - rheumatic heart disease NOT an indication except in indigenous Australians. Regime : Amoxycillin 2g orally 1 hr pre-op e.g. dental (child 50mg/kg) Augmentin 2g IV just prior (or IM 30 mins prior) If hypersensitive to penicillin use clindamycin 600mg 1 hr before or IV infusion 600mg at incision. Vancomycin 1g IV over 60mins also option. Note - Procedures that require prophylaxis are in guidelines. Always required in dental extractions, avulsion, peridontal procedures. Respiratory procedures like branch if there is biopsy, I&D abscess, epidural through infected skin (! would one do this) Consider in dental procedure if long, periodontal disease. Not required in oral exam; infiltration of LA; intubation; urethral catheter; vaginal delivery; TOE; endoscopy

When do most patients with SAH rebleed? A. 0-24 hours B. 1-2 days C. 4-10 days D. 14-28 days

A 0 -24 hrs "Rebleeding after initial aneurysmal subarachnoid hemorrhage (SAH) can have substantial impact on overall patient outcome....older studies have suggested rebleeding occurs in about 4% of patients during the first day after initial aneurysmal bleed"

buprenorphine patch removed morning of surgery. What time till PLASMA reaches half original level A. 12 hours B. 18 hours C. 24 hours D. 30 hours E. 36 hours

A 12 hrs Norspan product information "After removal of a NORSPAN patch, buprenorphine concentrations decline, decreasing approximately 50% in 12 hours (range 10 - 24 h)"

Half life of tirofiban: A. 2hrs B. 8hrs C. 12hrs D. 24hrs E. 15 minutes

A 2 hours

Intubating over a bougie. Rotate ETT? A. 90 degrees anticlockwise B. 90 degrees clockwise C. 180 degrees

A 90 anticlockwise

White cylinder with grey shoulder? A. CO2 B. Air C. O2 D. N2O E. N2

A Carbon dioxide • oxygen (white) nitrous (blue) • nitrogen (black) • carbon dioxide (grey), helium (brown) argon (dark green) • medical breathing gas mixtures containing oxygen and an inert gas must be marked with alternating white and the second gas's colour on the shoulder e.g. black + white for air, brown + white for Heliox, etc.

Bowel surgery patient. Best method for intraoperative optimization of fluid therapy? A. Arterial pulse pressure contour analysis B. CVP C. PAOP D. UO

A Controversial topic (kate) Perhaps SVV or oesophageal doppler would be better? Update in Anaesthesia - Enhanced recovery after surgery - current trends in perioperative care "Accurate fluid management and resuscitation requires regular reassessment of physiological parameters and, where available, invasive haemodynamic monitoring. Historically this has been provided by pulmonary artery catheters, but these are increasingly being replaced by targeted stroke volume optimisation with oesophageal Doppler probes. Given the relative simplicity and lack of complications, where available the latter is the recommended method of guiding fluid administration in the operating room"

Acute visual loss after non-ocular surgery is most commonly caused by A. ischaemic optic neuropathy B. prolonged direct compression of the globe C. cortical blindness D. retinal artery occlusion E. electrolyte imbalance

A Ischaemic Postoperative blindness is a much rarer complication of anaesthesia; its incidence is 1 in 125 000 anaesthetics (0.0008%). - The most frequent cause is ISCAHEMIC OPTIC NEUROPATHY.

Oxycodone 20mg SR / Naloxone 20 mcg: A. Decreased constipation B. Reduced risk of drug misuse/abuse C. Respiratory depression

A This is Targin; modified released tablet. Naloxone binds to gut wall mu receptors; does not affect systemic analgesic effect of oxycodone due to pronounced first-pass effect and its very low oral bioavailability upon oral administration Constipation with targin compared with oxycodone CR was about 25% lower among people with a history of constipation, and 7% lower in an unselected group. NNT 4 Contraindicated in moderate or severe hepatic impairment Diarrhoea may be a possible effect of naloxone, especially at the beginning of treatment, and tends to be transient. Oral administration of naloxone is unlikely to result in a clinically relevant systemic effect due to a p

The time constant of the lung is calculated by a. Compliance x resistance b. Compliance plus resistance c. Compliance /resistance d. Resistance/compliance

A compliance x airway resistance units = seconds of time

Most safe side to insert subtenon block A. Inferonasal B. Inferotemporal C. Medial D. Superonasal E. Superotemporal

A inferonasal

Thallium scan: A. High negative predictive value B. High positive predictive value C. Not as good as a dobutamine stress echocardiography D. E.

A. AHA/ACCHA Guidelines: The 2 main techniques used in preoperative evaluation of patients undergoing noncardiac surgery who cannot exercise are to increase myocardial oxygen demand (by pacing or intravenous dobutamine) and to induce hyperemic responses by pharmacological vasodilators such as intravenous dipyridamole or adenosine. The most common examples presently in use are DSE and intravenous dipyridamole/adenosine myocardial perfusion imaging with both thallium-201 and technetium-99m. From the 2007 AHA guidelines, 'because of a very high sensitivity of abnormal stress nuclear imaging studies for detecting patients at risk for perioperative cardiac events, the negative predictive value of a normal scan has remained uniformly high at approximately 99% for MI or cardiac death'. IE. NEGATIVE = MORE LIKELY NOT GOING TO RUN INTO TROUBLE! Thallium scanning in coronary artery disease (CAD) using radioactive thallium-201(201-Tl) is a widely available technique which is sensitive, accurate and noninvasive. It detects CAD accurately in patients with: atypical chest pain and a positive exercise ECG or typical chest pain and a negative exercise ECG.

What is approximately the systolic blood pressure in an awake neonate (mmHg) A. 55 B. 70 C. 85 D. 100 E. 115

ANSWER B neonatte 50-90/25-60. Hr 80-200. > 1 yr SBP = 80 + (age x 2)

Which type of aortic dissection can be managed conservatively/non-operatively A. Debakey 1 B. Debakey 2 C. Stanford A D. Stanford B E. Stanford C

ANSWER D

Pregnant woman with ?MS. Develops SVT. Try vagal manouveres without success. A. Adenosine 6mg B. DCR C. Amiodarone D. Atenolol E. ?

A. Adenosine or shock if unstable BJA 2004 "SVT in Pregnancy" Drug use in labour or LSCS can precipitate SVT. Careful with synto, and ephedrine. Regional anaesthesia --> decreased filling and may precipitate. Ensure aortocaval tilt is used. Adenosine, a naturally occurring purine nucleotide, transiently depresses sinus node activity and slows atrioventricular conduction, and is effective in terminating SVT. It is rapidly metabolized with an elimination half-life of less than 10 s, making it ideally suitable for use in pregnancy. If adenosine fails, other antiarrhythmics may be indicated and the risk of their use should be weighed against the risk of continuing SVT. Beta-blockers have been used extensively in pregnancy, to treat maternal hypertension and cardiac problems, and are generally well tolerated. They are the agents of choice in Wolf-Parkinson-White syndrome, where AV nodal blocking drugs may lead to acceleration of conduction through the accessory pathway and the arrhythmia being sustained. Verapamil, a calcium channel-blocking agent, is as effective as adenosine in converting an SVT to sinus rhythm. Peripheral vasodilation and negative inotropy are unwanted side-effects. There are reports of its safe use in pregnancy for treatment of SVT. Digoxin has been used in all stages of gestation for maternal and fetal indications without causing harm. Amiodarone is best avoided because of its potential teratogenic effects and reports of fetal toxicity, but again there are reports of its safe use during pregnancy. Synchronized electrical cardioversion may necessary for SVT resistant to pharmacological therapy, particularly if hypotension develops. Direct current electrical shock has been used at all stages in pregnancy without significant complication. The current reaching the fetus is thought to be negligible. However, transient fetal dysrhythmia has been described, and monitoring of fetal heart rate during maternal cardioversion is advisable. Implantable devices have been successfully used during pregnancy for malignant tachyarrhythmias. Both temporary and permanent endocardial pacing has been used in pregnancy, although mainly in the treatment of bradyarrhythmias.

Diastolic dysfunction is NOT caused by: A Adrenaline B Aortic stenosis C Hypertension D myocardial fibrosis

A. Adrenaline Left ventricular (LV) diastolic dysfunction refers to abnormalities of diastolic distensibility, filling, or relaxation, regardless of whether LV ejection fraction (LVEF) is normal or abnormal and whether the patient is symptomatic or not. The major causes of diastolic dysfunction include: chronic hypertension hypertrophic cardiomyopathy aortic stenosis coronary artery disease restrictive cardiomyopathy (a rare condition in which the heart muscle is infiltrated, and made stiff, by abnormal cells, protein, or scar tissue. The most common cause of restrictive cardiomyopathy is amyloidosis, a disease in which protein-like substance is deposited within the body's tissues. Other causes include sarcoidosis and haemochromatosis.) aging (Whether age alone causes stiffening of the ventricles, or whether such stiffening is related to "subclinical hypertension" or some other definable medical condition, is not yet worked out.)

Child-Pugh score. Components ? A. Bilirubin / albumen / INR (yes INR, not PT), ascites, encephalopathy B. Various other options including AST/ALT, GGT, PT C. ? D. ? E. ?

A. Child-Pugh: Clinical or biochemical variable Points scored for increasing abnormality 1 / 2 / 3 The Pugh modification of Child's classification is used to estimate the risk of mortality in patients with liver disease undergoing surgery. Points from each variable are added to make the total score. A total score of 5 or 6 is considered Child's class A and is associated with a low operative mortality risk (<5%); a total score of 7-9 (Child's class B) carries a moderate risk (25%) and total score of 10-15 (Child's class C) carries a high risk (>50%). Although this classification was originally used in patients undergoing portosystemic shunts, the variables included have been shown to be predictive of outcome for all types of abdominal surgery in patients with liver disease. Other predictors of poor outcome include malnutrition, emergency surgery, sepsis, and blood loss.

Cell saver. Which does NOT get filtered ? A. Foetal cells B. Free Hb C. Platelets D. Clotting factors E. Microaggregates of leukocytes

A. Fetal cells. But they've shown it's no worse than labour/delivery and the UK College recommends for LSCS now. CEACCP: Red cells are retained. Plasma, platelets, heparin, free haemoglobin, and inflammatory mediators are discarded with the wash solution. Concerns about amniotic fluid embolism, fetal debris contamination, and rhesus sensitization previously limited such use. However to date, there have been no proven cases of amniotic fluid embolism caused by reinfusion of salvaged blood in the literature. NICE guidelines support use. Use 2 sucker; one for amniotic fluid; other for blood. Use leucodepletion filters. In pregnancies involving an Rh-negative mother and an Rh-positive fetus, a Kleihauer test should be performed in the immediate post-partum period. This will allow the calculation of the appropriate dose of anti-D immunoglobulin (usually 125 IU ml21 of fetal blood) required.

Pre-eclamptic woman, BP 170/110, headache, proteinuria 1.2g. Which of the following NOT to use for control of her hypertension: A. Magnesium B. SNP C. GTN D. Hydralazine E. Metoprolol

A. Magnesium - not for control of HTN Severe PET as > 160/110 + headache and proteinuria. Management of PET: - early diagnosis, control of BP, prevention of convulsions and timely delivery, strict fluid balance BP control: Maintain MAP 100-140 (130/90-170/110). Sudden drop can compromise placental perfusion. Drugs used: - Hydralazine 5mg increments or infusion - Labetalol 50mg increments IV or 100mg PO q30min - Methyldopa 1-3g PO per day - Nifedipine 20mg PO (drops BP ++ with magnesium) - SNP infusion - may cause excessive hypotension, good for emergencies, risk of cyanide toxicity to fetus - GTN - as above, risk of methaemoglobinaemia Magnesium is NOT for control of BP. It is for prevention of fits and treatment. 4g loading + 1g/h infusion

A 4 year old child with Arthrogrophysis multiplex congenita for dental surgery. Jaw rigidity post induction. Likely cause ? A. Temporomandibular joint involvement/ TMJ rigidity B. Inadequate depth of anaesthesia C. Inadequate muscle relaxation/ inadequate sux D. Masseter spasm E. ?

A. TMJ rigidity No increased risk MH. Arthrogryposis multiplex congenita refers to a variety of conditions that involve congenital limitation of joint movement. Intelligence is relatively normal except when the arthrogryposis is caused by a disorder or syndrome that also affects intelligence. 2 major types of arthrogryposis multiplex congenital classic has symmetric contractures in limbs; limbs. Distal is hands and feet. Etiology Impaired in utero movement for > 3 wk can result in AMC. - Physical limitation of movement (eg, due to uterine malformations, multiple gestations, oligohydramnios) - Maternal disorders (eg, multiple sclerosis, impaired uterine vascularity) - Fetal disorders (eg, neuropathies; myopathies, including muscular dystrophies; connective tissue abnormalities; impaired fetal vascularity; anterior horn cell disease) Symptoms and Signs Deformities are prominent at birth; not progressive; but an underlying condition may be. Joints are contracted in flexion or extension. Endotracheal intubation during surgery may be difficult because children have SMALL IMMOBILE JAWS Other abnormalities that rarely accompany arthrogryposis include microcephaly, cleft palate, cryptorchidism, and cardiac and urinary tract abnormalities. *

Which of the following is suggesting of an inhaled foreign body in a child on chest x ray a. Foreign body visible in front of airway b. Hyper-expanded hemithorax c. Collapse

ALL RCH CPG - Inhaled Foreign Body Look for: • an opaque foreign body • segmental or lobar collapse • localised emphysema in expiration (ball valve obstruction) • The CXR may be normal Rovin JD, Rodgers BM. Pediatric foreign body aspiration. Pediatrics in Review 2000 Most common X-ray findings with inhaled foreign body: • Normal - no abnormality • Gas Trapping (due to ball-valve effect of foreign body with respiration) • Mediastinal shift • Atelectasis • Lobar collapse/consolidation

130 AA22 ANZCA version [2005-Apr] Q106 The commonest initial presenting feature in anaphylaxis is A. coughing B. desaturation C. hypotension D. rash E. wheeze

ANSER C

A neonate will desaturate faster than an adult at induction because A. FRC decreased more B. Faster onset of induction agents C. More difficult to pre-oxygenate D. E.

ANSWER

In a patient with severe rheumatoid arthritiswhich radiological finding is most consistent with severe atlantoaxial instability (? C1/C2 instability) A. A 9mm gap between the anterior arch of C1 and the odontoid peg B. Increased saggital diameter C. Posterior atlantodental interval of > 14mm D. Midpart of C1 over C2 E. Tear drop sign of C2

ANSWER

?What cannot be used for tocolysis in a 34/40 pregnant woman: A. Clonidine B. Indomethacin C. Magnesium D. Salbutamol E. Nifedipine

ANSWER - A *

LSCS for foetal distress, meconium stained liquor. Management of baby A. Intrapartum suctioning B. Intrapartum suctioning and post partum tracheal suction C. Post partum tracheal suctioning D. Routine neonatal care E. Intubate

ANSWER ?C Rapid assessment: -pink and breathing-> routine care -apnoea/flat -> tracheal suction then CPAP 5 PIP 30 with air with neopuff mask -continue neopuff is adequate ventilation, HR>100, pink,Preductle Sat>90% -HR<60: commence CPR, atropine -Consider intubation if prem, difficulty with neopuff mask, expected prolonged respiratory support, HR<60

105. New- The cause of hypoxia in one lung ventilation A. Blood flow through non ventilated lung B. Impairment of hypoxic pulmonary vasoconstriction C. Ventilation perfusion mismatched (?)

ANSWER A

110. New- Indicates autonomic neuropathy except A. Sinus arrthymias B. Gastric reflux C. Postural hypotension

ANSWER A

40. NEW. What makes tramadol less effective? A: ondansetron B: prochlorperazine C: metoclopramide

ANSWER A

41. Post CEA on ward, patient seizes. BP has been hard to control. What to do to prevent further seizures? A: Add another antihypertensive B: Start antiplatelet drugs C: Start anticonvulsants D: Do angio and stent E: Nimodipine

ANSWER A

42. Repeat- Post local anaesthetic block in difficult intubate patient- patient seizure. What would you give? A. Midazolam 5mg B. thiopentone C. propofol D. Suxamethonium

ANSWER A

54. Intraop hyperfibrinolysis- how to diagnose (euglobulin lysis time NOT an option in the answer) A. TEG B. PT C. APTT

ANSWER A

74. Repeat- Most safe side to insert subtenon block A. Inferonasal B. Inferotemporal C. Medial D. Superonasal E. Superotemporal

ANSWER A

88 MN21 [1985] [1986] [1987] [1988] [Mar93] [Apr98] (type A) Which of the following is NOT a feature of long-standing paraplegia above T6? A. Flaccidity of the leg muscles B. Poikilothermia C. Mass autonomic reflex D. Hyperkalaemia after Suxamethonium administration E. Labile blood pressure

ANSWER A

91 TMP-Jul10-049 The EARLIEST sign of hypocalcaemia is: A. Tingling of face and hands B. Chvostek's sign C. Carpopedal spasm D. ? E. ?

ANSWER A

92 RH12b [Mar92] To operate on the anterior 2/3rds of the ear you would need to block: A. Mandibular n B. Maxillary n C. Vagus n D. Greater auricular n

ANSWER A

97 TMP-Jul10-064 [Aug10] Pregnant woman presents with narrow complex tachycardia HR 190, stable BP 100/60. No response to vagal manoevures. Management? A. Adenosine 6mg B. DCR C. Amiodarone D. Atenolol E. ?

ANSWER A

A 29 year old female undergoes craniotomy for posterior fossa tumour. Which of the following is an absolute contraindication to the sitting position A. Patent ventriculo-atrial shunt B. Previous back surgery C. Pacemaker D. Small patent foramen ovale E. Oesophageal stricture contraindicated for transoesophageal echocardiogram

ANSWER A

A 35kg 5 year old girl is having elective surgery for suturing of a superficial leg laceration. After induction with N2O/Sevoflurane/O2 and in absence of any visible veins you have placed an appropriately sized LMA. Following this her SpO2 immediately drop to 90%. What is your initial management? A. Remove LMA and increase inspired Sevoflurane concentration B. Increase inspired Sevoflurane concentration through the LMA C. Give sublingual Suxamethonium D. Give intramuscular Atropine E. Give intramuscular Suxamethonium

ANSWER A

A 54 year-old patient is on warfarin for AF. They have a history of alcohol abuse and liver failure with a bilirubin of 28 and an albumin of 30. He also has a history of DVT following a flight. What is his CHADS2 score A. 0 B. 1 C. 2 D. 3 E. 4

ANSWER A

A patient has a terminal malignancy. His family doesn't want you to tell the patient about his diagnosis and prognosis. Your decision to inform him is an example of A. Autonomy B. Beneficence C. Confidentiality D. Non-maleficence E. Utilitarianism

ANSWER A

A patient presents for dilation of a pharyngeal stenosis post laryngopharyngectomy 12 months earlier. After inducing anaesthesia you site a size 7 reinforced ETT in the stoma. Over the next 30 minutes the patient gradually desaturations. Despite hand bag ventilation and an increased FiO2 of 1 the saturations remain at 88%. This is due to A. Endobronchial intubation B. Aspiration C. Tension Pneumothorax D. Circuit leak E. Blockage of ETT with secretions

ANSWER A

The time constant of the alveoli is A. Resistance multiplied by compliance B. Resistance divided by compliance C. Resistance plus compliance D. Resistance minus compliance E.

ANSWER A resistance x compliance

Advantage of supraclavicular block over an interscalene nerve block for shoulder surgery A. Less phrenic nerve block B. Easier landmarks in obese patient C. Arm can be in any position for block D. Less risk pneumothorax E. Better cover for shoulder surgery

ANSWER A

After 3 litres of normal saline the dilutional anaemia is initially offset by A. Increased cardiac output B. Increased oxygen extraction C. Capillary vasodilation D. ? E. ?,

ANSWER A

After intubating for an elective case you connect up the circuit and notice that you are unable to ventilate and observe high airway pressures. The next most appropriate step is to: A. Auscultate the lungs B. Release the APL valve C. Remove the endotracheal tube and bag mask ventilate D. Turn on the ventilator E. Low positive end expiratory pressure

ANSWER A

An 18 month old infant is undergoing a routine spontaneously breathing GA with an LMA. They have a sudden onset of SVT with a heart rate of 220 and a BP of 84/60 with an ETCO2 of 32 and SpO2 of 98. The best management strategy is A. Adenosine 100mcg/kg B. DCR 2J/kg C. DCR 4J/kg D. Amiodarone 5mg/kg E. CPR

ANSWER A

Central sensitization, resulting in prolongation of post-operative pain, is caused by: A. Increased intra-cellular gene expression B. Increased intra-cellular magnesium C. Low frequency activation of A-delta fibres D. Primary activation of N-methyl-D-aspartate receptor E. Increased glycine as a major neurotransmitter

ANSWER A

During cardiac catheterisation (?) patient become BP 80/60, HR 110, CVP 16. What is the next most important investigation A. Echocardiogram B. CXR C. Electrocardiogram

ANSWER A

During scoliosis surgery with monitoring of somatosensory evoked potentials, which tract are they mainly monitoring? A: Dorsal column B: Spinothalamic tract C: Lateral Corticospinal tract D: Cerebrospinal tract E: Anterior horn cells

ANSWER A

Greatest predictor of AF post CPB A. advanced age B. history of hypertension C. history of CVA D. history of CCF E. prolonged CPB

ANSWER A

Lumbarsacral nerve does not supply: A. Subcostal nerve B. Ilioinguinal n C. Iliohypogastric n D. Femoral n E. Genitofemoral n (?)

ANSWER A

PZ130 Which drugs below does not need dose adjustment in renal failure patient A. Buprenorphine B. Morphine C. Tramadol D. ? E. ?

ANSWER A

Post CEA on ward, patient seizes. BP has been hard to control. What to do to prevent further seizures? A: Add another antihypertensive B: Start antiplatelet drugs C: Start anticonvulsants D: Do angio and stent E: Nimodipine

ANSWER A

Reverse splitting of the second heart sound occurs with: A. LBBB B. Pulmonary hypertension C. Acute pulmonary embolus D. ASD E. Severe MR

ANSWER A

Subarachnoid haemorrhage patient. What percentage rebleed in the first 24hours A. <5% B. 5-10% C. 10-15% D. 15-20% E. >20%

ANSWER A

Suxamethonium dosage higher in neonates compare to adult because A. Increased volume of distribution B. Increased pseudocholinesterase activity C. More receptors D. Higher cardiac output (?) E. Decreased sensitivity of nicotinic ACH receptors to suxamethonium F. Faster diffusion away from neuromuscular junction

ANSWER A

TMP-104 [Mar10] [Aug10] Stellate ganglion A. Anterior to scalenius anterior B. ? C. ? D. ? E. ?

ANSWER A

TMP-Jul10-024 A 78 year old man with past difficult intubation for arm surgery. Supraclavicular block with 25 mls 0.5% bupivacaine. Shortly after begins convulsing. INITIAL management? A. Midazolam 5mg B. Intralipid 20% 1.5 ml/kg C. Thiopentone 150mg D. Suxamethonium 50mg E. Propofol 50mg

ANSWER A

The best clinical indicator of SEVERE Aortic stenosis A. Presence of thrill B. Mean Gradient 30mmHg C. Area 1.2 cm2 D. Slow rising pulse and ESM radiating to carotids E. Shortness of breath

ANSWER A

The desflurane vaporizer is heated because of its A. High SVP B. High boiling point C. Low SVP D. High MAC E. Low MAC

ANSWER A

The lumbar plexus supplies all of the following EXCEPT: A. Subcostal nerve B. Obturator nerve C. Lateral cutaneous femoral nerve D. Long saphenous nerve E. Iliohypogastric nerve

ANSWER A

The predominant pathology seen in restrictive heart disease is A. Diastolic dysfunction B. Systolic dysfunction C. Valvular dysfunction D. ? E. ?

ANSWER A

The test to diagnose pulmonary embolism A. CT pulmonary angiogram B. Echocardiogram C. Electrocardiogram D. Ventilation-perfusion scan

ANSWER A

Transfusion related acute lung injury (TRALI) A. can be caused by all homologous blood components, but particularly FFP (fresh frozen plasma) B. is associated with significantly elevated pulmonary artery pressure C. is the commonest cause of morbidity associated with blood transfusion D. should be treated with high dose steroids E. typically presents 24 hours following transfusion

ANSWER A

Trauma patient best indicator of good resuscitation (?)- A. Lactate level B. Heart rate C. Blood pressure D Acidosis (?)

ANSWER A

Which have been shown to decrease vasospasm post aneurysm? All EXCEPT: A: Antiplatelet drugs B: Nimodipine C: HHH therapy D: E:

ANSWER A

Which of the following decrease during pregnancy A. Functional Residual Capacity B. Forced Expiratory Volume in one second C. Tidal Volume D. Respiratory Rate E. Vital Capacity

ANSWER A

Which of the following is an advantage of a bronchial blocker over a double lumen tube A. Able to isolate separate lobes B. Significantly easier to deflate non-ventilated lung C. Better suited to pneumonectomy D. Less pressure on bronchial tissue E. Lower incidence of tube malpositioning

ANSWER A

Which of the following is the best predictor of a difficult intubation in a morbidly obese patient A. Pretracheal tissue volume (old question - neck circumference) B. Mallampati score C. Thyromental distance D. BMI E. Severity of OSA

ANSWER A

You are anesthetizing a patient for a laparotomy who has a history of pulmonary hypertension. Regarding the patients anaesthetic management A. An alpha-agonist is the inotrope of choice B. Hypothermia is protective against a rise in pulmonary artery pressure C. Isoflurane will tend to decrease pulmonary artery pressure D. Ketamine is an appropriate anaesthetic agent E. ? RHF

ANSWER A

You see a young man prior to surgery. He describes a history of throat swelling and difficulty breathing both spontaneously and in association with minor dental procedures. His brother has had similar episodes. The most likely mechanism is: A. C1-esterase deficiency B. Factor V deficiency C. Low bradykinin levels D. Mast cell degranulation E. Tryptase release

ANSWER A

Your registrar gives a Duchenne patient 1mg/kg of suxamethonium. What are you most worried about? A: hyperkalaemia B: rhabomyolysis C: MH

ANSWER A

A patient undergoes a femoral-popliteal bypass and has a mildly elevated troponin on day 1 post-operatively. They are otherwise asymptomatic with no other signs/symptoms of myocardial infarction and have an uneventful recovery. What do you do? A. Arrange for a cardiology follow-up and outpatient angiogram because he is at increased risk of future mycocardial infarction B. Arrange coronary angiogram as an inpatient prior to discharge C. Inform the patient that while the result is real the significance is questionable D. Repeat in a weeks time as a second troponin is a better indicator of long-term myocardial infarction risk E. Ignore the result as it is likely a laboratory error

ANSWER A 1. Troponin elevation after non-cardiac surgery occurs commonly. MI as measured by postoperative troponin elevation, is the most common major vascular complication after non-cardiac surgery. (In the placebo group of the POISE Trial, a large randomized controlled trail of over 8000 patients, 1.4% of patients suffered vascular death, 0.5% suffered stroke, 0.5% suffered nonfatal cardiac arrest, and 5.7% suffered myocardial injury in the first 30 days) 2. Patients who suffer postoperative troponin elevation are at increased risk of death after their surgery. (VISION study large international prospective cohort study evaluating major vascular complications in patients ≥45 years of age undergoing non-cardiac surgery that requires hospital admission. Peak postoperative TnT measurement in the first 3 days after surgery was an independent predictor of 30-day mortality. 1/6 with a peak TnT measurement >0.30 ng/ml died within 30 days of surgery. Myocardial injury detected through TnT elevations may explain 42% of deaths that occur after surgery) 3. Clinical symptoms usually will not identify these patients. -The majority of postoperative troponin elevations (74%) occur within 48 hours of surgery. In this period most patients are receiving analgesic medication, and 65% of patients with a troponin elevation do not experience any ischemic symptoms. Importantly, the mortality rate of patients with post-operative troponin elevation without symptoms is not different from those who do experience symptoms.

Most likely change on CTG with anaesthesia for non-obstetric surgery at 32 wks: A. Loss of beat to beat variability B. No change C. Late decelerations D. Variable decelerationss E. Uterine contractions

ANSWER A # Normal CTG under GA = Loss of beat to beat variability, no decelerations. # Normal CTG under neuraxial block without sedation = No change

Amniotic fluid embolism. Cause of death in first half hour ? A. Pulmonary hypertension B. Malignant arrhythmia C. Pulmonary oedema D. Hypovolaemic shock E.

ANSWER A *

A patient known to have porphyria is inadvertently administered thiopentone on induction of anaesthesia. In recovery the patient complains of abdominal pain prior to having a seizur and losing consciousness. Which drug should NOT be given A. Pethidine B. Diazepam C. Haematin D. Suxamethonium E. Pregabalin

ANSWER A - Pethidine. Decreases seizure threshold Porphyria = group of disease with enzyme defect in harm synthesis leading to accumulation of porphyrins. Porphyrins (purple) are organic substances with particular sturcuture; porphyrins with iron called harm. There are hepatic & erythoropoietic types of porphyria. Only the heaptic types affect anaesthesia - Acute intermittent porphyrins - common & variaegate & heridatory coroporphyria. Issue - Anaesthetic drugs + peri-op stress has potential to precipitate a porphyria crisis. Porphyria Crisis Women 30-40 Causes - stress, drugs, infection, alcohol, menses, pregnancy, starvation, dehydration Haematin - iron containing compound Abdo pain, vomiting, peripheral neuropathy, autonomic dysfuction, cranial nerve palsies, mental distrurbances, coma, convulsions, pyrexia. - May be first attack -get family hx. May be normal biochem b/w attacks Mx - Premedication - benzo - Minimise fasting - IVF - Regional anaesthesia? Ok for epidural but can confuse peripheral signs. Avoid in acute attack. - Art line _ Treat HTN with esmolol - Convulsions - treat with diazepam, propofol or Mg. In crisis Withdraw precipitant Reverese factors that stimulate ALA synthetase which is initial product of haem formation ====> give harm aragonite I. Acute seizure or status epilepticus during an acute attack Termination of an acute convulsion should be with an intravenous benzodiazepine such as lorazepam, diazepam or clonazepam. The choice of diazepam may be controversial but it is almost certainly safe as a single intravenous dose. Under no circumstances should phenytoin or phenobarbitone be used. Where benzodiazepine treatment fails, paraldehyde or magnesium sulphate should be considered. Where general anaesthesia is required propofol is the drug of choice. Porphyria Europre UNSAFE - Barbiturates, etomiate - Enflurane - Cemetidine, metoclopramide - Phenytoin SAFE - Propofol - Sux -Pethidine - Lorazepam, midaz CONTROVERSIAL OCH Page 212 Diazepam - ok in one off dose

#NOF patient with aortic stenosis. What is a sign/ investigation/ symptom that shows the most severity? A: Thrill in Aortic area B: Murmur in lower left sternal edge C: Murmur radiating to carotids D: History of ischaemic heart disease or coronary artery disease E: history of angina/ syncope

ANSWER A A - True - "A precordial thrill may be felt, especially on leaning forward in expiration. Its presence is reasonably specific for severe aortic stenosis"

76. Repeat- Diastolic dysfunction Not caused by A. Adrenaline B. Myocardial fibrosis C. Aortic stenosis D. Hypertension

ANSWER A Adrenaline increases the efflux of Ca, therefore aiding relaxation (lusitropy)

PZ115 ANZCA version [2005-Sep] Q123 Correct statements regarding fondaparinux include each of the following EXCEPT A. it has a structure unrelated to heparin B. it is administered once daily C. it is a synthetic, selective Factor Xa inhibitor D. it is recommended for DVT prophylaxis in major orthopaedic surgery E. the dosage does NOT need to be adjusted for age and sex

ANSWER A As per Yentis pg 214 * A - derived from the factor Xa-binding moeity of unfractionated heparain (FALSE, hence ANSWER) * B - single daily dose (TRUE) * C - "It is a synthetic and specific inhibitor of activated Factor X (Xa)" [1] (TRUE) * D - only licensed for DVT prophylaxis in orthopaedic surgery (TRUE) * E - implies standard dose for all (TRUE)

133. New - Advantages of off-pump CABG over on-pump CABG A. decreased transfusion rate B. decreased mortality C. decreased cost D. increased graft patency E. less cognitive impairment F. less stroke

ANSWER A CEACCP Hett 2006 Benefits of OPCABG -mortality reduced from 2.9% -> 2.3% and complication rate 12% -> 8% (conflicting results from other studies) -Most studies have shown a reduction in the need for transfusion and other blood products (effect of extracorporeal circulation and hypothermia) -Incidence of stroke is less (no manipulation of aorta resulting in macro and micro embolii) -reduced rise in inflammatory markers No difference -mortality and morbidity is unchange -incidence of AF is similar -short term patency rate are comparable. There is no info on long term patency -no difference in neurological dysfunction

PZ128 Patient on cisapride. What drug NOT to give in recovery? A. Tramadol B. ? C. ?

ANSWER A Cisapride is a prokinetic *agonist at muscarinic (M2) and some serotonergic (5HT4) receptors, and as an antagonist at other serotonergic (5HT3) receptors. *increases SM tone, strength and co-ordination Risk of Prolong QTc *inhibit K+ channels *higher doses * reduced metabolism via Cytochrome P450 (e.g. macrolides, azole antifungals, grapefruit juice) * other QTc drugs (e.g. quinidine, sotalol).

What is the chemical used in sodalime to indicate exhaustion? A. ethyl violet B. potassium permanganate C. ?

ANSWER A Components of Sodalime NaOH 5% Ca(OH)2 94% Water Ethyl violet as indicator Binders

A full size C oxygen cyclinder (size A in New Zealand) has pressure regulated from A. 16000kpa to 400kpa B. 16000kpa to 240kpa C. 11000kpa to 400kpa D. 11000kpa to 240kpa E. ?

ANSWER A Cylinders for gases are filled to 13700kPa (2/3 of rated pressure). (5000/13700) x 440L = ~150 ref: Chris Thompson Gas Supplies notes All cylinders get filled to 13700 kPa (full tank) Size A Size B Size C capacity 400L Size D Size E Size F Size G

39 ECG- which does NOT have abnormal Q waves: A: Digoxin toxicity B: Anterior myocardial infartion C: Previous AMI D: LBBB E: Wolff-Parkinson-White syndrome

ANSWER A Digoxin does not cause Q waves

Malignant hyperthermia. The number of people in the community at any given time with a predisposition is called the: A. Prevalence B. Incidence C. ? D. E.

ANSWER A Incidence measures the rate of occurrence of new cases of a disease or condition. Prevalence measures how much of some disease or condition there is in a population at a particular point in time.

41. NEW. 75 year old male with normal renal function for an endoluminal aortic repair. What is the best protection to prevent the development of renal dysfunction? A: NaCl B: NAC C: mannitol D: dopamine E: dialysis

ANSWER A Methods to reduce AKI after contrast (?is this adaptable to EVAR) -Witholding nephrotoxic drugs: NSAIDs, aminoglycosides, metformin etc Volume expansion -well established in AKi -evidence indicated normal saline or CSL is more effective than half normal saline -consider bicarbonate solutions -maintain UO>2ml/kg Dialysis or hemofiltration -no evidence for prophylactic dialysis with normal renal function -evidence in CRI -may be confounded by hemofiltration ensures adequate intravascular volume Pharmacological -no approved agents for prevention of AKI -Ascorbic acid 3g nocte before and 2g bd -multicentre RCT, placebo controlled showed reduced incidence -NAC not consistently shown to be effect, meta analysis showed no effect Fenoldopam, dopamine, calcium-channel blockers, atrial natriuretic peptide, and L-arginine have not been shown to be effective in the prevention of contrast-induced AKI. Furosemide, mannitol, and an endothelin receptor antagonist are potentially detrimental

Thallium scan: A. High negative predictive value B. High positive predictive value C. Not as good as a dobutamine stress echocardiography D. ? E.

ANSWER A NPV 95% PPV 30% Probably the same as dobutamine stress echo

55. New- 75yo patient seen for femoral bypass surgery, no significant cardiac risk factor. He will be admitted 3 days prior to operation. You decided NOT to start on beta blocker and you are justified because: A. There is increase mortality and morbidity B. There is not enough time to safely start beta blocker C. The beta blocker may make the patient claudication worst D. ?

ANSWER A POISE TRIAL

A diagnostic test has a sensitivity of 90% and a specificity of 99% in detecting a certain disease. From this we can conclude that A. the false positive rate of this test is 1% B. the false negative rate of this test is 1% C. the positive predictive value of this test is 90% D. the negative predictive value of this test is 90% E. this test would be a useful screening test for this disease

ANSWER A Sensitivity = TP / (TP + FN) Specificity = TN / (TN + FP) Positive Predictive Value = TP / (TP + FP) Negative Predictive Value = TN / (TN + FN) False Positive Rate = 1 - spec False Negative Rate = 1 - sens False positive rate = FP / (TN + FP) = 1 - spec False negative rate = FN / (TP + FN) = 1 - sens

65. Repeat- Most common signs of malignant hyperthermia- A. tachycardia

ANSWER A The signs and symptoms of the acute episode are: * Increased CO2 production (the most sensitive indicator) * Tachycardia * Muscular rigidity * Increased body temperature (relatively late) * Metabolic and respiratory acidosis * Masseter spasm * Tachypnoea (if spontaneous respiration) Late signs are: * Complex arrhythmias * Cyanosis * Hypotension * Electrolyte abnormalities * Rhabdomyolysis Differential diagnosis (Ali et al, 2003 (http://www.sciencedirect.com/science/article/B6WBC-49WH6NS-4/2/e600ed74c16e6edf8d10dd86be60e0fb)): * Thyroid storm * Neuroleptic malignant syndrome * Iatrogenic overheating * Heat illness * Pheochromocytoma * Sepsis * Cocaine, ecstasy overdose * Hypoxic encephalopathy * Faulty equipment for measuring temperature, carbon dioxide * Intrathecal injection of inappropriate radiological contrast agent * Sudden cardiac arrest in a patient with occult myopathy

ECG- which does NOT have abnormal Q waves: A: Digoxin toxicity B: Anterior myocardial infartion C: Previous AMI D: LBBB E: Wolff-Parkinson-White syndrome

ANSWER A Type A (left atrioventricular connections: positive atrioventricular connections) positive delta wave positive R wave V1. Type B (right atrioventicular connections: negative delta wave in V1 similar to a Q wave.

89 TMP-Jul10-048 Amniotic fluid embolism. Cause of death in first half hour ? A. Pulmonary hypertension B. Malignant arrhythmia C. Pulmonary oedema D. E.

ANSWER A first severe pulmonary vasoconstriction ,Rt ventricular failure ,then Lt ventricular failure and pulmonary oedema ,embolism is probably a misleading name,its actual pathology is anaphylaxis to foetal tissues

What is NOT true for PDPH following puncture A: Prophylactic bed rest B: Catheter in intrathecally C: D: E:

ANSWER A in Section 9.6.5 Acute Pain Management: Scientific Evidence (2005) and Update (2007) Incidence of headache following dural puncture 0.4-24% * postural in nature * commoner in patients under 50 yrs * commoner in parturients * significanntly less common in males than non-pregnant females (level 1 evidence - 2007 update) * 90% resolve spontaneously within 10 days Incidence may be reduced by using: (level I) * 26 gauge or smaller needle (NNT=13) * use of needle with a non-cutting bevel (NNT=27) No evidence that bed rest is beneficial in preventing PDPH (Level I) * PDPH may causes difficulty mobilising, and headache may then subside with bed rest * Non-opioid and opioid analgesics may provide temoporary relief * Preventive role of fluid therapy unclear (Level I) NO evidence to support the use of: * Sumatriptan (Level II) * ACTH * Epidurally administered saline, dextran, fibrin glue or neuraxial opioids IV and oral caffeine (both level II) are: * effective in treating PDPH * do not reduce blood patch rate Epidural blood patches: * are common practice but further high quality trials are required to determine efficacy (level I) * significant symptomatic relief obtained in 75-95% of patients given a 15 mL blood patch (level IV, three studies) * conflicting evidence regarding use of prophylactic blood patches - one trial showed decreased incidence of PDPH (level III) Autologous epidural blood patches may be contra-indicated in: * leukaemia * coagulopathy * infection, including HIV

Which has the weakest evidence for prevention of postoperative infection A. Intraoperative low inspired O2 B. Intraoperative blood transfusion C. Intraoperative hypothermia D. Intraoperative hyperglycaemia E. Cigarette smoking

ANSWER A low inspired O2 probably least influential on postoperative infection. (Inconsistent data to show increase in infection rate) Other factors all increase postoperative infection E (cigarette probably highest risk for postoperative infection)

A full size C oxygen cyclinder (size A in New Zealand) has pressure regulated from A. 16000kpa to 400kpa B. 16000kpa to 240kpa C. 11000kpa to 400kpa D. 11000kpa to 240kpa E. 7600kpa to 240kpa

ANSWER A BOC

In attempting to make a precise diagnosis of parathyroid adenoma, you would expect all of the following are found in hyperparathyroid disease EXCEPT: A. decreased urinary calcium B. extraosseous calcifications C. increased plasma calcium D. increased urinary phosphate E. renal calculi

ANSWER A extraosseous calcifications can happen due to pseudogout ( calcification of extraarticular cartilage). Google search on hardcore PTH page tells me "there is no correlation between blood calcium and urine calcium in patients with hyperparathyroidism" can have high or low - kate. WIll just go with A. bones, stones, abdominal moans and psychic groans. from hypercalcaemia

The incidence of Fat Embolism Syndrome following a unilateral closed femoral fracture is A. 0 -3% B. 4 - 7% C. 8 -11% D. 12 - 15% E. 16 - 19%

ANSWER A CEACCP fat embolism

A patient known to have porphyria is inadvertently administered thiopentone on induction of anaesthesia. In recovery the patient complains of abdominal pain prior to having a seizure and losing consciousness. Which drug should NOT be given? A. Pethidine B. Diazepam C. Haematin D. Suxamethonium E. Pregabalin

ANSWER A Pethidine: norpethidine precipitate seizures Diazepam: first line in treating seizure Haematin: antidote for AIP Suxamethonium: short acting and metabolised without enzyme induction Pregabalin: drug of choice for long term control of seizure in AIP patients

When a 3 lead ECG is applied correctly in the CS5 position you will monitor lead II when you suspect which of the following conditions A. Anterior ischemia B. Inferior ischemia C. Lateral ischemia D. Atrial ischemia E. Posterior ischemia

ANSWER A The central subclavicular (CS5) lead is particularly well suited for the detection of anterior myocardial wall ischemia. The right arm (RA) electrode is placed under the right clavicle, the left arm (LA) electrode is placed in the V5 position, and the left leg electrode is in its usual position to serve as a ground. Lead I is selected for detection of anterior wall ischemia, and lead II can be selected for monitoring inferior wall ischemia or for the detection of arrhythmias. If a unipolar precordial electrode is unavailable, this CS5 bipolar lead is the best and easiest alternative to a true V5 lead for monitoring myocardial ischemia. http://web.squ.edu.om/med-Lib/MED_CD/E_CDs/anesthesia/site/content/v03/030340r00.HTM

Electrocardiogram in the Cs5 configuration. What are you looking at when monitoring lead I. A. anterior ischaemia B. atrial C. inferior D. lateral E. septal

ANSWER A The central subclavicular (CS5) lead is particularly well suited for the detection of anterior myocardial wall ischemia. The right arm (RA) electrode is placed under the right clavicle, the left arm (LA) electrode is placed in the V5 position, and the left leg electrode is in its usual position to serve as a ground. Lead I is selected for detection of anterior wall ischemia, and lead II can be selected for monitoring inferior wall ischemia or for the detection of arrhythmias. If a unipolar precordial electrode is unavailable, this CS5 bipolar lead is the best and easiest alternative to a true V5 lead for monitoring myocardial ischemia. Thys DM, Kaplan JA: The ECG in Anesthesia and Critical Care. New York, Churchill Livingstone, 1987.&&%%//dp11i9uvzjqmt.cloudfront.net/2/images/upload-flashcards/38/59/36/3385936_m.jpg

Malignant hyperthermia. The number of people in the community at any given time with a predisposition is called the: A. Prevalence B. Incidence C. D. E.

ANSWER A - Prevalence *

Definitive evaluation of malignant hyperthermia (MH) susceptibility does NOT include observing A. Abnormalities on magnetic resonance imaging (MRI) spectroscopy B. Calcium release from B lymphocytes in response to caffeine stimulation C. Certain mutations in the ryanodine receptor gene D. Myofibrillar necrosis on muscle biopsy plasma E. Creatine kinase (CK) levels above 800 units/L

ANSWER A A : Nuclear magnetic resonance spectroscopy measure ATP and pH in muscles non-invasively NOT MRI B,C,D,E all true and mentioned in Australian Anaesthesia 2005 Although large numbers of RYR1 mutations are known, linkage studies have determined that MH susceptibility is linked to the RYR1 locus in only about 50% of affected families

TEG tracing given, post cardiac surgery. Had quite slim tail (ie fibrinolysis) but broader 'shoulders'. A: Fibrinolysis B: Hypofibrinogenaemia C: Platelet dysfunction D: Heparin effect E: Surgical bleeding

ANSWER A *

A 4 year old child booked for minor surgery is seen in pre-admission clinic where a murmur is detected. Which feature will warrant further investigation A. Loudness 4/6 B. Decreases on inspiration C. Vibratory quality D. Ejection systolic murmur E. Louder on supine

ANSWER A - 4 implies thrill

Which piece of airway equipment is designed for use with a fibreoptic bronchoscope A. Aintree B. Cook's airway exchange catheter C. Frova introducer D. Gum elastic bougie E.

ANSWER A - Aintree Aintree intubation catheter (by Cook) 56 cm, ventilation hole at tip. Comes with rapi-fit connectors (one for bag mask, other by jet) 19Fr gauge, semi pliable catheter. Designed to accommodate fiberoptic scope with scope tip able to fit down end and remain flexible. Smallest tube will be 7mm ETT so choose carefully. Load scope through catheter then attach with tape to stop slipping Insert scope and catheter into LMA. Remove scope then railraod tube over aintree.

Abnormal Q waves occur in all the following EXCEPT A. Digitalis toxicity B. LBBB C. Recent transmural MI D. Wolff-Parkinson-White E. Previous MI

ANSWER A - Digitalis

In the management of torsades de pointes (polymorphic ventricular tachycardia), all the following drugs may be useful EXCEPT A. amiodarone B. isoprenaline C. lignocaine D. magnesium E. phenytoin

ANSWER A - amiodarone

A patient with known metastatic lung cancer is found to have hypercalcaemia all of the following would help excretion of calcium except A. Bisphosphates B. Calcitonin C. Frusemide D. Sodium Chloride E. IV crystalloids

ANSWER A - inhibits osteoclastic activity, others promote renal excretion Initial therapy: FLUIDS & DIURETIC Hydration, increasing salt intake, and forced diuresi (hydration is needed because many patients are dehydrated due to vomiting or renal defects in concentrating urine). Increased salt intake -> increase body fluid volume -> increase urine sodium excretion, -> increases urinary calcium excretion. {Na + Ca handled similarly in kidney } After rehydration -> a loop diuretic such as furosemide can be given to permit continued large volume intravenous salt and water replacement while minimizing the risk of blood volume overload and pulmonary oedem -> loop diuretics tend to depress renal calcium reabsorption thereby helping to lower blood calcium levels -can usually decrease serum calcium by 1-3 mg/dL within 24 h -caution must be taken to prevent potassium or magnesium depletion BISPHOSPHONATES -high affinity for bone, especially areas of high bone-turnover. Taken up by osteoclasts and inhibit osteoclastic bone resorption (1st gen) etidronate (2nd gen) tiludronate, IV pamidronate, alendronate (3rd gen) zoledronate and risedronate Patients in renal failure and hypercalcaemia should have a risk-benefit analysis before being given bisphosphonates, since they are relatively contraindicated in renal failure. CALCITONIN Blocks bone resorption and also increases urinary calcium excretion by inhibiting renal calcium reabsorption. Usually used in life-threatening hypercalcaemia. Dose is 4 Units per kg via subcutaneous or intramuscular route every 12 hours, usually not continued indefinitely

Which of the following drugs is least likely to cause hypoxia in ARDS A. Noradrenaline B. Milrinone C. Isoprenaline D. Isoflurane E. SNP

ANSWER A - just accepted kate

A 54 year old man, is on warfarin for atrial fibrillation, has a history of alcohol abuse and liver failure with an albumin of 30 and a bilirubin of 28. What is his CHADS 2 score? A. 0 B. 1 C. 2 D. 3 E. 4

ANSWER A 0 CCF/Hypertension/Age >75/ Diabetes/ Stroke,TIA, TED previously (this scores 2)

Which is the most powerful predictor of atrial fibrillation post cardiac surgery. A. Age B. History of hypertension C. History of CVA D. History of diabetes E. Time on Bypass

ANSWER A Age Procedural risk factors -valve surgery - pulmonary vein venting - bicaval venous cannulation -longer cross-clamp times. Patient risk factors - male gender ? more at risk of getting but CVA risk higher in females - renal dysfunction -CHF -Left atrial enlargement,

A 2 year-old child has just undergone strabismus surgery. They had an URTI 1/52 prior to surgery. They had an uneventful general anaesthetic with a 4.5mm cuffed ETTwas extubated and sent to recovery. 20 minutes later they develop respiratory distress. Their saturations are 96% on room air, and there is noticeable tracheal tug. What is the most appropriate initial management that will help with their respiratory distress A. Apply CPAP via a facemask B. Propofol 1mg/kg C. Dexamethasone 0.4mg/kg D. Gas induction and reintubate E. Nebulized adrenaline (1:1000) 0.5mL/kg

ANSWER A CPAP via facemask

Which general anaesthetic agent contributes the most to green house gas? A. Desflurane B. Isoflurane C. Sevoflurane D. Propfol E. N2O

ANSWER A Desflurane (article on respective Greenhouse gas contributions)

An 80 year old man undergoes a unilateral lumbar sympathectic blockade. THe most likely side effect that he experiences is: A. Genitofemoral neuralgia B. Haematuria C. Postural hypotension D. Lumbar radiculopathy E. Psoas haematoma

ANSWER A Genitofemoral neuralgia

Systemic vascular resistance index (SVRI) is calculated from A. systemic vascular resistance multiplied by body surface area B. systemic vascular resistance divided by body surface area C. mean aortic and central venous pressure difference divided by cardiac output D. cardiac index divided by the mean aortic and central venous pressure difference E. none of the above

ANSWER A SVRI = SVR x BSA Cardiac index = CO/BSA SVR = 80 x (MAP - RAP)/CO (800 - 1200 dynes - sec.cm-5) Systemic Vascular Resistance Index (SVRI) 80 x (MAP - RAP)/CI (1970 - 2390 dynes.sec.cm-5/m2)

The volatile agent most likely to be associated with carbon monoxide production when used with a soda lime scrubber is: A. Desflurane B. Isoflurane C. Sevoflurane D. Halothane E. Enflurane

ANSWER A desflurane

Nitrous oxide anaesthesia may cause all of the following EXCEPT A. An increased incidence of myocardial ischaemia B. Decreased leukocyte chemotactic response C. Elevation of plasma homocysteine levels D. Megaloblastic anaemia E. Reversible inhibition of methionine synthetase

ANSWER A is new answer..otherwise reversible ... ENIGMA "Our findings support the safety profile of nitrous oxide use in major non-cardiac surgery. Nitrous oxide did not increase the risk of death and cardiovascular complications or surgical-site infection, the emetogenic effect of nitrous oxide can be controlled with antiemetic prophylaxis, and a desired effect of reduced volatile agent use was shown."Methionine synthase also known as MS, MeSe, MetH is responsible for the regeneration of methionine from homocysteine; requires Vitamin B12 (cobalamin) as a cofactor. Nitrous oxide oxidizes the cobalt atom on vitamin B12, inactivating methionine synthase and caus- ing a dose-dependent increase in plasma homocysteine concentrations for days after surgery

The best clinical indicator of severe aortic stenosis is A. Presence of a thrill B. Mean pressure gradient of 30mmHg C. Area 1.2cm2 D. Slow rising pulse E. ESM radiating to the carotids

ANSWER A murmur > 4 has high specificity?? for severe AS (kate google)

Which of the following is the best predictor of a difficult intubation in a morbidly obese patient A. Pretracheal tissue volume B. Mallampati score C. Thyromental distance D. BMI E. Severity of OSA

ANSWER A neck circumference based on 2002 paper. Use > 40cm

The most appropriate method for improving oxygenation during one lung anaesthesia, after institution of an FiO2 of 1.0, is application of A. 5cm H2O CPAP to the non-dependent lung B. 10cm H2O CPAP to the non-dependent lung C. 5cm H2O PEEP to the dependent lung D. 5cm H2O CPAP to the non-dependent and 5cm H2O PEEP to the dependent lung E. intermittent re-inflation to the non-dependent lung

ANSWER A or D Management 1. increase FiO2 100% 2. Check position and function of DLT 3. Ensure muscle relaxation 4. Suction 5. Adequate cardiac ouput 6. CPAP to non dependent, surgical 7. PEEP to dependent, ventilated lung 5cmH2O (may increase shunt through non dependent lung, therefore CPAP first) 8. Intermittent inflation of surgical side 9. PA clamp

A patient undergoes a femoral-popliteal bypass and has a mildly elevated troponin on day 1 post-operatively. They are otherwise asymptomatic with no other signs/symptoms of myocardial infarction and have an uneventful recovery. What do you do? A. Arrange for a cardiology follow-up and outpatient angiogram because he is at increased risk of future myocardial infarction B. Arrange coronary angiogram as an inpatient prior to discharge C. Inform the patient that while the result is real the significance is questionable D. Repeat in one week's time as a second troponin is a better indicator of long-term myocardial infarction risk E. Ignore the result as it is likely a laboratory error

ANSWER A- repeat

108 NV42 [Apr07] [Jul07] What do C6/7 motor function do A. flex/extension of fingers B. flex /extend wrist C. shoulder ext rotation / abduction D. elbow pronation/supination E. flexion at elbow

ANSWER B

114 Can01-113 What nerve supplies sensation to the larynx above the vocal cords: A. internal branch of superior laryngeal nerve B. external branch of superior laryngeal nerve C. recurrent laryngeal nerve D. glossopharyngeal nerve E. palatotonsillar nerve

ANSWER B

119.92.Pulsus paradoxus is: (the Q was something like - severe asthmatic - when take BP you would find) A. Reduced BP on inspiration unlike normal (ie normally increased on insp) B. Reduced BP on inspiration exaggerated from normal C. Reduced BP on expiration unlike normal D. Reduced BP on expiration exaggerated from normal E. ?

ANSWER B

124 AZ (Q120 Aug 2008) Preoperative assessment shows a malampati (ML) score of III and thyromental distance (TMD) of < 6cm. A grade 3 to 4 on Cormark and Lehanes is predicted. Compared to the ML score, the TMD is: A less sensitive, less specific B less sensitive, more specific C more sensitive, less specific D more sensitive, more specific E equal sensitivity an specificity

ANSWER B

135. New - A 60 year old man 24 hours post CABG is confused, oliguric, with BP 80/40, pulse 120. The most appropriate and useful investigation is A. electrocardiogram B. echocardiogram C. chest x-ray D. arterial blood gas E. coronary angiogram

ANSWER B

136. Iron deficiency A. decreased serum ferritin, increased serum iron B. decreased serum ferritin, absence of bone marrow iron C. decreased serum ferritin, normal serum iron D. increased serum ferritin, decreased serum iron E. increased serum ferritin, decreased total iron binding capacity

ANSWER B

36yo male with sickle cell anaemia Hct 0.3 with close foot fracture, what is true A. Transfusion 2 pint packed cell preop B. Spinal can be done

ANSWER B

48. AC62b ANZCA version [2003-Apr] Q144, [2004-Apr] Q98, [2004-Aug] Q44, [Jul06] Q23 The most commonly reported cause of awareness during general anaesthesia for a non-obstetric procedure is A. equipment failure B. human error C. lack of premedication D. recreational drug use E. the use of total intravenous anaesthesia

ANSWER B

51. New- Post cervical spine op, there is bulging noted under the incision sit:E. Patient desaturated, combative, keep pulling off the oxygen facemask. Next course of action A. Rapid sequence induction B. Gas induction C. Needle aspiration of the bulge at the neck

ANSWER B

52. New- What drug known to cause prolong QT and risk of Torsades de Pointes A. Metoclopramide B. Droperidol C. Tranexamic acid

ANSWER B

99 MC157 [Mar10] [Aug10] An 18 yo with Fontan circulation undergoing exploratory laparotomy. On ICU ventilation, saturation is 70%. Which ventilator parameter would you INCREASE to improve his saturation? A. Bilevel pressure B. Expiratory time C. Inspiratory time D. Peak inspiratory pressure E. PEEP

ANSWER B

A 15yo girl with a newly diagnosed mediastinal mass presents for lymph node biopsy under general anaesthesia. The most important investigation to perform preoperatively is. A. CXR B. CT chest C. MRI chest D. PET scan E. Transthoracic echocardiogram

ANSWER B

A new test has been developed to diagnose a disease. To determine the SPECIFICITY of this new test it should be administered to A. A mixed series of patients some known to be suffering from the disease and some known to NOT be suffering from it B. A series of patients known to NOT be suffering from the disease C. A series of patients known to NOT be suffering from the disease and an estimate of the prevalence of the disease in the population obtained D. A series of patients known to be suffering from the disease E. A series of patients known to be suffering from the disease and an estimate of the prevalence of the disease in the population obtained

ANSWER B

Flow with the O2 flush button pressed and volatile agent turned on will give you: A: 20-30l/min O2 B: 30-70l/min O2 C: volatile agent + 30l/m O2 D: volatile agent + 40l/m O2 E: volatile agent + 50l/min O2

ANSWER B

Main heat loss in anaesthetic for neonate A. vasodilatation B. radiation C. convection D. conduction E. evaporative

ANSWER B

Neonate to drug addicts found by grandmother in the house, brought into ed, mildly jaundice, slight tachycapnic. ABG PH 7.54, PaCO2 46, pO2 74, HCO 13 A. Septicaemic B. Pyloric stenosis C. Opiod overdose D. Meningitis E. Hepatitis

ANSWER B

New blood pressure measuring device is developed. Best was to compare it to the current gold standard A. SCUSUS B. Bland Altman Plot C. Kendall Coefficient of Concordance D. Pearsons coefficient E. Friedmans ??

ANSWER B

Patient ingested 500mg/kg aspirin. In ICU, the most effective method to remove aspirin A. IV fluid B. Haemodialysis C. Sodium bicarbonate infusion D. Frusemide

ANSWER B

Patient with history of Long QT syndrome treated with long term propanolol. How do you know the treatment is effective A. Normal QTc B. No further prolongation of QT in response to valsalva manoeuvre C. ? D. ? E. ?

ANSWER B

Post delivery neonate did not breath post stimulation by midwife, not vigorous, heart rate drop from 140 to 90bpm. Next step of action A. 100% oxygen B. Positive pressure ventilation C. Intubation D. CPR E. Adrenaline

ANSWER B

What is the best indicator of pending respiratory depression when using a morphine PCA A. Respiratory rate B. Sedation score C. Reduced saturations

B

Prothrombin VX useful in perioperative period to correct the coagulopathic defect of all except A. Isolated factor II deficiency B. Isolated factor VII deficiency C. Isolated factor IX deficiency D. Isolated factor X deficiency E. Warfarin

ANSWER B

Regarding Thallium Stress Testing in predicting perioperative cardiac events A. A positive result requires further investigation with a pulmonary artery catheter B. It has a high negative predictive value C. It has a low negative predictive value D. It has a high positive predictive value E. Thallium Stress testing is considered inferior to Dobutamine Stress Echo

ANSWER B

Respiratory function in quadriplegics is improved by A. abdominal distension B. an increase in chest wall spasticity C. interscalene nerve block D. the upright position E. unilateral compliance reduction

ANSWER B

TMP-Jul10-036 Which is NOT a disadvantage of drawover vaporiser versus plenum vaporiser: A. Temperature compensation B. Cannot use sevoflurane C. Small volume reservoir D. Flow compensation E. ?

ANSWER B

TMP-Jul10-051 Visual loss with pupillary reflexes retained. Likely cause ? A. Retinal detachment B. Occipital mass (! I don't think this option was on the 2011 paper??) C. Frontal mass D. Chiasmal mass E. Optic neuritis

ANSWER B

The MAC awake:MAC ratio of sevoflurane is closest to A. 0.22 B. 0.34 C. 0.45 D. 0.76 E. 1.00

ANSWER B

The carotid sinus derives its nerve supply from the A. vagus nerve B. glossopharyngeal nerve C. ansa cervicalis (hypoglossi) D. middle cervical ganglion E. stellate ganglion

ANSWER B

This grossly abnormal CXR is A. right basal pneumothorax B. right hydropneumothorax C. artifact D. right pleural effusion E. right R lower lobe atelectases

ANSWER B

What is the best predictor of poor prognosis with aortic stenosis? A. Chest pain B. Paroxysmal nocturnal dyspnoea C. Syncope D. Palpitations E. Fatigue

ANSWER B

What is the most accurate method of determining foetal heart rate in a neonate A. Palpation of an umbilical vein pulse B. Auscultation with a stethoscope C. Palpation of the femoral artery D. Pulse oximetry E. ?

ANSWER B

You are anaesthetizing a pregnant woman for neuro-radiological coiling. At what gestation is it important to monitor uteroplacental sufficiency? A. 22 weeks B. 24 weeks C. 26 weeks D. 28 weeks E. 32 weeks

ANSWER B

You are called to the labour ward to assist in the manual removal of a retained placenta in a healthy woman. The obstetrician asks you to administer intravenous glycerol trinitrate. An initial safe dose that you would expect to be effective, would be A. 5mcg B. 50mcg C. 250mcg D. 500mcg E. 1000mcg,

ANSWER B

he anterior branch of the femoral nerve supplies everything but: A: pectinius B: rectus femoris C: Medial thigh D: anterior thigh E: sartorius

ANSWER B

During one lung ventilation hypoxaemia can occur. The cause for this is: A. Loss of hypoxic pulmonary vasoconstriction B. Perfusion of the unventilated lung C. Ventilation perfusion mismatch of the ventilated lung D. Atelectasis of the ventilated lung E. Upper lobe collapse of the ventilated lung

ANSWER B *

AC155 [Apr07] A patient with severe COPD on home oxygen is having an excision of a submandibular tumour under local anaesthesia. The best way to prevent fire in the operating room is: A. seal the surgical site from the patients airway with adhesive drapes B. use bipolar instead of monopolar diathermy C. decr FIO2 to maintain sats 97% D. use alcoholic chlorhex instead of iodine E. add nitrous oxide to the inhaled gases to reduce the FiO2 and provide sedation

ANSWER B * A. seal the surgical site from the patients airway with adhesive drapes - potentially correct: "Oxygen is heavier than air, and can therefore accumulate under surgical drapes. This accumulation may be reduced by the use of 'incise drapes' that protect the wound from high oxygen concentrations and by tenting surgical drapes to dilute oxygen with room air." (Muchatuta and Sale, Fires and explosions, Anaesth&IC, 2007, 8:11) The answer will depend on what the examiners mean by adhesive drapes * B. use bipolar instead of monopolar diathermy - definitely correct: "The cutting mode of diathermy is more likely to ignite fuels than the coagulation mode, and fires are more likely with monopolar diathermy than bipolar" (Muchatuta and Sale, Fires and explosions, Anaesth&IC, 2007, 8:11) * C. decr FIO2 to maintain sats 97% - almost certainly incorrect: "Strategies to reduce the risks posed by high oxygen concentrations include (the) judicious use of oxygen (using the lowest oxygen concentration that provides acceptable haemoglobin oxygen saturations" (Muchatuta and Sale, Fires and explosions, Anaesth&IC, 2007, 8:11) The key here is acceptable haemoglobin oxygen saturations. "Long term continuous oxygen therapy should be considered for patients with stable chronic lung disease, particularly COPD, who have an arterial PO2 (PaO2) consistently less than or equal to 55 mm Hg when breathing air, at rest and awake. ...Flow rate should be set to maintain PaO2 > 60mmHg (8 kPa) (oxygen saturation level, measured by pulse oximetry [SpO2]> 90%) during waking rest." (McDonald et al, Adult domiciliary oxygen therapy. Position statement of the Thoracic Society of Australia and New Zealand, MJA 2005; 182: 621-626) Note that PO2=55mmHg is equivalent to SaO2 88% * D. use alcoholic chlorhex instead of iodine - definitely incorrect: "Alcohol-based antibacterial skin preparations are one of the more common causes of surgical fires since the withdrawal of flammable anaesthetic agents. They can pool on the body surface (especially umbilicus and suprasternal notch), be wicked into surgical drapes and produce flammable vapours that can accumulate beneath the drapes." (Muchatuta and Sale, Fires and explosions, Anaesth&IC, 2007, 8:11) * E. add nitrous oxide to the inhaled gases to reduce the FiO2 and provide sedation - definitely incorrect: "Nitrous oxide also supports combustion and is broken down to produce oxygen, nitrogen and heat." (Muchatuta and Sale, Fires and explosions, Anaesth&IC, 2007, 8:11)

127 RH26b ANZCA version [2004-Apr] Q126, [Jul07] To achieve maximum anaesthesia with minimal risk of trauma to veins, the tip of a needle used for a medial peribulbar injection should be advanced no further past the equator of the globe than A. 5 mm B. 10 mm C. 15 mm D. 20 mm E. 25 mm

ANSWER B 10mm past equator as CEACCP says should not go past posterior border of globe.

Petit mal epilepsy - Which is true? (or words to that effect) A: Most common in child <2 years old B: Can precipitate seizures by hyperventilating (+/- deliberately???) C: Often seizures last for more than 30 seconds D: Rarely familial E: Isoelectric EEG during seizure (burst suppression)

ANSWER B Absence seizures -abrupt and sudden onset of impairment of consciousness (blank stare, possible upward rotation of eyes) -usually brief <20sec -EEG generalised spike and slow wave discharges -hyperventilation triggers in 90% -treatment mainly valproate (second line lamotrigine) -should not use carbamazepine, vigabatrin, tiagabine, phenytoin, gabapentin and pregabalin --> not effective in treatment

72 year old has had hip replacement surgery and 3 days postop has a pulmonary embolus. He is fully heparinised, but still dyspnoeic, clammy, BP 80/40, pulse 120 and CVP 18. The most appropriate next step is A. IVC filter B. Refer him for a pulmonary embolectomy C. Supportive (fluids and inotropes) D. Thrombolysis E. Warfarin

ANSWER B Absolute Previous intracranial bleeding at any time, stroke in less than 3months,closed head or facial trauma within 3 months,suspected aortic dissection ,ischemic stroke within 3 months(except in ischemic stroke within 3hours time), active bleeding diathesis, uncontrolled high blood pressure (>180 systolic or >100 diastolic),known structural cerebral vascular lesion viz av malformations. [edit] Relative Current anticoagulant use, invasive or surgical procedure in the last 2 weeks, prolonged cardiopulmonary resuscitation (CPR) defined as more than 10 minutes, known bleeding diathesis, pregnancy, hemorrhagic or diabetic retinopathies, active peptic ulcer, controlled severe hypertension.

During the neurosurgical management of a cerebral aneurysm. The drug to administer to facilitate permanent clip placement is? A. Nimodipine B. Adenosine C. Mannitol D. Hypertonic Saline E. Thiopentone

ANSWER B Novel treatment but definitely used to facilitate permanent clip placement when temporary clip unable to be used or slips etc.

You are pre assessing A 70 year old patient treated for congestive cardiac failure. They are able to shower themselves and complete other ADLs but get dyspneoa on mowing the lawn. They are New York Heart Association classification A. Class 1 B. Class 2 C. Class 3a D. Class 3b E. Class 4

ANSWER B if any symptoms at all, then can't be class 1

Post scoliosis repair, decreased movement bilaterally in the legs with decreased pain and temperature sensation but spared joint position sense and vibration. What is at fault? A: Posterior spinal arteries B: Anterior spinal arteries C: Epidural haematoma D: Misplaced pedicle screw E: Lateral cord syndrome

ANSWER B Anterior Spinal Cord Syndrome - Complete motor paralysis below the level of the lesion due to interruption of the corticospinal tract - Loss of pain and temperature sensation at and below the level of the lesion due to interruption of the spinothalamic tract - Retained proprioception and vibratory sensation due to intact dorsal columns. Joint capsule, tactile and pressure receptors send a signal through the posterior root ganglia up through the gracile fasciculus for lower body sensory impulses and the cuneate fasciculus for upper body impulses Typical causes include acute disc herniation or ischemia from anterior spinal artery occlusion. *

Autonomic dysreflexia. Which ONE is true? A: 50% of patients with a level below T6 B: Unlikely if below T10 C: Can be prevented?? D: Can be precipitated by light touch E. ?

ANSWER B Autonomic dysreflexia - clinical emergency characterized by massive disordered autonomic response to certain stimuli. Clinical manifestations: Increase in BP by at least 20% headache flushing sweating chilld nasal congestion piloerection pallor Severe HTN may lead to 1) raised ICP -> seizures 2) Cardiac complicatinos MI, pulm oedema Incidence is 50-70% in those with lesion above T6 Usually at chronic stage > 1 yr since injury Reports with lesion as low as T10 but unlikely May be precipitated by bladder distension, bowel distension, acute abdominal pathology, UTI, skeletal fracture, pressure areas, pain, sexual activity, uterine contractions. Pathophyisology - Noxious stimuli below lesion travels up spinla cord to sympathetic chain. 2) Sympathetic chain activation causes massive vasoconstrivtion + HTN (part below T6 as sphancnic involved) 3) BaroR intact -> sends signals to cortex via IX, X. 4) Messages from brain may cause bradycardia but further inhibition of sympathetic response below lesion does not occur resulting in unopposed HTN. *

A patient with HOCM presents with dyspnoea and angina on exertion. Which of the following is the best agent to treat these symptoms A. Glycerol trinitrate B. Metoprolol C. Morphine D. Hydrochlorthiazide E. Salbutamol

ANSWER B Cardiomyopathy - Dilated - systolic dysfxn - HOCM - diastolic -Restrictive - diastolic dysfxn Diastolic dysfxn may be impaired active relaxation or decreased compliance or both Goals with HOCM - Aim to avoid contributing to dynamic obstruction -Maintain SVR - Avoid volume loss incl diuresis - Maintain preload Betablockers to reduced HR and contractility -decreased myocardial demand -increase supply -reduced SAM

Which herbal supplement reacts with tramadol? A. Ephedra B. St John's wort

ANSWER B Ephedra: cardiovascular instability: hypertension, angina acutely and catechol depletion chronically. St J Wort: P450 induction. Interaction with tramadol. Increased sensitivity to anaesthesia. Very important is the induction of metabolism of immunosuppressants, antiretrovirals and anticonvulsants.

Which is the best predictor of poor prognosis with aortic stenosis? A. chest pain B. paroxysmal nocturnal dyspnoea C. syncope

ANSWER B Mean survival Onset of angina 2-4.7 yrs Onset of syncope 0.8-3.7 yrs Onset of heart failure 0.5-2.8 yrs (uptodate)

TURP - patient under spinal. Confused. ABG: Na+ 117 / normal gas exchange. Treatment ? A. 10 ml 20% Saline as fast push IV B. 3% NS 100 ml/h C. Normal saline 200 ml/h D. Frusemide 40 mg IV E. Fluid restrict 500 ml/day

ANSWER B Patient is symptomatic -3% Saline at 1-2ml/kg/hr (stop as soon as symptoms resolve) Correction should be no more than 2 mmol/L per hour initially for 3 to 4 hours, then about 1 mmol/L per hour afterwards. In 24 hours, correction should be no more than 12 mmol/L. -use frusemide -fluid restriction OCH

If a test is negative, what proportion will not have the disease: A. Sensitivity B. Specificity C. Positive Predictive Value D. Negative Predisctive Value

ANSWER B Sensitivity and specificity are a function of a test, but positive and negative predictive values change depending on prevelance. * Sensitivity = TP/(TP+FN), "proportion of positive cases correctly identified" * Specificity = TN/(TN+FP), "proportion of negative cases correctly identified" * PPV = TP/(TP+FP) * NPV = TN/(TN+FN)

The below would increase A-a oxygen gradient Except A. Increase FIO2 B. Decrease FIO2 C. Decrease cardiac output D. Increase shunt

ANSWER B A-a gradient * increases 5-7mmHg for every 10% increase in FiO2 * increases with age * increases with increased shunt * V/Q mismatch * defect in diffusion

35. New. What percentage of patients with SAH are troponin positive? A: <5% B: 15-30% C: 40-60% D: 70-90% E: 100%

ANSWER B Contin Educ Anaesth Crit Care Pain (2008) 8 (2): 62-66. states 20%" (accepted - kate) COMPLICATIONS * - Severe life threatening during course 40% of pts. - Hydocephalus - 25% - Seizures 1-7% (anticonvulsants - worse outcomes) - Delayed neurological deficit - Cardiovasclar effects of SAH? Massive sympathetic surge + catecholamine release -> LV dysfxn/cardiogenic shock; RWM abnormalities; Canon T waves, ECG changes - (27-100% TWI, ST changes, Q waves, prolonged QT). ~ 10% go on to have symptomatic arrhythmia. NO relationship between ECG changes and ECHO findings. Troponin rise - lower peak than in cardiac issue, 20% of pts (as per MCQ ref to CEACCP 2008)

Regarding intra-osseous cannulation in paediatric during resus for shock/cardio arrest, a correct statement is: A. distal tibial above medial malleolus is preferred due to easy access B. drug reaction time is the same as central venous route C. 12G used to ensure adequate flow D. bicarbonate cannot be infused due to bone damage E. fat embolism is common complication

ANSWER B A- no proximal tibial or proximal humerus B- ? yes C- no- RCH guidelines say use 18G cannula, GOSH guidelines say use 16G cannula D -can't find a reference E- can happen but is rare

Which organ is least tolerant of ischaemia following removal for transplantation A. Cornea B. Heart C. Liver D. Kidney E. Pancreas

ANSWER B BJA 2012 108 (51) i29-i42 Organ transplantation: historical perspective and current practice Heart>Lung> liver~pancreas>kidney 4:6:12:18 hours respectively

50yo lady, attempted suicide attempt, jumped from 5th floor building. She does not open her eyes or vocalise and there is no response to pressure on her nail-bed. What is her GCS? A. 2 B. 3 C. 5 D. 8 E. 12

ANSWER B M- 1 V-1 E-1 So GCS 3- B is answer

MRI Telsa 3, least likely to cause harm A. Cochlear implant B. mechanical heart valve C. Implanted intrathecal pump D. Recently placed aortic stent E. shrapnel fragment

ANSWER B - mechanical heart valve MRI versatile imaging tool without ionising radiation. Hydrogen nuclei are flipped in & out of alignment by magnetic pulses in a powerful magentic field; this produces small radio frequencies which are used by a computer to create cross sectinal /3D images. Goals of anaesthesia in MRI - Provide immobility to obtain best pictures - Maintain pat safety & comfort during procedure Hazards From MRI Machine 1.Magnetic field Most scanners use superconducting magnet to generate a high density magentic field which is always present. Strengths between 0.5 and 3 T (30 000 times the Earth's geomagnetic field). Around MRI - fringe field. stregth describe in Gauss. Near scanner > 5G -> ferromagnetic objects become projectiles. 2. Acoustic noise - vibrations caused by changing magnetic fields; pts need air protection 3. Eddy currents - from oscillating magnetic field during picture acquisition risk burns to pt. Can disrput & damage electrical equipment (inactivate pacemakers) 4. Helium escape - used to shut magent down, needs to be vented. Monitoring/Equipment 1. Physical & magentically restricted from pt. Ensure airway severe. Coil around pt. 2. Need MRI compatible pumps, monitors, pulse ox. 3. Avoid metal in equipment, tape pilot balloo away form face. Avoid reinforced tube. ECG leads ensure short, braided, electrodes togethter to reduce interference. Location & transport 1. Remote - all the issues of this. Medication 1. Reaction to contrast medium - (nephrogeni systemic fibrosis) Need organization to get done! lots of setup time. http://www.ranzcr.edu.au/search-result?searchword=MRI%20safety%20guidelines&searchphrase=all - Guideline says that some aneurysm clips have been ok at 3 Tesla. - Pacemaker still very CI

6 week old baby is booked for elective right inguinal hernia repair. An appropriate fasting time is A. 2 hours for breast milk B. 4 hours for formula C. 5 hours for breast milk or formula D. 6 hours for solids E. 8 hours for solids, 4 hours for all fluids.

ANSWER B Poor question as ANZCA guideline changeover age is 6/52. RCH guidelines are different to ANZCA guidelines (shown below) For healthy adults having an elective procedure, limited solid food may be taken up to six hours prior to anaesthesia and clear fluids totalling not more than 200 mls per hour may be taken up to two hours prior to anaesthesia. For healthy children over six weeks of age having an elective procedure, limited solid food and formula milk may be given up to six hours, breast milk may be given up to four hours and clear fluids up to two hours prior to anaesthesia. For healthy infants under six weeks of age having an elective procedure, formula or breast milk may be given up to four hours and clear fluids up to two hours prior to anaesthesia. Only medications with a little water if required as ordered by the anaesthetist should be taken less than two hours prior to anaesthesia.

A pregnant lady is undergoing neuroradiological coiling of a cerebral aneurysm. At what gestational age should you monitor foetal heart rate to ensure adequate uteroplacental blood flow A. 20 weeks B. 24 weeks C. 28 weeks D. 30 weeks E. 32 weeks

ANSWER B Variability starts at ~25 weeks

Atrial Septal Defect murmur is heard due to blood flow through A. Tricuspid valve B. Pulmonary valve C. Mitral valve D. Aortic valve E. Atrial Septal Defect

ANSWER B "ASD with moderate-to-large left-to-right shunts result in increased right ventricular stroke volume across the pulmonary outflow tract creating a crescendo-decrescendo systolic ejection murmur. This murmur is heard in the second intercostal space at the upper left sternal border" (not sure of this source but will accept)

The percentage of post dural puncture headaches that would resolve spontaneously by 1 week is closest to A. 90% B. 70% C. 50% D. 30% E. 10%

ANSWER B - 75% resolve in first week Vandam LD, Dripps RD. Long term follow up of patients who received 10,098 spinal anesthetics. JAMA 1956,161:586-90 A PDPH is usually a self-limiting process. If left untreated, 75% of them will resolve within the first week and 88% will have resolved by 6 weeks.

An adult male preoperatively complains of pain similar to his angina. Initial treatment is all below except: A. Aspirin B. heparin C. morphine D. nitrates E. oxygen

ANSWER B - Heparin

The reason that desflurane requires a heated vapour chamber can be best explained by its: A. Low saturated vapour pressure B. High saturated vapour pressure C. High boiling point D. Low molecular weight E. Very low solubility

ANSWER B - High saturated vapour pressure

Lowest extension of thoracic paravertebral space A. t10 B. t12 C. l2 D. l4 E. s1

ANSWER B - T12 CEACCP - The thoracic paravertebral space begins at T1 and extends caudally to terminate at T12. Although PVBs can be performed in the cervical and lumbar regions, there is no direct communication between adjacent levels in these areas. Most PVBs are therefore performed at the thoracic level.

Post intubation, you manual ventilate and noted patient high airway pressure. What would you do next A. Open the APL valve B. Auscultate the lung C. Switch to ventilator

ANSWER B - auscultate lung

Complications of mediastinoscopy include all of the following except A. Air embolism B. Cardiac laceration C. Pneumothorax D. Recurrent laryngeal nerve palsy E. Tracheal compression

ANSWER B - cardiac laceration

Which of the following are NOT useful in the management of Torsades de Pointes A. Isoprenaline B. Procainamide C. DCCV D. Electrical pacing E. Magnesium

ANSWER B - check rationale

Which antihypertensive is not safe to use in pregnancy A. Aspirin B. Enalapril C. Metoprolol D. Hydralazine E. Nifedipine

ANSWER B - enalapril

A healthy 20 year old patient undergoing nasal surgery under general anaesthesia has the nose packed with gauze soaked in 0.5% phenylephrine and a submucosal injection of lignocaine with 1:100,000 adrenaline. Over the next 10 minutes the blood pressure rises from 130/80 to 220/120 mmHg and the heart rate from 60 to 100 beats per minute. The LEAST appropriate management of this situation would be to A. administer glyceryl trinitrate B. administer esmolol C. administer labetalol D. administer sodium nitroprusside E. deepen anaesthesia with isoflurane

ANSWER B - esmolol is cardioselective B1 blocker so will leave alpha agonism unabated. Labetalol has alpha/beta agonism

A 60 year old man with normal LV function is having coronary artery bypass grafting. After separation from the bypass machine he becomes hypotensive with ST elevation in leads II and aVF. The Swan Ganz Catheter showed a PCWP of 25 and CVP of 15 with normal PVR and SVR. The TOE is likely to show: A. Early mitral inflow > inflow during atrial systole B. Inferior wall hypokinesis C. Severe MR D. TR and RV dilatation E. LV cavity obliteration at the end of systole

ANSWER B - inferior wall hypokinesis

A 30 year old lady has a vaginal forceps delivery without neuroaxial blockade. The next day she is noted to have loss of sensation over the anteriolateral aspect of her left thigh. There are NO motor symptoms. The is best explained by damage to the left sided: A. Lumbosacral trunk B. Lateral cutaneous nerve of the thigh C. Pudendal nerve D. L2/3 Nerve root E. Sciatic nerve

ANSWER B - lateral cutaneous nerve Lumbosacral Plexus divided into lumbar + sacral parts. The lumbosacral trunk is part of this , just the part where L5-S1 contribute to it. Lat cut nerve of thigh- sensory only, from L3. Pudendal nerve comes from sacral plexus (S2/S3/S4) + cutaneous correlations are posterior thigh, perineal, genital areas L2/L3 nerve root supplies -femoral (motor to anterior thigh) + obturator nerves (motor to medial thigh) and sensory innervation of thigh and leg - notable exception is anterior lateral region supplied by lat cutaneous nerve of thigh. Lumbar : Some idiots in Georgis Like Falling Off Logs Sub costal T12 Iliohypogastric - x Ilioinguinal - sensory below T12 Genitofemoral - sensory to inner thigh via femoral branch Lateral femoral cutaneous - sensory only Femoral - anterior thigh motor + sensory via anterior femoral cutaneous & saphenous Obturator - medial thigh motor + sensory medial thigh Lumbosacral trunk - Sacral Plexus is : Some Idiots Sit Pondering Perversions Superior gluteal - Inferior gluteal - Sciatic L4-S3-> Common peroneal (motor to lateral + anterior leg; sensory to anterolateral leg) + tibial ( motor to posterior thigh + posterior leg; sensory to posterior leg) Post femoral cutaneous -> sensory to post thigh Pudendal

Current guidelines regarding cardiopulmonary resuscitation include all of the following EXCEPT A. Allow equal time for chest compression and relaxation B. Give 2 rescue breath before commencement of CPR C. Chest compression at 100bpm D. Chest compression should be at least 5cm depth E. Chest compression to breath ratio at 30:2

ANSWER B - no need for breaths first. BLS guidelines, resus.org.au

What is the test is decreased in Iron deficiency anaemia? A. microcytosis B. serum ferritin C. serum iron D. transferrin E. total iron binding capacity

ANSWER B - serum ferritin Iron Deficiencey & Treatment

In paediatric trauma the Broselow tape is used to estimate A. Blood loss B. Weight and drug dosages C. Urine output D. Abdominal girth E. Head circumference

ANSWER B - weight & drugs

A 70 year old man with non-valvular atrial fibrillation is taken off his warfarin for 7 days prior to surgery and has no bridging therapy. His daily risk of stroke is A. 0.001% B. 0.01% C. 0.1% D. 4% E. 10%

ANSWER B - yearly risk 4% (anticoag decreases to 2%) 0.01 is daily risk ----4/365

Magnesium for treatment of pre-eclampsia. What is the therapeutic level? A: B: 3 - 5 C: 5 - 7 D: E:

ANSWER B 2 -3. 5 mmol/L

What is approximately the systolic blood pressure in an awake neonate (mmHg) A. 55 B. 70 C. 85 D. 100 E. 115

ANSWER B 70

?Features of severe pre-eclampsia include all except: A. Foetal growth retardation B. Peripheral oedema C. Systolic BP more than 160 D. Thrombocytopenia E. Severe proteinuria

ANSWER B ?? or A Systemic disorder associated with complex pathology incl endothelial derrangements in placentofetal unit . Part of a spectrum of disorders known HTN disease of pregnancy. Complicates 2-3% of pregnancies. Priority of ante-natal care after 20 weeks is diagnosis. Treatment is delivery of baby & placenta. Many associated complications. Defined: 1) HTN after 20 weeks gestation > 140 systolic > 90 diastolic with proteinuria ( > 300mg in 24 hr) Severe pre-eclampsia (as per ADHB guidelines 2012) - Persistent severe HTN above 170 systolic, > 110 distolic .....OHS says > 160/110 - Oliguria < 100mL in 4 hrs - Creat > 0.9 umol/L - Neurological involvement -Pulm odema or resp compromise - Plt < 100 - Epigastric or RUQ pain; Hepatic rupture Symptoms & Signs of Pre-eclampsia - HTN + proteinuria - Persistent severe headache - Abdo pain incl epigastri, liver -Visual disturbances - blurred vision - Hyperreflexia - Severe swelling of hands/feet/face of sudden onset - Creat high - Low platelets - Elevated transaminases Complications -CNS - Seizure, stroke, cerebral oedema, blindness - Resp - Pulm oedema, laryngeal oedema - CVS - HTN - Renal - Tubular or cortical necrosis - GI - Jaundice, HELLP, hepatic rupture - Blood - DIC, homeless - Placenta - infarction, abruption - Baby - IUGR, pre-term birth, death Risk Factors - older age> 40 - First pregnancy - BMI > 35 at booking - Multiple pregnancy ( is this twins or multi??) - Personal or family hx or preeclampsia - Diseaese - chronic HTN, diabetes, renal disease Treatment 1. Definitive mx is delivery of fetus + placenta -

A reduction in DLCO can be caused by; A. Asthma B. COPD C. Left to right shunt D. Pulmonary haemorrhage E. Bronchitis

ANSWER B COPD asthma unlikely; shunt no

A 70 year old patient is being treated for congestive cardiac failure. They are able to shower themselves and complete other ADLs but get dyspneoa on mowing the lawn. They are New York Heart Association classification A. Class 1 B. Class 2 C. Class 3a D. Class 3b E. Class 4

ANSWER B Class II NYHA Functional Classification Originated in 1928 to provide common language. FX activity is a determinant of outcome so clinically useful as prognostic marker. I Cardiac disease but no symptoms & no limitation in ordinary physical activity. II Mild symptoms (SOB +/or angina ) with slight limitation during normal activity III Marked limitation in activity due to symptoms even during less than ordinary activity (walk 20m) IV Severe limitations. Rest symptoms. Mostly bed bound.

A 25 week post conceptual age infant is being ventilated in the Neonatal Intensive Care Unit. To reduce the risk of retinopathy of prematurity, they are being ventilated to a target oxygen saturation of 85-89% instead of 91-95%. This is associated with: A. Increased acute lung injury B. Increased mortality C. Increased sepsis D. Reduced intracerebral haemorrhage E. Reduced necrotizing enterocolitis

ANSWER B Increased mortality BOOST 2 trial NEJM

A 50 year old lady is seen at the pre-operative assessment clinic she is on 150mg/day methadone, what is the most likely ECG change to be found in her pre-op ECG? A. Prolonged PR interval B. Prolonged QTc C. ST depression D. U wave E. Tented T-waves

ANSWER B QT prolongation methadone prolongs the cardiac action potential by inhibiting cardiac potassium channels.

Periop clinic reviewing a patient with chronic/ end stage renal failure. Her calcium found to be low. He most certainly have A. Primary hyperparathyroidism B. Secondary hyperparathyroidism C. Tertiary hyperparathyroidism

ANSWER B Secondary hyperparathyroidism refers to the excessive secretion of parathyroid hormone (PTH) by the parathyroid glands in response to hypocalcemia (low blood calcium levels) and associated hypertrophy of the glands. This disorder is especially seen in patients with chronic renal failure. It is often—although not consistently—abbreviated as SHPT in medical literature.-------------------------------- ------------------------------------------------------------------------------------------------ Primary hyperparathyroidism causes hypercalcemia (elevated blood calcium levels) through the excessive secretion of parathyroid hormone (PTH), usually by an adenoma (benign tumors) of the parathyroid glands. Secondary hyperparathyroidism refers to the excessive secretion of parathyroid hormone (PTH) by the parathyroid glands in response to hypocalcemia (low blood calcium levels) and associated hypertrophy of the glands. Tertiary hyperparathyroidism is a state of excessive secretion of parathyroid hormone (PTH) after a long period of secondary hyperparathyroidism and resulting hypercalcemia. It reflects development of autonomous (unregulated) parathyroid function following a period of persistent parathyroid stimulation.

The organ that is least tolerant of ischaemia, after removal for transplant, is: A. Cornea B. Heart C. Kidney D. Liver E. Pancreas

ANSWER B heart

Your patient has smoked cannabis prior to arrival in the OT. Pt taking cannabis might lead to: A. Intraoperative Bradycardia B. Decreased anaesthetic requirement C. Increased nausea and vomiting D. Increased risk of awareness E. Decreased BIS reliability

ANSWER B if acutely intoxicated

Acromegaly due to excess of growth hormone. Why is it difficult to do a direct laryngoscopy? A: Distorted facial anatomy B: Macroglossia C: Glottic stenosis D: Prognathe mandible E: Arthritis of the neck

ANSWER B or C Acromegaly : either pituitary tumour or extra-pit (breast, ovary, lung and and pancreas) •usually characteristic facies with marked prognathism, frontal bossing and prominent supra-orbital ridges •clinical features develop slowly, and usually present in the 4th decade, with a.local effects : pituitary tumour → visual field defects and increasing ICP b.GH effects (i) overgrowth of bones : big limbs, skull, kyphoscoliosis (ii) overgrowth of CT - coarse skin with skin tags, macroglossia and carpal tunnel syndrome and hyperhidrosis c.CVS effects - ATS, HT, cardiomegaly with cardiomyopathy → CCF d.Other Endocrine disturbances : DM, hypothyroid, impotence, amenorrhea, galactorhhea •Difficult airways : both ventilation and intubation, for a number of reasons oBig jaw, tongue and soft tissues → facemask ventilation difficult oNose may be obstructed because of hypertrophy of turbinates oLonger distance between incisors and glottis oGlottis may ne narrowed by CT overgrowth involving vocal cords, crico-arytenoid joints and recurrent larungeal nerve (sub-glottic diameter may also be reduced → use smaller ETT) •Dx : GH levels - fasting elevated AND doesn't suppress with glucose load plus MRI •Rx : to reduce GH levels to normal, minimise local effects and replace other hormones oSurgery, irradiation odrugs (bromocriptine or octreotide (long acting somatostatin analogue inhibits GH secretion))

2yr child post op following stabismus surgery. ETT 4.5 used. Awake, stridor and tracheal tug. Immediate action? A. inhalational induction B. CPAP with facemask C. propofol 1mg/kg D. dexamethasone 0.4mg/kg E. adrenaline nebuliser 1:1000 05ml/kg

ANSWER B- showing signs of laryngospasm

128 PR04 ANZCA version [1985] [Mar95] [Apr97] [Jul97] [Apr98] [Jul98] [2002-Aug] Q11, [2003-Apr] Q39, [2005-Sep] Q46, [Mar06] Q25 The percentage of the population which is heterozygous as regards pseudocholinesterase, thus having a dibucaine number between 30 and 80, is A. 0.04% B. 0.4% C. 4.0% D. 14.0% E. 40.0%

ANSWER C

132. New - When intubating over a bougie / awake fibreoptic, which direction do you rotate the tube to stop it catching on structures in the glottis A. no change from normal B. 90 degrees clockwise C. 90 degrees counterclockwise D. 180 degrees E. try either direction

ANSWER C

32. NEW. 75 year old with non-valvular AF usually on warfarin has their warfarin stopped for one week. What is their daily risk of stroke? A: 1% B: 0.1% C: 0.01% D: 4% E: 10%

ANSWER C

40 The QT interval may be prolonged by each of the following EXCEPT A. high intra-thoracic pressure B. hypothermia C. magnesium sulphate D. suxamethonium E. volatile anaesthetic agents

ANSWER C

57.EZ80 Repeat- A line isolation monitor protects against microshock A. only if the warning current is set at 10mA B. only if the warning current is set at 30mA C. under no circumstances D. only if the equipment used is grounded E. only if it monitors all the equipment in the region

ANSWER C

98 Which patients do not get pulmonary hypertension a. ASD b. Chronic thromboembolism c. Tetralogy d. MR e. MS

ANSWER C

A 30-year old patient, who takes paroxetine, has suffered a traumatic amputation. The most appropriate medication to reduce her developing chronic post-operative pain is: A. amitriptyline B. dextromethorphan C. gabapentin D. tramadol E. pethidine

ANSWER C

A 50 year old male in recovery after an anterior cervical fusion developing increasing respiratory distress, bulge under original incision, combative, repeatedly removing oxygen mask, SpO2 96%. What is the most appropriate management A. Aspirate the collection with a 19G needle and syringe B. Awake fibreoptic intubation with minimal sedation C. Direct laryngoscopy and intubation after sevoflurane/O2 gaseous induction D. Direct laryngoscopy and intubation after propofol/suxamethonium induction E. Intubation via intubating LMA,

ANSWER C

A child with intra-operative blood loss. A cardiac arrest is most likely because of A. A delay in delivery of blood from the blood bank B. Inadequate intravenous access C. Underestimated intra-operative blood loss D. Underestimated pre-operative hypovolaemia E. Complication of transfusion

ANSWER C

A middle-aged male with severe mitral stenosis having general anaesthesia for repair of fractured ulna/radius. Ten minutes into the case you notice a tachyarrythmia with his HR 130 and BP 70/-. He is normally in sinus rhythm. What do you do A. Adenosine B. Amiodarone C. Shock D. Volume E. Metaraminol

ANSWER C

A neonate is born with meconium stained liquor but is vigorous and crying. The reason for not suctioning the pharynx is A. Hypertension B. Hypotension C. Bradycardia D. Tachycardia E. ?

ANSWER C

A patient is having an electrophysiological study and ablation for atrial fibrillation. Suddenly the blood pressure drops to 76/38 mmHg with the heart rate at 110 in sinus rhythm. What is the best investigation to confirm the cause of hypotension? A. Troponin B. ST-segment elevation C. Transoesophageal echocardiography D. Coronary Angiogram E. Electrocardiogram

ANSWER C

A septic patient has a CVP of 12mmHg a blood pressure of 80/40mmHg and a pulse rate of 90/minute. Which is the best agent to treat their hypotension A. Dopamine B. Dobutamine C. Noradrenaline D. Adrenaline E. Levosimenden

ANSWER C

Anaesthetic Machine is left on all weekend with flow rate of oxygen at 6L/min. A Desflurane vaporiser is placed on it on Monday morning without changing the CO2 absorber. What is the most likely toxic product produced A. Ca(OH)2 B. Carbon dioxide C. Carbon monoxide D. Compound A E. Compound B

ANSWER C

Baby with Tracheo-oesophageal fistula found by bubbling saliva and nasogastric tube coiling on Xray. BEST immediate management? A. Bag and mask ventilate B. Intubate and ventilate C. position head up, insert suction catheter in oesophagus (or to stomach?) D. Place prone, head down to allow contents to drain E. Insert gastrostomy

ANSWER C

Commonest valvular heart disease seen in pregnancy is A. Aortic stenosis B. Aortic regurgitation C. Mitral Stenosis D. Mitral regurgitation E. Tricuspid reguritation

ANSWER C

Compared with a plenium vaporizer what is NOT a disadvantage of draw-over vaporizer (repeat but still not quite remembered correctly) A. Basic temperature compensation B. Basic flow compensation C. Cannot use sevoflurane D. Small volume reservoir E. ?

ANSWER C

Finding on haemophilia A patient A. Female haemarthrosis B. Male haemarthrosis C. Normal PT, abnormal APTT D. Abnormal PT, normal APTT

ANSWER C

Fluoroscopy in the operating theatre increases the exposure of theatre personnel to ionising radiation. Best method to minimise one's exposure to such radiation is to A. have dosimeter checked at least 6-monthly B. limit exposure time to radiation C. maximal distance from radiation source D. stand behind transmitter of C arm E. wear protective garments

ANSWER C

Infra-renal aortic cross-clamping usually results in A. decreased cardiac contractility B. decreased coronary blood flow C. decreased renal blood flow D. minimal change in cardiac output E. increased heart rate

ANSWER C

Patient complains of numbness in the anterior 2/3 of tongue after GA with LMA. Most likely nerve injured is A. Glossopharyngeal B. Facial nerve C. Mandibular D. Superior vagus E. Maxillary nerve

ANSWER C

TMP-Jul10-004 Exponential decline / definition of time constant (with various options) A. time for exponential process to reach log(e) of its initial value B. Time until exponential process reaches zero C. Time to reach 37% of initial value D. Time to reach half if its initial value E. 69% of half life

ANSWER C

Very sick patient on CVVHF. On norad, changed to adrenaline with no improvement in haemodynamic variables. What is your next step? A: change to another inotrope B: check their response to a synacthen test C: give hydrocortisone

ANSWER C

What is the most important immediate treatment for a cardiac arrest due to ventricular fibrillation in a patient with Hypertrophic Obstructive Cardiomyopathy A. Adrenaline B. Amiodarone C. Defibrillation D. Intubation, ventilation and oxygenation E. Praecordial Thump

ANSWER C

When stimulating the ulnar nerve with a nerve stimulator which muscle do you see twitch A. Opponens abducens B. Abductor pollicis brevis C. Adductor pollicis brevis D. Extensor pollicis E. Flexor pollicis brevis

ANSWER C

You are 2 hours into an operation. 3L of IV Crystalloid has been given. There has been minimal blood loss. The dilutional anaemia is compensated by: A. Cellular anaerobic metabolism B: Capillary vasodilation C: Increased cardiac output D: Increased tissue oxygen extraction E: Rightwards shift of the Oxygen - Haemoglobin dissociation curve

ANSWER C

You are doing an awake fibreoptic intubation and having difficulty identifying the anatomy of where you are. Then you observe a trifurcation. The lobe of the lung to which this airway is connected is A. LUL B. Lingula C. RUL D. RML E. RLL

ANSWER C

You see a man in his 60s in clinic 1 week prior to laparoscopic cholecystectomy. He has dilated cardiomyopathy with an ejection fraction of 30% but does not get dyspnoeic with normal activities of daily living. What is the most appropriate management of his heart failure? A. Amiodarone 100mg bd B. Digoxin 250mcg daily C. Enalapril 2.5mg bd D. Metoprolol 100mg bd E. Diltiazem slow release 240mg daily,

ANSWER C

Young asthmatic male in emergency department. RR 26, pCO2 27, SAO2 92%, struggling talking in sentences. Given nebulised salbutamol, and ipratropium, 200mg IV hydrocortisone. After 30 minutes - no improvement. Further management: A. IV salbutamol B. IV aminophylline C. IV magnesium D. Intubate and ventilate E. ???IV adrenaline?

ANSWER C

a

ANSWER C

How quickly does the CO2 rise in the apnoeic patient ? A. 1 mmHg per min B. 2 mmHg per min C. 3 mmHg per min D. 4 mmHg per min E. 5 or ?8 mmHg per min

ANSWER C *

MH61 A 35yr old African-American with sickle cell and fractured ankle for ORIF. Hb 90, Haematocrit 0.3. A: Transfuse 2 units packed cells (?pre-op) B: Let him cool passively to low/normal temperature C: Spinal is safe D: Avoid thiopentone E: Tourniquet is absolutely contra-indicated

ANSWER C A - WRONG: "Although widely practiced, prophylactic erythrocyte transfusion remains a treatment with appreciable complications whose potential benefits have not been clearly demonstrated by a prospective, randomized clinical trial". Also a study in orthopaedic patients "did not detect a prophylactic effect from preoperative transfusion" (Sickle Cell Disease and Anesthesia. Anesthesiology 2004; 101:766-85) B - WRONG: "Although hypothermia would tend to retard sickling because of a left shift of the oxygen dissociation curve, hypothermia is often identified as a precipitant of perioperative SCD complications". "avoidance of patient hypothermia is a basic objective for most anesthetics" (Same reference) C - CORRECT: "The use of regional anesthesia therefore does not appear to be contraindicated in SCD" (Same reference) and OHA p202 agrees D - ??? but seems unlikely E - WRONG according to OHA p202 -JC

What is the major cause of death in a patient with perforation of the pharynx, oesophagus or trachea? A: failure to intubate B: failure to ventilation C: sepsis

ANSWER C Core topics in airway management 2005 chap 23 airway mortality associated with anaesthesia and medico legal aspect pp173-176 Perforation of pharynx/oesophagus or trachea The cause of death is usually sepsis following the development of mediastinitis or cervical abscess. The mortality rate from mediastinitis is about 20%. Acute airway obstruction or respiratory failure can result from surgical oedema or pneumothorax. Only half of the perforations are recognized (pneumothorax or subcutaneous air) at the time. Perforation can occur during easy intubation, but is strongly associated with difficult intubation. Emedicine: http://emedicine.medscape.com/article/425410-overview#showall The esophagus lacks a serosal layer and is, therefore, more vulnerable to rupture or perforation. Once a perforation (ie, full-thickness tear in the wall) occurs, retained gastric contents, saliva, bile, and other substances may enter the mediastinum, resulting in mediastinitis. Esophageal perforation remains a highly morbid condition. Mortality rates are reported from 25-89% and are based predominantly on time of presentation and etiology of perforation. If treatment is instituted within 24 hours of symptoms, mortality rates are 25%; rates rose to above 65% after 24 hours and 75-89% after 48 hours.

75yo woman with an ejection systolic murmur presents for elective total knee joint replacement. Focussed transthoracic echocardiogram is performed. The feature most consistent with severe aortic stenosis is: A. Mean gradient across aortic valve of 30mmHg B. Peak gradient across aortic valve of 40mmHg C. Peak velocity across aortic valve of 4.2m/s D. Aortic valve area of 1.2cm2 E. Calcification and restriction of the aortic valve

ANSWER C Echocardiographic features of severity in AS: mean gradient > 40mmHg peak velocity > 4m/sec Valve area < 1.0 cm squared The definition of severe AS is based on natural history studies of patients with unoperated AS, which show that the prognosis is poor once there is a peak aortic valve velocity of >4 m per second, corresponding to a mean aortic valve gradient >40 mm Hg. In patients with low forward flow, severe AS can be present with lower aortic valve velocities and lower aortic valve gradients. Thus, an aortic valve area should be calculated in these patients. The prognosis of patients with AS is poorer when the aortic valve area is <1.0 cm2. At normal flow rates, an aortic valve area of <0.8 cm2 correlates with a mean aortic valve gradient >40 mm Hg.

An elderly lady has a closed neck of femur fracture and presents to ED. She is in chronic AF and on warfarin. INR is 2.6 and she is not bleeding. It is 9am and she is scheduled for repair the following day. According to current guidelines, how should her warfarin be reversed? A. Prothrombinex 25IU/kg immediately and then 2 units FFP immediately prior to surgery B. No immediate treatment then 2 units FFP immediately prior to surgery C. Vitamin K 1mg IV immediately D. Vitamin K 10mg IV immediately E. Withhold warfarin

ANSWER C INR <1.5 = proceed with surgery URGENT SURGERY INR <1.9 FFP INR <5.0 FFP + Vit K 1-3mg slow IV INR < 9 FFP Vit K 2-5mg slow IV SEMI URGENT INR <1.9 Vikt K 1mg oral INR< 5.0 Vit K 1.25mg oral, repeat INR INR <9.0, Vit K 2.5-5.0mg, repeat INR ELECTIVE SURGERY Bridging therapy if high risk of VTE Risk factors 1. Acute thrombosis, on treatment > 1month 2. Mechanical heart valve 3. Severe myocardial dysfunction 4. Atrial fibrillation Acute thrombosis <1month IVC filter

95 TMP-121 [Apr08] [Aug08] Levosimendan: A. Increases contractility and myocardial oxygen consumption B. Increases SVR C. Binds to troponin C and induces a conformational change D. Increases contractility by increasing calcium influx E. Causes coronary vasodilation but NOT peripheral vasodilation

ANSWER C Levosimendan causes conformational changes in cardiac troponin C during systole, leading to sensitisation of the contractile apparatus to calcium ions It increases contractility without increasing oxygen requirements Causes coronary and systemic vasodilation

101. New- Young pregnant patient with moderate mitral stenosis, normal LV function. The best delivery method A. Epidural anaesthesia LSCS B. Spinal with LSCS C. Epidural analgesia and normal vaginal delivery D. GA LSCS E. Normal vaginal delivery with remifentanil PCA

ANSWER C MS is a fixed output valvular disease -transmitral gradient is proportional to CO squared, therefore increasing CO by 50%, will increase the gradient 2.3 fold -generally pregnancy will increase NYHA class by One Aims are to keep the patient at they are -maintain preload -normal HR -maintain afterload MS should be evaluated before pregnancy -prophylactic percutaneous mitral balloon valvotomy -NHYA 2-4 or high PTH high risk of complications and death Treat antenatal with diuretic and beta blockers -care should be taken to avoid hypovolaemia -ACEI are contraindicated in pregnancy Labour and post partum are the most dangerous times -Epidural to blunt sympathetic response (attenuate HR, CO, therefore minimised increasing transmitral gradient) -Most delivery, sudden increase in preload with autotransfusion can lead to APO -Cautious use of Syntocinon during 3rd stage. Vasodilation can cause hypotension with compensatory tachycardia leading to overdistention of LA (due to MS) and AF => APO -AF should be electrically cardioverted.

Reverse splitting of the 2nd heart sound is caused by A. acute pulmonary embolism B. ASD C. complete LBBB D. severe MR E. pulmonary HT

ANSWER C Split during inspiration : Normal Split during expiration = Reverse splitting *Aortic stenosis *hypertrophic cardiomyopathy *left bundle branch block (LBBB) * ventricular pacemaker Split during both inspiration and expiration = fixed split S2 * atrial septal defect (ASD) * ventricular septal defect (VSD).

There is evidence to avoid BIS <40 for more than 5minutes because A. Safe cost B. Increase incident of hypotension C. Increase post op mortality D. Decrease volatile (?) for poor cardiac output patient E. Decrease the incidence of awareness

ANSWER C The effect of bispectral index monitoring on long-term survival in the B-Aware Trial. Anesth Analg 2010 BACKGROUND: When anesthesia is titrated using bispectral index (BIS) monitoring, patients generally receive lower doses of hypnotic drugs. Intraoperative hypotension and organ toxicity might be avoided if lower doses of anesthetics are administered, but whether this translates into a reduction in serious morbidity or mortality remains controversial. The B-Aware Trial randomly allocated 2463 patients at high risk of awareness to BIS-guided anesthesia or routine care. We tested the hypothesis that the risks of death, myocardial infarction (MI), and stroke would be lower in patients allocated to BIS-guided management than in those allocated to routine care. METHODS: The medical records of all patients who had not died within 30 days of surgery were reviewed. The date and cause of death and occurrence of MI or stroke were recorded. A telephone interview was then conducted with all surviving patients. The primary end point of the study was survival. RESULTS: The median follow-up time was 4.1 (range: 0-6.5) years. Five hundred forty-eight patients (22.2%) had died since the index surgery, 220 patients (8.9%) had an MI, and 115 patients (4.7%) had a stroke. The risk of death in BIS patients was not significantly different than in routine care patients (hazard ratio = 0.86 [95% confidence interval {CI}: 0.72-1.01]; P = 0.07). However, propensity score analysis indicated that the hazard ratio for death in patients who recorded BIS values <40 for >5 min compared with other BIS-monitored patients was 1.41 (95% CI: 1.02-1.95; P = 0.039). In addition, the odds ratios for MI in patients who recorded BIS values <40 for >5 min compared with other BIS-monitored patients was 1.94 (95% CI: 1.12-3.35; P = 0.02) and the odds ratio for stroke was 3.23 (95% CI: 1.29-8.07; P = 0.01). CONCLUSIONS: Monitoring with BIS and absence of BIS values <40 for >5 min were associated with improved survival and reduced morbidity in patients enrolled in the B-Aware Trial.

A patient has suffered flash burns of the upper half of the left upper limball of the left lower limb and the anterior surface of the abdomen. The approximate percentage of the body surface which has been burned is A. 18% B. 23% C. 32% D. 41% E. 48%

ANSWER C 4.5+18+9 =31.5

You place a thoracic epidural for a patient having an elective open AAA repair. There are 4cm in the epidural space and you aspirate blood. What is the most appropriate management plan: A. inject 5 mL of saline, and if you can no longer aspirate blood, leave in place and use B. inject 5 mL lignocaine 2% with adrenaline. If there is no rise in HR be happy that it is not intravascular and secure in place and use C. Remove and postpone surgery for 24 hours D. Remove and place epidural 1 level higher E. Remove and postpone surgery for 4 hours

ANSWER C NYSORA textbook of regional anaesth & pain Mx (online)

Pierre-Robin sequence is characterized by cleft palatemicrognathia and A. Craniosynostosis B. Macroglossia C. Glossoptosis D. Microstomia E. ?,

ANSWER C Posterior displacement or retraction of the tongue Prone to upper airway obstruction

A young female patient with anorexia nervosa had just started eating again. After three days she develops dyspnea and is found to have cardiac failure. Which of the following is the most important to correct A. Potassium B. Chloride C. Phosphate D. Glucose E. Sodium

ANSWER C The refeeding syndrome is marked by hypophosphatemia, hypokalemia, hypomagnesemia, and volume overload. Hypophosphatemia is the hallmark of the syndrome and predominant cause of the refeeding syndrome . One case series found that hypophosphatemia requiring phosphate replacement occurred in 28 percent of adolescents hospitalized for treatment of anorexia nervosa (AN). The pathogenesis of hypophosphatemia begins when stores of phosphate are depleted during episodes of AN and starvation. When nutritional replenishment begins and patients are fed carbohydrates, glucose causes release of insulin, which triggers cellular uptake of phosphate (and potassium and magnesium). Insulin also causes cells to produce a variety of depleted molecules that require phosphate (eg, adenosine triphosphate (ATP) and 2,3-diphosphoglycerate), which further depletes the body's stores of phosphate. The lack of PHOSPHORYLATED intermediates causes tissue hypoxia and resultant MYOCARDIAL DYSFUNCTION & RESP FAILURE http://www.uptodate.com/contents/anorexia-nervosa-in-adults-and-adolescents-the-refeeding-syndrome

131. New - When stimulating the ulnar nerve with a nerve stimulator, which muscle do you see twitch? A. opponens abducens B. abductor pollicis brevis C. adductor pollicis brevis D. extensor pollicis E. flexor pollicis brevis

ANSWER C Ulnar nerve supplies the intrinsic muscles of the hand, except LOAF

In patient with ankylosing spondylitis which of the following is INCORRECT A. Amyloid renal infiltration is rarely seen B. Cardiac complications occur in < 10% C. Normocyctic anaemia occurs in 85% of cases D. Uveitis is the most common extra-articular presentation E. Sacroilitis is an early sign of presentation

ANSWER C All the others are true http://emedicine.medscape.com/article/332945-clinical#showall Extra-articular manifestations include the following: Uveitis Cardiovascular disease Pulmonary disease Renal disease Neurologic disease GI disease Uveitis Uveitis (also called iritis or iridocyclitis) is the most common extra-articular manifestation of AS, occurring in 20-30% of patients Clinically significant cardiovascular involvement occurs in fewer than 10% of AS patients. Aortitis of the ascending aorta may lead to distortion of the aortic ring, resulting in aortic valve insufficiency. Mitral valve insufficiency rarely occurs. Fibrosis of the conduction system may result in various degrees of atrioventricular block, including complete heart block. Pulmonary involvement- Restrictive lung diseae in late-stages Renal involvement - Amyloidosis is a very rare complication of AS in patients with severe, active, and long-standing disease.

A 40 year-old lady with a history of a bleeding diathesis presents for a tonsillectomy. What is the most likely cause? A. Factor V Leiden B. Protein S deficiency C. Haemophilia B D. Antithrombin III deficiency E. Protein C deficiency

ANSWER C Haemophilia B: x-link disease Other options hyper coagulable diseases F V Leiden Thrombophilia. FV Leiden is a variant (mutant) form of FV that causes increased clotting as FV Leiden cannot be inactivated by protein C

A 70 year-old male presents for right lower lobectomy. Preoperative spirometry shows FEV1 2.4L (4.2L predicted)FVC 4L (5L predicted). The predicted post-operative FEV1 is: A. 1.0L B. 1.3L C. 1.7L D. 1.9L E. 2.2L

ANSWER C 14/19x2.4=1.77

You insert a thoracic epidural in a patient for a liver resection with an upper abdominal incision. You have recently topped it up. On waking the patient appears weak, despite adequate reversal. He can breathe spontaneously and can flex his biceps but is not able to extend triceps. The level of the block is most likely to be: A. C5 B. C6 C. C7 D. C8 E. T1

ANSWER C 5-6 shoulder movt 6-7 elbow flexion 7-8 elbow extension 7-8 wrist flexion/extension 8-T1 finger movt see myotome dance :) http://www.youtube.com/watch?v=NHaZMFpKOMA

The insulation on the power cord of a piece of class 1 equipment is faulty such that the active wire is in contact with the equipment casing. What will happen when the power cord is plugged in and the piece of equipment is turned on A. The double insulation of the device will prevent macroshock when the outer casing is touched B. The electrical fuse will immediately break and disconnect the device from the power supply C. Equipotential earthing will prevent microshock from anyone who touches it. D. The Line Isolation Monitor will alarm and disconnect power to the device E. The RCD will immediately disconnect the device from the power supply

ANSWER C A- no double insulation is present on Class 2 equipment not Class 1 B- there isn't a fuse in the piece of equipment per se. C- Correct: From RCH Electrical Safety site: Class I equipment is fitted with a three core mains cable containing a protective earth wire. Exposed metal parts on class I equipment are connected to this earth wire. Should a fault develop inside the equipment and the exposed metal comes into contact with the mains, the earthing conductor will conduct the fault current to ground. Regular testing procedures ensure that earthing conductors are intact, as the integrity of the earth wire is of vital importance D- no the LIM is external to the piece of equipment, and DOES NOT disconnect the equipment from power, just alarms when equipotentiality of the two monitored circuits is disrupted, indicating current leakage, possibly through faulty equipment. E- RCD disconnects after 40ms, not immediately. In hospital RCD trips at leakage of 10mAmps. RCDs are present in the external circuit, not in the piee of equipment itself (walls/ towers of theatre)

According to the current ANZCA approved standards for labeling the appropriate colour label for an intraosseous infusion is A. Yellow B. Beige C. Pink D. Blue E. Red

ANSWER C IA - Red IV - Blue Neural/Regional - yellow Sub cut - Beige Other - Pink

Most cephalic interspace in neonate to perform spinal while minimising the possibility of spinal cord puncture A. L1-L2 B. L2-L3 C. L3-L4 D. L4-L5 E. L5-S1

ANSWER C NYSORA.com Spinal cord at L2/3 in neonate, dural sac at S3

You are performing a TAP block. If the needle is correctly positioned where will you deposit the local anaesthetic A. Beneath the peritoneum B. Into the transverse abdominus muscle C. Between the transverse adominus muscle and the internal oblique muscle D. Between the transverse abdominus muscle and the external oblique muscle E. Between the internal oblique and the external oblique muscle

ANSWER C http://www.nysora.com/files/2013/pdf/(v12p28-33)TAPBlock.pdf

What transfusion related complication is the commonest cause of mortality a. Bacterial infection b. TRALI c. ABO incompatibility d.

B

37 female presents to ED with headache and confusion. She is otherwise neurological normal and haemodynamically stable. Urine catheter and bloods takes. UO > 400ml/hr for 2 consecutive hours, Serum Na 123 mmol/l, Serum Osmolality 268, Urine Osmolality 85 The most likely diagnosis is A. Central diabetes insipidus B. Nephrogenic diabetes insipidus C. Psychogenic polydipsia D. Cerebral salt wasting E. SIADH

ANSWER C ■A low plasma sodium concentration (less than 137 meq/L) with a low urine osmolality (eg, less than one-half the plasma osmolality) is usually indicative of water overload due to primary polydipsia. ■A high-normal plasma sodium concentration (greater than 142 meq/L, due to water loss) points toward DI, particularly if the urine osmolality is less than the plasma osmolality [1]. ■A normal plasma sodium concentration is not helpful in diagnosis but, if associated with a urine osmolality more than 600 mosmol/kg, excludes a diagnosis of DI. -from UTD

You are anaesthetising a fit 50 year old woman for an elective laparoscopic cholecystectomy. In her pre operative assessment she has a normal cardiovascular exam and her BP is 115/75. You induce anaesthesia with 100mcg fentanyl, 100mg propofol and 50 mg rocuronium. Soon after induction her ECG looks like this (showed narrow complex tachycardia around 180-200/min - ie SVT). Her BP is now 95/50. What is the most appropriate management? A. adenosine B. amiodarone C. DC cardioversion D. GTN E. metaraminol

ANSWER C EMAC course book "DC cardioversion if serious signs, HR > 150, failed medical treatment"

73 Stellate ganglion block associated with all except: A. Ptosis B. Miosis C. Sweating D. Facial flushing E. Nasal stuffiness

ANSWER C Stellate Ganglion produces Horners syndrome * Ptosis * Miosis * Anhidrosis In addition, several other eye signs are present: * Conjunctival injection * Lacrimation

You are anaesthetising a 6 month-old infant for repair of a VSD. You perform an inhalational induction with 8% sevoflurane and 50% nitrous oxide. Several minutes later, whilst trying to secure IV access, the infant's oxygen saturations fall to 85%. The most appropriate next step in management: A. give a fluid bolus B. change from sevoflurane to isoflurane C. apply CPAP D. reduce the FiO2 E. reduce sevoflurane

ANSWER C (although some debate btwn C & E)

A 42 year old lady presents for right pneumonectomy with a left sided double-lumen tube. She is 132kg and 160cm. What depth, measured at the incisors, is likely to give the ideal position? A. 24cm B. 26cm C. 28cm D. 30cm E. 32cm

ANSWER C - 28 cm 1. largest DLT that will pass through the glottis 41 or 39 males, 37 for females. 2. Left sided tube preferred; less likely to block lobar bronchus, greater tolerance to shift 3. Height is the main determinant of correct insertion depth. Depth = 12 + height/10 (also written as 29 +/- cm for every 10cm from 170)

How quickly does the CO2 rise in the apnoeic patient ? A. 1 mmHg per min B. 2 mmHg per min C. 3 mmHg per min D. 4 mmHg per min E. 5 or ?8 mmHg per min

ANSWER C - 3 per min

ASD murmur heard at A. ASD B. Tricuspid valve C. Pulmonary valve D. Mitral valve E. Aortic valve

ANSWER C - Pulmonary Valve

A 45 year old obese man complains of headache, lower limb weakness and polyuria. On examination, his blood pressure is 150/70mmHg. He has a displaced apex beat. Bloods revealed Na145, K2.8, Cl101, HCO3 27. What is the most likely diagnosis A. Cushings B. Diabetes C. Primary hyperaldosteronism D. Hypothyroidism E. Phaeochromocytoma

ANSWER C - accepted as is Hyperaldosteronism: Primary usually from adenoma causes- salt retention, hypervolaemia (therefore hypertension), causing mildly increased serum Na (145-147) Potassium wasting from cortical collecting tubule, which causes muscle weakness (usually not til K <2.5) Hydrogen ion excretion from distal tubule Mild Mg wasting

What is the first sign/symptom seen with an inadvertent total spinal whilst performing caudal anaesthesia in a neonate A. Hypotension B. Bradycardia C. Desaturation D. Tachycardia E. Loss of consciousness

ANSWER C - desaturation

A type I diabetic is fasting pre-operatively and you decide to place them on an IV insulin infusion to optimize their perioperative glycaemic control. Their BSL is 7mmol/L. BY what mechanism does the insulin infusion decrease their BSL A. Stimulates glucose uptake into the liver B. Stimulates glucose uptake into skeletal muscle C. Inhibits glucose production in the liver D. Decreases glucose absorption from the gastrointestinal tract E. Inhibit glucagon release

ANSWER C - kate It does inhibit glucose-6-phosphatase in liver thus reduce glucose production

An 8 year old 30kg girl presents for resection of a Wilms tumour. Her starting haematocrit is 35% and you decide that your trigger for transfusion will be 25%. The amount of blood that she will need to lose prior to transfusion is A. 400mL B. 500mL C. 600mL D. 700mL E. 800mL

ANSWER C - kate variation uses 80mL/kg not 70 as child EBV (80mL/kg) = 2400mL HCT- target HCT/ HCT = 35-25/35 = 0.28 0.28 x 2400 = 680mL

PiCCO determines cardiac output utilizing A. Thermodilution B. Pulse contour analysis C. Thermodilution and pulse contour analysis D. ? Doppler E. ?

ANSWER C - thermodilution + Pulse contour

?A 40 year old man with Marfan's has undergone a thoracoabdominal aneurysm repair. 48 hours post procedure there is blood noted in his CSF drain and he is obtunded. Your next course of action is: A. Coagulation studies B. CSF microscopy and culture C. CT Head D. MRI Head E. MRI Spine

ANSWER C ? subdural

138. A 62 year old man has chronic renal failure. You notice his total serum calcium is 2.05 mmol/L. This is because he has A. high serum vitamin D B. hypoparathyroidism C. primary hyperparathyroidism D. secondary hyperparathyroidism E. tertiary hyperparathyroidism

ANSWER D

44. NEW. Multiple attempts to place ETT during difficult intubation causing pharyngeal and oesophageal perforations. Most likely cause of death? A: Failure to intubate B: Failure to ventilate C: Bleeding D: Sepsis

ANSWER D

59.RB53 Repeat- Post dural punture headache A. 24hour bed rest B. Prone position worst C. Increase incidence with insertion of spinal catheter D. Hearing loss

ANSWER D

A female patient with a history of COPD presents for lung volume reduction surgery which of the following is a contraindication for surgery (? indicates a poor prognosis) A. Age > 60 years B. Chronic asthma C. Evidence of bullous disease on CT scan D. FEV < 25% E. Long-term prednisolone 10mg/day

ANSWER D

56. New- You see a man in his 60s in clinic 1 week prior to laparoscopic cholecystectomy. He has dilated cardiomyopathy with an ejection fraction of 30%, but does not get dyspnoeic with normal activities of daily living. What is the most appropriate management of his heart failure? A. amiodarone 100mg bd B. digoxin 250mcg daily C. enalapril 2.5mg bd D. metoprolol 100mg bd E. diltiazem slow release 240mg daily

ANSWER C CEACCP 2008 Systolic dysfunction is the major pathological component of DCM; medical therapy is as for chronic heart failure. A preoperative history of heart failure is an important risk factor for postoperative cardiac complications; risk increases when clinical signs are present before surgery. Optimal management of this heart failure should occur before an operation. Angiotensin inhibitors ACEIs have been shown to improve survival in all grades of left ventricular failure with angiotensin II inhibitors indicated in those who do not tolerate ACEI. Whether these drugs should be given on the day of surgery is contentious. ACEIs are known to cause intraoperative hypotension, but this is often short-lived and gener- ally responds to i.v. fluids and conventional treatment. If the decision is taken to omit ACEIs, then they should be recommenced as soon as possible after surgery. b-Blockade The use of b-blockers in the treatment of chronic heart failure has been shown to improve mortality, and is well established. In patients undergoing surgery and considered at high risk of coronary events, b-blockers have been shown to improve outcomes. There is, however, no evidence that they improve perioperative outcome in patients with acute heart failure. Diuretics Spironolactone is an aldosterone antagonist that has been shown to decrease mortality by 30% in those with severe (New York Heart Association class III or IV) chronic heart failure already on ACEIs and loop diuretics. Loop diuretics are useful in relieving symp- toms, despite being unproven in prolonging survival. Biventricular pacing In addition to medication, some patients gain benefit from biventri-cular pacing. These patients lack coordination of ventricular con- traction, due to delayed pathways of conduction through damaged myocardium. Re-synchronization of left and right ventricular contraction by coordinated stimulation from leads in both ventricles (that improves stroke volume) is supported by recommendations from NICE (May 2007).

Pringles procedure for life threatening liver haemorrhage includes clamping of: A. Hepatic artery B. Hepatic vein C. Portal pedicle - (should be called hepatic pedicle prob/ poor memory of question) D. Aorta E. Splenic Artery

ANSWER C Hepatoduodenal ligament clamping. max time 20 mins The Pringle manoeuvre is a surgical manoeuvre used in some abdominal operations. A large atraumatic haemostat is used to clamp the hepatoduodenal ligament (free border of the lesser omentum) interrupting the flow of blood through the hepatic artery and the portal vein and thus helping to control bleeding from the liver. More commonly, in the absence of soft clamp, manual compression of the hepaticoduodenal ligament is performed.

A 25 year male with a history of asthma who is usually on fluticonasone and salbutamol nebs presents with an acute exacerbation. On examination you see he is distressedRR 26 bpm. On auscultation: poor air entry and polyphonic wheeze bilaterally. ABG: pH 7.45, pCO2 27, pO2 75, HCO3 24. He has been treated with salbutamol and ipratropium nebules and intrasvenous hydrocortison:E. What is the next step in his treatment A. Inhaled helium/oxygen B. IV aminophylline C. IV magnesium D. IVsalbutamol E. Intubation and ventilation

ANSWER C IV Magnesium 2g over 10 mins

Incidence and severity of vasospasm following sub-arachnoid haemorrhage is seen maximally at A. 0-24 hrs B. 2-4 days C. 6-8 days (note 5-7 days in the old question) D. 7-10 days E. 2 weeks

ANSWER C Peak incidence of vasospasm is 4-10 days. Surgery previously delayed until this was over but now nimodipine used. (so would choose C here ) Risk of rebleed highest in first 24 hrs; aim f clip/coil is to secure aneurysm & prevent rebleed. Sub Arachnoid Haemorrhage World Federation of Neurosurgeons Grading influeces morbidity/moratlity; used to determine operation or early coiling (1-3) or delay iterventions (4-5) 1 GCS 15 2 GCS 13-14 3 GCS 13-14 4 GCS 7-12 5 - GCS 3-6

Ulcerative colitis is associated with all of the following EXCEPT: A. Cirrhosis B. Iritis C. Psoriasis D. Arthritis E. Sclerosing cholangitis

ANSWER C Psoriasis (associated with Crohn's )

In the Revised Trauma Score, the initial assessment parameters include Glascow Coma Scale, Blood Pressure, and : A. Heart Rate B. Saturation C. Respiratory Rate D. Urine Output E. Temperature

ANSWER C RR

Which of the following statements regarding patients with ankylosing spondylitis is FALSE? A. amyloid renal infiltration is rarely seen B. cardiac complications occur in less than 10% of cases C. normochromic anaemia occurs in over 85% of cases D. sacroileitis is an early sign of presentation E. uveitis is the most common extra-articular manifestation

ANSWER C anaemia not a feature usually

A 40 y.o. female newly diagnosed ITP. Retinal detachment for surgery in 2 days. Platelets 40 and blood group A+. Management of her ITP: A. Administer Anti-D antibodies 6 hrs pre op B. Admister desmopressin one hour pre op C. Administer methylpred and IVIg 2 days pre op D. Recheck platelet count morning of surgery and if not dropped continue E. Platelet transfusion morning of surgery

ANSWER C but not exactly right acc to UTD- For management of life-threatening bleeding, treatment options include: ■Platelet transfusions ■IVIG (1 g/kg, repeated the following day if the platelet count remains <50,000/microL) ■Pulse methylprednisolone (1 g intravenously, repeated daily for three doses) ■Recombinant human factor VIIa no mention of desmopressin, and no mention of Anti-D for use in the acute phase.

According to guidelines endorsed by ANZCA, the label of an intra-osseous infusion should be A. beige B. blue C. Pink D. Red E. yellow

ANSWER C pink beige = subcutaneous blue = venous pink = other red = arterial yellow = perineural/nerve catheter

During scoliosis surgery with monitoring of somatosensory evoked potentials which tract are they mainly monitoring A. Anterior horn B. Anterior corticospinal tract C. Dorsal column D. Spinothalamic tract E. Lateral corticospinal tract,

ANSWER C somatosensory are posterior or dorsal columns; these are supplied by posterior spinal arteries. The anterior or ventral spinal column has the motor tracts that can be measured by evoked motor potentials. Pyramidal = lateral corticospinal; Extrapyrimidal = vestibulospinal; reticulospinal; oliviospinal Corticospinal is a descending pathway. Spinothalamic tract = anteriolateral or ventrolateral tract. Consists of: -> Lateral Spinothalamic = Pain + temperature -> Anterior Spinothalamic = Crude touch

The thoracodorsal nerve arises from A. A Medial cord of brachial plexus B. Lateral cord of brachial plexus C. Posterior cord of brachial plexus D. Dorsal scapular nerve E. Long thoracic nerve

ANSWER C thoracodorsal nerve comes from posterior cord of brachila pelxus, supplies lat dorsi.

111. Repeat- Best indicator of return function of laryngeal muscle A. Sustained head lift 5 sec B. Sustained leg lift 5 sec C. TOF 0.9 D. DBS no fade E. Tetanus 50Hz

ANSWER D

A patient with a head injury is found to have a unilateral dilated pupil with no direct or consensual response to light. What is the most likely diagnosis A. Global injury B. Optic nerve injury C. Horners syndrome D. Transtentorial herniation E. Injury to the pons

ANSWER D

An 85 year old patient with a bleeding disorder (? haemophilia) suffers a fractured neck of femur (#NOF). You discuss the possibility of a needing a blood transfusion but despite your explanation they refuse because they are scared of CJD infection post transfusion. Subsequently you decide not to proceed with the case because of the high risk of bleeding. The ethical principle that this is an example of is A. Paternalism B. Coercion C. Justice D. Beneficience E. Autonomy

ANSWER D

Blunt liver trauma can be treated non surgically if A. No peritoneal signs B. Low Grade injury on CT scan C. Severe COPD D. Haemodynamically stable E. US confirms <500mls peritoneal fluid collection (i thought this was a paracentesis result)

ANSWER D

Circuit disconnection during spontaneous breathing anaesthesia A. Will be reliably detected by a fall in end-tidal carbon dioxide concentration B. Will be detected early by the low inspired oxygen alarm C. Will be most reliably detected by spirometry with minute volume alarms D. May be detected by an unexpected drop in end-tidal volatile anaesthetic agent concentration E. Can be prevented by using new single-use tubing

ANSWER D

During apnoeic oxygenation under light anaesthesia, the expected rise in PaO2 would be: A. 0.5 mmHg per min B. 1 mmHg per min C. 2 mmHg per min D. 3 mmHg per min E. 5 mmHg per min

ANSWER D

During endovascular aneurysm repair, GA is preferred due to: A. risk of uncontrolled haemorrhage B. renal ischaemia is painful C. aorta traction is painful D. long duration of apnoea is needed E. contrast used can cause CVS instability

ANSWER D

For a nurse monitoring an opioid PCA, the earliest sign of respiratory depression is; A. Number of boluses of PCA per hour B. Respiratory rate C. Oxygen saturation D. Sedation score E. Pupil size

ANSWER D

Hypercalcaemia due to hyperparathyroidism is associated with A. A shortened PR interval B. A prolonged QTc interval C. Muscle rigidity D. Polyuria and polydipsia E. Increased glomerular filtration rate

ANSWER D

In a patient with porphyria, the drug most likely to cause an acute episode is A. morphine B. propofol C. propanidid D. phenytoin E. atropine

ANSWER D

In pregnancy the dural sac ends at: A. T12 B. L2 C. L4 D. S2 E. S4

ANSWER D

Isoflurane is administered in a hyperbaric chamber at 3 atmospheres absolute pressure using a variable bypass vaporizer. At a given dial setting and constant fresh gas flow, vapour will be produced at: A. the indicated vapour concentration B. three times the indicated vapour concentration C. one third the partial pressure obtained at 1 atmosphere D. the same partial pressure as is obtained at 1 atmosphere E. three times the partial pressure obtained at 1 atmosphere

ANSWER D

Laser endotracheal tubes A. More resistant to ignition when covered in blood B. Resistant to electrosurgical cautery C. Wont ignite when touched by laser D. Have larger external diameter for same internal diameter relative to standard PVC tubes E. Have double cuffs which are resistant to puncture by laser

ANSWER D

Post operative left pneumonectomy. What to do with underwater seal drain? A: Nurse patient in R lateral decubitus position B: Expect to see bubbles C: Suction every hour for 5 minutes D: Unclamp drain once an hour for 5 minutes, leave clamp on for the rest of the time E: Leave on free drainage

ANSWER D

Prolonged Trendelenburg (head-down) positioning causes which of the following A. No change in intracranial pressure B. No change in intraocular presssre C. No change in pulmonary venous pressure D. Increased myocardial work E. Increased pulmonary compliance

ANSWER D

Recognised weaknesses of systematic reviews include all of the following EXCEPT A. publication bias B. duplicate publication C. study heterogeneity D. inclusion of outdated studies E. systematic review author bias

ANSWER D

Regarding endotracheal tubes used in laser surgery: A. They are more resistant to combustion when the cuff is covered in blood B. Resistant to ignition from electrocautery C. The cuff is resistant to ignition if hit by the laser D. Have an external diameter which is larger than a normal PVC endotracheal tube (compared to the internal diamater) E. Have 2 cuffs which are resistant to combustion

ANSWER D

Regarding rotameters A. The bobbin is contained in a tube with parallel sides B. There is laminar flow at high flows C. The height of the bobbin is directly proportionate to the pressure drop across the bobbin. D. There is a constant pressure difference across the bobbin at all flows. E. Resistance increases at high flows

ANSWER D

TMP-Jul10-062 In pregnancy the dural sac ends at: A. T12 B. L2 C. L4 D. S2 E. S4

ANSWER D

The commonest postoperative complication in a patient with a # NOF is A. UTI B. Pneumonia C. Myocardial Infarction D. Delirium E. ?

ANSWER D

What happens when you place a magnet over a biventricular internal cardiac defibrillator A. Switch to asynchronous pacing B. Damage the internal programming C. Nothing D. Switch off antitachycardia function E. Switch of rate responsiveness

ANSWER D

When performing laryngoscopy using a Macintosh blade, your best view is of the patient's epiglottis touching the posterior pharyngeal wall. Using the Cormack and Lehane scale this is grade A. 1 B. 2 C. 3a D. 3b E. 4

ANSWER D

Which of the following is most effective way to reduce renal failure in AAA surgery A. Fluid bolus prior to aortic clamping B. Fluid bolus after aortic clamp release C. Frusemide D. Minimize cross-clamp time E. Mannitol

ANSWER D

Which of the following should be used by a lay person to indicate that they should commence CPR A. Absence of central pulse B. Absence of peripheral pulse C. Loss of consciousness D. Absence of breathing E. Obvious airway obstruction

ANSWER D

Why should NSAIDs be avoided in pregnant women >30 weeks gestation? A. cause neonatal acute renal failure B. increased antepartum haemorrhage C. increased rate of pre-eclampsia D. cause closure of the fetal ductus arteriosus E. increase preterm labour

ANSWER D

You are performing an awake fibreoptic intubation through the nose, on an adult patient. In order, the fibrescope will encounter structures with sensory innervation from the following nerves A. facial, trigeminal, glossopharyngeal B. facial, trigeminal, vagus C. glossopharyngeal, trigeminal, vagus D. trigeminal, glossopharyngeal, vagus E. trigeminal, vagus, glossopharyngeal,

ANSWER D

Coeliac plexus block. What is the complication? A: Erectile dysfunction B: Constipation C: Hypertension which resolves spontaneously D: Paralysis E:

ANSWER D 3 common transient adverse effects 1. Local pain (96%) 2. Diarrhoea (44%) 3. Hypotension (38%) Other complications include o Paraplegia o Parasthesia o Bleeding → retroperitoneal haematoma o Epidural injection; Subarachnoid injection o Intravascular injection o Pneumothorax; Chylothorax Neurology caused by - Direct injury to spinal cord or somatic nerves - Spinal cord ischaemia ---> Spasm or thrombosis of the artery of Adamkiewicz → spinal cord ischaemia T8 to L4 *

A patient is cooled to 33 degrees Celcius in an attempt to improve neurological outcome after out-of-hospital ventricular fibrillation cardiac arrest. The evidence for this treatment comes from A. Case Reports B. Case Control Studies C. Systematic Review D. Randomized Control Trial E. Pseudo-randomized Trial

ANSWER D 2010 AHA guidelines for CPR and emergency cardiovascular care Circulation 2010 122:S768-786 (kate seen; agree) NEJM 2002 346:549-566 Mild therapeutic hypothermia to improve the neurologic outcome after cardiac arrest (RCT) Treatment of comatose survivors of out-of-hospital cardiac arrest with induced hypothermia. N Engl J Med. 2002 346:557-563 (Pseudo randomized trial)

Circuit disconnection during spontaneous breathing anaesthesia A. will be reliably detected by a fall in end-tidal carbon dioxide concentration B. will be detected early by the low inspired oxygen alarm C. will be most reliably detected by spirometry with minute volume alarms D. may be detected by an unexpected drop in end-tidal volatile anaesthetic agent concentration E. can be prevented by using new, single-use tubing

ANSWER D A. FALSE: Gas analyzer is still connected to patient despite disconnection of circuit, therefore ETCO2 will be normal B. FALSE: Disconnection allows entrainment of room air which will not be detected by low inspired O2 alarm C. FALSE : Depends where the spirometry is taken from, but some spirometry taken near the filter and so a disconnect at the machine will not be detected D. TRUE : Any disconnect will allow entrainment of room air into the circuit and allow escape of the volatiles, thus a drop in the ET-agent. E. FALSE: Single use tubing will not prevent a disconnection. Apnea/disconnect alarms may be based on 1. Chemical monitoring (lack of end tidal carbon dioxide) 2. Mechanical monitoring * Failure to reach normal inspiratory peak pressure, or * Failure to sense return of tidal volume on a spirometer 3. Visual monitoring * Failure of standing bellows to fill during mechanical ventilator exhalation * Failure of manual breathing bag to fill during mechanical ventilation (machines with fresh gas decoupling- the Apollo, Fabius GS, Narkomed 6000) 4. Auditory monitoring - lack of breath sounds in precordial, lack of sound from ventilator cycling, etc. 5. Optic monitoring - Failure of the hanging bellows to fill completely (the "garage door" electronic eye sensor on the Julian)

38 ANZCA Version [Jul07]Q.150 You are asked to see a 60 y.o. male 2 days following a cervical laminectomy because he has new new neurological symptoms in his right arm. The surgical team think these may be due to poor patient positioning. The sign that would most help differentiate c C8-T1 nerve root injury from an ulnar nerve injury is A. loss of sensation in the index finger B. loss of sensation in the little finger C. weakness of the abductor digiti minimi muscle D. weakness of the abductor pollicis brevis m E. weakness of the first dorsal interosseous m.

ANSWER D A. FALSE: neither ulnar nerve or C8T1 supply, therefore non differentiating B. FALSE. both ulnar and C8T1 supply therefore non differentiating C. FALSE: both affected by C8T1 and ulnar therefore non differentiating D. TRUE: T1 and median nerve supply, but ulnar does not E. FALSE. both supplied by c8T1 and ulnar nerve therefore non differentiating

You are doing a supraclavicular brachial plexus block on an awake 35 year-old lady who is healthy with no significant past medical hsitory. Soon after injecting 20mLs of 0.375% ropivicaine she becomes agitatedhas a seizure and loses consciousness. Your 1st step in management is A. Intralipid 20% 1.5ml/kg bolus B. Midazolam C. Propofol D. Establish airway and give 100% O2 via a facemask E. Feel for radial pulse and give 100mcg adrenaline,

ANSWER D AAGBI protocol. Signs of toxicity - Sudden alteration in mental status, severe agitation, LOC +/- tonic/clinic seizures. - CV collapse - sinus brafy, conduction blocks, asystole, VT. Stop injecting, call for help, maintain airway with 100% O2, control seizures with benzo (or propofol), assess CVS throughout. In arrest - start CPR, manage arryhthmias; Intralipid 20% 1.5mL/kg over 1 min Start infusion of intralipid 20% at 15ml/kg/hr. Max of 2 repeat doses, 5 min between bolusus. Max 12mL/kg (this will be 100mL boluses in 70kg with max 840mL) Transfer Watch for pancreatitis *

A G1P0 patient with a dilated cardiomyopathy and an ejection fraction (EF) of 35% presents for a caesarean section. The benefits of regional vs general anaesthesia are A. Decreased heart rate B. Decreased systolic blood pressure C. Increased ejection fraction D. Increased myocardial contractility E. Decreased preload

ANSWER D Anaesthetic goals in patients cardiomyopathy -avoidance of drugs which reduce contractility -maintenance of normovolaemia -prevention of increased ventricular afterload Answer on card already........but i agree.... Overall avoiding the cardiodepressants effects of a general anaesthetic is the advantage of a slowly titrated epidural. Regional - will improve myocardial performance by reducing the left ventricular after load; - move the patient down the Starling curve by reducing the preload.

The cause of early mortality (early - within 30 minutes) in a pregnant women with amniotic fluid embolism is A. Bronchospasm B. Hypovolaemia C. Malignant Arrhythmia D. Pulmonary Hypertension E. Pulmonary Oedema

ANSWER D CEACCP amniotic fluid embolus 2007

A new antiemetic decreases the incidence of PONV by 33% compared with conventional treatment. 8% who receive the new treatment still experience PONV. The no of patients who must receive the new treatment instead of the conventional before 1 extra patient will benefit is A. 3 B. 4 C. 8 D. 25 E. 33

ANSWER D 25 but don't know the formulae

The half life of the active metabolite of levosimendan is: A: 1hr B: 8hr C: 24hr D: 3 days E: 7 days

ANSWER D Clin Pharmacokinet. 2007;46(7):535-52. Levosimendan has been developed for the treatment of decompensated heart failure and is used intravenously when patients with heart failure require immediate initiation of drug therapy. It increases cardiac contractility and induces vasodilatation. The pharmacokinetics of levosimendan are linear at the therapeutic dose range of 0.05-0.2 microg/kg/minute. The short half-life (about 1 hour) of the parent drug, levosimendan, enables fast onset of drug action, although the effects are long-lasting due to the active metabolite OR-1896, which has an elimination half-life of 70-80 hours in patients with heart failure (New York Heart Association functional class III-IV). Although levosimendan is administered intravenously, it is excreted into the small intestine and reduced by intestinal bacteria to an amino phenolpyridazinone metabolite (OR-1855). This metabolite is further metabolised by acetylation to N-acetylated conjugate (OR-1896). The circulating metabolites OR-1855 and OR-1896 are formed slowly, and their maximum concentrations are seen on average 2 days after stopping a 24-hour infusion. The haemodynamic effects after levosimendan seem to be similar between fast and slow acetylators despite the fact that the enzyme N-acetyltransferase-2, which is responsible for the metabolism of OR-1855 to OR-1896, is polymorphically distributed in the population. Levosimendan reduces peripheral vascular resistance and has direct contractility-enhancing effects on the failing left ventricle. It also improves indices of diastolic function and seems to improve the function of stunned myocardium. Despite an improvement in ventricular function, levosimendan does not increase myocardial oxygen uptake significantly. An increase in coronary blood flow and a reduction in coronary vascular resistance have been observed. Levosimendan reduces plasma brain natriuretic peptide (BNP) and N-terminal pro-BNP (NT-proBNP) levels substantially, and a decrease in plasma endothelin-1 has been observed. Levosimendan also exerts beneficial effects on proinflammatory cytokines and apoptosis mediators. The effects of a 24-hour levosimendan infusion on filling pressure, ventricular function and BNP, as well as NT-proBNP, last for at least 7 days.

After coronary artery bypass graft surgery, the FRC is A. increased 40% B. increased 20% C. unchanged D. decreased 20% E. decreased 40%

ANSWER D Compared to preop -open sternotomy 55% increase -closure decrease 10% -day 1 decrease 20%

Complication of celiac plexus block A. Hypertension B. Failure of erection C. Constipation D. Paraplegia E. L3,4 lumbar pain

ANSWER D Complications • Severe hypotension may result, even after unilateral block. • Local pain during procedure • Diarrhoea • Intramuscular injection into the psoas muscle. • Bleeding due to aorta or inferior vena cava injury by the needle. • Intravascular injection (should be prevented by checking the needle position with radio-opaque dye). • Upper abdominal organ puncture with abscess/cyst formation. • Paraplegia from injecting phenol into the arteries that supply the spinal cord (prevented by checking the needle position with radio-opaque dye). • Lumbar nerve root irritation (injected solution tracks backwards towards the lumbar plexus). • Sexual dysfunction (injected solution spreads to the sympathetic chain bilaterally) = inability to ejaculate • Pneumothorax • Shoulder/chest/pleuritic pain/hiccupping - diaphragmatic irritation • Haematuria from renal pouncture

In an arterial line system

ANSWER D Damping : Progressive diminution of amplitude of oscillations in a resonant system by dissipation of energy. In arterial BP may be caused by kinking, clots, bubble, tubing, viscous drag of fluid.

83. AB58 ANZCA version [Jul07] Q141 A young woman with type 1 von Willebrand disease presents for a dilatation and curettage. She is a Jehovah's Witness. You consider administering intravenous desmopressin in an attempt to reduce haemorrhage. Which of the following statements regarding desmopressin is FALSE? A. it is a synthetic substance and is acceptable to Jehovah's Witnesses B. it is likely to reduce haemorrhage in this patient C. it should be given 30 minutes prior to surgery as an infusion D. its duration of effect is approximately 5 days E. the intravenous dose is 0.3 mcg.kg-1

ANSWER D Elimination half life 2.4-4-4 hrs

113. New- A nulliparous woman in labour for 8 hours with epidural analgesia has a fever 37.6 degrees. The most likely reason for this is A. altered thermoregulation B. chorioamnionitis C. urinary tract infection D. inflammatory response E. neuraxial infection

ANSWER D Epidural associated fever is common, ranging from mild hyperthermia to overt fever. Risk Factors 1. Nullparious 2. Prolonged labor 3. PROM Mechanisms postulated 1. Inflammation: most accepted explanation, unknown if it is infectious or non infectious (women with fever and epidural do not have evidence of chorioamonitis on histology) 2. Altered thermoregulation: hyperventilation during labor is diminished, resulting in reduced heat loss. 3. Effect of opioids: opioids suppress IL-2 formation "LAbour Epidural & Maternal Fever" www.anesthesia-analgesia.org/content/111/6/1467.full.pdf *

LUSCS for failure to progress. Spinal is inserted uneventfully. Next day the patient has foot drop. The most likely cause is? A. epidural haematoma B. lumbosacral palsy C. sciatic nerve palsy D. common peroneal palsy E. ?

ANSWER D Epidural haematomas rare (1:168,000 from review in Anaesthesiology 2006). Obstetric palsies are much more common than complications related to neuraxial blocks. Postpartum foot drop - damage to the lumbosacral trunk - common peroneal nerve. Lumbosacral trunk (L4, L5) is compressed between the ala of the sacrum and the descending fetal head. It may also occur during a forceps delivery. The result is a unilateral foot drop with loss of sensation and/or paraesthesia along the lateral calf and foot. Common peroneal nerve - lithotomy + lateral postition are risk factors nerve is potentially compressed at fibular head. Loss of dorsiflexion/everison of foot so get equinovarus foot) Sensation anterolateral border of leg; dorsum of digits. Sciatic nerve injury L4-S3 - stretch compression, ischaemia, lithotomy (flexion of hip, abduction, extension of leg). Common peroneal more affected than tibial component as more superficial. Injury = paralysis of hamstrings & muscles below knee so poor knee flexion, foot drop & altered sensation (Except for medial aspect of foot/leg).

A man attending his daughter's wedding is involved in fight with his son-in-law. He does not know where he is opens eyes to voice, but removes tie when instructed. What is his GCS? A. 10 B. 11 C. 12 D. 13 E. 14,

ANSWER D GCS13 E3V4M6

You are called to see a 30 year old man with rapidly deteriorating asthma. Following appropriate medical management an endotracheal tube is inserted and he is ventilated with a mechanical ventilator with a tidal volume of 600ml and a rate of 12 breaths per minute. Five minutes later the blood pressure is unrecordable and external cardiac massage is commenced. Arterial blood is taked and shows ph 7.08, pCO2 96 mmHg, pO2 36 mmHg, SpO2 46% and bicarbonate 27 mmol/L. He is administered adrenaline, salbutamol, pancuronium, bicarbonate and calcium gluconate. The ECG shows sinus rhythm at a rate of 60 beats per minute. The patient remains pulseless and cyanosed with fixed dilated pupils and distended neck veins. The most appropriate management is to A. cease resuscitation B. administer further adrenaline C. insert bilateral intercostal drains D. cease ventilation for 30 seconds and resume at a slower rate E. increase peak inspiratory pressure

ANSWER D - cease ventilation for 30 seconds

69. An essential criteria for diagnosis of left bundle branch block (LBBB) on ECG is A. RSR in V1 B. Minimum duration QRS of 0.2 secs C. Deep slurred S wave in V6 D. Loss of septal Q waves in V5 and V6 E. T waves opposite direction to main direction of QRS

ANSWER D Normally septum activated L ->R producing small Q waves lateral leads. In LBBB, the normal direction of septal depolarisation is reversed (becomes R-> L left), as the impulse spreads first to the RV via the right bundle branch and then to the LV via the septum. Extends the QRS duration to > 120 ms and eliminates the normal septal Q waves in the lateral leads./Users/kateromeril/Desktop/RBBB-V12-590x235.jpg Overall direction of depolarisation (from right to left) produces tall R waves in the lateral leads (I, V5-6) and deep S waves in the right precordial leads (V1-3), and usually leads to left axis deviation. As the ventricles are activated sequentially (right, then left) rather than simultaneously, this produces a broad or notched ('M'-shaped) R wave in the lateral leads. L) BBB QRS duration of > 120 ms Dominant S wave in V1 Broad monophasic R wave in lateral leads (I, aVL, V5-V6) Absence of Q waves in lateral leads (I, V5-V6; small Q waves are still allowed in aVL) Prolonged R wave peak time > 60ms in left precordial leads (V5-6) In RBBB, activation of the right ventricle is delayed as depolarisation has to spread across the septum from the left ventricle. LV activated normally, meaning that the early part of the QRS complex is unchanged. Delayed right ventricular activation produces a secondary R wave (R') in the right precordial leads (V1-3) and a wide, slurred S wave in the lateral leads. Delay also gives rise to secondary repolarization abnormalities, with ST depression and T wave inversion in the right precordial leads. In isolated RBBB the cardiac axis is unchanged, as left ventricular activation proceeds normally via the left bundle branch. R) BBB Broad QRS > 120 ms RSR' pattern in V1-3 ('M-shaped' QRS complex) Wide, slurred S wave in the lateral leads (I, aVL, V5-6)

During an elective thyroidectomy a patient develops symptoms consistent with the diagnosis of "thyroid storm" which of the following treatment options is NOT appropriate A. Carbimazole B. Beta-blocker C. Propythiouracil D. Plasmaphoresis E. Hydrocortisone

ANSWER D Plasmapheresis has been used to treat thyroid storm in adults as a last resort. Other options are initiated much earlier on. PTU or carbimazole can be used. Treatment: ABC's IV Hydration, cool patient IV propranolol (.5mg increments)/esmolol to control heart rate until less than 100. Propylthiouracil 250mg Q6 hours orally or by NG tube Sodium Iodide 1 gram over 12 hours correction of any precipitating events (infection) Cortisol is recommended if there is any coexisting adrenal gland suppression Mortality rate is approximately 20%

Prior to seeking consent from family/relatives for DCD, it is important to confirm which of the following? A. Not a coroners case B. Pt will have a cardiac death within 90 minutes in the absence of life-support C. Potential organ recipient's identified and are available D. Patient's wishes have been considered E. Decision confirmed by an external committee

ANSWER D A- no, if the coroner consents then the organs will be removed before autopsy B- no time limit set on the process of death by cessation of circulation, but some organs more vulnerable to warm ischaemic time (heart>lungs>liver>kidneys>corneas) C- not necessary for process of consent from family/relatives to occur D- yes absolutely E- not necessary

Two days post upper spinal surgery, patient notices parathesia of the right arm, surgeon thinks this is an ulnar nerve palsy due to poor positioning. What sign will distinguish a C8-T1 nerve root lesion from an ulnar nerve neuropathy? A. parasthesia in little finger B. parasthesia in the distribution of the interscalene nerve C. weakness in adductor digiti minimi D. weakness in abductor pollicis brevis E. weakness in lateral interosseus

ANSWER D A- no, ulnar nerve territory B- no such nerve (that I can find!) C- if they meant ABDUCTOR digiti minimus then no, as supplied by ulnar, as are all intrinsic muscles of the hand except lateral two lumbricals & the three muscles of the thenar eminence (abductor pollicis brevis, flexor pollicis brevis, opponens brevis; median nerve innervates these) D. YES If abductor pollicis brevis weak, then must be C8/T1 lesion, as this muscle supplied by MEDIAN nerve but shares the nerve root origin of C8/T1 with the ulnar nerve.

When performing laryngoscopy using a Macintosh blade your best view is of the patient's epiglottis touching the posterior pharyngeal wall. Using the Cormack and Lehane scale this is grade A. 1 B. 2 C. 3a D. 3b E. 4

ANSWER D Cormack and Lehane system should be subdivided further into 3a and 3b (denoting, respectively, only epi-glottis visible and only epiglottis visible but adherent to the posterior pharyngeal wall). 2a (part of the cords visible) and 2b (only the arytenoids or the very posterior origin of the cords visible)

A 3 year old child has suffered a fractured arm. What is the most appropriate way to assess her pain? A. the reported severity from the child B. the reported severity from the parent C. the reported severity from the nursing staff D. using the FLACC scale E. the Wong-Baker Faces scale

ANSWER D FLACC until about 4-5 years old, then Wong baker faces can be used for children able to self-report (ANZCA Acute Pain Mx 2010)

You are inserting a left sided double lumen tube into a 140kg 160cm woman. At what depth measured at the incisors is it most likely to be in the correct position A. 25cm B. 26cm C. 27cm D. 28cm E. 29cm

ANSWER D 29 cm +/- 1 cm for every 10 cm change in height from 170

What is the most cephalad intervertebral space at which a spinal can be sited in a neonate where the risk of damage to the spinal cord is minimal A. L1/2 B. L2/3 C. L3/4 D. L4/5 E. L5/S1

ANSWER D Spinal cord ends at L3 in neonate 1 years of age back to L1-2 http://www.luigivicari.it/med/wp-content/uploads/2012/10/R02Y2012N01A0078.pdf

Which of the following causes the most heat loss in a neonate? A: conduction B: convection C: evaporation D: radiation E: vasodilation

ANSWER D *

Patient presents with carcinoid syndrome and developes hypotension intraoperatively. Best drug to treat it is: A. Noradrenaline B. Adrenaline C. Metaraminol D. Octreotide E. Ephedrine

ANSWER D 1) Tumours that produce peptides & amines; mostly benign. Related to endocrine tissue; from embryonic gut tissue -> so can end up in lungs, pancreas, gonads, GI tract. Size of tumour correlates with survival 2) Release vascoctive substances into systemic circulation leading to carcinoid syndrome. 3) Mediators are metabolised in liver => need tumour to have non-portal drainage to get the syndrome (or liver met causing it) 4) Vasoactive substanvce - serotonin, bradykinin, histamine, substance P, prostaglandin, VIP. ASYMPTOMATIC pt - simple carcinoid disease, no anaesthetic issues. 10% only will have symptoms => carcinoid SYNDROME Carcinoid crisis - exaggeration of symptoms. anesthetic, surgery, interventional rad may present with bleeding PERI-OP Symptoms - Related to tumour (bowel obstruction, malnutrition, dehydration, anaemia, electrolytes) - Related to vasoactive substances Hx - flushing (head, neck, torso) diarrhoea -> dehydration electrolyte disturbance, hyperglycaemia bronchospasm (20%); hypotension, tachycardia, R heart failure sec to endocardial fibrosis of tricuspid + pull (don't reach the left heart as metabolised in lung!) => ECHO **Octreotide to reduce tumour hormonal activity*** ANAESTHESIA 1. severe HYPOTENSION or HYPERTENSION so aim to avoid pain or contributors to these 2. Fluid & electrolyte shift - lrge bore IV, fluid warmers 3. Bronchospasm 4. Blood loss if liver + high r heart pressures Pre-op 1. benzo; octreotide 100mcg SC 1 hr before 2. Monitoring - art line, CVP +/- PAC 3. Induction - prevent pressor response to tube Maintenance 1. Avoid all histamine releasing drugs (case studies only) 2. Avoid catecholamines (e.g. NA -> kallikrien in tumour activated-> synthesis of bradykinin- > paradoxical further vascodilation or may get exam HTN. => In practice, cautious administration of small doses of phenylephrine has been found helpful in some patients. 3. For BP esmolol for HTN; octreotide 20mcg for hypotension; vasopressin may be useful but evidence small Post-op 1. HDU 48 hrs. May wake slowly (? serotonin related) 2. Fentanyl rather than morphine 3. May need further octreotide.

During a cerebral aneurysm clippingthe anaesthetist can assist with the placement of the clip by giving the patient which drug immediately prior to clipping A. Nimodipine B. Thiopentone C. Hypertonic saline D. Adenosine E. Mannitol

ANSWER D adenosine

MVA trauma patient arrives in ED BP100/60 HR 100 with the following CXR (''I thought it looked like an aortic dissection/rupture with a widened mediastinum''). The most appropriate next investigation would be: A. Aortography B. CT Chest C. MRI D. TOE E. TTE

ANSWER D - Carrie says CT chest...is pt stable for CT scan? To do TOE need sedation so CT angio most correct prob -kate

Chronic alcohol use. Which is not an associated complication ? A. Pancreatitis B. Atrial fibrillation C. Macrocytosis D. Nephrotic syndrome E. Hypertriglyceridaemia

ANSWER D - nephrotic syndreom

Why is codeine not used in paediatrics? A. Poor taste B. High inter-individual pharmacokinetic variability C. Not licensed for <10 year old D. not as effective as adult when given in ?weight adjusted dose? E. ?

B

20 yr old male presents to ED with 30% burns from a fire. His approx weight is 80kg. Based on the Parkland formula, how much fluid is required in the first 8hrs from time of injury? A. 2.4L N/S B. 3.6L N/S C. 3.6L Hartmann's D. 4.8L N/S E. 4.8L CSL

ANSWER D 4.8L Normal Saline Parkland Formula 4 mL/kg x BSA burned 1/2 in first 8/24, remainder in next 16/24 Above is set, use UO + CVS response to guide ongoing If myoglobin/hb in urine increase input & alkalanise & promote diuresis Hartmann's is preferred option Need maintenance in addition to calculated rests fluid Burns > 25% ==> marked inflame response, increased capillary permability & general oedema. Rule of 9s - good for adults Each Upper Limb 9 Each Lower limb 18 Perineum 1 Head 9 Thorax/abdo 9 Back 9 Palmar surface of pt hand & fingers = 1% Lund & Bower Chart for more detailed/accurate Type of Burns Superficial - epidermis or superficial dermis Deep burns - deep dermis, no cap refill due to damage to blood vessels or full thickness (entire dermis +/- underlying strutures - white, waxy, loss of pinprick sensation) Carbon Monoxide Posioning - reduced capacity of Hb to carry oxygen so get tissue hypoxia but PaO2 normal. sats monitor can't detect COhb and will over-read oxygen saturation so use c0-0ximeter. Half life is 240 minutes RA but reduced to 40 min when breathing 100% oxygen. Keep going as get secondary peak of CO at 24hrsas it comes off cytochromes

Earliest sign of a high block in a neonate post awake caudal: A. Increased HR B. Increased BP C. Reduced HR D. Desaturation E. Loss of consciousness

ANSWER D Desaturation Central blocks do not affect LV fun + are virtually free of measurable haemodynamic effects, at least up till 8 yrs old. Neonates have low sympathetic tone; the decrease in sympathetic input may be balanced by concurrent decrease in vagal activity. Contraindications to Neuroaxial block in children 1) Coag disorderes (acquired of therapeutic) 2. Severe infection - meningitis, sepsis 3. Hysrocephaly 4. Allergy to LA 5. Certain chemotherapy's e.g. cisplatin can cause subclnicla meurological lesions which may be aggravated by neuroaxial 6. Uncorrected hypovolaemia 7. Cutaneous or subcut lesions at site of puncture 8. Parental refusal Volume of CSF varies according to age. larger in neonates 10mL/kg cf 2 mL/kg adult. Distribution of drug will vary with age. ==> larger doses of LA are required for spinals in infants & children.

A 20 year old man was punched in the throat 3 hours ago at a party. He is now complaining of severe pain, difficulty swallowing, has a hoarse voice and had has some haemoptysis. What is your next step in his management? A. Awake Fibreoptic Intubation B. CT scan for laryngeal fractures C. Direct laryngoscopy after topicalising with local anaesthetic D. Nasopharyngoscopy by an ENT surgeon E. Soft tissue xray of the neck

ANSWER D ENT surgeon nasopharyngscopy

The diagnosis of neuroleptic malignant syndrome requires the presence of A. Diaphoresis B. Elevated plasma creatine kinase C. Hypertension D. Muscle rigidity E. Tachycardia

ANSWER D Muscle rigidity

The POISE trial showed that the perioperative administration of metoprolol XR resulted in decreased A. Perioperative mortality B. Hypotension C. Congestive Cardiac Failure D. Myocardial Infarction E. Stroke

ANSWER D Myocardial Infarction

75. Compared to retrobulbar block, peribulbar block is associated with A. More bleeding B. More risk to optic nerve C. More akinetic eye D. Less block to orbicularis oculi

ANSWER D but do they mean more block to orbicularis occulis?? katePeribulbar (extraconal) has more gradual onset of akinesis/analgesia; lesser potential for significant complications (retrobulbar haemoorrhage & globe penetrarino less likely if use single inferolateral injection & avaoid superior aspect of orbit.) Peribulbar high volume; no needfor facial nerve block Retrobulbar block may require a separate facial nerve block for lid akinesia.

What happens when you place a magnet over a biventricular internal cardiac defibrillator A. Switch to asynchronous pacing B. Damage the internal programming C. Nothing D. Switch off antitachycardia function E. Switch of rate responsiveness

ANSWER D but usually retains PPM functions. Will depends on the device What does a magnet do? On pacemakers effect is variable. Often - > convert it to asynchronous fixed pacing mode, (pacing spikes march through with no regard for intrinsic rhythm). NOT recc. If to be used check pre-op. On ICD - Diff to PPM. Disable tachyarrythmia detection + therapy which are enabled when magnet removed. Rarely alter bradycardia pacing fxn. Unreliable though, may not have response so caution.

80 year old female for open reduction and internal fixation of a fractured neck of femur. Fit and well. You notice a systolic murmur on examination. Blood pressure normal. On transthoracic echo, she has a calcified aortic valve, with aortic stenosis with a mean gradient of 40mmHg. How do you manage her: A. Instigate low dose beta blockade B. Defer, and refer to a cardiologist C. Perform a transoesophageal echo to get a better look at the valve D. Proceed to surgery with no further investigation E. Perform a dobutamine stress echo

ANSWER D emergency surgery and doesn't need intervention anyway.

A healthy 25 year old woman is 18 weeks pregnant. Her paternal uncle has had a confirmed episode of malignant hyperthermia. She has never had susceptibility testing. Her father and siblings have not been tested either. The best test to exclude malignant hyperthermia susceptibility before she delivers is A. Genetic test father B. Genetic test woman C. Muscle biopsy sibling D. Muscle biopsy father E. Muscle biopsy woman

ANSWER D muscle biopsy father condition is autosomal dominant and best to avoid any anaesthetic if possible during pregnancy. Kate - below from emed + MH USA. Muscle biopsy - can;t be done under LA; unsure about being done under spinal. Best test is biospy in this actual patient. Malignant hyperthermia (MH) is a life-threatening clinical syndrome of hypermetabolism involving the skeletal muscle. Triggered in susceptible individuals primarily by the volatile inhalational anesthetic agents and the muscle relaxant succinylcholine, though other drugs have also been implicated as potential triggers Inheritance is autosomal dominant with variable penetrance. Typically located on chromosome 19 involving the ryanodine receptor.More than 30 mutations account for human MH. Genetic testing is available to establish a diagnosis, but the caffeine halothane contracture test (CHCT) remains the criterion standard. Dantrolene, the antidote, decreases the loss of calcium from the sarcoplasmic reticulum in the skeletal muscle and restores normal metabolism 1970s - mortality of an acute MH reaction was greater than 70%. Currently, the mortality of acute MH is less than 5%. Caffeine halothane contracture test (CHCT) is the criterion standard for establishing the diagnosis of malignant hyperthermia (MH). The test is performed on freshly biopsied muscle tissue at 30 centers worldwide; The patient must travel to one of the testing sites. The CHCT involves extraction of 2 g of muscle, usually from the thigh. This tissue must be tested immediately after extraction; its usability for testing lasts only a few hours. A negative CHCT result is the only way to prove that a patient is not susceptible to MH. UK: Testing (screening) of a new case is by a muscle biopsy called an in-vitro contracture test (IVCT). This involves exposing muscle tissue samples while still living to halothane and caffeine following a standard protocol devised by the European MH Group. Normal muscle will relax when exposed to halothane whereas MH muscle will contract and this can be measured. MH muscle is more sensitive to caffeine and will contract at much lower concentrations than normal muscle. Test can't be done on pregnatn woman or pt on steroids Molecular genetic testing (DNA testing) is less expensive and less invasive than the muscle contracture test. It is 25-30% sensitive and is highly specific for MH susceptibility. The DNA is extracted from cells in a blood sample. However, a negative genetic test result does not mean that a person is MH-negative. A CHCT is needed to determine MH-negative status.

You are performing an awake fibreoptic intubation, through the nose, on an adult patient. In order, the fibrescope will encounter structures with sensory innervation from the following nerves A. facial, trigeminal, glossopharyngeal B. facial, trigeminal, vagus C. glossopharyngeal, trigeminal, vagus D. trigeminal, glossopharyngeal, vagus E. trigeminal, vagus, glossopharyngeal

ANSWER D trigeminal, glossophayrngeal, vagus

During an elective thyroidectomy a patient develops symptoms consistent with the diagnosis of "thyroid storm" which of the following treatment options in NOT appropriate A. Carbimazole B. Beta-blocker C. Propythiouracil D. Plasmaphoresis E. Hydrocortisone

ANSWER D- least practical

Transient neurological (radicular) syndrome ONLY occurs with A. Hyperbaric local anaesthetics B. Intrathecal lignocaine C. Lithotomy positioning D. Following complete resolution of motor blockade E. When there has been a dense motor block with spinal anaesthetic

ANSWER D. Usually occurs later same day or next day

A patient's competence to give informed consent is determined by all the following EXCEPT: A. Ability to communicate a choice B. Ability to apply reasoning C. Ability to understand consequences D. The provision of significant information E. ??

ANSWER D? depending on what E was, as this question is querying the PATIENTs' competence, not their capacity (I think), and to give informed consent the degree of information given IS a factor.

2ml of 0.75% ropivacaine is injected for an interscalene block. Soon after the patient loses consciousness. The most likely place of inadvertent injection is A. Subdural B. Internal jugular vein C. Common carotid artery D. External jugular vein E. Vertebral artery

ANSWER E

85 TMP-Jul10-044 Called to emergency department to review a 20 year old male punched in throat at a party. Some haemoptysis / hoarse / soft voice. Next step in management: A. CT to rule out thyroid cartilage fracture B. XR to rule out fractured hyoid C. Rapid sequence induction / laryngoscopy / intubation D. Awake fibreoptic intubation E. Nasendoscopy by ENT in emergency department

ANSWER E

?An 18 month old boy presents for surgery for an incarcerated inguinal hernia. On examination you note that he has had an URTI for approximately one week. Your advice regarding surgery should be A. Postpone the surgery for two weeks B. Proceed with surgery under spinal anaesthetic C. Proceed with surgery with a full course of antibiotics to treat the URTI D. Undertake surgerybut avoid the use of an ETT E. Proceed with surgery with careful monitoring,

ANSWER E

A 70 year old lady suffered a subdural haematoma. She is currently confused and the neurosurgeon wants to take her to theatre for urgent decompression. She is a vague historianbut from notes you find out she had ablation and pacemaker put in 7 months ago - DDD mode. Cardiac technician in 1 hour away and the surgeon wants to proceed. What do you do A. Postpone until cardiology review B. Postpone until pacemaker checked by technician C. Postpone until temporary pacemaker inserted D. Proceed after having implemented external pacing E. Proceed with magnet available

ANSWER E

After ingestion of 500mg/kg aspirin, the most efficient therapy to enhance the elimination is A. normal saline infusion B. bicarbonate infusion C. mannitol D. frusemide E. haemodialysis

ANSWER E

Pulsus Paradoxusin cardiac tamponade, the blood pressure decreases A. Every second beat B. In expiration when increase is normal C. In expiration more than normal subjects D. In inspiration when increase is normal E. In inspiration more than normal subjects,

ANSWER E

Regarding post dural puncture headache all of the following are true, EXCEPT A. If puncture with the tuohy needle during epidural insertion, subsequent blood patch is 30-50% effective B. Caffeine is often used to treat mild headache C. Subdural haemorrhage can occur rarely D. E. Unlikely to be post dural puncture headache if the headache is only in the occipital area

ANSWER E

The clinical sign that a lay person should use to decide whether to start CPR is: A. Absent central pulse B. Absent peripheral pulse C. Loss of consciousness D. Obvious airway obstruction E. Absence of breathing

ANSWER E

The most likely cause of death after pharyngeal esophageal or tracheal perforation is A. Air embolus B. Hemorrhage C. Failure to intubate D. Failure to ventilate E. Sepsis

ANSWER E

You are anaethetising a lady for elective laparoscopic cholecystectomy, who apparently had an anaphylactic reaction to rocuronium in her last anaesthetic. There has not been sufficient time for her to undergo cross-reactivity testing. What would be the most appropriate drug to use: A. rocuronium B. suxamethonium C. pancuronium D. atracurium E. cisatracurium

ANSWER E

You are putting in an Internal Jugular CVC. Which manoeuvre will cause maximum venous distension of the jugular vein? A. Continuous Positive Airway Pressure (No value given) B: Breath hold at end-expiration C: Manual compression at the base of the neck D: Trendelenburg position E: Patient performs a valsalva

ANSWER E

Popliteal block placed from the lateral approach: A: Passes through semimembranosus B: Has eversion of the foot as the end point C: Has increased failure rate compared to a posterior approach D: ? E: Can be performed supine or prone

ANSWER E A FALSE, semmembranosus is muscle on the back of thigh B. FALSE Common peroneal : dorsiflesion and eversion Tibial nerve: plantar flexion and inversion C FALSE intermediate block, easier to perform, higher success rate D ? E. TRUE

102 PP84b ANZCA version [2005-Sep] Q141 Tracheo-oesophageal Fistula (TOF) A. is associated with cardiac anomalies in approximately 60% of cases B. is associated with oesophageal atresia in approximately 20% of cases C. is more common in males than females D. is usually left sided E. does not usually require contrast studies for diagnosis

ANSWER E A. Cardiac in 20% B. Atresia in 80-90% C. 1:1 D. right

If type and Rh specific blood is given to a patient, how safe is the transfusion A: ? B: ? C: 97% D: 98.6% E: 99.8%

ANSWER E Administration of ABO-RH compatible blood is very safe & additional procedures do not confer only small additional safety. Relative safety of using ABO-Rh blood without further antibody screen or cross match. ABO 99.4% ABO-Rh 99.8%. ABO - Rh plus negative antibody screen 99.94%, ABO + Rh + negative Ab screen + Coombs test - 99.95% Coobs adds so little it's not used, instead a panel of antibodies is used to detect the rarer antibodies e.g. duffy, kell. *

A 30 year old multi trauma patient one week post injury has severe ARDS. He is currently ventilated at 6ml/kg tidal volume, PEEP of 15cm H20 and pa02/Fi02 is less than 150. The next step to improve oxygenation is: A. increase PEEP to 20cmH20 B. increase tidal volume to 10mls/kg C. initiate nitrous oxide therapy D. commence high flow oscillatory ventilation E. ventilate in the prone position

ANSWER E PROSEVA study Among patients with severe ARDS, does prone positioning reduce all-cause mortality at 28 days when compared to supine-only positioning? Among patients with severe ARDS (P:F ratio <150 mmHg), prone positioning reduces 28-day mortality. (17% reduction in all cause mortality)

What is the cerebral perfusion pressure if MAP 80, CVP 5, both at the level of the RA with the tragus 13 cm above the RA? A. 62 mmHg B. 65 C. 75 D. 80

B 1mmHg = 1.36cmH20 80 x 1.36 = 108.8cmH20 108.8 - 13 = 95.8cmH20 95.8/1.36 = 70.4mmHg (i.e. the MAP at level of the head) CPP = MAP-CVP CPP = 70-5 = 65mmHg

A 5 year-old child with recently diagnosed Duchenne muscular dystrophy has an inhalation induction with sevoflurane for closed reduction of a distal forearm fracture. No other drugs have been given. 10 minutes later the child suffers a cardiac arrest. After a further 5 minutes a venous blood sample shows a potassium level of 8.5mmol/L. The most likely mechanism for the hyperkalaemia is: A. Acute renal failure B. Cardiomyopathy C. Crush injury D. Malignant hyperthermia E. Rhabdomyolysis

ANSWER E Anaesthetic Issues 1) Resp insufficiency 2) Cardiac dysfxn - failure & arrhythmias (avoid hypovolaemia due to fixed cardiac output state) 3) Risk of increased blood loss due to smooth muscle + platelet dysfxn 4) Anaesthetic agents - Volatiles have been implicated in causing rhabdomyolysis so TIVA preferred + new circuit/machine. - Depolarising agents (sux) cause hyperkalaemia + rhabdomyolysis from effect on extrajunctional receptors. - Non-depolarising agents have a prolonged effect so decrease dose. Anaesthesia & Analgesia 2009 CONCLUSION: We did not find an increased risk of MHS in patients with DMD or BD. Exposure to volatile anes-thetics in patients with muscular dystrophy may be associated with life-threatening rhabdomyolysis and therefore should be used cautiously, and when the benefits of their use outweigh the possible risks. Succinylcholine administration is associated with life- threatening hyperkalemia and should be avoided in patients with DMD and BD. Smith's Anesthsiology for infants "Dystrophin is also found in cells other than skeletal muscle and has an apparent role in organizing protein complexes in the membrane and stabilizing the membrane. Absence of the protein leads to membrane instability which causes eventual muscle cell deterioration. Associated defects seen in patients with DMD and BMD dystrophies include cardiomyopathy, cardiac conduction defects and, occasionally, mild mental retardation. The presence of the protein in other cell types provides an explanation for the involvement of the nervous system in affected patients. Dystrophin also is important in organizing the post-synaptic acetylcholine receptors. In its absence, abnormalities occur both in the types of receptors and in their number and location. In the absence of dystrophin there is also an increase in expression of acetylcholine receptor subunits as well as changes in interacting proteins. The membrane instability coupled with changes in acetylcholine receptors may explain the sensitivity of the muscle to succinylcholine and volatile anesthetics. *

Subtenon's block. What is the worst position to insert block? A. Inferonasal B. Inferotemporal C. Superonasal D. Supertemporal E. Medial / canthal

ANSWER E Answer E as might strike medial rectus insertion Tenon's Capsule is adense, fibrous layer of elastic tissue surrounding the eye + extraoccular muscles in the orbit. Sun tenon space originates at limbus; extends posterior to optic nerve. Rectus muscle divides the tenon space into anterior& post parts. Delivery of LA to this space allows spread along the extraocular muscle sheaths, diffusion into retrobulbar space, spread into fascia around lids + to nerves in this space. A sub-Tenon's block can be done in any of the 4 quadrants (options A-D above). These positions avoid the insertions of the rectus muscles which pierce the Tenon's capsule and insert into the sclera. Thus, for example, if you tried to do a block superiorly you would hit the superior rectus insertion.

The autonomic supply of the ciliary ganglion is such that it A. Receives its sympathetic nerve supply from the cervical ganglion B. Receives its parasympathetic nerve supply from the trochlear nerve C. Is located inferiorly in the orbit D. Is at risk from injury during peribulbar nerve block E. Receives parasympathetic nerve supply from the Edinger Westphal Nucleus

ANSWER E Ciliary ganglion - parasympathetic ganglion located in the posterior orbit. Preganglionic axons from the Edinger-Westphal nucleus travel along the oculomotor nerve and form synapses with these cells. The postganglionic axons run in the short ciliary nerves and innervate two eye muscles: - sphincter pupillae (constricts the pupil) - ciliaris - accommodation. Both of these muscles are involuntary - they are controlled by the autonomic nervous system. Ciliary ganglion one of four parasympathetic ganglia of the head and neck. (The others are the submandibular ganglion, pterygopalatine ganglion, and otic ganglion).

The insulation on the power cord of a piece of class 1 equipment is faulty such that the active wire is in contact with the equipment casing. What will happen when the power cord is plugged in and the piece of equipment is turned on A. The double insulation of the device will prevent macroshock when the outer casing is touched B. The electrical fuse will immediately break and disconnect the device from the power supply C. Equipotential earthing will prevent microshock D. The Line Isolation Monitor will alarm and disconnect power to the device E. The RCD will rapidly disconnect the device from the power supply

ANSWER E Class 1 - single insulation, has an earth connector that will divert energy if insulation fails and a fuse or RCD should disconnect power supply. Class 2 - double insulation Class 3 - SELV - safety extra low voltage, ie: voltage not high enogh to cause significant harm Can I change in line

MZ82 Ehlers-Danlos syndrome. Most important to specifically do all EXCEPT: A: Avoid hyperextension of the neck B: Damage to the teeth C: Avoid joint hypermobility D: Gastro oesophageal reflex E: Strict temperature regulation

ANSWER E ISSUES 1. GORD and gastritis 2. Early satiety and delayed gastric emptying 3. High, narrow palate and dental crowding 4. Peridontal disease (friability, gingivitis, gum recession) 5. Joint laxity. Subluxations and dislocations are common and represent the major manifestation of the condition. All sites can be involved, including the extremities, vertebral column, costo-vertebral and costo-sternal joints, clavicular articulations, and temporomandibular joints. Stoelting Anesthesia and Co Existing Disease 5th Edition ED is associated with skin fragility, easy bruisability and OA. Common presentations are arterial dissection or intestinal rupture. Anaesthetic consideration: CVS: Mitral regurgitation, cardiac conduction defect/abnormality. Resp: Tracheal dilatation, increased incidence of pneumothorax. Low airway pressures Careful laryngoscopy to avoid trauma Haematological: easy bruising, increase risk of bleeding. Avoid IM injection, instrumentation of nose or oesophagus. MSK: hyperelasticity, joint hypermobility. Regional anaesthesia: resistance to LA and increase risk of bleeding.

At what valve area do you begin to get symptoms at restwith mitral stenosis? A. 4.5 cm2 B. 3.5 cm2 C. 2.5 cm2 D. 1.5 cm2 E. 1.0 cm2,

ANSWER E open anaesthesia & Merck Manual http://www.merckmanuals.com/professional/cardiovascular_disorders/valvular_disorders/mitral_stenosis.html The normal area of the mitral valve orifice is 4 to 5 cm2. An area of 1 to 1.5 cm2 or mean transmitral gradient 5 to 10 mm Hg reflects moderate MS and often causes exertional symptoms. An area < 1 cm2 or mean gradient > 10 mm Hg represents severe stenosis and may cause symptoms during rest. However, the relationship between the area of the valve orifice and symptoms is not always consistent.

The most important effect of Lugol's iodine administration before thyroid surgery is A. reduce incidence of thyroid storm B. reduce incidence of vocal cord palsy C. increase likelihood to identify and preserve parathyroid glands D. pigmentation of thyroid gland to help identify thyroid gland E. reduce vascularity of thyroid gland.

ANSWER E (OCH)

Performed a brachial plexus block. Normal sensation still remains in medial forearm. Which part of brachial plexus is most likely to have been missed A. Posterior cord B. Anterior division C. Median brachial cutaneous nerve D. Ulnar nerve E. Inferior trunk

ANSWER E (checked by kate)

The electrical requirement that distinguishes a "cardiac protected area" from a "body protected area" is the A. isolation transformer B. line isolation monitor C. equipment has a maximum leakage current of 500 microamperes D. residual current device E. equipotentiality

ANSWER E RCH biomedical : electrical safety page good guide on Class I - 3 prongs; earth prong Class II - 2 prongs, insultaed RCD -A residual-current device (RCD), an electrical wiring device that disconnects a circuit whenever it detects that the electric current is not balanced between the energized line and the return (neutral) conductor. In normal circumstances, these two wires are expected to carry matching currents, and any difference usually indicates a short circuit or other electrical anomaly is present. Even a small leakage current can mean a risk of harm or death due to electric shock if the leaking electric current passes through a human being; disconnect the conducting wires quickly enough to prevent serious injury from such shocks. A RCD does not provide protection against unexpected or dangerously high current when current is flowing in the usual wires in the circuit, therefore they cannot replace a fuse or protect against overheating or fire risk due to overcurrent (overload) or short circuits if the fault does not lead to current leakage. Therefore RCDs are often used or integrated as a single product along with some kind of circuit breaker. LIM - alarms when leakage current is too high Body Protected - Room set up to prevent conduction to pt. A Body Protected Electrical Area is intended to provide increased protection against electric shock. The use of a residual current device (RCD) will ensure that if any contact is made between a live conductor and earth (either directly or as a result of electrical leakage) 'macro shock electrocution' is very unlikely. Cardiac Proteced - room set up to prevent condcution of current to HEART and associated microshock. This is areas where procedures are undertaken with an electrically exposed heart. Where patient circuits are either within or in close proximity to the heart. In these areas specially protected equipment and area protection are required.

St John's Wort (Hypericum perforatum) potentiates the effects of A. Dapigatrin B. Heparin C. Warfarin D. Aspirin E. Clopidogrel

ANSWER E St John's Wort enhances CYP3A4 so increase conversion of clopidogrel to active form -Decrease Dabigatrin concentration -Decrease warfarin -No known interaction with aspirin or heparin

What potentiates/interacts with adenosine A. Aspirin B. Warfarin C. Clopidogrel D. Dabigatran E. Dipyrimadole

ANSWER E Dipyridamole prevents the breakdown of adenosine

During caesarean section a meconium stained floppy apnoeic baby is delivered. When the midwife gives you the babyit is apnoeic, cyanotic with heart rate of 90 bpm. What do you do next A. Give naloxone B. Dry and stimulate C. Start chest compressions D. Give positive pressure ventilation E. Suction the trachea,

ANSWER E NZRC Guidelines Routine intrapartum oropharyngeal and nasopharyngeal suctioning is not recommended for infants born with either clear or meconium-stained amniotic fluid [Class A, LOE II8, 9] There is insufficient evidence to recommend a change in the current practice of endotracheal suctioning of non-vigorous infants born through meconium-stained amniotic fluid [Class B, expert consensus opinion10-12]. If it is done, it should be performed only: Immediately after birth. If an experienced practitioner and all needed equipment are immediately available. Before the onset of breathing or crying and in infants with decreased muscle tone. It should not be done if the infant is vigorous and breathing or crying.13 It should be done once, and then any subsequent resuscitation that is needed should be commenced expeditiously [Class B, expert consensus opinion]. All meconium-exposed infants should be considered in need of careful assessment at birth and over the next 1-2 days [Class A, expert consensus opinion].

Regarding Le Fort fractures: A. External signs correlate with internal skeletal damage B. Le Fort fractures don't usually occur in combination (for example I and II) C. Patients with a Le Fort I fracture should NOT undergo nasal intubation D. Patients with a Le Fort II fracture should have evaluation of the base of skull prior to nasal intubation E. Le Fort III fracture is associated with fracture of the cribiform plate

ANSWER E A- no B- no, they often occur in combination C- Le Fort 1 is an isolated lower Maxillary fracture and often need nasal intubation to facilitate assessment of teeth occlusion by surgeons D- yes, but more commonly associated with Le Fort III fractures E- Yes and most likely correct answer. Cribiform plate is on ethmoid bone and can be fractured in extensive midline injuries.

A 63yo woman with chronic AF has a history of hypertension, Type 2 Diabetes Mellitus and has previously had a CVA. What is her annual risk of stroke without anticoagulation? A. <1% B. 1.9% C. 2.8% D. 4% E. 8.5%

ANSWER E CHADs2 score: Hypertension- 1 Diabetes- 1 Prev. stroke -2 score= 4, yearly risk = 8.5% 0-1.9 1- 2.8 2- 4.0 3- 5.9 4- 8.5

A man with a history of Parkinsons disease has undergone uncomplicated general anaesthetic for a knee replacement but develops post-operative nausea and vomiting (PONV). He received 4mg dexamethsone intraoperatively as prophylaxis. What would you use to treat his PONV in recovery? A. Dexamethasone B. Prochloperazine C. Metoclopramide D. Droperidol E. Ondansetron

ANSWER E No point in giving more dexamethasone Other options all have dopamine antagonist effects, they will worsen his parkinsons

A test has a sensitivity + specificity of 90% for a disease with a prevalence of 10%. What is the positive predictive value? A. 10% B. 50% C. 82% D. 90% E. 99%

ANSWER E The former is simply the product of the sensitivity and the prevalence: Sensitivity (S)= Number who have disease(D) and who screened +ve/ number who have disease Prevalence (P)= Number who have disease/ total number screened PPV= SxP/ (SxP)+(1-specificity)x(1-prevalence) = 900/ 900+ (1-90)x(1-10) =9.09

Neurosurgery operation in the sitting position. MAP 80mmHgCVP 5mmHg, the transducers are located 13cm below the external auditory meatus. What is the CPP A. 62 mmHg B. 65 mmHg C. 67 mmHg D. 72 mmHg E. 75 mmHg

ANSWER E Trick question? The height difference doesn't matter because it cancels itself out from arterial to venous side, as the hydrostatic column is the same as the transducers are at the same position. CPP = MAP - (greater of ICP or CVP). We have to use CVP as we don't get told ICP = 80 - 5 =75

You are anaesthetizing a patient with chronic renal failure for removal of a Tenkoff catheter and have intubated using rocuronium at a dose of 1.2mg/kg. You are immediately unable to intubate or ventilate and you decide to reverse the patient with sugammadex. What dosage would you use A. 2mg/kg B. 4mg/kg C. 8mg/kg D. 12mg/kg E. 16mg/kg

ANSWER E Medsafe Data sheet - same dose in mild + moderate renal failure but not recommended for use in pots with severe renal failure incl dialysis - KR

Which of the following is of the least benefit in the treatment of severe anaphylaxis? A. Cardiopulmonary bypass B. Nebulised salbutamol C. IV crystalloid D. IV vasopressin E. Subcutaneous adrenaline

ANSWER E - SC adrenaline. - Peripheral perfusion will be very poor. - There is no reference to SC Adrenaline in ANZAAG/ANZCA guidelines to anaphyalxis

You are anaesthetizing a 50 year old man who is undergoing liver resection for removal of metastatic carcinoid tumour. He has persistent intraoperative hypotension despite fluid resuscitation and intravenous octreotide 50 ug. The treatment most likely to be effective in correcting the hypotension is: A. Adrenaline B. Dobutamine C. Levosimenden D. Milrinone E. Vasopressin

ANSWER E - VASOPRESSIN Vasopressin to treat refractory hypotension CEACCP

Which of the following is an absolute contraindication to electroconvulsive therapy A. Cochlear implant B. Epilepsy C. Pregnancy D. Raised Intracranial Pressure E. Recent myocardial infarct

ANSWER E - kate The cardiovascular response is secondary to activation of the autonomic nervous system. Beginning with the electrical stimulus, there is an initial parasympathetic discharge lasting 10-15 s. This can result in bradycardia, hypotension, or even asystole. A more prominent sympathetic response follows during which time cardiac arrhythmias occasionally occur. Systolic arterial pressure may increase by 30-40% and heart rate may increase by 20% or more, generally peaking at 3-5 min. Myocardial oxygen consumption, as determined by the rate-pressure product (RPP), therefore increases. Simultaneously, seizure activity increases tissue oxygen consumption, potentially reducing myocardial oxygen supply. Myocardial ischaemia and infarction can therefore occur, particularly with pre-existing disease. Left ventricular systolic and diastolic function can remain decreased up to 6 h after ECT. Cardiac rupture has also been described. Despite the physiological effects, and its frequent use in elderly people with significant co-morbidity, ECT is a low-risk procedure. The mortality rate of ∼1 per 10 000 patients (1 per 80 000 treatments) is similar to that of anaesthesia for minor surgical procedures.3 Cardiovascular (arrhythmia and myocardial infarction) and, to a lesser extent, pulmonary (laryngospasm and aspiration) complications are the most common causes of death and serious morbidity. CEACCP

What is the most effective method of minimizing acute kidney injury following an elective open abdominal aortic aneurysm repair? A. give IV crystalloid as a 'preload' before cross-clamp B. give IV mannitol before cross-clamp C. give IV frusemide before cross-clamp D. give preoperative N-acetylcysteine E. minimize aortic cross-clamp time

ANSWER E - minimise aortic xclamp time

Patient with subdural haematoma, on warfarin. INR 4.5. Needs urgent craniotomy. Vit K given already by ED resident. What further do you give for urgent reversal of this patient's INR? A. Factor VII B. Cryoprecipitate C. FFP D. Prothrombinex E. FFP + prothrombinex

ANSWER E - need both in critical bleeding incl intracranial as per Australasian 2013 guidelines (see similar question) 50IU/kg prothrombinX; 150mL - 300ml FFP. if you had no prothrombinex then use 15ml/kg FFP.

Following a left sided pneumonectomy, a left intercostal drain is placed and connected to an underwater drainage system. In the postoperative period A. A leakage of air is expected from the drain B. The patient should be nursed in the right lateral decubitus position C. The underwater seal drain should be left on continuous free drainage D. The underwater seal drain should be left on continuous free drainage, and connected to wall suction for 5 minutes every hour E. The underwater seal drain should remain clamped and be released for a short period every hour

ANSWER E - remain clamp, unclamp briefly each hr

A male patient has a haemoglobin of 8g/dL and reticulocyte count 10%. The most likely diagnosis is A. Untreated pernicious anaemia B. Aplastic anaemia C. Acute leukaemia D. Anaemia of chronic disease E. Hereditary spherocytosis

ANSWER E -spherocytosis has increased reticular count. Spherocytosis is an auto-hemolytic anemia (a disease of the blood) characterized by the production of spherocytes - red blood cells (RBCs), or erythrocytes, that are sphere-shaped, rather than bi-concave disk shaped. Spherocytes are found in hereditary spherocytosis and autoimmune hemolytic anemia.[1] The normal fraction of reticulocytes in the blood depends on the clinical situation but is usually 0.5% to 2.5% in adults and 2% to 6% in infants. A reticulocyte percentage that is higher than "normal" can be a sign of anemia, but this depends on the health of a person's bone marrow. When there is an increased production of red blood cells to overcome chronic or severe loss of mature red blood cells, such as in a haemolytic anemia, people often have a markedly high number and percentage of reticulocytes. A very high number of reticulocytes in the blood can be described as reticulocytosis.[citation needed] Abnormally low numbers of reticulocytes can be attributed to chemotherapy, aplastic anemia, pernicious anemia, bone marrow malignancies, problems of erythropoietin production, various vitamin or mineral deficiencies (B9, B12, iron), disease states (anemia of chronic disease) and other causes of anemia due to poor RBC production.[citation needed]

An elderly gentleman on warfarin has suffered a subdural haematoma. His INR on admission was 4.5. The resident in Ed has already given him 2.5mg of Vit K. To reverse his coagulopathy prior to urgent surgery you should give him A. Factor VIIa B. FFP C. Cryoprecipitate D. Prothrominex E. Prothrombinex and FFP

ANSWER E 2013 NZ blood service • This is based on guidelines produced by the Australasian Society of Thrombosis and Haemostasis (ASTH) in 2013. •Warfarin - it is also associated with a risk of major bleeding of 1-3% per year • Prothrombinex-VF is a three factor prothrombin complex concentrate. It contains Factor II, IX and X with only trace levels of factor VII. Works in 15 min. 50IU/kg in life threatening Recent studies have demonstrated that in most clinical settings Prothrombinex-VF will effectively reverse the anticoagulant effect of warfarin WITHOUT the need for combination treatment with FFP. • Limited data is currently available on the effectiveness of Prothrombinex-VF in reversing warfarin in patients with LIFE THREATENING or CRITICAL (including intracranial haemorrhage) and COMBINED use of both Prothrombinex- VF and FFP (150-300ml) is therefore recommended. - Vit K esesntial 5-10mg IV - If using only FF then 15mL/kg dose.

Awareness

Ability to respond purposefully to intra-op stimuli. MAC > 0.8 - very good chance of no recall. But ensure age adjusted. High risk Cardiac Obstetric Use of NMB Prev awareness Difficult airways Lower risk Children

Relative humidity of fully saturated air at 20degree and 37 degrees- A. 40%

Absolute humidity @20deg 17g @37deg 44g 17/44 = 40%

Regarding tryptase level testing for suspected anaphylaxisall are true EXCEPT: A. Levels peak within 1 hour B. Increased with anaphylactoid and anaphylactic reactions C. 99% of the body's stores are found in mast cells D. Levels of > 20ng/mL are suggestive of anaphylaxis E. Test should be repeated at 24-48 hours

ANSWER E ?? Any increase above baseline is considered as possible anaphylaxis > 20 mcg/L baseline consider systemic mastocytosis > 11.5 mcg/L Half life 2 hrs with peak at 1 hr. Tryptase at 1, 4, 24 hrs Mast cell tryptase (MCT) is enzyme secreted by tissue bound mast cells, mediates smooth muscle relaxation. It is released in anaphlactic runs; stimulates pro-inflamm mediators. Better marker than histamine - half life too short. 99% tryptase found in mast cells. Normal level 0.8 - 1.5mg/mL Anaphlyaxis is a severe life-threatenign , generalized or systemic hypersensitivity reaction. Sub-divided into allergic + non-allergic. Allergic - IgE, IgG or complement mediated. Non-allergic - Mast cell + basophil degranulation caused by direct drug action with no immune trigger. Skin prick testing only picks up IgE mediated reactions - > so drug can still be trigger. "Anaphylaxoid" term now obsolete. Increase less marked in no allergic states but baseline varies so relative change more significant. Normal levels do NOT exclude anaphylaxis as can see this in basophil or complement mediated reactions + fluid can dilute MCT. > 20ng/mL likely IgE rxn

A 70 year old man with slow atrial fibrillation is reviewed for insertion of a permanent pacemaker. He is otherwise well. He is on warfarin with an INR of 2.2. Prior to PPM insertion do you A. Cease warfarin and commence dabigatran B. Cease warfarin and commence Enoxaparin C. Cease warfarin and recommence post procedure D. cease warfarin and commence heparin E. Continue warfarin

ANSWER E Continue Warfarin (BRUISE trial 2013 NEJM warfarin vs bridging heparin; those on heparin had increased intrapocket haematoma)

In adult cardiopulmonary resuscitation in the community include all of the following EXCEPT: A. Allow equal time for chest compression and relaxation B. Chest compression at 100bpm C. C. Chest compression should be at least 5cm depth D. D. Give 2 rescue breath before commencement of CPR E. Chest compression to breaths ratio at 30:2

ANSWER E D?(kate)

A 70 year old man with severe mitral stenosis and normally in sinus rhythm is going for an ORIF of fractured radius and ulna. Soon after induction of GA, he develops a tachyarrhythmia with BP 70/40mmHg and HR 130bpm. The most appropriate immediate action is A. Amiodarone B. Adenosine C. IV fluid bolus D. Adrenaline E. Direct cardioversion

ANSWER E DC cardioversion

A 25 y.o. male has a traumatic brain injury on a construction site. GCS 7. Intubated on site and transported 1 hour to hospital. Haemodynamically stable and no other injuries. Most appropriate pre hospital fluid: A. 4% albumin B. Dextran 70 in 0.9%N/saline C. 6% hydroxyethyl starch D. Ringers lactate E. 0.9% N/saline

ANSWER E N Saline

In regards to systemic sclerosis, what is the least likely cardiac manifestation? A. accelerated coronary artery disease B. atrioventricular conduction block C. myocarditis D. pericardial effusion E. valvular regurgitation

ANSWER E Valvular Although accelerated coronary artery disease is NOT a feature (vasospastic lesions occur), myocardial infarction is a greater risk than in the general population. (UTD)

What is the mechanism of central sensitisation? A. Increased intracellular magnesium B. Antagonism of the NMDA receptor C. Glycine is the major neurotransmitter involved D. Recurrent a-delta fibre activation E. Alteration in gene expression

ANSWER E central; if NMDA involved increase in Calcium causes displacement from NMDA R & glutamate binds (agonism) ; Adelta are peripheral; glycine is an inhibitory transmitter.

You wish to compare a new method of BP measurement with the gold standard. The best way to do this is: A. CUSUM analysis B. Friedman's test C. ? D. Pearson's correlation E. Bland-Altman plot

ANSWER E- repeat Bland and Altman plots are extensively used to evaluate the agreement among two different instruments or two measurements techniques. Bland and Altman plots allow us to investigate the existence of any systematic difference between the measurements (i.e., fixed bias) and to identify possible outliers. The mean difference is the estimated bias, and the SD of the differences measures the random fluctuations around this mean.

Elderly lady post operatively with painful eye. Differential between narrow angle glaucoma and corneal abrasion A. ? B. C. D. E. Relieved by topical local anaesthetic

ANSWER E- will be relieved by topical anaesth if Corneal abrasion.

St John's wort will reduce the effect of A. aspirin B. clopidogrel C. dabigatran D. heparin E. warfarin

ANSWER E. warfarin. TGA Alert- St John's wort induces CytochromeP450 enzymes and so decreases the effect of Warfarin, digoxin, theophylline Increases serotonergic effects of SSRIs with increased risk of sideffects. Potentiates clopidogrel as it is a prodrug which requires conversion by p450 enzyme to become active.

Anaesthetising an obese patient. Acelerometer on TOF 0.9. Could dose suxamethonium on ideal body weight or total body weight. With respect to 1mg/kg IBW vs. TBW you will see: A: shorter onset and faster twitch recovery B: shorter onset and similar twitch recovery C: shorter onset and slower twitch recovery D: similar speed of onset with similar speed of twitch recovery E: similar onset and longer recovery F: similar onset and faster twitch recovery

ANSWER F (kate - accept; learning point dose sux at 1mg/mg total body weight); most other drugs use ideal body weight). A & A February 2006 vol. 102 no. 2 438-442 We studied 45 morbidly obese (body mass index >40 kg/m2) adults scheduled for gastric bypass surgery. ...In Group I, patients received SCH 1 mg/kg ideal body weight, in Group II 1 mg/kg lean body weight, and in Group III 1 mg/kg total body weight....After SCH administration, endotracheal intubating conditions were scored. The recovery from neuromuscular block was recorded for 20 min. There was no difference in the onset time of maximum neuromuscular blockade among groups, but maximum block was significantly less in Group I with ideal body wwight The recovery intervals were significantly shorter in Groups lower dose....Our study demonstrates that for complete neuromuscular paralysis and predictable laryngoscopy conditions, SCH 1 mg/kg total body weight is recommended.

68. Repeat- Patient cough during interscalene block- insertion needle should be directed-

ANSWER posterior Stimulation of diaphragm : phrenic nerve : needle tip is anterior to plexus Stimulation of trapezoid : needle tip is posterior to plexus

Renal handling of solutes

Active transport - glucose Active secretion - imp for those substances not filtered incl. protein bound substances eg Biles salts, furosemide, PAH, salicylate, creatinine, dopamine, morphine

Carbonic anhydrase inhibitor Acetazolamide (diamox)

Acute reduction in intraocular pressure - glaucoma Diuretic (but weak) - altitude sickness Enzyme inhibitor. Prevents reaction of H2CO3 to HCO3+ and H+......in tubule this reduces HCO3- production so it is not absorbed. Less Na absorbed. Gives diuresis and acidic blood

Platelet adhesion Platelet Activation

Adhesion - Platelets bind to collagen via Ia receptor - vWF binds to platelets and tissue (acts as bridge) - IIb/IIIa on platelet exposed -> allows platelet to platelet binding. Activation - Shape change from disc to pesudopods but this needs ADP to increase Ca++ - Exposure of PF3 -> activation of IX - Release of granules - serotonin (vasoconstriction), fibrinogen, PDGF. Antagonists COX inhibitors - Cyclooxygenase inhibition results in less thromboxane A2 so less adhesion. Tirofiban & abciximab - Inhibit IIb/IIA complex Prostacyclin (PGI2) - inhibits activation Clopidogrel (thienopyridine) - decreases ADP release Dipyridamole (phosphodiesterase inhibitor) - decreases Ca

Alpha Receptora

Adrenergic GPCR which are present throughout body pre-synaptically & post-synaptically. Natural legends are adrenaline and noradrenaline. Alpha 1 receptors - smooth muscle usually Alpha 2 Receptors - classically described as pre-synaptic vascular but receptors are throughout body including brain, platelets, heart. Subtypes of these = A/B/C so effect will depend on if the GPCR is a stimulatory or inhibitory. Clonidine alpha 2 to alpha 1 = > 220: 1 Dexmedetomidine alpha 2 to 1 => 1600: 1

Tocolytics

Agents that act on the uterus to decrease uterine tone. Indications - pre-term labour (aim is for time for steroids) - facilitate delivery in c-section - facilitate access in fetal surgery 1. B agonists (salbutamol, ritodrine) Act via GPCR to decrease muscle contraction 2. Ca channel antagonists (Nifedipine ) Decrease Ca entry into cells 3. Magnesium Decrease Ca entry by decreasing activity at voltage gated channels 4. Nitrates 5. Oxytocin receptor antagonists - atosiban Competitive inhibition

MTP

Aim to give RBC and FFP in 1 to 1 ratio Massive bleeding with shock 3 units O neg or type specific MTP Box 1 - 2 U RBC/ 2 U FFP MTP Box 2 - 4 U RBC/4U FFP/Platelets MTP Box 3 - 3U cryo/ 4 RBC/4FFP Consider FVIIa if > 10 units but need pH > 7.2 Consider tranxaemic acid, Calcium

What is venous admixture

Amount of MIXED VENOUS BLOOD required to mix with pulmonary capillary blood to produce to the observed alveolar-arterial difference. (May not be equal to the actual shunted blood as this may not have the same O2 contact as mixed venous blood) Physiological Sources Bronchial circulatoin > 1% Thebesian veins which drain wall of LV 0.3% Pathological sources Congenital heart disease Increased venous drainage form tumours Areas of V/Q mismatch (shunt) Qs/Qt = conc end capillary - conc arterial blood/conc end capillary blood - conc venous blood

Serotonin

Aka 5HT (5 hydroxytryptamine) Naturally occurring amine which is produced in CNS, platelets and enterochromaffin cells in gut (90%). Important neurotransmitter but has a wide range of functions incl: - Increasing gut motility, water/electrolyt secretion n gut - Mediating nausea - vasoconstriction - modulation of inhibitory pain pathways - mood/arousal/muscle tone/temp/behaviour - appetite - sleep/wake cycle Metabolised by MAO, re-uptake inhibited by SSRIs, TCA. Many receptors metabotropic & ionotropic. Some receptors are inhibitory, other excitatory. Sumitriptan 5HT1 agonist (inhibitory action) Ergotamine 5HT5 agonist Ondansetron 5HT3 antagonism; agonism of this causes sedation, anxiolysis, emetic

In salicylate poisoning what do yo do to urine?

Alkalinise it so that more of the drug (which is a weak acid) becomes ionised and can be excreted. COOH <-> COO- + H+ Drive equilibrium to right by adding alkaline to mop up hydrogen ions.

Nicotine

Alkaloid from plant Binds to nicotinic receptors which are inotropic receptors throughout body in sympathetic nervous system + at neuromuscular junctions. Nicotinic receptors in adrenal medulla cause release of adrenaline. Adrenaline release causes increase in HR, BP, RR, glucose release. Peripherally it will cause vasoconstriction. Endogenous agonist for nicotinic receptors is acetylcholine however nicotine was used experimentally to distinguish nicotinic from muscarinic receptors (which also bing acetylcholine but are part of the parasympathetic system & activated through GPCR mechanisms). Nicotine causes release of dopamine in the CNS. Stimulant + relaxant. It is highly addictive.

Doxazosin

Alpha receptor antagonist used for BPH, HTN. In BPH it relaxes smooth muscle at bladder outlet.

Vecuronium

Amino steroid of medium duration + medium onsest Binds to nicotinic ACh receptors. Dose 0.1 mg/kg Causes decrease in HR + SVR Metabolised in liver

Pancuronium

Aminosteroid long acting muscle relaxant Dose 0.1mg/kg No histamine relase Increase HR due to binding to muscarine receptors causing vagolysis

Rocuronim

Aminosteroid neuromuscular blocking drug. Binds competitively to nicotinc ACh receptors at NMJ and prevents ACh from binding. Receptors are mostly post-synaptic . ED 90 = 0.3mg/kg Dose for intimation is 2 x ED90 = 0.6mg/kg Anaphylaxis Low histamine release At large doses only do you get some vagolysis with increase in HR Metabolised in liver, 40% excreted unchanged. 20% renal elimination . Caution in liver failure. Not an MH trigger in animals. Antidote

Henry's Law

Amount of gas dissolved in a solvent is proportional to the partial pressure above the solvent at constant temp

Latent heat of vapourisation

Amount of heat required to convert 1 kg of a solid into liquid phase without a change in temperature

Oxygen Flux

Amount of oxygen DELIVERED to tissues per unit time Normall 850 - 1200mL/min O2 flux = CO x oxygen content -> = CO x Hb x sats x 1.34 + 0.3 x PaO2 Oxygen extraction is 250 mL -> ~ 25% useage In pregnancy oxygen flux is 10% higher (hb mostly saturated so an increase in TV + RR add very little, PO2 slightly higher at 105mmHg The cardiac output in 30% higher so this is what is important in providing additional oxygen.

Humidity

Amount of water vapour in air ABSOLUTE humidity - is the mass present in given volume. The amount will be determined by the temp. With increasing temp the more H2O can be present. RELATIVE is the ratio of water present to water present when air saturated. At 20C .......17grams per m3 At 34C....... 34 grams At 37C ........44 grams

In pregnancy the dural sac ends at: A. T12 B. L2 C. L4 D. S2 E. S4

Ans D

Management of rhabdomyolysis - best option? A. Haemodialysis B. Bicarbonate C. Frusemide D. IV fluids E.

Ans D

Pain

An unpleasant sensory & emotional experience that is associated with actual or potential tissue damage or described in terms of such damage.

How does anaesthesia affect sleep

Anaesthesia - Suppresses REM sleep - Alters REM sleep at day 3 post op with increase in disordered sleep breathing??? Sleep deprivation will potenetiate propofol + isoflurane. Dexmedetomidine produces sleep state similar to normal (acts at locus coerulus)

IV paracetamol: A. late plasma levels around the same as oral B. highly protein bound C. ?30%? renally excreted D. VD 10L/kg E.

Ans A

Stellate ganglion A. Anterior to scalenius anterior B. ? C. ? D. ? E. ?

Ans A The inferior cervical ganglion lies at the level of the disc space between the 7th cervical and 1st thoracic vertebrae; in 80% of subjects it is fused with the 1st thoracic ganglion to form the stellate ganglion. It represents the coalescence of the 7th and 8th cervical ganglia. Relations of stellate ganglion Anterior: The structures anterior to the ganglion include the skin and subcutaneous tissue, the sternocleidomastoid and the carotid sheath. The dome of the lung lies anterior and inferior to the ganglion. Medial: The prevertebral fascia, vertebral body of C7, oesophagus and thoracic duct lie medially. Posterior: Structures posterior to the ganglion include the longus colli muscle, anterior scalene muscle, vertebral artery, brachial plexus sheath and neck of the first rib.

A 60 year old female is undergoing hysterectomy. Gabapentin reduces postoperative: A. Nausea B. Vomiting C. Sedation D. Pruritus E. Constipation

Ans A Acute pain management - scientific evidence - third edition 2010 p91 After hysterectomy and spinal surgery specifically, gabapentin improved pain relief and was opioid-sparing, nausea was less in patients after hysterectomy, and there was no difference in sedation.

Patient post anterior cervical fusion. Patient in recovery. Confused and combative. Nurse concerned about haematoma Taken to theatre: Most appropriate way of securing airway: A. Gas induction / laryngoscopy / intubate B. Awake tracheostomy C. Awake fibreoptic intubation using minimal sedation D. Thiopentone, suxamethonium, direct laryngoscopy and intubation E. retrograde intubation

Ans A CEACCP V7 No3 2007 Re-intubation may be difficult. The patients must be managed in a semi-sitting posture. Awake fibreoptic intubation is sometimes a good option, but direct laryngoscopy after sevoflurane and oxygen induction (with judicious doses of propofol) may be easier. The gum-elastic bougie is often vital and an LMA (+fibrescope,+gum elastic bougie or Aintree catheter) or an ILMA-guided technique may save the day. The use of succinylcholine in myelopathic patients is hazardous because of abnormal potassium shifts.

Acute renal failure. Which is not an indication for dialysis ? A. Hyperkalaemia B. Metabolic alkalosis C. Hypernatraemia D. Uraemic pericarditis E. APO

B ?? Ans C

How do you minimise risk of intravenous cannulation with epidural insertion ? A. Injection saline through epidural needle before catheter insertion B. Lie patient lateral C. Do CSE D. Thread catheter slowly E.

Ans A Detection of intravascular epidural catheter placement A review 2007 The injection of saline into the epidural space prior to threading the catheter is reported to decrease the incidence of intravascular placement; use 20ml *

The most appropriate method for improving oxygenation during one lung anaesthesia, after institution of an FiO2 of 1.0, is application of A. 5 cm H2O CPAP to the non-dependent lung B. 10 cm H2O CPAP to the non-dependent lung C. 5 cm H2O PEEP to the dependent lung D. 5 cm H2O CPAP to the non-dependent and 5cm H2O PEEP to the dependent lung E. intermittent re-inflation to the non-dependent lung

Ans A (for CEACCP they prefer CPAP 5 cmH2O to non-dependent lung first) ANSWER C (for Miller) Another point of view Note - In Miller's, chapter on thoracics (by Slinger) the steps are * Fio2 of 1.0 * Check position of DLT (using FOB) * ensure adequate CO, reduce volatile to <1MAC * Apply recruitment maneouvre * Apply 5cm H2O PEEP to dependent lung * Apply 1-2cm H2O CPAP to non-dependent lung after recruitment maneouvre * Intermittent Reinflation * Partial ventilation techniques including oxygen insufflation, HFJV, mechanical obstruction of blood flow to non-dependent lung

A 60 year old man describes orthopnoea. On examination: pansystolic murmur (at LSE)/ displaced apex beat. Likely diagnosis ? A. Mitral regurgitation

Ans A - MR *

Male undergoing trans-sphenoidal surgery. Now Na+ 155mmol/l and thirsty with polydypsia. Treatment: A. Desmopressin (DDAVP) B. Fluid restrict C. Aldosterone.

Ans A - desmopressin DDAVP for diabetes insipidus Central or nephrogenic Typically polyuria is compensated for by thirst and excess consumption of fluid to match output Usual features are: Hypertonic plasma with hypotonic urine Plasma ~ high normal > 290 - 300 Urine 50 - 200 mosm/L High Volume Urine output: > 4 - 6 L/day or > 3ml/kg for 4 - 6 consecutive hours TREATMENT Determines if central or nephrogenic • 10mcg DDAVP nasally (give for 12-24hrs) • 1mcg DDAVP S/C or IV Central - will be associated with 50% increase in urine osmolality *

Chest xray shown of patient post Left pneumonectomy with heart swung to left side. Management: A. Increase PEEP B. Roll onto right side C. Turn on suction to left pleural catheter D. Lung biopsy E.

Ans B

Which is NOT a disadvantage of drawover vaporiser versus plenum vaporiser: A. Temperature compensation B. Cannot use sevoflurane C. Small volume reservoir D. Flow compensation E.

Ans B

Burns dressings. The following is proven to be of analgesic benefit: A. Morphine gel B. Biosynthetic dressings (the answer per pain book) C. Dexmedetomidine IV D. Lignocaine IV E. Cognitive/Distraction technique

Ans B Acute pain management - scientific evidence - third edition 2010 p250 The choice of dressing has an effect on time to healing and pain during dressing change; biosynthetic dressings have been found to be superior. Nitrous oxide (N2O), ketamine and IV lignocaine infusions (Jonsson et al, 1991 Level IV) have also been used to provide analgesia for burn procedures (see Sections 4.3.1, 4.3.2 and 4.3.5), however a Cochrane review reported that more trials were required to determine the efficacy of lignocaine (Wasiak & Cleland, 2007 Level I).

What's the area burnt in the following man? Half of left upper arm, all of left leg and anterior abdomen. A. 27% B. 32% C. 42% D. E.

Ans B Arm is 9% half an arm is 4.5 Leg 18% Anterior abdomen 9% 4.5 + 18 + 9 = 31.5%

Which of the following is NOT an absolute contra-indication for MRI? A. cochlear implant B. heart valve prosthesis C. ICD D. pacemaker E. intracranial clips

Ans B CAECCP Anaesthesia for magnetic resonance imaging 2003 Absolute contra-indications to MRI include implanted surgical devices such as cochlear implants, cardiac pacemakers or implanted defibrillators, intra-ocular metallic foreign bodies or ferromagnetic neuro-vascular surgical clips. Many implanted prosthetic devices are non-ferromagnetic. General surgical clips, joint prostheses, artificial heart valves and sternal wires are generally safe as they are fixed by fibrous tissue

A 50 year old man with multiple fractures. The BEST parameter to monitor volume resuscitation is: A. Heart rate B. LVEDV C. PCWP D. RVEDV E. Changes in R atrial pressure during inspiration

Ans B LVEDV - accepted as is

Patient with aortic dissection. Blood pressure 150/90. Best drug to control BP: A. Captopril B. Esmolol C. GTN D. Hydralazine E. SNP

Ans B beta blocker esmolol CEACCP 2009 The primary goal is to reduce the force of left ventricular con-traction without compromising perfusion, thus reducing shear forces and preventing further extension of the dissection or possible rupture. Beta-blockers (e.g. esmolol, metoprolol) and labetalol (beta- and alpha-blocker) can be used. If further reduction in BP is required, sodium nitroprusside, glyceryl trinitrate, or hydralazine are appropriate. Beta-blockers should be given first before vasodilators, as the reflex catecholamine release due to vasodilatation may increase left ventricular contractions. *

Compared to lignocaine, bupivacaine is A. Twice as potent B. Three times as potent C. Four times as potent D. Five times as potent E. Same potency

Ans C

What's the most appropriate mode for neuromuscular monitoring during aneurysm clipping? A. TOF count B. TOF ratio C. Post tetanic count D. ? E. ?

Ans C Anaesthesia for cerebral aneurysm repair Roger Traill TOF 0 PTC >10

Liposuction. Infiltration of lignocaine with 1:200,000 adrenaline. Peak plasma concentration of lignocaine occurs at: A. 1 hour B. 3 hours C. 18 hours D. 24 hours E. 30 mins

Ans C CAECCP Novel techniques of LA infiltration Tumescent analgesia The high hydrostatic pressure within the tissues is also thought to be responsible for the delayed systemic absorption and hence delayed and reduced peak plasma concentrations of local anaesthetic, despite the very large doses being used. Doses as high as 22-57 mg kg-1 of lidocaine. Tumescent technique for regional anesthesia permits lidocaine doses of 35 mg/kg for liposuction. J Dermatol Surg Oncol 1990 Peak plasma lidocaine levels occurred 12-14 hours after beginning the infiltration.

Indication for percutaneous closure of ASD A. Primun < 3cm B. Primun > 3cm C. Secundum < 3 cm D. Secundum > 3cm E. sinus venosus ASD

Ans C CEACCP Anaesthesia for percutaneous closure of atrial septal defects 2008 Only an ostium secundum ASD is suitable for percutaneous closure. If the defect is very large (>3 cm) or complicated (associated with other abnormalities), or an incomplete rim is detected, referral for surgical closure is indicated.

The intercostobrachial nerve: A. Arises from T2 trunk B. Is usually blocked in brachial plexus block C. Supplies antecubital fossa D. can be damaged by torniquet E. Arises from inferior trunk

Ans D A- False. Arise from la. Cut. Branch of 2nd intercostal nerve (originally from T2 but not from the trunk. ) B- False. It joins the medial cutaneous nerve of the arm which comes from the medial cord, but does not form part of the brachial plexus, and is not blocked in brachial plexus blocks. C- False. Supplies medial side of upper arm, and joins medial cutaneous nerve of arm which supplies medial side of upper arm down to the elbow. D- True. Any nerve compressed by a tourniquet can be damaged. Would have to be high up he arm close to axilla to compress it. E- False. Not part of the brachial plexus, or a branch from it. Arises from lat. Cut. Branch of 2nd intercostal nerve.

Chronic alcohol use. Which is not an associated complication ? A. AF B. hypertriglyceridemia C. Macrocytosis D. Nephrotic syndrome E. Pancreatic Ca

Ans D CEACCP Alcohol and the Anaesthetist

A device that detects a 10mA difference in active and neutral leads and causes turning off of the circuit within 40 ms. This is a: A. Class 1 device B. Equipotential earthing C. Line isolation monitor D. Residual Current Device E. Fuse

Ans D CEACCP electrical safety in the operating theatre A. Class 1 Device: Any conducting part of Class I equipment accessible to the user, such as the metal casing, is connected to earth by an earth wire. If a fault occurs, this allows the live supply to come into contact with an accessible part, current flows down the earth wire. This new circuit has a lower resistance, resulting in an increased current which melts the protective fuses and breaks the circuit, removing the source of potential electrocution. B. Equipotential earthing: the terminals of each piece of equipment in a stack can be connected to each other bringing them all to the same potential. C. Line isolation monitor: measures the potential for current flow from the isolated power supply to the ground. i.e. active and neutral should have the same current. If there is a fault, a device is grounded then the current through to neutral would decrease. There is then a current able to flow through the line isolation monitor and sounds an alarm. D. Residual current device: If the current in the live and neutral conductors is the same, the magnetic fluxes cancel themselves out. However, if they are different (due to excessive current leakage) there is a resultant magnetic field. This induces a current in the third winding causing the relay to break the circuit. E. Fuse: a material that melts with increased current and breaks the circuit.

A 60yo man with anterior mediastinal mass, during induction for mediastinoscopy....lose cardiac output, decreased saturations, drop in ETCO2. Management: A. Adrenaline B. CPR C. CPB D. Place prone E.

Ans D Management of the patient with a large anterior mediastinal mass: recurring myths Curr Opin Anaesthesiol 20:1-3 2007 Should try wake up the patient. Intraoperative life-threatening airway compression has usually responded to one of two therapies: 1. Repositioning of the patient (it should be determined before induction if there is one side or position that causes less symptomatic compression) 2. Rigid bronchoscopy and ventilation distal to the obstruction (this means that an experienced bronchoscopist and rigid bronchoscopy equipment must always be immediately available in the operating room during these cases). For patients with life-threatening cardiovascular compression after induction that does not respond to lightening the anesthetic the only therapy is immediate sternotomy and surgical elevation of the mass off the great vessels.

Thermoneutral zone in 1 month old infant ? A. 26 - 28 degrees Celcius B. 28 - 30 degrees Celcius C. 30 - 32 degrees Celcius D. 32 - 34 degrees Celcius E. 34-46 degrees celcius

Ans D - 32 to 34 degrees Thermoneutral zone is the environmental temp range over which metabolic rate is kept at a minimum & within which temp regulation is achieved by non evaporative physical means alone. In thermoneutral zone temp maintained by vascular changes alone - this is less developed autonomic nervous system. Naked adult - 27-31. *

Type of dissection - which is classically for NON-operative management: A. DeBakey Type I B. DeBakey Type II C. Stanford A D. Stanford B E. Stanford C

Ans D - Stanford B CEACCP 2009 In acute type B aortic dissections, surgical intervention is only indicated if there is persistent or recurrent intractable pain, aneurysm expansion, peripheral ischaemic complications, and rupture. This is because surgical repair has no proven superiority over non-surgical treatment in stable type B dissection patients. Stanford A involves ascending aorta but may extend into arch and descending aorta Type B involves descending aorta only, dissection distal to left subclavian artery origin DeBakey - Type I involves ascending aorta, arch, descending aorta Type II - confined to ascending aorta Type III - descending aorta distal to left subclavian artery (IIIa up to diaphragm, IIIb beyond) *

What is the most common way to measure end tidal gas concentrations on our anaesthetic machines? A: mass spectometry B: Raman scattering C: ultrasonic D: infrared E: piezoelectric

Ans D -infrared CEACCP respiratory gas analysis Draeger use infrared cheapest

Anaphylaxis to rocuronium. Which is most likely to cause coss-reactivity ? A. Vecuronium B. Pancuronium C. Atracurium D. Cisatracurium E. None of the above -cross reactivity too variable to predict

Ans E Ref: AAGBI Anaphylaxis 2009 60% of anaphylaxis in anaesthesia = NBMD Miv- and atracurium are associated with histamine / non-allergy reaction Sux is most likely, Roc close behind Cross sensitivity is relatively common, probably because of their quaternary ammonium group. If anaphylaxis to an NMBA is suspected, the patient should undergo skin prick testing with all the NMBAs in current use. If a patient demonstrates a positive skin prick test (SPT) to an NMBA, the patient should be warned against future exposure to all NMBAs if possible. If it is mandatory to use an NMBA during anaesthesia in the future, it would seem appropriate to permit the use of an NMBA which has a negative skin test, accepting that a negative skin test does not guarantee that anaphylaxis will not occur. Rocuronium in NZ population ...... *

Innervation of larynx A. The internal branch of the superior laryngeal nerve supplies the lingual surface of the epiglottis B. In cadaveric position the cords are fully abducted C. The RLN supplies all intrinsic muscles of the larynx D. The glossopharyngeal nerves are sensory to the laryngeal mucous membrane above the level of the cords E. Cuff compression of recurrent laryngeal nerve against thyroid can cause palsy

Ans E Regional and topical anesthesia for endotracheal intubation Airway innervations Nasal cavity: greater and lesser palatine nerves and anterior ethmoidal nerve. Trigeminal nerve palatine nerves innervates: nasal turbinates and most of the nasal septum. Olfactory nerve anterior ethmoidal nerve innervates: nares and anterior third of the nasal septum. Oropharynx: vagus, facial and glossopharyngeal nerves Trigeminal nerve: lingual branch of the mandibular division supply the anterior 2/3 of tongue Glossopharyngeal: travels anteriorly along the lateral surface of the pharynx with 3 branches 1. Lingual branch: sensory posterior 1/3 of tongue, vallecula, anterior surface of the epiglottis 2. Pharyngeal branch: wall of the pharynx 3. Tonsillar branch: tonsils Vagus: 1. Internal branch of superior laryngeal nerve: This branch originates from superior laryngeal nerve lateral to the greater cornu of the hyoid bone a. Sensation: base of the tongue, posterior epiglottis, aryepiglottic folds and arytenoids. b. Motor: Cricothyoid 2. Recurrent laryngeal nerve: a. Sensation: vocal folds and trachea sensation b. Motor: All intrinsic laryngeal muscles except cricothyroid

What is the SVR in a patient with MAP 100mmHg, CVP 5, PCWP 15, CO 5L/min? A. ?0.8 B. ?3 C. 520 D. 1280 E. 1520 dynes.sec/cm-5

Ans E SVR=80(MAP-CVP)/CO SVR=80(100-5)/5 =1520

With regards to obstructive sleep apnoea (OSA), which of the following statements is INCORRECT? A. hypoxaemia is the main stimulus to arousal B. the main method of treating this syndrome is with Continuous Positive Airway Pressure (CPAP) C. this syndrome is the most likely diagnosis in patients presenting with excessive daytime sleepiness D. this syndrome occurs in up to 5% of adults E. this syndrome rarely has an obstructive component

Ans E Sleep apnoea syndrome may be obstructive or central. In OSA - obstruction the problem. Obstructive most common form of sleep apnoea syndrome. 4% men; 2% women.

Male with a Haemoglobin of 8G% and reticulocyte count 10%. Possible diagnosis: A. Untreated pernicious anaemia B. Aplastic anaemia C. Acute leukaemia D. Anaemia of chronic disease E. Hereditary spherocytosis

Ans E Spherocytosis Only this have increased reticulocyte count

TMP-Jul10-042 Patient post anterior cervical fusion. Patient in recovery. Confused and combative. Nurse concerned about haematoma. Taken to theatre: Most appropriate way of securing airway: A. Gas induction / laryngoscopy / intubate B. Awake tracheostomy C. Awake fibreoptic intubation using minimal sedation D. Thiopentone, suxamethonium, direct laryngoscopy and intubation E. Retrograde intubation

Ans: A CEACCP V7 No3 2007 Re-intubation may be difficult. The patients must be managed in a semi-sitting posture. Awake fibreoptic intubation is sometimes a good option, but direct laryngoscopy after sevoflurane and oxygen induction (with judicious doses of propofol) may be easier. The gum-elastic bougie is often vital and an LMA (+fibrescope,+gum elastic bougie or Aintree catheter) or an ILMA-guided technique may save the day. The use of succinylcholine in myelopathic patients is hazardous because of abnormal potassium shifts.

Elderly patient. Indications for pre Femoro-Popliteal Bypass angiogram include all EXCEPT: A. Severe heart failure B. Suspicion of L main disease C. Symptomatic tachyarrhythmia D. Unstable angina E. Stable angina with positive thallium

Ans: C ABD they are all straight forward indications for coronary angio. Positive thallium suggest patient may require revascularisation and coronary anatomy is required. AHA ACA *

Hydralazine

Anti-hypertenisve drug used for rapid lowering of BP Work on arteries & decreases Ca++ availability causing vasodilation. Dose 5-10mg IV Also in tablets for chronic HTN + CHF Used for pre-eclampsia but does cross placenta and cause fatal tachycardia

Leakage Current

Any current which returns by a pathway that is not neutral wire All electrically operated equipment has some leakage. Devices which have parts in contact with patient man have the that part isolated from rest of device so the leakage current is low but still present so microshock can occur. Current < 100micro amps ok but safest is leakage < 10microA which is required std for intracardiac devices

81. New- Pregnant patient seatbelt, driver- involved in car accident. Suddenly developed severe central chest pain, HR 110, BP 154/80, RR 26, Sat 100%. The most likely cause? A. Sternal fracture B. Aortic dissection C. Pneumothorax D. Rib fracture E. Myocardial infarction

Aortic dissection Consensus *

153. Regarding adenosine as an anti-arrhythmic, which is correct? a. potentiates theophylline b. potentiates dipyridamole c. alpha blockers should not be used d. beta blockers should not be used e. it is as effective as verapamil 5mg

B

70 y/o postop in recovery following hip surgery. Develops severe chest pain, ST elevation. Immediate mx: A Beta blocker B Aspirin C GTN infusion D Heparin infusion

B

A machine with a soda lime absorber was left on overnight with oxygen running at 6 litres per minute. In the morning a desflurane vaporiser is connected. What toxic substance may be produced? A. Substance A B. Carbon monoxide C. Carbon dioxide D. Calcium hydroxide E. Substance B

B

A patient has chronic AF on Dabigatran 150mg, with normal renal function. They are booked for elective ortho surgery. How do you manage Dabigatran. a) Cease 3 days prior, no bridging therapy b) Cease 7 days prior, no bridging therapy c) Bridging therapy d) Check INR pre-op & proceed if < 1.5

B

A patient having a craniotomy has the CVP/arterial transducers at the level of the right atrium. The head is 13cm above the level of the heart. If the MAP is 80mmHg and the CVP is 5mmHg what is the cerebral perfusion pressure in mmHg a. 60 b. 62 c. 65 d. 70 e. 75

B

A post-op child being given 2.5%D + 1/2NS on the ward seizes, is intubated and ventilated and transferred to ICU. Sodium is 116. What do you do? A. Give phenytoin B. Give hypertonic saline C. Give normal saline D. Give frusemide E. Give normal saline

B

Child with ?pyloric stenosis and 3 days of vomiting. Which bloods would you expect? [Not given pH] A. Na 130 K 4.5 Cl 90 Bic 17 B. Na 130 K 2.5 Cl 87 Bic 24 C. D. E.

B

Histamine release in anaphylaxis does NOT cause: A. Tachycardia B. Myocardial depression C. Coronary artery vasodilatation D. Prolonged PR interval E. Decreased impulse conduction

B

In a penetrating chest injury what part of the heart is most likely to be injured a. Left ventricle b. Right ventricle c. Right coronary artery d. Right atrium e. Sinus node

B

Labour epidurals increase maternal and foetal temperature. This results in neonatal A. Increased sepsis B. Increased investigations for sepsis C. increased non shivering thermogenesis D. Increased need for resuscitation E. Cerebral palsy

B

Lowering intra-ocular pressure by applying pressure to the globe (e.g. Honan balloon) is typically contraindicated in a patient having A. a revision corneal graft B. a revision trabeculectomy C. an extra-capsular lens extraction D. a redo vitrectomy E. repeat retinal cryotherapy

B

Maternal cardiac arrest. In making the diagnosis of amniotic fluid embolism, large amount of PMNs surrounding foetal squamous cells are A. Pathognomonic B. Supportive C. Only found at postmortem D. Irrelevant E. Incidental

B

Most likely to result in myocardial infarction: A. intraop myocardial ischaemia B. post op myocardial ischaemia. C. D. E.

B

Myasthenia gravis, Eaton Lambert Syndrome What happens with exercise? a. MG better, EL worse b. EL better, MG worse c. Both EL and MG get worse d. Both EL and MG get better

B

Preoperative autologous blood donation results in less: A. Cost B. Incompatible transfusion C. Less blood wastage D. Less unnecessary transfusion

B

Respiratory function in quadriplegics is improved by A. abdominal distension B. an increase in chest wall spasticity C. interscalene nerve block D. the upright position E. unilateral compliance reduction

B

Salicylate poisoning: A. Respiratory acidosis B. Metabolic acidosis C. Increased CO2 production D. High output renal failure E. Hypothermia/hypethermia

B

The MAIN indication for biventricular pacing is A. complete heart block B. congestive cardiac failure C. VF D. E.

B

The PREDOMINANT pathophysiological effect of restrictive cardiomyopathy is: A. B. Diastolic dysfunction C. D. E.

B

The commonest post operative complication in a patient with a # NOF is a. UTI b. Pneumonia c. Delirium d. Myocardial infarction

B

The muscles of the upper eyelid receive a somatic nerve supply from the A. oculomotor nerve and a parasympathetic supply from the superior vagus nerve B. oculomotor nerve and a sympathetic supply from the superior cervical ganglion C. ophthalmic division of the trigeminal nerve and a parasympathetic supply from the superior vagus nerve D. ophthalmic division of the trigeminal nerve and a sympathetic supply from the superior cervical ganglion E. ophthalmic division of the facial nerve only

B

Uneventful surgery, someone goes into rapid AF and is hypotensive. What is the correct treatment? a. 50J unsynchronised monophasic shock b. 200J unsynchronised monophasic shock c. 500mcg digoxin slowly d. 5mg verapamil e. 6mg adenosine

B

VT features: A monophasic V6 B QRS > 0.14 C Right axis deviation

B

What gestation to monitor uteroplacental flow A 20 weeks B 24 weeks C 28 weeks D 32 weeks E 36 weeks

B

Lateral approach to popliteal block. A. Passes through semimembranosis B. May be performed supine or prone C. Adequate for ankle surgery D. Less effective in comparison to posterior approach E. Eversion is an endpoint for nerve stimulation

B ?? B & E (Preferably tibial stimulation observe as it is more medial) • No, passes through biceps femoris. Semimembranosus is on the medial side of the nerve • Usually performed supine • Need to block femoral branches too for the medial aspect of ankle • Can't find anything on this - wouldn't think so? • According to NYSORA end-point is twitching of the foot, which can be either dorsiflexion/eversion (from common peroneal branch of sciatic), or plantarflexion/inversion (from tibial branch of sciatic).

Which drug has the best evidence for reducing blood loss in trauma? A. Aminocaproic acid B. Novo 7 C. Prothrombinex D. Tranexamic acid E. Aprotinin

B ?? D CRASH-2 Trial

Dose of GTN IV to relax uterus? a. 25 mcg b. 50 mcg c. 250 mcg d. 500 mcg e. 750 mcg

B ANZJOG Volume 37, Issue 1, pages 20-24, February 1997 : 50 - 200 mcg , 200mcg is mentioned a few places. I will check the O&G textbooks for a proper reference. --Methoxyflurane 06:45, 21 April 2012 (CDT)

What is the most accurate method of determining fetal heart rate in a neonate a. Palpation of an umbilical vein pulse b. Auscultation with a stethoscope c. Palpation of femoral pulse d. Pulse oximetry

B ARC Neonatal Resuscitation • "Heart rate can be determined by listening to the heart with a stethoscope (most reliable) or in the first few minutes after birth, by feeling for pulsations at the base of the umbilical cord" • Pulse oximetry can provide and accurate and continuous display of the heart rate within about a minute of birth" ?more accurate than auscultation

What is the ratio of MAC awake:MAC of sevoflurance a. 0.2 b. 0.34 c. 0.5

B Cerebral Awakening Concentration of Sevoflurane and Isoflurane Predicted During Slow and Fast Alveolar Washout. A & A November 1993 vol. 77 no. 5 1012-1017 Make awake value for sevo = 0.34

Lap chole on citalopram. What is NOT relatively contraindicated? A. Omeprazole B. Clonidine C. Pethidine D. Tramadol E. ?Midazolam

B Citalopram shares CYP3A4 metabolism with midazolam Citalopram shares CYPC19 metabolism with omeprazole Tramadol and pethidine with citalopram may be relatively contraindicated due to serotonin syndrome concerns

Which of the following is a contra-indication to a left DLT a. Left pneumonectomy b. Tumour in the left main stem bronchus c. R sided bronchopleural fistula d. the patient has a shunt >10% e. the left lung is to be collapsed

B Contraindications to a Left DLT • Intra-luminal tumour of l main bronchus • Left bronchial stent in situ • Left tracheo-bronchial disruption • Left pneumonectomy • Left lung transplant

Which patient do you not put a left-sided Robert-Shaw DLT into? A. Left pneumonectomy B. Left main bronchial lesion C. There is a right-sided broncho-pleural fistula D. The patient has s hunt > 10% E. The left lung is to be collapsed

B Contraindications to a Left DLT • Intra-luminal tumour of l main bronchus • Left bronchial stent in situ • Left tracheo-bronchial disruption • Left pneumonectomy • Left lung transplant

Which is NOT a disadvantage of drawover vaporiser versus plenum vaporiser? A. Temperature compensation B. Cannot use sevoflurane C. Small volume reservoir D. Flow compensation E.

B Diadvantages 1. Poor temperature compensation (OMV suffer from a reduction in vapour ouput at lower temperatures) 2. Difficult to use sevoflurane (sevo requires high concentrations, therefore require extra wicks to maximise output, however loss of latent heat of vaporisation rapidly lowers its performance. Two OMVs are required to provide adequate sevo conc for induction) 3. Small volume reservior (OMV contains 50ml, which empties rapidly) 4. Basic temperature compensation 5. Less easy to observe spont vent with self inflating bag 6. cumbersome in paedatric use Advantages of draw over 1. easy to set up 2. in circuit 3. low resistance 4. mobile 5. non agent specific 6. no need for pressurized gas supply 7. robust, reliable 8. easily serviced

A C size oxygen cylinder (A size in New Zealand) reads 5000kPa. How much oxygen remains? A. 50 Litres B. 150 litres C. 500 D. 750 E. 1500

B Full size C (13700 kPa) = 420 L So 5000/13700 x 420 = 153L

A size C oxygen cylinder that reads 5000kpa contains approximately how many litres of oxygen a. 100 b. 150 c. 200 d. 350 e. 600

B Full size C (13700 kPa) = 420 L So 5000/13700 x 420 = 153L

Estimate GCS post head-trauma.

B GCS 8 E2 V2 M4 Best Eye Response. (4) 1. No eye opening. 2. Eye opening to pain. 3. Eye opening to verbal command. 4. Eyes open spontaneously. Best Verbal Response. (5) 1. No verbal response 2. Incomprehensible sounds. 3. Inappropriate words. 4. Confused 5. Orientated Best Motor Response. (6) 1. No motor response. 2. Extension to pain. 3. Flexion to pain. 4. Withdrawal from pain. 5. Localising pain. 6. Obeys Commands.

After a procedure with an LMA in situ a patient complains of loss of sensation to the anterior part of the tongue. What nerve is likely damaged? a. Facial b. Lingual c. Greater palatine d. Glossopharyngeal

B Innervation of the Tongue Anterior 2/3rds of tongue • Somatic afferent: lingual nerve branch of V3 of the trigeminal nerve • Taste: chorda tympani branch of facial nerve (carried to the tongue by the lingual nerve) Posterior 1/3rd of tongue • Somatic afferent and taste: Glossopharyngeal nerve CN IX Motor • All intrinsic and extrinsic muscles of the tongue are supplied by the hypoglossal nerve • Except for one of the extrinsic muscles, palatoglossus, which is innervated by CN X

Which nerves need to blocked to anaesthetise the hard palate: A. Superior labial nerve and greater palatine nerve B. Greater palatine nerve and nasopalatine nerve C. Inferior orbital nerve and nasopalatine nerve D. Glossopharyngeal nerve and... E. Anterior ethmoidal nerve and...

B http://ozradonc.wikidot.com/anatomy:focused-hard-palate "The hard palate is innervated by branches of the maxillary nerve, both of which initially pass through the pterygopalatine ganglion. The greater palatine nerve descends through the greater palatine foramen with its companion artery, and runs anteromedially to supply the mucosa of the posterior hard palate. The nasopalatine nerve descends through the incisive foramen to supply the most anterior parts of the hard palate"

Meconium stained liquour but neonate delivered is vigorous. Rationale for NOT suctioning the neonate? A. May aspirate meconium B. May cause bradycardia C. May cause hypertension

B It does not alter outcome & may cause harm - Trauma to oropharynx - Bradycardia - Laryngospasm - Prolong cyanisis - Delay onset of spont breathing Intubate + suction if non responsive (in non VIGOROUS it would appear that either ok but guidelines not very clear for me) Resus Council Neonate Guidelines * Neonatal Resus 3:1 Priority is ventilation to establish FRC Use room air inititally; ensure good breaths with chest expansion before increasing FiO2. Babies may use 80cmH2O to expand lungs Use 30cmH2O as initial breath (25 if preterm); aim is to use the minimum that gives lung (chest wall ) expansion. At least 5cm H2O Most reliable for HR is chest auscultation. Consider umbilical cord but of NOT heard don;t worry as unreliable. Use pulse ox R) hand - request if IPPV to doing resus. Sats 60% -> 90& over 10 minutes. Avoid hyperoxaemia Normal HR ~ 130 min (110 -160/min ). Start compressions if < 60/min. Use two thumb technique lower 1/2 sternum, compress 1/3 AP diameter (less tiring than fingers). 90 compressions/min with half second pause after the breath. Resus guidelines use 80mL/kg for blood volume Canulate umbilical vein adrenaline IV dose 10-30mcg/kg ETT adrenaline 50-100mcg and still might not be enough Fluid 10mL/kg Normal saline pH of foetus (arterial umbilical sample) pH 7.27, BE -4; PO2 17; PCO2 52

You see a patient in the pre-op clinic. He is on propranolol for treatment of long QT syndrome. Which of the following will give the best reassurance that his treatment is effective? a. normal QT interval on resting ECG b. no change in QT interval with valsalva c. HR less than 60 d. no arrhythmias on 24h holter monitor e. decreased QT interval in response to a valsalva manoeuvre

B Long QT syndrome and anaesthesia. BJA 2003 "The mainstay of treatment of congenital LQTS since 1975 has been beta‐block" "The dose of β‐blocker is determined by ensuring a reduction in maximal heart rate on treadmill exercise testing to 130 beats min-1 or less... where exercise testing is impractical, there should ideally be no change in the QT interval in response to a Valsalva manoeuvre" "The QTc is unchanged despite efficacy of treatment, although QTD is higher in patients who do not respond to β‐block

Regarding remifintanil, which is incorrect? a. high potency b. metabolised by pseudocholinesterase c. muscle rigidity in high doses d. weakly active metabolite e. short context sensitive half time

B Metabolised by non-specific tissue and plasma esterase

Young female having cholecystectomy. Venous air embolus: A. Mechanical ventilation and PEEP is part of treatment strategy B. Most likley to occur at initial gas insufflation, but can occur at any time C. Inert gas (argon, xenon) is safer D. E.

B Miller 7th ed. p. 2188 "This complication develops principally during the induction of pneumoperitoneum, particularly in patients with previous abdominal surgery."

Regarding mixed venous blood oxygen saturation, which statement is correct? a. it is collected from the right atrium b. it is used to calculate cardiac output c. it can be used to accurately measure the mixed venous pO2 d. it has no impact on the A-a gradient e. it is usually 40%

B Normal range for SvO2 is 60 - 80% Is collected from PA CO can be calculated by a modification of the Fick equation, where VO2 = (CO x CaO2) - (CO x CvO2 ) Where CO = Cardiac Output, Ca = Oxygen concentration of arterial blood and Cv = Oxygen concentration of mixed venous blood VO2 = Arterial Oxygen Transport - Venous, Oxygen Transport = (CO x CaO2 x 10) - (CO x CvO2 x 10) = CO x (CaO2 - CvO2) x 10 = CO (Hb x SaO2 x 13.8) - CO (Hb x SvO2 x 13.8) = CO x Hb x 13.8 x (SaO2 - SvO2) VO2 = CO x Hb x 13.8 x (SaO2 - SvO2) VO2 = Oxygen consumption CO = cardiac output SaO2 = arterial O2 sats SvO2 = venous O2 sats

A 23 year old man takes his buprenorphine patch off prior to surgery. When can you expect the plasma level to be half? a. 6 hours b. 12 hours c. 24 hours d. 36 hours e. 48 hours

B Norspan product information "After removal of a NORSPAN patch, buprenorphine concentrations decline, decreasing approximately 50% in 12 hours (range 10 - 24 h)"

Chest Xray findings in a child who has inhaled a foreign body: A. Opaque mass overlying the airway B. Hyper-expanded lung fields C. Unilateral pulmonary oedema D. Collapsed lung base E. Mediastinal shift

B RCH CPG - Inhaled Foreign Body Look for: • an opaque foreign body • segmental or lobar collapse • localised emphysema in expiration (ball valve obstruction) • The CXR may be normal Rovin JD, Rodgers BM. Pediatric foreign body aspiration. Pediatrics in Review 2000 Most common X-ray findings with inhaled foreign body: • Normal - no abnormality • Gas Trapping (due to ball-valve effect of foreign body with respiration) • Mediastinal shift • Atelectasis • Lobar collapse/consolidation

Child after gas induction, LMA insertion without IV cannula- desaturate to 90%. Next step of action? A. Bag with LMA insitu B. Bag without LMA insitu

B Remove offending stimulus 100% O2 with CPAP, deep anaesthetic Optimize airway with gentle jaw thrust Morgani maneuver Drugs Propfol 0.5mg/kg Sux 0.5mg/kg atrop 0.2mic/kg IM sux if no IV Refractory latngospasm: SLN block, Transtracheal lig through cric mem

You are asked by an Obstetrician to help relax a uterus in labour and deliver for manual removal of placenta. What is a safe and effective dose of IV GTN to be delivered? A. 5 mcg B. 50 mcg C. 250 mcg D. 400 mcg E. 500 mcg

B Safety of Intravenous Glyceryl Trinitrate in Management of Retained Placenta. Australian and New Zealand Journal of Obstetrics and Gynaecology. 2008. Vol 37 Issue 1 "A retrospective chart review of all anaesthesia consultations for retained placenta at 28 weeks' gestational age or more was performed to determine the safety and efficacy of intravenous glyceryl trinitrate therapy in the management of retained placenta. Of the 33 patients who received glyceryl trinitrate, 1 received a total dose of 50 μg and the remainder received 100 to 200 μg. All placentas were extracted within 4 minutes of the first bolus. The systolic and diastolic blood pressures and the haematocrit fell by a mean of 8.1 ±5.3 mmHg, 6.0 ± 3.5 mmHg, and 2.6 ± 1.7%, respectively (mean difference ± SD, p<0.05 for each). The pulse rose by a mean of 7.7 ±4.5 bpm (p<0.001). Only 1 patient required ergometrine for continued atony. None of the patients required transfusions or operative therapy other than dilatation and curettage. The use of glyceryl trinitrate in doses of 200ug or less for retained placenta appears efficacious and safe, and may obviate the need for general anaesthesia for uterine relaxation"

A 35kg 5 year old girl is having elective surgery for suturing of a superficial leg laceration. After induction with N2O/Sevoflurane/O2 and in absence of any visible veins you have placed an appropriately sized LMA. Following this her SpO2 immediately drop to 90%. What is your initial management? A. Increase inhaled sevoflurane concentration with LMA in situ B. Increase inhaled sevoflurane concentration after removing LMA C. Intralingual suxamethonium (no dose stated) D. Intramuscular suxamethonium (no dose stated) E. Intramuscular atropine (no dose stated)

B Stem incomplete? Looks like they are suggesting laryngospasm.

In acute liver injury what causes the highest risk of bleeding a. Thrombocytopenia b. Coagulopathy c. Portal hypertension d. Platelet dysfunction e.

B can't find anything on this Miller • "Perioperative hemorrhage in patients with significant liver dysfunction may occur because of bleeding diatheses or the complications of portal hypertension (or both)" • Portal hypertension → splenomegaly → thrombocytopaenia • "In patients with acute liver failure, plasmapheresis may have potential benefit because it promotes rapid correction of coagulopathy while minimizing volume overload"

37. A pregnant lady is undergoing neuroradiological coiling of a cerebral aneurysm. At what gestational age should you monitor foetal heart rate to ensure adequate uteroplacental blood flow A. 20 weeks B. 24 weeks C. 28 weeks D. 30 weeks E. 32 weeks

B see ACOG statement Intraoperative electronic fetal monitoring may be appropriate when all of the following apply: — The fetus is viable. — It is physically possible to perform intraoperative electronic fetal monitoring. — A health care provider with obstetric surgery privileges is available and willing to intervene during the surgical procedure for fetal indications. — When possible, the woman has given informed consent to emergency cesarean delivery. — The nature of the planned surgery will allow the safe interruption or alteration of the procedure to provide access to perform emergency delivery. If previable - doppler prior to and after procedure If viable - intraoperative, or pre and post CTG

Labour epidurals increase maternal and foetal temperature. This results in neonatal A. Increased sepsis B. Increased investigations for sepsis C. increased non shivering thermogenesis D. Increased need for resuscitation E. Cerebral palsy

B A great core temperature is tolerated with an epidural block

Time to reach peak plasma concentration after injection of 2% lignocaine with adrenaline into epidural space A. 20 min B. 30 min C. 40 min D. 50 min E. 60 min

B Br J Clin Pharmacol 1996; 42: 242245 Peak lignocaine concentration ~30 mins

Arterial blood gases (ABGs): pH 7.12, PO2 100, PCO2 65, HCO3 20.3, BE -10. Consistent with? A. Chronic renal failure B. Malignant hyperthermia C. Diabetic ketoacidosis D. End-stage respiratory failure E. Ethylene glycol toxicity

B Combined metabolic and respiratory acidosis = MH! a. CRF wouldn't have high CO2 c. DKA = low CO2 to breathe off d. Would be metabolic compensation i.e. high bicarb e. Would try to breathe off CO2 to compensate for non-anion gap metabolic acidosis

Visual loss with pupillary reflexes retained. Likely cause ? A. Retinal detachment B. Occipital mass C. Frontal mass D. Chiasmal mass E. Optic neuritis

B Cortical blindess = visual loss but with retained pupillary reflexes and normal fundoscopy. Caused by pathology in the occipital lobes. This damage is most often caused by loss of blood flow to the occipital cortex from either unilateral or bilateral posterior cerebral artery blockage (ischemic stroke). A patient with cortical blindness often has little or no insight that they have lost vision, a phenomenon known as Anton's Syndrome or Anton-Babinski syndrome. http://www.iveyeye.ca/pdfs/acute_visual_loss.pdf Retinal detachment - an extensive retinal detachment involving the macular area would produce acute visual loss and this patient will complain of flashing lights followed by a large number of floaters and then a shade or blind covering the visual field. An afferent pupillary defect is usually present. The diagnosis is confirmed by ophthalmoscopy through a dilated pupil, and retina appears elevated with folds and the choroidal background is indistinct. Optic Neuritis: Optic Neuritis is inflammation of the optic nerve and is usually associated with multiple sclerosis in a significant number. The visual acuity is markedly reduced and an afferent pupillary defect is present. The optic disc initially appears hyperemic and swollen. The visual acuity usually recovers; however, repeated episodes of optic neuritis may lead to permanent loss of vision.

Nerve block for anaesthesia over anterior 2/3 of ear? A. C2 B. Mandibular nerve C. Maxillary nerve D. Ophthalmic nerve E. Vagus

B Four sensory nerves supply the external ear: (1) greater auricular nerve, (2) lesser occipital nerve, (3) auricular branch of the vagus nerve, and (4) auriculotemporal nerve. (Emedicine) The greater auricular nerve is a branch of the cervical plexus. It innervates the posteromedial, posterolateral, and inferior auricle. The lesser auricular nerve innervates a small portion of the helix. The auricular branch of the vagus nerve innervates the concha and most of the area around the auditory meatus. The auriculotemporal nerve is a branch of the mandibular branch of the trigeminal nerve. It innervates the anterosuperior and anteromedial aspect of the auricle. The external auditory canal and tympanic membrane have separate innervation. http://emedicine.medscape.com/article/82698-overview

Plasma glucose level compared to blood glucose level A. 32% higher B. 14% higher C. Same D. 14%lower E. 32% lower

B Wiki: Glucose levels in plasma (one of the components of blood) are generally 10%-15% higher than glucose measurements in whole blood (and even more after eating).

A 55 year-old man presents to the emergency department with an obviously infected heel ulcer - BP 100/60PR 110/minute, temperature 35.8, Na 125, K 2.7, BSL 55, Creatinine 180. Which do you give first/most urgently A. Antibioitcs B. Crystalloid C. Insulin D. Potassium E. Adrenaline

B HONK management

Which gives the BEST seal? A LMA classic B Proseal C Intubating LMA D. ? E. ?

B C according to anesth analg 2008 A Comparison of Seal in Seven Supraglottic Airway Devices Using a Cadaver Model of Elevated Esophageal Pressure B according to product info CEACCP Supraglottic Airway Deices - Recent Advancements From the LMA website / product information: LMA Classic - Seal pressure up to 20cm H20 LMA Supreme™ - measured oropharyngeal leak pressures up to 37 cm H2O LMA Flexible™ - oropharyngeal seal pressures up to 20 cm H2O LMA Fastrach - Seal pressures up to 20 cm H2O LMA ProSeal™ - leak pressures up to 32 cm H2O (Oesophageal seat = ILMA; Concerning the risk of aspiration, the use of devices with an additional esophageal drainage lumen might be superior for use in patients with an increased risk of aspiration. The Combitube, Easytube, and intubating laryngeal mask Fastrach showed the best capacity to withstand an increase of esophageal pressure.)

Post accidental dural puncture with epidural needle. Headache. Which does NOT fit ? A. Epidural blood patch 30-50% effective B. Unlikely to be related to epidural if purely occipital headache C. Caffeine mildly effective in reducing headache D. Subdural haematoma can rarely occur with PDPH E. (?something about photophobia)

B CEACCP 2001 The headache does not occur in any specific distribution in the cranium. *

Preoperative assessment shows a Mallampati (ML) score of III and thyromental distance (TMD) of < 6cm. A grade 3 to 4 on Cormark and Lehane is predicted. Compared to the ML score, the TMD is A less sensitive, less specific B less sensitive, more specific C more sensitive, less specific D more sensitive, more specific E equal sensitivity an specificity

B (this makes sense to me, kate; need to check)

Initial dose of IV GTN to relax the uterus is? A. 5 mcg B. 50 mcg C. 200 mcg

B - 50 mcg

Normal systolic BP at birth? A. Something less than 70 mmHg B. 70 mmHg C. 85 mmHg D. Something more than 85 mmHg E. 115 mmHg

B - 75mmHg NETS Victoria - Neonatal Handbook (which references Nuntnarumit P, Yang W, Bada-Ellzey HS. Blood pressure measurements in the newborn. Clin Perinatol 1999)\d\d//d3cgb598vs7bfg.cloudfront.net/images/upload-flashcards/554968/540518_m.jpg

CHADS2 score. Which is not a feature? A. Age B. Gender C. Diabetes mellitus D. Stroke E. CCF

B - Gender CCF +1 HT +1 Age > 75, +1 DM +1 Stroke/TIA/Thromboembolic event, +2

Fat:blood coefficients? A. N2O~Des > Sevo > Iso B. N2O > Des > Sevo~Iso

B - N2O > Des > Sevo~Iso Blood:Gas are very similar for Des and N2O, fat:blood very different\d\d//d3cgb598vs7bfg.cloudfront.net/images/upload-flashcards/868076/587661_m.jpg

Young woman with subarachnoid haemorrhage, hyponatraemia and increased urinary sodium (did not specify if high sodium concentration or total amount lost). What is likely cause? A. cerebral salt wasting syndrome B. SIADH C. HHH therapy D. Excess NS administration E. diabetes insipidus

B - SIADH CEACCP 2008 CSWS is usually caused by brain injury/trauma or cerebral lesion, tumor, or hematoma. CSWS is a diagnosis of exclusion and may be difficult to distinguish from the syndrome of inappropriate antidiuretic hormone (SIADH), which develops under similar circumstances and also presents with hyponatremia. The main clinical difference is that of total fluid status of the patient: CSWS leads to a relative or overt hypovolemia whereas SIADH is consistent with a normal to hypervolemic range. Random urine sodium concentrations tend to be lower than 100 mEq/L in CSWS and greater in SIADH. If blood-sodium levels increase when fluids are restricted, SIADH is more likely. The biochemical criteria for CSWS are: (i) low or normal serum sodium (ii) high or normal serum osmolality (iii) high or normal urine osmolality (iv) increased haematocrit, urea, bicarbonate, and albumin as a consequence of hypovolaemia. However, these criteria are often inconclusive. In CSWS, total daily urine sodium excretion is greater than intake, whereas it is usually equal to intake in SIADH, that is, overall sodium balance is negative in CSWS and generally neutral in SIADH.

What is NOT a contraindication to MRI? A. Pulmonary artery catheter B. Arterial line C. Scissors D. Coiled ECG cable E. Laryngoscope

B - arterial line Blue Book 2005 "Invasive blood pressure transducers: These are not ferromagnetic, and are safe to use. Transducer cables should be kept out of the magnet bore, so as to avoid imagedistortion" MRIsafety.com "There is at least one report of a cardiovascular catheter (Swan-Ganz Triple Lumen Thermodilution Catheter) that "melted" in a patient undergoing MR imaging. This catheter contained a wire made from a conductive material that was considered to be responsible for this problem. Thus, there are realistic concerns pertaining to the use of similar devices in patients undergoing MR examinations"

Complications of mediastinoscopy include all of the following EXCEPT A. air embolism B. cardiac laceration C. pneumothorax D. recurrent laryngeal nerve palsy E. tracheal compression

B - cardiac laceration Plummer et al. Anaesthesia for telescopic procedures in the thorax BJA 1998 Major complications of mediastinoscopy • Haemorrhage • Pneumothorax • Recurrent laryngeal nerve injury • Air embolism • *Compression of vessels • Aorta → reflex bradycardia • Innominate artery ( R brachiocephalic trunk) • Right carotid → hemiparesis • Right subclavian → loss of right radial pulse • Compression of trachea

Inverted P waves in lead II may be caused by? A. transposed lower limb leads B. junctional rhythm C. hypothermia D. left axis deviation E. inferior myocardial infarction

B - junctional rhythm

Nerve block for anaesthesia over anterior 2/3 of ear? A. C2 B. Mandibular nerve C. Maxillary nerve D. Ophthalmic nerve E. Vagus

B - mandibular nerve (origin of auriculotemporal nerve)

Perform a brachial plexus block however the medial forearm is NOT numb. Which nerve has been missed? A. Medial antebrachial cutaneous nerve B. Medial brachial cutanous nerve C. Intercostal brachial nerve D. Axillary superior lateral cutaneous nerve

B - median antebrachial cutaneous nerve A Smallest branch of brachial plexus, arises from medial cord, derived from C8-T1 C8-T1 - Inferior or lower trunk - anterior division - medial cord\d\d//d3cgb598vs7bfg.cloudfront.net/images/upload-flashcards/554968/540516_m.jpg

Main heat loss in anaesthetic for neonate A. vasodilatation B. radiation C. convection D. conduction E. evaporative

B - radiation

Emergency caesarean section for foetal distress (and foetal acidosis on scalp probe?). what is best option to raise gastric pH preop: A)Oral Na Citrate B)Ranitidine IV C)Ranitidine oral D)Omeprazole IV E)Omeprazole oral F)Metoclopramide 20 IV

B - ranitidine IV Ranitidine more effective than PPI in 1. reducing volume 2. increasing pH Antacids neutralise pH only, no volume effect.

What is the maximum dose of Intralipid during LA toxicity resus? 8ml/kg A. 10 B. 12 C. 16

B -12

Half-life of mast cell tryptase? a) 1 hour b) 3 hours c) 5 hours d) 7 hours e) 24 hours

B -best Half life 2 hrs with peak at 1 hr. Mast cell tryptase (MCT) is enzyme secreted by tissue bound mast cells, mediates smooth muscle relaxation. It is released in anaphlactic runs; stimulates pro-inflamm mediators. Better marker than histamine - half life too short. 99% tryptase found in mast cells. Normal level 0.8 - 1.5mg/mL Anaphlyaxis is a severe life-threatenign , generalized or systemic hypersensitivity reaction. Sub-divided into allergic + non-allergic. Allergic - IgE, IgG or complement mediated. Non-allergic - Mast cell + basophil degranulation caused by direct drug action with no immune trigger. Skin prick testing only picks up IgE mediated reactions - > so drug can still be trigger. "Anaphylaxoid" term now obsolete. Increase less marked in no allergic states but baseline varies so relative change more significant. Normal levels do NOT exclude anaphylaxis as can see this in basophil or complement mediated reactions + fluid can dilute MCT. > 20ng/mL likely IgE rxn

Following an eclamptic seizure the dose of MgSO4 is? A. 1 gram B. 4 grams C. 8 grams

B 4 grams The regimen recommended by the Collaborative Eclampsia Trial is 4-5 g MgSO4 intravenously over 5 min, followed by infusion of 1 g/hr. For a second siezure, give a further 2 g bolus, and increase infusion to 2g/hr

Interscalene block, patient hiccups...where do you redirect your needle? A. Anterior B. Posterior C. Caudal D. Cranial E. Superficial

B It is stimulating the phrenic nerve *

Best agent to decrease both gastric volume and gastric acidity? A. Omeprazole B. Ranitidine C. Metoclopramide D. Cisapride E. Na citrate

B Ranitidine (IV if option exists)

Interscalene block after injection of 2ml bupivacaine- patient seizure. Most likely injected to A. Dural cuff B. Vertebral arteries C. Internal carotid arteries D. Jugular veins E Subarachnoid

B Vertebral artery

Air bubble leads to decreased: A. Damping coefficient B. Resonant frequency

B resonant frequency (chang)

Congenital Upper Airway Obstruction

Baby PINK when crying -> increased inspiratory pressure; Blue at rest. eg choanal atresia, narrow pharynx

An 18 yo with Fontan circulation undergoing exploratory laparotomy. On ICU vent, sats 70%. Which ventilator parameter would you INCREASE to improve his sats? A. Bilevel pressure B. Expiratory time C. Inspiratory time D. Peak inspiratory pressure E. PEEP

B. Expiratory time CEACCP Low respiratory rates, short inspiratory times, low PEEP, and tidal volumes of 5-6 ml kg21 usually allow adequate pulmonary blood flow, normocarbia, and a low PVR.

LUSCS for failure to progress. Spinal is inserted uneventfully. Next day the patient has foot drop. The most likely cause is? A. epidural haematoma B. lumbosacral palsy C. sciatic nerve palsy D. common peroneal palsy E.

B. It's never us ;) CEACCP 2003: "Postpartum foot drop is caused by damage to the lumbosacral trunk or, less frequently, the common peroneal nerve. The lumbosacral trunk (L4, L5) is compressed between the ala of the sacrum and the descending fetal head. It may also occur during a forceps delivery. The result is a unilateral foot drop with loss of sensation and/or paraesthesia along the lateral calf and foot." "Common peroneal nerve damage may occur due to improper or prolonged positioning during lithotomy and the sensory deficit may be limited to the dorsum of the foot." Epidural haematomas extremely rare (1:168,000 from review in Anaesthesiology 2006; 105: 394)and obstetric palsies are much more common than complications related to neuraxial blocks. Sciatic nerve injury would cause a foot drop but would also affect knee flexion (hamstrings) and all muscles in lower leg and foot. Common peroneal nerve palsy less likely in this case as there is no mention of stirrups or 'excessive knee holding'

Septic elderly man. Given lots of obs but essentially mixed venous oxygen sat 65%, lactate 4, MAP low. Mx. A transfuse B fluid bolus C Noradrenaline

B. Points to inadequate perfusion --> needs resuscitation, especially if septic. Then maybe to squeezers (which can worsen lactate). Oxygen Fluid resuscitation

An indication for NON-operative management of blunt liver trauma in adults is A. absence of peritoneal signs B. a haemodynamically stable patient C. a haemopertitoneum of LESS than 500 ml D. a LOW grade injury on CT scan E. severe chronic obstructive airway disease

B. a haemodynamically stable patient

Thiopentone

Barbiturate anaesthetic agent. Used for induction of anaethesia, anticonvulsant and cerebral protection. Recovery due to resdistribution. Highly lipid soluble with saturated liver kinetics at high concentration giving long context sensitive half life if given by infusion. Low hepatic extraction ratio, metabolism depends n enzymes not blood flow. Dose 5mg/kg. LOC in ~ 1 minute. Lasts ~ 7min. Caution in elderly, hypovolaemia & porphyria. Causes hypotension, Prepared as Na thiopentone which is alkaline, pH 10.5. Faster onset in acidotic patient as the lower pH drives the equilibrium to more unionised drug which x BBB. Necrosis on intra arterial injection & precipitation if given with acidic drug (muscle relaxant, ringer's lactate), arterial blood.

Amiodarone

Benzofuran derivative class III anti- arrythmic for use with, SVT incl WPW & VT Complex action with effects on all classes. It has less effect on BP or LV contractility than other anti arhythmic drugs. 100mg/200mg tabs or 150mg solution requiring dilution with dextrose MOA 1. Block K+ channels - Increases duration of action potential - Increases refractory period 2. Higher doses block Na currents 3. Antagonism of alpha & beta receptors

Atracurium

Benzylisoquinilonium ester 10 isomers of which cis-atracurium is the main active component. 0.5mg/kg; onset 90 seconds, duration ~ 40 min Hoffman degradation (temp/pH) & non specific plasma esterases (laudanosine is byproduct, causes seizures in rats) Histamine release -> decrease in BP

Coagulation

Biological amplification system with step by step activation of plasma proteins. Results in lrge thrombin production which forms haemostats plug (clot) of fibrin + platelets. 1. Initiation - Baseline circulating clotting factor diffuse in smalla mounts into interstitium - Endothelial damage allows FVII, II, X to migrate & contact TF on macrophages - Activated FVIIa now activates FX -> FII to produce thrombin (IIa) 2. Amplification Endothelial damage allows further factors (vWF, VIII) & platelets through so platelets get activated & this creates large amount of thrombin. 3. Propogation TF & VIIa activate IXa -> large amount of thrombin produced.

Angiotensin R Blockers - Losartan -Candesartan

Block angiotenisin receptors on blood vessels 1. Dilate smooth muscle 2. Down regulate sympathetic response 3. Block aldosterone secretion 4. Inhibit CV remodelling Benefit of no cough as bradykinin doesn't build up

Factors that will decrease uterine blood flow

Blood flow = perfusion pressure / resistance Perfusion determined by arterial & venous Arterial pressure - decreased by Arteriolar dilation eg spinal Hypotension eg haemorrhage Supine position Pump dysfxn Venous pressure increased by Aortocaval cpmpression Uterine contractions Increase uterine tine (oxytocinin) Valsalva maenouvres - expulsion effects Seizures Increase in resistance Catecholamines caused by stress or exogenously adrenaline, phenylephrine, ephedrine Compression of arteries by contraction

Auto regulation of renal blood flow

Blood flow ~ 1250mL/min Regulated between 80 & 180mmHg for functional not metabolic demands. Between these values GFR can be maintained at ~ 125mL/min

Why do we avoid nitrous oxide (N2O) with pneumothorax?

Blood: gas partition coefficient is higher than nitrogen so more nitrous can leave the blood and enters the body cavity than nitrogen can be resorbed. It will double size of pneumothorax in 10 mins. Also avoid for - intracranial surgery - Eye surgery - intestinal gas - Pulm blens - Middle ear surgery

Neonatal resuscitation. Meconium present. Neonate comes out floppy. What is most appropriate action? a) Dry and stimulate b) CPR c) Suction

C

Establishing Medullary Conc Gradient

Body has no active pumps in which to control water uptake so we must use osmosis for water movement. This requires est of gradients for water to move down. Kidney reabsorb water by creating a hyperosmolar medullary interstitium which attracts water from the renal tubule to intertstitium. In order for this to function 1, The medulla must be made hyperosmolar 2. This gradient must be maintained by removing the water that has been reabsorbed but keep solute trapped. The mechanims for this are the counter current mulitplier & counter current exchange system Key features - Ascending & descending loops of Henle in parallel & close proximity - Flow is in opposite direction - Vasa recta remove water

How does endothelium differ in throughout body?

Brain - No fenestrations, tight BBB Arteries - Elongated cells + prominent stress fibres to cope with shear stress Kidneys/glands/intestine - Lrge no of fenestrations Lymph nosed - High walls encourage circulation from blood to lymph nodes.

2 yo 15kg child following seizure on surgical ward. Admitted with appendicitis and perforation. 60ml/hr of ½ N. Saline 5%dextrose Na+ 119 K+ 4.5 HCO3- 19 Cl- 90 Best treatment would be A. Desmopressin B. Frusemide C. 3% normal saline D. Normal Saline E. Fluid restrict

C

A

C

A 74 year old pedestrian is hit by a car and suffers multiple rib fractures. His CXR shows an area in the left lower zone with an air fluid level abutting the left heart border and the left chest wall. This is likely to represent a. a hiatus hernia b. a subpulmonic pneumothorax c. a diaphragmatic rupture

C

A line isolation monitor protects against microshock A. only if the warning current is set at 10mA B. only if the warning current is set at 30mA C. under no circumstances D. only if the equipment used is grounded E. only if it monitors all the equipment in the region

C

ASA grading was introduced to A. predict intraop anaesthetic risk B. Predict intraop surgical and anaesthetic risk C. Standardise the physical status classification of patients D. Predict periop anaesthetic risk E. Predict periop anaesthetic and surgical risk

C

An infant is anaesthetised and ventilated using an endotracheal tube and circle breathing system with CO2 absorber. The item which causes the most resistance to breathing is the A. airway pressure limiting (APL) valve B. circuit hosing C. endotracheal tube D. heat and moisture exchange filter E. inspiratory and expiratory valves

C

An inpatient becomes hyponatraemic 48 hours post op and has a seizure. The most appropriate treatment is a. Fluid restriction b. Normal saline ?ml/hr c. Hypertonic saline d. Salt tables

C

Baby with Tracheo-oesophageal fistula found by bubbling saliva and nasogastric tube coiling on Xray. Best immediate management? A. Bag and mask ventilate B. Intubate and ventilate C. position head up, insert suction catheter in oesophagus (or to stomach?) D. Place prone, head down to allow contents to drain E. Insert gastrostomy

C

Best agent to decrease gastric volume AND increase gastric pH before semi-urgent procedure A. Omeprazole B. Cimetidine C. Ranitidine D. Sodium citrate E.

C

Carbon dioxide is the most common gas used for insufflation for laparoscopy because it A. is cheap and readily available B. is slow to be absorbed from the peritoneum and thus safer C. is not as dangerous as some other gases if inadvertently given intravenously D. provides the best surgical conditions for vision and diathermy E. will not produce any problems with gas emboli as it dissolves rapidly in blood

C

Factors associated with post-operative ulnar nerve palsy include all of the following EXCEPT A. male gender B. sternal retraction for cardiac surgery C. cardiopulmonary bypass for cardiac surgery D. internal jugular vein catheterisation E. diabetes mellitus

C

Finding on haemophilia A patient A. Female haemarthrosis B. Male haemarthrosis C. Normal PT, abnormal APTT D. Abnormal PT, normal APTT

C

Levosemendin: A. Increases contractility and myocardial oxygen consumption B. Increases SVR C. Binds to troponin C and induces a conformational change D. Increases contractility by increasing calcium influx E. Causes coronary vasodilation but NOT peripheral vasodilation

C

Required for diagnosis of Neuroleptic Malignant Syndrome A. Diaphoresis B. ↑ CK C. Rigidity D. Hypertenion E. ↑ HR

C An international consensus study of neuroleptic malignant syndrome diagnostic criteria using the Delphi method. J Clin Psychiatry. 2011 Sep Consensus was reached on the fifth round regarding the following criteria: recent dopamine antagonist exposure, or dopamine agonist withdrawal; hyperthermia; rigidity; mental status alteration; creatine kinase elevation; sympathetic nervous system lability; tachycardia plus tachypnea; and a negative work-up for other causes. The panel also reached a consensus on the relative importance of these criteria and on the following critical values for quantitative criteria: hyperthermia,>100.4°F or>38.0°C on at least 2 occasions; creatine kinase elevation, at least 4 times the upper limit of normal; blood pressure elevation,≥25% above baseline; blood pressure fluctuation,≥20 mm Hg (diastolic) or≥25 mm Hg (systolic) change within 24 hours; tachycardia,≥25% above baseline; and tachypnea,≥50% above baseline According to DSM-V Rigidity is a must have characteristic

One lung ventilation and hypoxaemia. After 100% O2 and FOB next step is: A. CPAP 5cm top lung B. CPAP 10cm top lung C. PEEP 5cm bottom lung D. CPAP 5cm top + PEEP 5cm bottom E. ?

C Ans A CAECCP prefer CPAP to non-ventilated lung first Miller is the reverse

involving: Numb tongue and impaired taste sensation post LMA anaesthesia. A. Facial Nerve B. Mandibular division of CNV C. Lingual Nerve

C Anterior 2/3rds of tongue • Somatic afferent: lingual nerve branch of V3 of the trigeminal nerve • Taste: chorda tympani branch of facial nerve (carried to the tongue by the lingual nerve) Posterior 1/3rd of tongue • Somatic afferent and taste: Glossopharyngeal nerve CN IX Motor • All intrinsic and extrinsic muscles of the tongue are supplied by the hypoglossal nerve • Except for one of the extrinsic muscles, palatoglossus, which is innervated by CN X

Stellate ganglion blockade causes a. Conjunctival injection b. Dry eyes c. Decreased axillary sweating

C Apparently stellate ganglion block can be used to treat excessive axillary sweating (hyperhidrosis) - lousianna Pain Specialists

What splitting ratio gives a 3% concentration of isoflurane a. 1/5 b. 1/9 c. 1/13 d. 1/20 e. 1/23

C Dorsh & Dorsh "The ratio of bypass gas to gas going to the vaporizing chamber is called the splitting ratio and depends on the ratio of resistances in the two pathways" http://www.anesthesia2000.com/physics/Chemistry_Physics/physics17.htm Isoflurane 3% = 1:13

FFP dose to increase fibrinogen by 1 mg/l a. 10 mls/kg b. 20 mls/kg c. 30 mls/lg d. 40 mls/kg

C Efficacy of standard dose and 30 ml/kg fresh frozen plasma in correcting laboratory parameters of haemostasis in critically ill patients. British Journal of Haematology. 2004 In this study administration of 33ml/Kg of FFP increased Fibrinogen by 1g/L "In group 2, all seven of the patients who had had low coagulation factor levels before FFP had levels above 30 IU/dl post-FFP (33 ml/kg). The median increment for group 2 for the individual coagulation factors was between 17 and 44 IU/ dl. The fibrinogen increased by a median of 1 g/l, although this was not significantly different from group 1"

Appropriate postoperative maintenance fluid in a child [can't recall situation, but something to do with head injury]: A. 3% and 1/3 NS B. 1/2 NS C. Normal Saline D. Hartmanns E. Hartmanns with glucose

C In a head injury you would not want to give excess water, if that is part of the stem.

Loading dose of IV paracetamol in x kg child? A. (homeopathic) B. (closer) C. 15mg/kg D. 20mg/kg E. 30mg/kg

C LOADING DOSES FOR PARACETAMOL 15 for IV 20 for PO 30 for PR

New national labelling standards endorsed by ANZCA. What colour should the label on a subcutaneous ketamine infusion be? A. Red B. Blue C. Beige D. Yellow E. Pink

C National Recommendations for User-applied Labelling of Injectable Medicines, Fluids and Lines Intra-arterial = Red Intravenous = Blue Epidural / Intrathecal / Regional = Yellow Subcutaneous = Beige 723 Other routes = Pink

The pain of the first stage of labour is transmitted by: A. Grey rami communicantes B. T10-L1 anterior roots C. The hypogastric plexus D. Inhibitory nerves to the internal vesical sphincter E. Parasympathetic nerves

C OpenAnaesthesia "Pain travels via sympathetic nerve fibers (going through the inferior hypogastric plexus on the way to the sympathetic chain) that originate from the T10-L1 segments of the spinal cord

A home handyman leaves his electricity turned on whilst fiddling with wires [repairing a power outlet]. He has a RCD. What happens if he touches the neutral and ground wires? A. Nothing will happen B. Receives macroshock C. Protected from macroshock by RCD D. Protected from microshock by domestic fuse E Receives microshock

C RCD will protect him from macroshock RCD will trip with a 30 mA leakage - works on principle that current flowing to and from an appliance (in live and neutral wires) is equal. Current of 100mA required to cause VF when applied to surface of body. Only 0.05 - 0.1 mA required to cause VF when applied directly to myocardium (=microshock)

A 60kg female is given 50 mg of rocuronium, she is unable to be intubated, what dose of sugamadex is required to reverse the rocuronium a. 240 b. 800 c. 960

C Reversal of shallow neuromuscular = 2 mg/kg Reversal of profound neuromuscular blockade = 4 mg/kg Immediate reversal of neuromuscular blockade = 16 mg/kg

In the Revised Trauma Score includes GCS, Blood pressure and what other parameter? a. HR b. Saturation c. Respiratory rate d. Urine output

C The Revised Trauma Score (RTS) is a physiologic scoring system, based on the initial vital signs of a patient. A lower score indicates a higher severity of injury\d\d//d3cgb598vs7bfg.cloudfront.net/images/upload-flashcards/554968/540121_m.jpg

Thoracodorsal nerve arises from? A. Upper trunk B. Middle trunk C. Posterior Cord D. Medial Cord E. Lateral Cord

C The thoracodorsal nerve is a branch of the posterior cord of the brachial plexus, and is made up of fibres from the posterior divisions of all three trunks of the brachial plexus. It derives its fibers from the sixth, seventh, and eighth cervical nerves. Anatomy for Anaesthetist - 'Branches of the Posterior Cord' "The nerve to latissimus dorsi (thoracodorsal nerve) (C6-8) arises between the upper and lower subscapular nerves. It accompanies the subscapular vessels along the posterior axillary wall and supplies latissimus dorsi"\d\d//d3cgb598vs7bfg.cloudfront.net/images/upload-flashcards/994538/1671035_m.png

A 60kg 17 year old female given 50 mg rocuronium for RSI. You can't intubate or ventilate. What is the appropriate dose of sugammadex? A. 300mg B. 600mg C. 920mg D. 1300mg

C Well actually 960mg Reversal of shallow neuromuscular = 2 mg/kg Reversal of profound neuromuscular blockade = 4 mg/kg Immediate reversal of neuromuscular blockade = 16 mg/kg

Isoflurane vaporiser giving 3%. What is the splitting ratio? A. 1:3 B. 1:9 C. 1:13 D. 1:20 E. 1:27

C Well, almost Dorsh & Dorsh "The ratio of bypass gas to gas going to the vaporizing chamber is called the splitting ratio and depends on the ratio of resistances in the two pathways" http://www.anesthesia2000.com/physics/Chemistry_Physics/physics17.htm Isoflurane 3% = 1:14

Middle-aged male with severe MS having general anaesthesia for repair of fractured ulna / radius. 10 minutes into the case you notice a tachyarrythmia with his HR 130 and BP 70. He is normally in sinus. What do you do? A. Adenosine B.Amiodarone C. Shock D. Volume E. Metaraminol

C With increasing stenosis, passive filling of LV becomes more difficult and atrial contraction becomes more important. This contribution of atrial kick increases from 15-40%. So such an arrhythmia will be poorly tolerated and needs raid reversion to SR

A 30yo Male. 5 hour operation. Arms abducted to 60 degrees. Head turned slightly to left side. Post op numb palm/thumb/index finger/middle finger and lateral half of ring finger. Numb ventral forearm. Weak finger grip. Weak elbow flexion. Most likely nerve injured? A. median nerve B. musculocutanous nerve C. upper trunk of brachial plexus D. ?brachial plexus stretch E.

C A. Median nerve doesn't supply elbow flexion (this is upper arm - brachialis (musculocutaneous), brachioradialis (radial), biceps (musculocutaneous) B. coracobrachialis, brachialis and biceps brachi but not median nerve sensory C. Think it must be to encompass m/cutaneous + median D. ?

You get a TOE on a patient with aortic stenosis. What is the finding most likely to indicate that the valve needs replacement? A: Average pressure gradient 30mmHg B: Valve area 1.2cm(squared) C: dyspnoea

C ACC/AHA 2007 guideline on perioperative cvs evaluation and care for noncardiac surgery In symptomatic aortic stenosis, elective noncardiac surgery should generally be postponed or canceled. Such patients require aortic valve replacement before elective but necessary noncardiac surgery. If the aortic stenosis is severe but asymptomatic, the surgery should be postponed or cancelled if the valve has not been evaluated within the year. On the other hand, in patients with severe aortic stenosis who refuse cardiac surgery or are otherwise not candidates for aortic valve replacement, noncardiac surgery can be performed with a mortality risk of approximately 10%. If a patient is not a candidate for valve replacement, percutaneous balloon aortic valvuloplasty may be reasonable as a bridge to surgery in hemodynamically unstable adult patients with aortic stenosis who are at high risk for aortic valve replacement surgery and may be reasonable in adult patients with aortic stenosis in whom aortic valve replacement cannot be per-formed because of serious comorbid conditions.

Off-label use of a drug refers to all of the following EXCEPT: A. Different age-group B. Different indication C. Different concentration D. Different route of administration

C Off-label use of medicines: consensus recommendations for evaluating appropriateness. MJA 2006; 185 (10): 544-548 "Examples include use in a different indication, patient age range, dose or route" Use of "Off Label" or Drugs beyond Licence in Pain Medicine. ANZCA Faculty of Pain Medicine "The term "off-label use" may pertain to an unapproved indication, route of administration, age group, or dose. The term does NOT relate to any prescribing conditions outlined by the PBS"

Carcinoid syndrome - finding on examining heart: A. Fine inspiratory crepitations B. Systolic murmur at apex C. Systolic murmur at left sternal edge D. Murmur at apex with opening snap E. Pericardial rub

C Pulmonary stenosis (90%), tricuspid insufficiency (47%), and tricuspid stenosis (42%) are encountered most commonly. OCH & CEACCP 1) Tumours that produce peptides & amines; mostly benign. Related to endocrine tissue; from embryonic gut tissue -> so can end up in lungs, pancreas, gonads, GI tract. Size of tumour correlates with survival 2) Release vascoctive substances into systemic circulation leading to carcinoid syndrome. 3) Mediators are metabolised in liver => need tumour to have non-portal drainage to get the syndrome (or liver met causing it) 4) Vasoactive substanvce - serotonin, bradykinin, histamine, substance P, prostaglandin, VIP. ASYMPTOMATIC pt - simple carcinoid disease, no anaesthetic issues. 10% only will have symptoms => carcinoid SYNDROME Carcinoid crisis - exaggeration of symptoms. anesthetic, surgery, interventional rad may present with bleeding PERI-OP Symptoms - Related to tumour (bowel obstruction, malnutrition, dehydration, anaemia, electrolytes) - Related to vasoactive substances Hx - flushing (head, neck, torso) diarrhoea -> dehydration electrolyte disturbance, hyperglycaemia bronchospasm (20%); hypotension, tachycardia, R heart failure sec to endocardial fibrosis of tricuspid + pulm (don't reach the left heart as metabolised in lung!) => ECHO **Octreotide to reduce tumour hormonal activity*** ANAESTHESIA 1. severe HYPOTENSION or HYPERTENSION so aim to avoid pain or contributors to these 2. Fluid & electrolyte shift - lrge bore IV, fluid warmers 3. Bronchospasm 4. Blood loss if liver + high r heart pressures Pre-op 1. benzo; octreotide 100mcg SC 1 hr before 2. Monitoring - art line, CVP +/- PAC 3. Induction - prevent pressor response to tube Maintenance 1. Avoid all histamine releasing drugs (case studies only) 2. Avoid catecholamines (e.g. NA -> kallikrien in tumour activated-> synthesis of bradykinin- > paradoxical further vascodilation or may get exam HTN. => In practice, cautious administration of small doses of phenylephrine has been found helpful in some patients. 3. For BP esmolol for HTN; octrotide 20mcg for hypotension; vasopressin may be useful but evidence small Post-op 1. HDU 48 hrs. May wake slowly (? serotonin related) 2. Fentanyl rather than morphine 3. May need further octreotide.

Aneurysm sugery. Propofol/remifentanil/NMDR. DOA monitoring (Entropy). MAP 70 , HR 70/min, State entropy 50, Response entropy 70. What do you do? A. ? B. Metaraminol C. Check TOF D. Nothing E. Increase TCI.

C RE of 70 compared to SE of 50 means paralysis is wearing off

Young female patient for tonsillectomy with history of bleeding tendency. Which is the most likely cause? A. Factor V Leiden B. Protein C deficiency C. Haemophilia B (Christmas disease) D. Antithrombin III deficiency E. Lupus anticoagulant

C Rest are pro-coagulant and she would clot clot clot. Genetic Bleeding disorders • Haemophilia A, B, C • von Willebrand disease • Factors II, V, VII, X deficiency (common pathway proteins) • Factor XIII and fibrinogen deficiency Hypercoagulable diseases • A ntithrombin III deficiency • Protein C and S deficiency Acquired Prohaemorrhagic • L iver diseases • Drugs: vitamin K deficiency, warfarin, heparin • Haemodilution and massive transfusion • Disseminated intravascular coagulation • Hyperfibrinolysis • Venom-induced coagulopathy Prothrombotic • Heparin-induced thrombocytopenia • Antiphospholipid antibody syndrome ('lupus anticoagulant') • Microvascular thrombosis: thrombotic microangiopathy, coumarin-induced skin necrosis

Stellate ganglion block is associated with all EXCEPT: A. Ptosis B. Miosis C. Sweating D. Facial flushing E. Nasal stuffiness

C Stellate Ganglion produces Horners syndrome * Ptosis * Miosis * Anhidrosis In addition, several other eye signs are present: * Conjunctival injection * Lacrimation

You intubate a young male patient for a left thoracotomy with a 39FG Robertshaw tube. When you inflate both cuffs and ventilate the bronchial lumen you get left sided ventilation. When you attempt to ventilate the tracheal lumen the pressures are very high and you get no air entry. Yet when you deflate BRONCHIAL cuff you can ventilate BOTH left and right lungs through the tracheal lumen. The most appropriate step to take next is: A. Change to a 41FG tube B. Change to a 37FG tube C. Deflate both cuffs and insert further cm and recheck D. Deflate both cuffs and withdraw a few cm and recheck E. Pull ETT out and start again.

C bronchial cuff in the trachea

Regarding PS9 safe provision of anaesthesia for Colonoscopy: A. Medical Practitioner to providing sedation with a skilled assistant who is not assisting the proceduralist. B. Medical practitioner alone C. Specialist Anaesthetist D. Skilled nurse with airway experience E. Skilled bogan

C for "Anaesthesia", not necessary for "sedation" or "analgesia" ?? A Guidelines on Sedation and/or Analgesia for Diagnostic and Interventional Medical, Dental or Surgical Procedures (PS9 2010) "5.1 - Except for very light conscious sedation and/or analgesic techniques such as inhaled nitrous oxide or low dose oral sedation, there must be a minimum of three appropriately trained staff present: the proceduralist, the medical or dental practitioner administering sedation and monitoring the patient, and at least one additional staff member to provide assistance to the proceduralist and/or the practitioner providing sedation as required"

The average expected depth of insertion of an oral endotracheal tube, from the lip, in a normal newborn infant is A. 7.5 cm B. 8.5 cm C. 9.5 cm D. 10.5 cm E. 11.5 cm

C weight + 6

Which is true of Eaton-Lambert syndrome that differentiates it from Myasthenia Gravis A. Immune antibodies against post-synaptic ion channels B. Associated with thymoma C. Repeated exercise causes weakness to initially improve D. Good response to edrophonium E. Resistant to non-depolarizing muscle relaxants

C A: It is antibodies to voltage gated ca channels pre-synaptic B: associated with small cell lung cancer C: Repeated exercise improves muscle strength D: poor response to anticholinergics E: It is sensitive to both depolarising and non-depolarising blockers

Hypotension post propofol induction in elderly patient. More pronounced than in younger patient. Reason ? A. Concentric LVH associated with ageing and therefore preload dependent B. Because of increased lean body mass C. Decreased cardiac output with ageing D. Increased sensiticity to all anaesthetic agents, thus relative overdose is common E. Decreased liver blood flow with ageing, decrease drug clearance and increased drug concentration

C (kate, uncertain of best answer) Main issue is autonomic impairment - decreased baroreceptor response. iN addition impair cardiac & smooth muscle contractility (except etomidate). In addition contracted volume with reduced protein binding leads to higher free drug concentration; prolonged arm-brain circulation means drugs should be given slowly; reduced beta receptor response. Reduced heaptic clearance - slower recovery. *

A 60kg female is given 50 mg of rocuronium, she is unable to be intubated, what dose of sugamadex is required to reverse the rocuronium a. 240 b. 800 c. 960

C - 16mg/kg Reversal of shallow neuromuscular = 2 mg/kg Reversal of profound neuromuscular blockade = 4 mg/kg Immediate reversal of neuromuscular blockade = 16 mg/kg

With regard to Digoxin toxicity which of the following is NOT a feature? a. ventricular bigeminy b. sinus arrest c. atrial flutter d. atrial tachycardia with variable block

C - Atrial Flutter Clinical Features • GIT: Nausea, vomiting, anorexia, diarrhoea • Visual: Blurred vision, yellow/green discolouration, haloes • CVS: Palpitations, syncope, dyspnoea • CNS: Confusion, dizziness, delirium, fatigue Electrocardiographic Features • Digoxin can cause a multitude of dysrhythmias, due to increased automaticity (increased intracellular calcium) and decreased AV conduction (increased vagal effects at the AV node) • The classic dysrhythmia associated with digoxin toxicity is the combination of a supraventricular tachycardia (due to increased automaticity) with a slow ventricular response (due to decreased AV conduction), e.g. 'atrial tachycardia with block'. Other arrhythmias associated with digoxin toxicity are: • Frequent PVCs (the most common abnormality), including ventricular bigeminy and trigeminy • Sinus bradycardia or slow AF • Any type of AV block (1st degree, 2nd degree & 3rd degree) • Regularised AF = AF with complete heart block and a junctional or ventricular escape rhythm • Ventricular tachycardia, including polymorphic and bidirectional VT

Plasma Proteins Function

C - Coagulation O- Oncotic T - TRansport E - Enzymatic (pseudocholinesterase) I- Immune M- Metabolism

Which of the following does NOT occur following bilateral lung transplant? A. Impaired mucociliary clearance B. Impaired lymphatic drainage C. Impaired HPV D. the response to increased pCO2 is lost

C - Hypoxic vaso constriction is preserved Anesthetic Challenges in Patients After Lung Transplantation. The Internet Journal of Anesthesiology "Hypoxic pulmonary vasoconstriction is intact in the pulmonary allograft so during an episode of rejection, pulmonary blood flow may be directed away from the transplanted lung"

Indicative of severe Aortic Stenosis? A. Palpitations B. Fatigue C. Paroxysmal Nocturnal Dyspnoea D. Angina E. Syncope

C - PND

Required for diagnosis of Neuroleptic Malignant Syndrome A. Diaphoresis B. ↑ CK C. Rigidity D. Hypertenion E. ↑ HR

C - Rigidity

What is NOT a disadvantage of drawover vaporizer? A. Basic temperature compensation B. Basic flow compensation C. Not able to use sevoflurane

C - Sevo can be used in draw over vaporisers VARIABLE FLOW vaporizers (split the fresh gas flow) 1. Plenum - Higher internal resistance requiring gas flow above atmospheric pressure - As fresh gas flow increases it becomes more diff to achieve full saturation -> plenum use wick's to increase surface area then baffles to direct flow into vapourising chamber - But flow splitting valve has more resistance at low flows - Accurate conc over range of 0.25 - 15L/min 2. Drawover - Use fresh gas flow at atmospheric pressure driven by patient's efforts -> so can be portable & used where compressed gas unavailable. - May contain wick's but less effective as design must keep resistance to a minimum - Any flow splitting valve generates resistance and this is more significant at low flow rates -> thus more gas bypasses the vapourizing chamber -> draw over are less accurate at high or low gas flows. MEASURED FLOW Vapourizer (sep stream of vapor carrying gas which is added to fresh gas flow) - Ohmeda Datec for desflurane -

When stimulating the ulnar nerve with a nerve stimulator, which muscle do you see twitch? A. opponens abducens B. abductor pollicis brevis C. adductor pollicis D. extensor pollicis E. flexor pollicis brevis

C - adductor pollicis

When stimulating the ulnar nerve with a nerve stimulator, which muscle do you see twitch? A. opponens abducens B. abductor pollicis brevis C. adductor pollicis brevis D. extensor pollicis E. flexor pollicis brevis

C - adductor pollicis * OCH

Which is a specific PDE inhibitor? A. Theophylline B. Dipyridimole C. Milrinone

C - milrinone Theophylline = methylxanthine, nonselective phosphodiesterase inhibitor Dipyridimole = Can find mentions of PDE 5 PDE 10 and PDE 11 Milrinone = phosphodiesterase-3 inhibitor

Middle-aged male with severe MS having general anaesthesia for repair of fractured ulna / radius. 10 minutes into the case you notice a tachyarrythmia with his HR 130 and BP 70. He is normally in sinus. What do you do? A. Adenosine B.Amiodarone C. Shock D. Volume E. Metaraminol

C - synchronised DC shock As per ALS guidelines for haemodynamically compromised tacharryhthmias

A child with intra-operative blood loss. A cardiac arrest is most likely because of A. A delay in delivery of blood from the blood bank B. Inadequate intravenous access C. Underestimated intra-operative blood loss D. Underestimated pre-operative hypovolaemia E. Complication of transfusion

C - underestimated loss

What percentage of primiparous women experience a headache in the first week following delivery? a. 3-5% b. 5-15% c. 15-40% d. 40-65% e. 65-85%

C 39% of woman suffer from headache post partum, usually about day 2. Only ~4-5% PDPH. Risk for PDPH - known inadvertent dural puncture - prev headache hx -multiparity -increasing age CEACCP - Postpartum headache: diagnosis and management 2011 "Postpartum headache is described as a complaint of headache and neck or shoulder pain in the first 6 weeks after delivery.1 It is one of the most common symptoms with up to 39% of parturients experiencing headache in the first postpartum week" Other question - 75% of PDPH will resolve if untreate

Patient with aortic stenosis, the signs indicate poor prognosis A. Palpitation B. Radiation to carotid arteries C. Paroxysmal nocturnal dyspnoea D. Angina E. Syncope

C PND Survival ~ 2 yrs with dyspnea or other manifestation of CHD

Exponential decline / definition of time constant (with various options) A. time for exponential process to reach log(e) of its initial value B. Time until exponential process reaches zero C. Time to reach 37% of initial value D. Time to reach half if its initial value E. 69% of half life

C Time constant (Tau ) is used to describe the rate of change of an exponential process,; it's the time at which the process would have been completed if initial rate of change had continued, eg flow of air into a lung unit. Process is 95% completed after 3 time constants. 1 time constant 63% complete 2 86% 3 95%. The half life is ln2 x time constant. *

All of the following may be associated with ulcerative colitis EXCEPT A. cirrhosis B. iritis C. psoriasis D. arthritis E. sclerosing cholangitis

C psoriasis

Patient for pneumonectomy. Pre op FEV1 2.4. (Predicted 4.5L) FVC given as well. For R lower lobectomy. Postoperative predicted FEV1 ? A. 1.3 B. 1.5 C. 1.7 D. 1.9 E. 2.2

C. CEACCP article (2006), we can calculate a predicted post-op FEV1 (ppoFEV1) by calculating how many broncho-pulmonary segments of lung will remain after surgery. The starting number varies ...19 lung segments in total - 10 on the right (RUL=3, RML=2, RLL=5), and 9 on the left (LUL=3, Lingula=2, LLL=4). Given that the right lung is physically and functionally larger than the left this makes sense. Therefore for a right lower lobectomy we will be removing 5 segments, and therefore be left with 14 segments. The ppoFEV1 will be 14/19 x 2.4L = 0.7368 x 2.4L = 1.77L.

A 50 y/o male diabetic admitted to intensive care with pneumonia. Intubated and ventilated. Extensive results given. BP 80/-, HR 120, CVP 4, PCWP 6, SvO2 69% PaO2 80, BE -4 pH 7.2. Management: A. Blood transfusion B. Bicarbonate infusion C. Fluid resuscitation D. Adrenaline infusion E. Insulin infusion

C. Fluids. Low CVP, hypotensive, tachycardic, low PCWP. Acidosis could be diabetic or lactate . No reason for Blood unless low Hb. Bicarb contraindicated in DKA as need to breathe off excess CO2 Adrenaline - if non-responsive to fluid Insulin - depends if hyperglycaemic

Cause of anaemia post partial gastrectomy? A. folate def secondary to steatorrhoea B. ongoing haemorrhage C. malabsorption of iron D. vit B12 def due to loss of intrinsic factor E. folate def due to lack of appetite

C. Malabsorption of iron Complications of post-gastrectomy syndrome include anemia as a result of vitamin B12 or iron malabsorption and osteoporosis. These problems generally occur months or even years after gastric surgery. Vitamin B12 malabsorption occurs when a protein known as intrinsic factor is either not produced by the stomach (this is a condition called pernicious anemia) or when the proximal stomach is resected (the portion of the stomach that produces intrinsic factor). In either case, that absence of intrinsic factor leads to the poor absorption of vitamin B12. Under normal circumstances intrinsic factor binds to vitamin B12 and assists with the absorption of this vitamin in the lower portion of the small bowel. When vitamin B12 is poorly absorbed, anemia and, in some cases, poor nerve function can occur. This generally does not happen for several years because vitamin B12 is stored in large amounts in the liver. Iron deficiency anemia develops because removal of the stomach often leads to a marked decrease in the production of gastric acid. This acid is necessary to convert dietary iron to a form that is more readily absorbed in the duodenum. Anemia usually does not occur for a few years after gastric surgery because iron is stored in moderately large amounts in the bone marrow, where red blood cells are produced......others suggest multifactorial decrease in iron absorption.....more common in a 2012 observational study Cleveland Clinic & google search *

You are on a humanitarian aid mission in the developing world. Drawover vaporiser apparatus described being used. Given 400 mm tubing, OMV or diamedica vaporiser, 200mm tubing attached to self-inflating bag. Which other ONE piece of equipment is ESSENTIAL to make this system functional? A. Halothane B. In-line Waters' Cannister [1] C. Non-rebreathing valve D. Oxygen source E. Ventilator

C. Need non rebreathing valve. Draw over can be used with spontaneous breathing, does not need compressed gases is transportable. Setup: Non rebreathing valve; self inflating bag; low resistance vapouriser, means of giving supplemental oxygen (T piece, length of tubing to act as oxygen reservoir) & source of O2 eg cylinder or concentrator. Air can also be used. *

A 7 kg Infant with tetralogy of fallot, post BT-shunt. Definitive repair at later date. Paralysed and ventilated. Sats 85% baseline, now 70%, best treatment: A. Increase FiO2 from 50 - 100% B. Esmolol 70 mcg C. Phenylephrine 35 mcg D. Morphine 1 mg E. 1/2 NS with 2.5% dex 70 mls

C. Phenylephrine - will increase SVR and L pressures, reducing R-L shunt. (increase FiO2 won't change if big shunt) http://lifeinthefastlane.com/2010/04/cardiovascular-curveball-009/ Frank Shann's dose recommendation for phenylephrine is 2-10 mcg/kg as a bolus, which would fit nicely with C (5mcg/kg).

A patient is having an electrophysiological study and ablation for atrial fibrillation. Suddenly the blood pressure drops to 76/38 mmHg, with the heart rate at 110 in sinus rhythm. What is the best investigation to confirm the cause of hypotension? A. Troponin B. ST-segment elevation C. Transoesophageal echocardiography D. Coronary Angiogram E. Electrocardiogram F. MRI

C. Transoesophageal echocardiography

The STRONGEST stimulus for ADH secretion: A. High serum osmolality B. Low serum osmolality C. Hypovolaemia D. High serum Na E.

C. Volume wins - sensitive to 1-2% change in osmolarity or 10% change in volume, but volume response is STRONGER Osmolality sensors are neuron sin the anterior hypothalamus that stimulate thirst & vasopressin stored in granules in post pituitary. *

Physiological changes in Non REM sleep

CNS - parasympathetic dominates, decreased LOC CVS - Decrease HR/BP/CO/SVR Resp - Regular breathing pattern Decrease response to hypercarbia, hypoxia Increase in airways resistance Decrease FRC; Decrease MV MSK - Decrease muscle tone Temp - shivering threshold decreased Circadian temp nadir 3 am Renal - Decreased GFR so small volume, conc urine. Increase in ADH Metabolism - Decrease cortisol (early hr trough ,peaks after waking) Decrease metabolic rate.

TEG - thromboelastogram components

CT - Clotting Time/R time: Time for initial clot formation. From start to 1mm height. Formation of fibrin. Affected by low platelets, low factors, anticoagulant CFT - Clot Formation time: From start until 2mm height. Affected by low fibrinogen, platelets, low FXIII alpha angle - From 1mm point. Affected by low fibrinogen, low/poor platelets, clotting factors MCF - Max Clot Firmness. Widest point. Affected by quality + amount of platelets & fibrinogen. Amount of Clot Lysis at 30 min and 60 min - Accelerated with agents that promote clot lysis.

Morphine Effects

CVS - Sympatholyticwith decrease in SVr -> decrease in BP (this is also histamine mediated) decrease HR at high doses due to action on vagal nuclei. (No direct myocardial depressant effect which pethidine has) CNS - sedative, analgesic at Mu R, resp depresssion, euphoria, N&V, pruritus, anxiolytic, hallucinations Resp - Decrease RR, pulm vasodilator; deceased response to hypoxia + hypercarbia, anti-tussive (central effect on cough centres) GI tract - Decreased motility -> Ileus, N&V, decreased transit time -> constipation, decreaed gastric emptying. Kidneys - excretion will be limited in renal failure so metabolites accumulate. Increase bladder tone & sphincter tone Endocrine - Inhibits ACTH release. Increase in ADH. Immune - Depressant Met - Histamine release Diaphoresis

Nifedipine

Ca channel blocker dihydropyridine Uses - Angina HTN - incl pregnancy Coronary vasospasm during angiography Raynauds Vasodilates arteries by inhibiting Ca channels. Has small negative inotropic effect Dose 10-20mg q8H Will cause flushing, eye pain, dizziness.

Digoxin

Cardiac glycoside used for treatment of - AF/A Flutter + CHF Has direct & indirect actions which increase inotropy & slow conduction. A slower HR in AF allows for more coronary blood flow + greater ventricular filling so this improves cardiac output. Less useful in peri-op period due to high sympathetic activity present. Direct action - Blocks Na/K ATPase which means Na is exchanged for Ca++ instead which increase excitability & inotropy. Indirect - Causes release of Ach which stimulate muscarinic receptors & slow conduction (PR interval is increased). Tablets 62.5mcg - 250mcg with IV 100 - 200mcg/mL Freq seen ECG changes - Prolong PR; ST depression; T wave flatenning; shorter QT interval May cause arrythymias, heart block, junctional rhythm. Anorexia, nausea, vomiting, diarrhowa, altered colour vision, headache, gynaecomastia Toxicity - > 2mcg/mL If > 20mcg/mL use F antibodies which have strong affinity for digoxin. Treat toxicity with phenytoin. 30% protein bound. Has large Vd. Dialysis NOT helpful Elimination reduced in renal failure Amiodarone & ACE-i increase levels

Tumour Lysis Syndrome

Caused by the breakdown of cells and release of intracellular products. Usually seen in contact of oncology treatment. Derrangements - Hyperkalemia. - Hyperphosphatemia. Like potassium, phosphates are also predominantly intracellular. Hyperphosphatemia causes acute kidney failure in tumor lysis syndrome, because of deposition of calcium phosphate crystals in the kidney parenchyma. - Hypocalcemia. Because of the hyperphosphatemia, calcium is precipitated to form calcium phosphate, leading to hypocalcemia. Symptoms of hypocalcemia include tetany, sudden mental incapacity, including emotional lability, myopathy -Hyperuricaemia - breakdonw of DNA - > breakdown of purines to urate NO sodium issue of note

TEG what is it, what advantages

Changes in viscoelastic properties over time of clot measured. Blood added to vial, pin suspended in good by torsion wire; pin rotated in blood (with or without additive like citrate). The rotation of the pin will be affected by the resistance of blood which in turn will be determined by clot formation. Increased clot, increased resistance. Advantages - Real time use - Point of care - Functional assay - Blood products can be titrated to results - Less blood product waste - Less risk of TACO & other transfusion reactions if less product used - Repeated tests allow real time measurement of interventions

Cardiovascular changes that occur with birth

Circulation of heart & lungs in paralllel with blood delivery & oxygen from low resistance placenta -> high resistant SVR with heart & lungs in series 1. Lung expansion & increase PO2 cause pulmonary vasoconstriction (mechanical forces, loss of HPV) 2. Cord clamping - Removes low resistance placenta, SVR increases - > BP increases 3. Closure of foramen oval - LA pressure > RA pressure 4. Closure of ductus arteriosus - Increase in oxygen partial pressure, increase in aortic pressure & decrease in prostaglandins - Initially physiological for 2/52 then mechanical - Transitional circulation is thus reversible with stress, hypoxia, acidosis 5. Closure of ductus venosum 6. Changes in regional blood flow, increase to lungs, kidneys, skin.

How do you classify shock?

Class I- IV based on volume of blood loss and other parameters such as RR, HR, BP, CNS, urine output + response to fluid Class I Loss of 15% Fluid responsive; HR < 100, RR 14-20 Class II Loss of 15-30% Fluid responsive HR > 100, RR 20-30 Anxious Class III 30-40% HR > 120, RR 30-40 Transiently fluid responsive Confused < 0.5mL/kg/hr of urine Class IV > 40% blood volume lost; will not be fluid respoinsive, HR > 140, RR > 40 with unrecordable BP and lethary anuria.

Vaughan Williams Classification

Classification of anti-arrythmics based on effect on electrical currents. Class I na channel blockers a. Prolong refractory period - procainamide b Shorten refractory period - lignocaine, phenytoin. c. No effect on refractory period flecainide Class II Beta blockers - Slow AV conduction Class III - K+ channel blockers Slow AV conduction + prolong refractory period (prolong QT) Amiodarone; Sotalol Class IV - Calcium channel blockers Verapamil, diltiazema, Prolong refractory period Problems with classification is that it does not include all antiarryhtmics e.g. adenosine/digoxin and drugs may have more than one class effect e.g. amiodarone. Used on isolated muscle studies. Effects on diseased tissue may not be the same. Lignocaine can be used in VT Phenytoin for digoxin toxicity Torsades use Mg

Factor VIIa & recombinant FVIIa = rVIIa (novoseven)

Clotting factor in the coagulation cascade which is activated by tissue factor & in turn activates F9 & FX. VIIa - plasma derived concentrate or recombinant - (derived from hamster cells). Recombinant FVIIa activates platelets independent of TF so factor Xa increase -> thrombin -> fibrin. Dose 100mg/kg Indications - Inhbitors of FVIII or FIX or FVII def

Responses of neonate to temp changes

Cold - Crying skin vasoconstricton Non shivering thermogenesis (brown fat) - beta 3 R; locations incl perinephric, scapular area Shivering via increased muscle activity Warm - crying, removing coverings Vasodilation sweating

Flumazenil

Competitive anatagonist at benzodiazepine modulating site on GABA a receptors. Use for benzo overdose Dose 100mcg up to 1 mg Effect in 1 min, last 15-40min so may need infusion SEs - HTN, anxiety, N&V, flushing, convulsions in epileptics

162. What is the least appropriate measure to minimize the use of homologous blood transfusion? a. isovolumetric haemodilution b. intraoperative cell salvage c. autologous blood banking d. lowering the 'trigger' for blood product administration

D

A lady with a Fontan's circulation for tricuspid atresia presents for caesarian section. What is the best way of maintaining her cardiac output? A. Trendelenburg B. Epidural contraindicated C. Allow pCO2 to rise to 50 to vasodilate her D. Short inspiratory time E. Allow hypovolaemia

D

An infant born at 32 weeks gestational age comes at 6 weeks for elective bilateral inguinal hernia repair. The parents expect to take him home that day. What do you tell them? A. He cannot have surgery until he is 3 months old B. They can take him home that day C. They can take him home with apnoea monitoring overnight D. He needs to stay in hospital for apnoea monitoring

D

Circuit disconnection during spontaneous breathing anaesthesia A. will be reliably detected by a fall in end-tidal carbon dioxide concentration B. will be detected early by the low inspired oxygen alarm C. will be most reliably detected by spirometry with minute volume alarms D. may be detected by an unexpected drop in end-tidal volatile anaesthetic agent concentration E. can be prevented by using new, single-use tubing

D

During prolonged trendelenburg positioning there is: A. No change in ICP B. No change in IOP C. Increased pulmonary compliance D. Increased myocardial work E. No increased pulmonary venous pressures

D

ECG recording devices have a high internal impedance. This is because a. it protects against microshock b. the display device has a high impedance c. it protects against interference from diathermy d. it limits interference from changes in thoracic impedance e. something about an earth loop

D

Endocarditis prophylaxis A Bicuspid valve B Congenital repair > 12 months ago C Rheumatic heart valve D Uncorrected cyanotic heart disease E MVP + ?MR

D

Fat: blood coefficient- N2O, Desflurane, Sevoflurane, Isoflurane A. N2O ~ D > S > I B. N2O > D > S > I C. D > N2O > S > I D. N2O > D > S ~ I E . D > N2O > I > S

D

Head Trauma patient with unilateral dialated pupil, whats the diagnosis ? A.Global injury B.Optic nerve injury C.Horners syndrome D.Transtentorial herniation E.

D

In a neonate the main resistance in a circle system with CO2 absorber A. APL valve B. Expiratory and inspiratory unidirectional valves C. tubing D. ETT E. HME filter

D

Left bundle branch block (LBBB) is associated with: A. B. C. D. Relative contraindication to CVL insertion E.

D

Long duration of surgery, arms stretched out, head turned 30 degrees to right. On waking patient has a neurological deficit. Sensory loss over ventral lateral palm and 3 fingers, some weakness of the hand, weakness of the wrist, some paraesthesia of the forearm and weak elbow flexions. Most likely injury is A. Median nerve B. Ulnar nerve C. C5 nerve root D. Upper cervical trunk E. Musculocutaneous

D

Myotome of C6-7 A. Shoulder flexion B. Shoulder extension C. Elbow flexion D. Wrist flexion and extension E. Finger flexion and extension

D

New onset atrial fibrillation in a 10 week pregnant lady. BP 150/90, HR 160, SaO2 92%. Moderate mitral stenosis on TTE, no thrombus seen. Emergency doctor gave her anticoagulant (not specified what). Most appropriate management: A. Verapamil B. Labetalol 20mg iv to 300mg C. Amiodarone 300mg IV D. Synchronised biphasic cardioversion with 70-100 Joule E. Oral digoxin -1000mcg then 500mcg 6 hrs later

D

Serotonin syndrome has been reported following SSRI coadministration with: A. Gingko B. Garlic C. Ginger D. St John's wort E. Vallerian

D

Unequal consolidation on CXR can be caused by all except: A. Pleural effusion B. Pulmonary infarction C. Pulmonary haemorrhage D. APO E. Pneumonia

D

When administering adrenaline and atropine via ETT dose compared with IV should be a. Same dose b. Double c. Quadruple d. Six times

D

Which hormone is not released during surgery? A. Glucagon B. ACTH/cortisol C. GH D. TSH

D

Wolf Parkinson White Syndrome: a. PR interval lenghtened b. [Something about delta wave] c. DCR is less effective d. Central IV access may precipitate arrythmias

D

You are asked to see a 60-year-old male 2 days following a cervical laminectomy because he has new neurological symptoms in his right arm. The surgical team think these may be due to poor patient positioning. The sign that would most help differentiate a C8-T1 nerve root injury from an ulnar nerve injury is A. loss of sensation in the index finger B. loss of sensation in the little finger C. weakness of the abductor digiti minimi muscle D. weakness of the abductor pollicis brevis muscle E. weakness of the first dorsal interosseous muscle

D

In what proportion of people is the SA node supplied by the R coronary artery? a. 5 b. 15 c. 40 d. 60 e. 85

D 60% from RCA, 40% from LCX

In a neonate the main resistance in a circle system with CO2 absorber A. APL valve B. expiratory and inspiratory unidirectional valves C. tubing D. ETT E. HME filter

D ??? B Valves accounted for about 2/3rds of resistance in one study. http://bja.oxfordjournals.org/content/83/1/50.full.pdf

Alcoholic patient undergoes unremarkable anaesthesia for explorative laparotomy for investigation of abdominal pain. No pathology is found. However, in recovery the following electrolyte disturbances found: Na 140 K 5.0 CL 115 HCO 18 What is the most likely cause A. Acute renal failure B. Lactatic acidosis C. Methanol ingestion D. Chloride [N/saline resuscitation] E DKA

D AG = (140 +5) - (115-18) AG = 12 So mildly elevated, but the rest are all high anion gap acidosis causes

The following electrolytes are from an alcoholic 12 hours post laparotomy. What is the diagnosis? Na 141, K 4, Cl 114, HCO3 19 A. DKA B. Methanol C. Lactate D. NaCl resus

D AG = (141 + 4 ) - (114 + 19) AG= 9

Acromegaly due to excess of growth hormone. Why is it difficult to do a direct laryngoscopy? A: Distorted facial anatomy B: Macroglossia C: Glottic stenosis D: Prognathe mandible E: Arthritis of the neck

D Acromegalic Features Influencing Intubation Performance Many typical acromegalic features are suggested to cause a difficult airway in these patients. The most discussed changes are: • macroglossia, • prognathism, • enlargement and distortion of glottic structures with additional folds, and hypertrophy of laryngeal and pharyngeal soft tissue.

Thermoneutral zone in 1 month old infant ? A. 26 - 28 degrees Celcius B. 28 - 30 degrees Celcius C. 30 - 32 degrees Celcius D. 32 - 34 degrees Celcius E. 34-46 degrees celcius

D Adult = 25-28 Neonate = 32-35

What is the average distance from the lips to carina in an average 70kg adult male? A. 21 cm B. 23 C. 25 D. 27 E. 29

D As per Miller chapter on DLT ?? C Appropriate depth of placement of oral endotracheal tube and its possible determinants in Indian adult patients. Indian J Anaesth. 2011 Sep-Oct; 55(5): 488-493 "The mean (SD) lip-carina distance, i.e., total airway length was 24.32 (1.81) cm and 21.62 (1.34) cm in males and females, respectively"

What is the maximum recommended dose of Intralipid in local anesthetic toxicity (ml/kg) a. 6 b. 8 c. 10 d. 12 e. 14

D Association of Anaesthetists Great Britain & Ireland (AAGBI) • Bolus 1.5ml/kg • Infusion 0.25ml/kg/min • Two further boluses 1.5ml/kg • Increase infusion to 0.5ml/kg/min • Apparently 12ml/kg max according to AAGBI

When performing cardioversion for new onset atrial fibrillation, what is the safe maximum time between onset and cardioversion without anticoagulation? A. 12 hours B. 24 hours C. 36 hours D. 48 hours E. 72 hours

D Cardioversion of Atrial Fibrillation for Maintenance of Sinus Rhythm: A Road to Nowhere. Circulation 2009 "The current treatment guidelines suggest it is permissible to cardiovert patients without continued anticoagulation in those for whom it is known that the duration of AF is <48 hours"

A man is working with electrical appliances at home with a residual current device. If he touches the active and the neutral (was it neutral or earth) wire he will suffer a. A microshock b. A macroshock c. Nothing happens because the fuse blows d. The RCD will protect him from macroshock

D Current normally flows along active and NEUTRAL. Touching these wires will result in macroshock with no protection, as it may be indistinguishable from normal operation. Current from active to EARTH should be protected by RCD's. http://www.anaesthesia.med.usyd.edu.au/resources/lectures/electricity.html

72. New- Modified Cormack and Lehane grade - You cannot see beyond the epiglottis and there is a little space between the epiglottis and the posterior pharyngeal wall (? remembered as epiglottis touching posterior pharyngeal wall) A. 2a B. 2b C. 3a D. 3b E. 4

D Grade I: Complete glottis visible. Grade IIa: Partial view of the glottis is visible Grade IIb: Only arytenoids or very posterior portion of vocal cords are visible. Grade III: Only epiglottis is visible, not the glottis Grade IV:Neither epiglottis nor glottis seen.

The line between the iliac crests in a neonate corresponds to: A. L2/3 B. L3/4 C. L4/5 D. L5/S1 E. S1/2

D Local and regional anaesthesia in infants. CEACCP 2004 "The intercristal line is at L5/S1 (L4 in adults), the termination of the spinal cord is at L3 (L1/2 in adults) and the termination of the dura is at S3/4 (S2 in adults)"

What is the distance from the lips to the carina in an 70kg adult male in cm a. 21 b. 23 c. 25 d. 27 e. 29

D Miller - See chapter on DLT. ? C Appropriate depth of placement of oral endotracheal tube and its possible determinants in Indian adult patients. Indian J Anaesth. 2011 Sep-Oct; 55(5): 488-493 "The mean (SD) lip-carina distance, i.e., total airway length was 24.32 (1.81) cm and 21.62 (1.34) cm in males and females, respectively"

Patient complains of numbness of the anterior third of his tongue following GA with LMA. Which nerve is involved? A. Glossopharyngeal B. Facial nerve C. Superior vagus D Mandibular n.

D More specifically the lingual nerve (branch on mandibular) Innervation of the Tongue Anterior 2/3rds of tongue • Somatic afferent: lingual nerve branch of V3 of the trigeminal nerve • Taste: chorda tympani branch of facial nerve (carried to the tongue by the lingual nerve) Posterior 1/3rd of tongue • Somatic afferent and taste: Glossopharyngeal nerve CN IX Motor • All intrinsic and extrinsic muscles of the tongue are supplied by the hypoglossal nerve • Except for one of the extrinsic muscles, palatoglossus, which is innervated by CN X

ANZCA professional documents, ketamine labels should be A. Pink B. Green C. Grey D. Yellow E. Black

D National Recommendations for User-applied Labelling of Injectable Medicines, Fluids and Lines Intra-arterial = Red Intravenous = Blue Epidural / Intrathecal / Regional = Yellow Subcutaneous = Beige 723 Other routes = Pink

National labelling standards endorsed by ANZCA. What colour should the label on a brachial plexus catheter infusion be? A. Red B. Blue C. Beige D. Yellow E. Pink

D National Recommendations for User-applied Labelling of Injectable Medicines, Fluids and Lines Intra-arterial = Red Intravenous = Blue Epidural / Intrathecal / Regional = Yellow Subcutaneous = Beige 723 Other routes = Pink

A man is working with electrical appliances at home with a residual current device. If he touches the active and the neutral (was it neutral or earth) wire he will suffer a. A microshock b. A macroshock c. Nothing happens because the fuse blows d. The RCD will protect him from macroshock

D RCD will protect him from macroshock RCD will trip with a 30 mA leakage - works on principle that current flowing to and from an appliance (in live and neutral wires) is equal. Current of 100mA required to cause VF when applied to surface of body. Only 0.05 - 0.1 mA required to cause VF when applied directly to myocardium (=microshock)

If a patient experiences parasthesia in the little finger during supraclavicular brachial plexus block, the needle is in proximity to the A. posterior cord B. middle trunk C. Ulnar nerve D. lower trunk E. medial cord

D Sensation in this area is C8distribution, supplied by the ulnar nerve, medial cord, lower trunk

Cerebral oximetry measures? A. Arterial saturation B. Mostly arterial saturation and some venous saturation C. Capillary saturation D. Mostly venous saturation and some arterial saturation E. Venous saturation

D The Official Journal of the Anesthesia Patient Safety Foundation. Spring 2009 "Cerebral oximetry differs from pulse oximetry in that tissue sampling represents primarily (70-75%) venous, and less (20-25%) arterial blood" Cerebral Oximetry: Monitoring the Brain as the Index Organ. Anesthesiology 2011 "Sco2 is weighted for approximately 70-75% venous blood"

Which type of aortic dissection is typically managed non-operatively? A. Debakey Type I B. Debakey Type II C. Stanford A D. Stanford B E. Stanford C

D eMedicine - Emergent Management of Acute Aortic Dissection Stanford classification "The Stanford classification divides dissections into 2 types, type A and type B. Type A involves the ascending aorta (DeBakey types I and II); type B does not (DeBakey type III). This system helps to delineate treatment. Usually, type A dissections require surgery, while type B dissections may be managed medically under most conditions."

In infants with congenital pyloric stenosis A. dehydration is associated with early hyponatremia B. plasma chloride levels seldom fall below 85 mmol.1-I C. renal conservation of hydrogen and potassium ions occurs D. the urine is initially alkaline, then may become acidic E. vomiting causes a loss of potassium ions

D • More common in males • Typically 3-6wks of age • Increased HCO3 load to distal tubule of kidney results in an alkaline urine initially. With Extreme K losses, H+ ions exchanged for K in kidney paradoxical acidotic urine • Main source of k loss from body is from the kidney →resulting hypokalaemia

A 75yo male with moderate aortic stenosis (valve area 1.1cm2).. Gets mild dyspnoea on exertion but otherwise asymptomatic. Needs hip replacement. A. Continue with surgery B. Beta block then continue C. Get myocardial perfusion scan D. Postpone surgery awaiting AVR E. Postpone surgery awaiting balloon valvotomy

D ACC AHA 2007 In symptomatic aortic stenosis, elective noncardiac surgery should generally be postponed or canceled. Such patients require aortic valve replacement before elective but necessary noncardiac surgery. If the aortic stenosis is severe but asymptomatic, the surgery should be postponed or cancelled if the valve has not been evaluated within the year. On the other hand, in patients with severe aortic stenosis who refuse cardiac surgery or are otherwise not candidates for aortic valve replacement, noncardiac surgery can be performed with a mortality risk of approximately 10%. If a patient is not a candidate for valve replacement, percutaneous balloon aortic valvuloplasty may be reasonable as a bridge to surgery in hemodynamically unstable adult patients with aortic stenosis who are at high risk for aortic valve replacement surgery and may be reasonable in adult patients with aortic stenosis in whom aortic valve replacement cannot be per-formed because of serious comorbid conditions.

In a rotameter the A. bobbin spins inside a tube that has parallel sides B. flow is laminar at high flow rates C. height of the bobbin is proportional to the pressure drop across the bobbin D. pressure drop across the bobbin is constant at varying flows E. resistance increases with increasing gas flow

D F = MA mass of bobbin and A is gravity both should be constant P = F/A area of the bobbin does not change

Myaesthenia gravis - features predicting need for post op ventilation EXCEPT A. Prolonged disease B. High dose Rx C. Previous respiratory crisis D. Increased sensitivity to NMB's E. bulbar dysfunction

D FVC < 2.9L Hx of resp disease Grade 3-4 MG >6 years of disease Dose > 750mg/day Pyridostigmine

At what valve area do you begin to get symptoms, at rest, with mitral stenosis? A. 4.5 cm2 B. 3.5 cm2 C. 2.5 cm2 D. 1.5 cm2 E. 1.0 cm2

D - 1.5cm^2 AHA guideline on valvular disease cited by Chang - a mitral valve area of greater than 1.5 doses not usually produce symptoms at rest. The challenge of valvular heart disease. Cleveland Clinic Journal Of Medicine, Volume 71, Number 6, June 2004 "Mitral stenosis is most commonly caused by damage to the mitral valve from rheumatic fever, after which there typically is a long period of asymptomatic progressive valve narrowing. Symptoms at rest are rare until the mitral valve area is less than 1.5 cm2"

Abnormal Q waves are NOT a feature of the ECG in A. an old myocardial infarction B. left bundle branch block C. recent transmural myocardial infarction D. digitalis toxicity E. Wolff-Parkinson-White syndrome

D - Digitalis toxicity

The cause of early mortality (early - within 30 minutes) in a pregnant women with amniotic fluid embolism is A. Bronchospasm B. Hypovolaemia C. Malignant arrhythmia D. Pulmonary hypertension E. Pulmonary oedema

D - Pulmonary HT

During prolonged trendelenburg positioning there is: A. No change in ICP B. No change in IOP C. Increased pulmonary compliance D. Increased myocardial work E. No increased pulmonary venous pressures

D - increased work

If a patient experiences parasthesia in the little finger during supraclavicular brachial plexus block, the needle is in proximity to the A. posterior cord B. middle trunk C. Ulnar nerve D. lower trunk E. medial cord

D - lower trunk. The medial cord also carries fibres for sensation of the little finger, but is further along, not in the supraclavicular region\d\d//d3cgb598vs7bfg.cloudfront.net/images/upload-flashcards/994538/1670552_m.jpg

70 year old post TKJR. On sub-cut heparin. Develops clinical DVT and platelets 40 (sounds like HITS type-II). Management A. Enoxaparin B. Fondoparinux C. Heparin by infusion D. Lepirudin E. Warfarin

D - no other direct thrombin inhibitors on list. Wikipedia - Lepirudin is an anticoagulant that functions as a direct thrombin inhibitor. Brand name: Refludan, Generic: Lepirudin Lepirudin may be used as an anticoagulant when heparins (unfractionated or low-molecular-weight) are contraindicated because of heparin-induced thrombocytopenia.

Recognised weaknesses of systematic reviews include all of the following EXCEPT A. publication bias B. duplicate publication C. study heterogeneity D. inclusion of outdated studies E. systematic review author bias

D - this is part of a literature search & part of a systemic review and not a weakenss in my opinin (kate)

Predictive factors for mortality in elderly patient (except): A. Aortic stenosis B. Diabetes mellitus C. Elevated Creatinine D. Cognitive dysfunction E. Type of surgery

D cognitive dysfunction Ref Minimising perioperative adverse events in the elderly BJA 2001 All are mentioned except cognitive dysfunction *

CTG [pictured] demonstrating late decelerations. Most likely cause? A. Fetal asphyxia B. Head compression C. Cord compression D. Uteroplacental insufficiency

D encompasses C with this answer NICE indications 1. Meconium staining of liquor 2. Maternal pyrexia - defined as 38.0 °C or 37.5 °C on two occasions two hours apart 3. The use of oxytocin for labour augmentation 4. Fresh bleeding developing in labour 5. At the woman's request 6. Abnormal FHR detected during intermittent auscultation: • FHR <110 beats per minute (bpm) • FHR >160 bpm • Any decelerations after a contraction 7. regional anaesthesia/analgesia. Continuous electronic fetal monitoring is recommended for at least 30 minutes during establishment of regional analgesia and after administration of a further bolus of local anaesthetic agent. In most UK centres, continuous CTG monitoring is performed after the insertion of a labour epidural. A combination of several abnormalities increases the likelihood of fetal distress. Suspicious or abnormal features include: • Baseline FHR outside normal range of 110 - 160 bpm • Baseline variability <5 bpm • Reduced or absent accelerations • Presence of decelerations HR variability Normal beat to beat changes in FHR; nomral tn 5-15 bpm. Fetal hypoxia may cause absent, increased or decreased variability. Other causes of decreased variability include: normal fetal sleep-wake pattern, prematurity and following maternal administration of certain drugs including opioids. Accelerations: Transient increases in FHR, defined as an increase in FHR >15 bpm for more than 15 seconds. When accelerations are present, the CTG is said to be reactive. Accelerations are often associated with fetal activity and are considered an indication that the fetus is healthy. Decelerations: Periodic, transient decreases in FHR, usually associated with uterine contractions. 4 types by shape and timing in relation to uterine contractions. • Early - Normal finding in the second stage of labour due to compression of fetal head during contraction. Occurs at the onset of the contraction baseline FHR recovers by the end of the contraction. They are not associated with a poor fetal outcome. Onset & depth mirrors contraction. • Late: Occur after peak of uterine contraction. Shape still uniform. Associated with decreased uterine blood flow and can occur as a result of: - Hypoxia - Placental abruption - Cord compression / prolapse - Excessive uterine activity - Maternal hypotension / hypovolaemia • Variable - describe FHR decelerations that are both variable in timing and size. They may be accompanied by increased variability of the FHR. They are caused by compression of the umbilical cord and may reflect fetal hypoxia. • Prolonged Prolonged decelerations/ bradycardia A deceleration with a reduction in FHR of greater than 30 bpm that lasts for at least 2 minutes. Caused by a decrease in O2 transfer to the fetus causes include: -Maternal hypotension - Umbilical cord compression - Uterine hypertonia

Stellate ganglion block. Needle entry next to SCM muscle at C6. Which direction to advance needle ? A. C3 B. C4 C. C5 D. C6 E. C7

D. C6 Cervical sympathetic trunk, cont of thoracic sympathetic trunk. Block may be used in pain syndromes amongst other things. Although the ganglion lies at the level of the C7 vertebral body, the needle is inserted at the level of C6 to avoid the piercing the pleura. *

Von Hippel-Lindau disease is associated with: A. increased risk of malignant hyperthermia B. meningiomas C. peripheral neuropathy D. pheochromocytomas E. poor dentition

D. Pheo "Management of anesthesia in patients with von Hippel-Lindau disease must consider the possible presence of pheochromocytomas" (Stoelting) von Hippel-Lindau disease (VHL) is a rare, genetic multi-system disorder characterized by the abnormal growth of tumors in certain parts of the body (angiomatosis). The tumors of the central nervous system (CNS) are benign and are comprised of a nest of blood vessels and are called hemangioblastomas (or angiomas in the eye). Hemangioblastomas may develop in the brain, the retina of the eyes, and other areas of the nervous system. Other types of tumors develop in the adrenal glands, the kidneys, or the pancreas. Symptoms of VHL vary among patients and depend on the size and location of the tumors. Symptoms may include headaches, problems with balance and walking, dizziness, weakness of the limbs, vision problems, and high blood pressure. Cysts (fluid-filled sacs) and/or tumors (benign or cancerous) may develop around the hemangioblastomas and cause the symptoms listed above. Individuals with VHL are also at a higher risk than normal for certain types of cancer, especially kidney cancer. Inheritance = Familial Autosomal dominant Features= Retinal angiomas, Haemangioblastomas, Cerebellar and visceral tumours (usually benign but can cause pressure effects) Associations = An increased incidence of Phaeochromocytoma - apparently 20%, Renal cysts, Renal cell carcinoma Anaesthesia= Treat hypertension occurring with phaeochromocytoma, Haemangioblastoma of spinal cord may limit use of spinal although epidural has been used for LSCS, Exagerrated hypertension with surgical stimulation or laryngoscopy = Treat with β blockers and/or SNP From recollection, Phaeos are associated with the Diseases of Von- Ie. Von Recklinghausen and Von Hippel Lindau.

Best measure of anticoagulant effect of Dabigatran? APTT Dilute thrombin time prothrombin time bleeding time TEG

DILUTE THROMBIN TIME APTT -raised at high dose PT - Not useful Dilute thrombin time - measures activity of Bleeding Time TEG - this prob won't give only dabigatran effect....google search seems to indicate tests being done to find TEG best assay.

How much air is the maximum to that should be used to inflate a 5 LMA classic cuff a. 15 b. 20 c. 25 d. 40 e. 45

D\d\d//d3cgb598vs7bfg.cloudfront.net/images/upload-flashcards/554968/540120_m.jpg

Acid

Daily H+ turnover 15 000 mmol Kidney can ~ 100mmol What contributes to acid in body - Volatile - CO2 (75%) - Metabollic/fixed - sulphuric, phosphoric, lactate ketoacids. Body maintains acid bas homeostasis by 1. Buffering - minimise changes 2. Compensation- aim to restor HCO3/CO2 ratio 3. Correction -corrects underlying prob

Porphyria

Defect in enzymes involved in harm synthesis leading to accumulation of precurors that are oxidised into porphyrins. Different types, autosomal dominant but varied expression. Can present as porphyria crisis with abdominal pain, pareripheral neuropathy, autonomic dysfunction, vomiting, cranial nerve palsies, coma, convulsion, pyrexa. Crisis caused by: stress, drugs, infection, alcohol, starvation, dehydration. Techniques Pre-medication Minimise fasting Avoid electrolyte abnormalities Regional - avoid in a crisis due to neuropathy Neuromuscular drugs safe Monitoring imp due to autonominc lability Treat - Withdraw precipitating drugs Decrease haem ALA synthestase (enzyme responsible for harm synthesis) with Haem arginine Treat infection ,dehydration, electrolytes

What are causes of post-op hypoxaemia

Demand outstripping supply; Arterial PaO less than normal of 75- 100mmHg (noting elderly norm is lower) Decreased supply - Inadequate oxygen delivery patient or delivery. - Hypoventilation - sedation, hypercarbia causing narcosis, hypocapnia from ihyperventilation intra-op causing decreased rest rate, central causes - opioids. - Airway - obstruction - oedmea, laryngospasm, - Lungs - consolidation, collapse (endobronchial intubation, aspiration, atelectasis) oedema, pneuomothorax. decreased FRC from anaesthesia, pain, bronchospasm, weakness from NMB, - Blood supply - PE, decreased cardiac output (pump, vessels, volume) Increase demand - Increased metabolic rate, shivering, sepsis, surgical stress response.

Bi-spectral index (BIS)

Depth of anaesthesia monitor which uses 4 electrodes across the forehead to generate an EEG. This EEG is analysed (processed) by using a proprietary algorithm and from this a value of between 0 and 100 is displayed on a monitor. Number is dimensionless. The value corresponds to depth of anaesthesia with value < 60 consistent with very low possibility of awareness. 100 awake Value 85 = sedation. 30-60 suitable for anaesthesia 0-30 burst suppression 0 no brain activity Analysis of EEG includes - Fast fourier analysis (sine waves broken down to component parts) - beta activity - Burst suppression Algorithm produced from a set of healthy controls who were anaesthetised. not validate with ketamine, nitrous invisbile, opioids minimal effect, artefacts (diathermy, EMG) Needs to be taken in context (sleep state low BIS value) Lag time 10 seconds

Entropy

Depth of anaesthesia monitoring system. A compressed EEG taken from 3 leads across the forehead and the resultant sine wave is analysed. Proprietary algorithm of processed EEG. Analysis of irregularity in EEG produces no. -> get which represents the person's depth of anaesthesia. State Entropy - EEG. 0 (no activity) - 91 (awake) Response Entropy - Higher freq components, includes EMG. 0-100 Underlying principle is that irreguarity will decrease with increasing depth of anaesthesia. Entropy of single sine wave = 0. From the EEG a value call state entropy is displayed. Value 0-90. > 60 is consistent with ananaesthetised patient appropriately ready for surgery. Target 40-60. In addition to an EMG from forehead is measured. This signal is turned into a value of between 0-100. The EMG signal looks at muscle movement on the forehead. the EMG is said to be more reactive in that there will be an EMG signal prior to the EEG value increasing.

Anaemia in chronic renal failure is characteristically A. due to haemolysis in the renal vascular bed B. normochromic and microcytic C. due to defective haemoglobin synthesis D. responsive to ion and folate therapy E. associated with increased 2,3-DPG levels in blood cells

E

The Nephron Descending Limb Thin ascending limb Thick ascending limb Collecting duct

Descending Limb - Fluid entering is isotonic ~ 300mmolL Water moves out down concentration gradient into interstitial Urine in tubules becomes more concentrated. Thin Ascending Limb - Impermeable to water - Na leaves so urine osmolality falls Thick asending limb - Na/K/Cl transported actively transports ions out, - Urine remaining becomes hypotonic. Collecting Duct - ADH makes epithelium permeable to water.

Capacitor

Device that stores electrical charge. 2 conducting plates separated by non conducting material Amount of charge that can be stored is capacitance, measure in Farads. 1 Farad = 1 coulomb of charge per 1 volt applied Does not let DC current pass.

Which of the following is not included in the DHATS2 AF thromboembolic risk scoring system a. Age b. Gender c. Diabetes d. Heart failure e. Previous TIA

Diabetes, Age, TIA, Sex.....hmm need to check

How do drugs reach placenta?

Diffusion. Fick's Law of diffusion Rate = A/T x k x P1-p2 where k = solubility/ square root of MW Factors associated with drugs - MW > 1000 D are too large - Must be unionized (NMBD have limited diffusion as hydrophilic & poorly soluble) - More lipid soluble drugs cross - Duration of exposure - Degree of protein binding

Autonomic nervous system

Division of the nervous system which regulates visceral activity & maintains homeostasis. Can be further divided into -Symapthetic - Parasympathetic - Enteral Afferent inputs and efferent pathways. All output from the CNS travels via autonomic paths except for the control of skeletal muscle which the somatic system looks after. Unlike the somatic system the motor neurone are outside the CNS & form ganglia. These may be chains along vertebrate column (sympathetic chain); near organs or be the adrenal gland. In body most organs have sympathetic & parasympathetic innervation; however blood vessels, sweat glands only have sympathetic; ciliary muscle of eye only parasympathetic.

Spinal Cord Wind Up

Dorsal horn of spinal cord has neurons associated with sensory processing Wind up occurs when 1. Sustained neural input activates protein kinase C 2. NMDA is phosphorylated 3. Mg plug is displaced & channel is now open 4. Calcium current can pass + transmits pain 5. Changes in genetic expression causes irreversible activation (marker cFOS) In lamellae V there are WDR (wide dynamic range) neutrons that are difficult to stimulate + only transmit in pathological situations. Called WDR as receive input form all pain fibres - chemical, thermal, mechanical. WDR receive convergent signnals so once activated pain may be experienced beyoond site of initial injury - hyperalgesia. Sprouting may occur and new connections made with neutrons in other lamella so simple touch is experienced as pain (allodynia) Need to control acute pain to prevent sustained neural input leading to chronic pain.

During interscalene block placement get medial movement of the scapula. This is secondary to stimulation of: A: long thoracic nerve B: dorsal scapula nerve C: suprascapular nerve D: supraclavicular nerve E: accessory nerve

Dorsal scapula nerve comes off C5 (usually) + innervates - Rhomboids to pull scapula back towards spine (medial) - Levator scapulae - to elevate scapula Interscalene - Brachial plexus block at roots to provide anaesthesia or analgesia for upper limb surgery. Used particularly in shoulder surgery. Superficial ~1-2 cm. Position patient with head away form block side - insertion is C6 at level of cricoid - lateral to sternocleidomastoid - between anterior & middle scalene muscles - landmark technique------direct needle towards contralateral elbow - caudal, dorsal, medial (DCM) Long thoracic nerve - comes off roots to serrateus anterior, caution in mastectomy/LN dissection as damage get winged scapular. Serratus anterior pulls scapular forward around the thorax (& antagonises rhomboids) or stabilised scapular against chest (working with rhomboids). Suprascapular - Comes of brachial trunk; innervation of some rotator cuff muscles that attach on the scapula. - supraspinatus (abduction of shoulder) -infraspinatus muscles (external rotation of humerus). Suprclavicular - Come off C3-C4 and cervical roots; top of arm sensation. Accessory - the common practice is to dismiss the cranial part altogether, referring to the accessory nerve specifically as the spinal accessory nerve; FXN - motor innervation to two muscles of the neck: SCM & trapezius. SCM tilts and rotates the head. Trapezius muscle - actions on the scapula, including shoulder elevation and abduction of the arm. TROUBLE SHOOTING Needle contacts bone at 1-2cm depth, no twitches -needle stopped by transverse process -insertion point is too posterior -reinsert needle more anteriorly Twitches the diahragm -stimulation of phrenic nerve -needle inserted anteriorly -reinsert more posteriorly Arterial blood -carotid artery puncture -angled too anterior -withdraw and apply pressure -redirect posteriorly Twitch of scapula -direct stim of serratus anterior muscle -direct stim of thoracodorsal nerve (supplied lat dorsi) -direct stim of dorsal scapular nerve (supplies rhomboid muscles and levator scapulae) -re-direct needle anteriorly *

Called to emergency department to review a 20 y/o male punched in throat at a party. Some haemoptysis / hoarse / soft voice. Next step in management: A. CT to rule out thyroid cartilage fracture B. XR to rule out fractured hyoid C. Rapid sequence induction / laryngoscopy / intubation D. Awake fibreoptic intubation E. Nasendoscopy by ENT in emergency department

E

Following a left sided pneumonectomy, a left intercostal drain is placed and connected to an underwater drainage system. In the postoperative period: A. a leakage of air is expected from the drain B. the patient should be nursed in the right lateral decubitus position C. the underwater seal drain should be left on continuous free drainage D. the underwater seal drain should be left on continuous free drainage, and connected to wall suction for 5 minutes every hour E. the underwater seal drain should remain clamped and be released for a short period every hour

E

Increased risk of post-partum haemorrhage in: A. Nulliparous patient B. Patient < 20 years old C. Factor V Leiden deficiency D. Oligohydramnios E. Prolonged labour

E

Recognised clinical associations with dystrophia myotonica include A. development of diabetes mellitus B. abnormal intestinal motility C. cardiomyopathy D. ovarian dysfunction E. all of the above

E

What does C6/7 do? A. B. C. D. E. wrist flexion/extension

E

What is NOT a feature of high spinal block? A. Hypotension B. C. Hard to speak D. Numb hands E. Tachycardia

E

What is the mechanism of central sensitisation? A. Increased intracellular magnesium B. Antagonism of the NMDA receptor C. Glycine is the major neurotransmitter involved D. Recurrent a-delta fibre activation E. Alteration in gene expression

E

126 AM41 ANZCA version [2004-Aug] Q15, [Mar06] Q11, [Jul07] The most frequently reported clinical sign in malignant hyperpyrexia is A. arrhythmia B. cyanosis C. sweating D. tachycardia E. rigidity

E ?? ANSWER D

What is the resus dose of atropine and adrenaline when given via ETT compared to IV A. x 0.5 B. No change C. x 2 D. x 4 E. x 6

E ?? C ARC Resus guidelines say 3-10x the dose if given via the endotracheal route A lot of other non-referenced websites say 2x the dose

Drug eluting stent 6 months old. On aspirin and prasugrel 10mg. Elective lap cholecystectomy for biliary colic. A. Do case while taking both. B. Do case while stopping both. C. Stop Prasugrel for 7 days, keep taking aspirin. D. Stop Prasugrel for some other different time E. Post-pone for 6 months

E ACC/AHA Pre-op: DRUG ELUTING: Thrombosis of DES may occur late and has been reported up to 1.5 years after implantation, particularly in the context of discontinuation of antiplatelet agents before noncardiac surgery. Discontinuation of antiplatelet therapy in the early-surgery group resulted in a 30.7% incidence of MACE (all fatal) versus a 0% incidence in early-surgery patients who continued dual antiplatelet therapy perioperatively. Overall, there was no difference in MACE between patients with bare-metal stents and those with DES. The study reported that all patients with MACE had discontinued antiplatelet therapy before surgery, whereas only 46% without MACE had done so. The study also stated there was no difference in surgical risk between patients in whom antiplatelet agents were discontinued and those in whom they were not. Excessive blood loss occurred in 2 patients, 1 of whom was receiving antiplatelet agents and 1 of whom was not. ** The panel concurred with the AHA/ACC guideline recommendation for 12 months of dual-antiplatelet therapy after DES implantation in patients who are not at high risk for bleeding. **1. Before implantation of a stent, the physician should discuss the need for dual-antiplatelet therapy. In patients not expected to comply with 12 months of thienopyridine therapy, whether for economic or other reasons, strong consideration should be given to avoiding a DES. BARE METAL: A thienopyridine (ticlopidine or clopidogrel) is generally administered with aspirin for 4 weeks after bare-metal stent placement. The thienopyridines and aspirin inhibit platelet aggregation and reduce stent thrombosis but increase the risk of bleeding. Rapid endothelialization of bare-metal stents makes late thrombosis rare, and thienopyridines are rarely needed for more than 4 weeks after implantation of baremetal stents. For this reason, delaying surgery 4 to 6 weeks after bare-metal stent placement allows proper thienopyridine use to reduce the risk of coronary stent thrombosis; then, after the thienopyridine has been discontinued, the noncardiac surgery can be performed. However, once the thienopyridine is stopped, its effects do not diminish immediately. It is for this reason that some surgical teams request a 1-week delay after thienopyridines are discontinued before the patient proceeds to surgery. In patients with bare-metal stents, daily aspirin antiplatelet therapy should be continued perioperatively. The risk of stopping the aspirin should be weighed against the benefit of reduction in bleeding complications from the planned surgery. In the setting of noncardiac surgery in patients who have recently received a bare-metal stent, the risk of stopping dual-antiplatelet agents prematurely (within 4 weeks of implantation) is significant compared with the risk of major bleeding from most commonly performed surgeries.

Which LMA has highest seal pressure? A. Classic B. Disposable supreme C. Flexible D. Intubating E. Proseal

E D according to anesth analg 2008 A Comparison of Seal in Seven Supraglottic Airway Devices Using a Cadaver Model of Elevated Esophageal Pressure B according to product info CEACCP Supraglottic Airway Deices - Recent Advancements From the LMA website / product information: LMA Classic - Seal pressure up to 20cm H20 LMA Supreme™ - measured oropharyngeal leak pressures up to 37 cm H2O LMA Flexible™ - oropharyngeal seal pressures up to 20 cm H2O LMA Fastrach - Seal pressures up to 20 cm H2O LMA ProSeal™ - leak pressures up to 32 cm H2O (Oesophageal seat = ILMA; Concerning the risk of aspiration, the use of devices with an additional esophageal drainage lumen might be superior for use in patients with an increased risk of aspiration. The Combitube, Easytube, and intubating laryngeal mask Fastrach showed the best capacity to withstand an increase of esophageal pressure.)

What is the dose of FFP required to increase fibrinogen levels by 1 g/L A. 2 ml/kg B. 5 C. 10 D. 20 E. 30

E Efficacy of standard dose and 30 ml/kg fresh frozen plasma in correcting laboratory parameters of haemostasis in critically ill patients. British Journal of Haematology. 2004 In this study administration of 33ml/Kg of FFP increased Fibrinogen by 1g/L "In group 2, all seven of the patients who had had low coagulation factor levels before FFP had levels above 30 IU/dl post-FFP (33 ml/kg). The median increment for group 2 for the individual coagulation factors was between 17 and 44 IU/ dl. The fibrinogen increased by a median of 1 g/l, although this was not significantly different from group 1"

What volume of FFP is required to increase fibrinogen level by 1g/L A. 2 ml/kg B. 5 C. 10 D. 20 E. 30

E Efficacy of standard dose and 30 ml/kg fresh frozen plasma in correcting laboratory parameters of haemostasis in critically ill patients. British Journal of Haematology. 2004 In this study administration of 33ml/Kg of FFP increased Fibrinogen by 1g/L "In group 2, all seven of the patients who had had low coagulation factor levels before FFP had levels above 30 IU/dl post-FFP (33 ml/kg). The median increment for group 2 for the individual coagulation factors was between 17 and 44 IU/ dl. The fibrinogen increased by a median of 1 g/l, although this was not significantly different from group 1"

In an infant, the intercristine line is at the level of a. L1-L2 b. L2-L3 c. L3-L4 d. L4-L5 e. L5-S1

E Local and regional anaesthesia in infants. CEACCP 2004 "The intercristal line is at L5/S1 (L4 in adults), the termination of the spinal cord is at L3 (L1/2 in adults) and the termination of the dura is at S3/4 (S2 in adults)"

Dural Sac ends at what level in a neonate? A. L1 B. L3 C. L5 D. S1 E. S3

E NYSORA "The dural sac in neonates and infants also terminates in a more caudad location compared to adults, usually at about the level of S3 compared to the adult level of S1"

20 kilogram child suffered 15% full thickness burns 6 hours ago. Optimum crystalloid fluid volume resuscitation for the first hour is A. 160 ml B. 260 ml C. 360 ml D. 460 ml E. 660 ml

E Parkland formula 4 x kg x % of bur half in first 8 hours and half in 16 hours Plus maintenance 4/2/1 rule RCH Protocol: 3 x kg x % = 900ml Half in 1st 8rs = 450ml But 6hrs already passed, so need to give 450ml over just 2hrs = 225 ml in first hour Plus need to give maintenance fluid via 4/2/1 rule = 60ml/hr x 7 = 420ml 225 +420 = 660ml closest answer

An elderly gentleman on warfarin has suffered a subdural haematoma. His INR on admission was 4.5. The resident in Ed has already given him 2.5mg of Vit K. To reverse his coagulopathy prior to urgent surgery you should give him A. Factor VIIa B. FFP C. Cryoprecipitate D. Prothrominex E. Prothrombinex and FFP

E Warfarin reversal: consensus guidelines, on behalf of the Australasian Society of Thrombosis and Haemostasis. MJA 2004 Cease warfarin therapy, give 5.0-10.0 mg vitamin K1 intravenously, as well as Prothrombinex-HT (25-50 IU/kg) and fresh frozen plasma (150-300 mL), assess patient continuously until INR < 5.0, and bleeding stops.§ OR If fresh frozen plasma is unavailable, cease warfarin therapy, give 5.0-10.0 mg vitamin K1 intravenously, and Prothrombinex-HT (25-50 IU/kg), assess patient continuously until INR < 5.0, and bleeding stops.§ OR If Prothrombinex-HT is unavailable, cease warfarin therapy, give 5.0-10.0 mg vitamin K1 intravenously, and 10-15 mL/kg of fresh frozen plasma, assess patient continuously until INR < 5.0, and bleeding stops.§

Peak plasma lignocaine level after epidural lignocaine. A 1 hour B 3 hours C 18 hours D 24 hours E 30 mins

E Br J Clin Pharmacol 1996; 42: 242245 Peak lignocaine concentration ~30 mins

In an acute malignant hyperthermia episode A. the serum creatine kinase level peaks within one hour B. the peak serum creatine kinase level is a good indicator of the amount of muscle involved C. elevated creatine kinase levels contribute to acute renal failure D. the serum myoglobin level does NOT peak for at LEAST 24 hours E. muscle rigidity occurs in 75% of cases

E - 75% muscle rigidityMasseter muscle rigidity (MMR) after succinylcholine administration may be associated with susceptibility to MH. Brief rigidity (< 20-30 s) is a normal response to succinylcholine. Rigidity lasting longer than 1 minute is seen as abnormal, and 20% of these patients will develop clinical MH within the next hour. Some patients have "jaws of steel," which make airway management difficult.

Subtenon's block. What is the worst position to insert block? A. Inferonasal B. Inferotemporal C. Superonasal D. Supertemporal E. Medial / canthal

E - Sub tenon can be done in all quadrants (A-D) these positions avoid the insertion of the resctus muscles, )

A 35yo G1P0 with a dilated cardiomyopathy presents for a Caesarean section. She has an ejection fraction of 35%. The benefits of a regional anaesthetic over a general anesthetic in this patient may include: A. decreased heart rate B. decreased systolic blood pressure C. increased ejection fraction D. decreased preload E. increased myocardial contractility

E - compared to GA the regional will maintain myocardial contractility. Cardiomyopathy 1) pre-existing - usually HOCM. (infection, sarcoidosis, IHD) 2) Peripartum Cardiomyopathy (aetiology unknown - theories of viral, autoimmune, exaggerated response to stress of pregnancy) Criteria usually - last month of pregnancy of 5 months post partum, absence of other cause. 1 in 3000. Obstructive eg HOCM Hypertrophied LV + inteventricular septum; MR. Factors that increase contractility (catecholamines, decrease in afterload or preload) will INCREASE outflow obstruction. Tachycardia -> decrease filling time, arrythmias not tolerated. Management of HOCM - Cont beta blockers (esmolol) - Cardiovert tachyarrhtymias - NO nitrates - decrease afterload - No indiacation for C-section unless deterioation - Monitor throughout art line/ECG - Regional prev an issue due to afterload reduction but pain relief good for sympathetics - Epidural slowly titrated -Ensure hydration maintained -Phenylephrine - less effects on heart as alpha agonist. Dilated cardiomyopathy/PPCM Reduced myocardial contractility. LV is usually hypokinetic, EF < 0.4, may have MR/TR. Right sided pressures are raised, likely to have pulmHTN. Any factors which depress myocardial contratility or increase afterload will compromise CV stability. Goals - maintain contractility + decrease afterload. If occurs antenatally optimise pt then deliver ASAP. Regional theoretically beneficial since depressant effects of GA avoided & afterload reduced. Treat hypotension with beta agonists (ephedrine).

Epidural block to T2 causes all of the following EXCEPT: A. Bradycardia B. Vasodilatation C. Reduced circulating catecholamines D. Dyspnoea E. Elevated PaCO2

E - higher CO2 Chang

When instructing ward staff on monitoring for respiratory depression in a patient using PCA (patient controlled analgesia) you would advise that early respiratory depression is best detected by monitoring A. frequency of boluses on PCA machine B. pulse oximetry C. pupil size D. respiratory rate E. sedation scores

E - sedation

Apnoeic oxygenation in obese patient can be increased by A. Sniffing position B. Prone C. Supine D. Lateral E. Head up

E Head up

In what proportion of people is the AV node supplied by the R coronary artery? a. 5 b. 15 c. 40 d. 40 e. 85

E ~ 90% SA node - RCA in 60%, LCx in 40% AV node - RCA in 90%, LCx in 10%

A male patient has a haemoglobin of 8g/dL and reticulocyte count 10%. The most likely diagnosis is A. Untreated pernicious anaemia B. Aplastic anaemia C. Acute leukaemia D. Anaemia of chronic disease E. Hereditary spherocytosis

E. Hereditary spherocytosis

Pulsus paradoxus in constrictive pericarditis: A. Decrease in BP Every second beat B. Decrease in BP In expiration when increase is normal C. Decrease in BP In expiration more than normal subjects D. Decrease in BP In inspiration when increase is normal E. Decrease in BP In inspiration more than normal subjects

E. In inspiration more than normal subjects

A 4 year old child with VSD (repaired when 2 years old) for dental surgery. What antibiotic prophylaxis do the guidelines recommend? A. Amoxycillyn orally B. Amoxycillin IV C. Cephazolin IV D. Amoxycillin / gentamicin E. No antibiotics required

E. Nil required; defect repaired. 2008 Australian Guidelines (still current) Prophylaxis only required for the following conditions which are considered HIGHEST RISK 1. Prosthetic cardiac valve or presence of material in valve repair 2. Prev infective endocarditis 3. Cardiac transplantation with subsequent valvulopathy 4. Congenital heart disease BUT only if - unprepared cyanotic defects incl shunts/conduits - repaired defects with prosthetic material in first 6 months since repair - repaired defects with residual defects close by (these don't get endothelialised) - rheumatic heart disease NOT an indication except in indigenous Australians. Regime : Amoxycillin 2g orally 1 hr pre-op e.g. dental (child 50mg/kg) Augmentin 2g IV just prior (or IM 30 mins prior)

125. New- Compare to Myasthenia gravis, which symptoms is more likely to be Eaton Lambert syndrome?

EATON LAMBERT SYNDROME Myasthenic syndrome Definition: autoimmune NM disorder characterized by IgG Ab to pre-synaptic Ca channels and decreased ACh release • Associated with malignancy 50-60% • Small cell lung Ca CLINICAL • Proximal muscles weakness, esp lower limbs • Strength improves with activity • Although 30% demonstrate fatigability • Myalgia • Tendon reflex is absent • Ocular/bulbar dysfunction • Autonomic dysfunction ANAESTHETIC IMPLICATIONS • Sensitive to both sux and NDMR TREATMENT • Not reverse by anticholinestases • Immunosuppression with steroid • Plasma exchange

U waves

ECG change seen in - Hypercalcaemia Hyperkalaemia - Hyperthyroid Possibly repolarisatnion of septum

NK1 receptor

Endogenous ligands include substance P which has role in pain pathways. Anti-emetic Aprepitant blocks NK receptor

Functions of endothelium

Endothelium is monolayer of flattened endothelial cells. Large surface area 1. Regulation of blood-tissue exchange 2. Regulate vasomotor tone -> NO, prostacyclin etc 3. Metbaolism - conversion of ACEI - ACE II. Inactivates bradykinin. 4. Secretion of antithrombotic/prothrombotic factors 5. Immunological defense - inflamed tissue captures leucocytes & helps migration 6. Angiogenesis 7. Atheroma formation (bad)

Control of Sleep

Evidence suggests it is an active process whereby some neutrons increase firing rate to produce active inhibition Wakefulness controlled by Reticular Activating System (RAS) anatomical area in brainstem with groups of visor neurons. Mulitple areas feed into RAS; neurotransmitters include Dopamine, NA, histamine, ACh (so inhibitors often cause sedation) Visual/Olfactory/Auditory pathways Trigeminal Ascending spinal sensory tracts Thalamus -> cortex -> RAS

Fresh Frozen Plasma

FFP. From single donor. Component of blood left after removal of RBC 1 unit of 250mL will increase clotting factors by 15%. Aim is to restore PT to < 1.5 of normal Comprises - Clotting factors FII, IX, X but low volumes of V, VII as heat labile. - albumin - Gamma globulin - Only low fibrinogen Plasma removed then frozen, needs to be thawed. Leucodepleted. Platelets removed. ABO compatability with AB universal donor and O for O patients From male donors to decrease risk of antibodies in plasma with may result from pregnancy and can cause TRALI

Renal Processes

Filtration ReAbsorption Secretinon Metabolism (ammonium, bicarbonate)

Work (measured in joules)

Force x distance And as...... Force = Pressure x area Distance = volume x area Work = pressure X volume

CSF formation

Formed choroid plexus lateral ventricles Interventricular foramen to 3rd ventricle Sylvian aquduct to 4th ventricle Lateral & median apertures to arachnoid villi, spinal cord + subarach space.

Glomerular filtration rate (GFR)

GFR = volume of filtrate produced per unit time is a clearance Normal 125mL/min or 180L/day GFR = Kf (P G - P B - oncotic pressure in glomerulus) Kf - glomerular ultrafiltration coefficient Pg = Glomerular hydrostatic pressures Pb - Bowman's casule hydrostatis pressure Note oncotic pressure in Bowman's capsule = 0 Kidneys filter 60 x plasma volume per day. Woman have lower GFR (10% less) Pressure in afferent arteriole 60mmHg; in glomerular capillary quite high at 45mmHg.

How are Starling Forces diff in kidneys?

Glomerulus - high filtration coefficient High protein reflection coffee as glomerular capillaries impermeable to proteins so oncotic pressure in filtrate is zero. High hydrostatic pressure in capillaries due to efferent arteriole. Oncotic pressure changes along capillary due to loss of water. In other capillaries the hydrostatic pressure drop means less fluid movement, in kidneys however the gradient changes as the oncotic pressure increases in capillary. There is a net outward flow across capillary.

GTN

Glyceryl trinitrate GTN is an organic nitrate used for the treatment of 1. Angina 2. LV failure 3. Intra-oprative BP lowering Available in patches, IV, SL, buccal. Tablets of ISMN have 100% OBA It principally works on venous system and large coronary arteries but does have some effect on arterial smooth muscle at large doses. Function by stimulating release of nitric oxide from nitrosothiol. Needs enzymatic process with supply of this thiol containing compounds to release NO. NO then works via cytoplasmic guanyl cyclase to prevent calcium mediated vasoconstriction. In the coronaries it decreases myocardial oxygen consumption, relieves vasospasm, improves blood flow to endocardium. Dose 300mcg SL; 50mcg IV SE - Produced methaemaglobin.

Erythropoietin

Glycoprotein hormone produced in live & kidney. Acts on immature erythrocytes in bone marrow. Secreted in response to decrease oxygen delivery (& so will occur with sympathetic stimulation causing decreased renal blood flow). Production occurs within hrs Recomninant mediacation used to promote erythropoiesis.

Staring's Forces in Lungs

Goal is for fluid to be retained in vasculature (which is opposite to nephron). Low pressure circulation means we have low hydrostatic pressure. Net driving pressure is initially outward then becomes into capillary at end. This prevents interstitial odema in lungs.

Hypertension Treatment

Goals - To decrease SVR to reduce after load - To decrease blood volume - To decrease CO by decreasing HR/SV 1. Diuretics - thiazide, loop, K+sparing 2. Vasodilators - alpha blockers - Ace-i/ARBs/renin inhibitors - Ca channel blockers - Ganglion blockers - Nitrodilators - GTN/ISMN - Direct acting arterial dilators (hydralazine) 3. Cardioinhibitory Beta blockers ca channel blockers 4. Centrally acting sympatholytics

When performing laryngoscopy using a Macintosh blade, your best view is of the patient's epiglottis touching the posterior pharyngeal wall. Using the Cormack and Lehane scale this is grade A. 1 B. 2 C. 3a D. 3b E. 4

Grade 3b as per MODIFIED CL score.\d\d//d3cgb598vs7bfg.cloudfront.net/images/upload-flashcards/554968/540517_m.jpg

Intrapleural pressure across lung Transpulmonary pressure = difference btw alveolar pressure + intrapleural pressure

Gradient of -10 at top to ~-3 as you move down lung. With large inspiration these pressures adjusted so top now -6 + bottom 3 so pressure at base exceeds alveolar pressure therefor alveoli are compressed and we have moved too far down compliance curve. Transpulmonary pressure is the difference between alveolar pressure - intrapleural pressure Usually at base with normal lung volume 0 --2.5 = +2.5cmH2O At base with low lung volume 0 -3 = - 3.5cm H2O

Immunology mediated heparin induced thrombocytopenia- intravascular thrombosis

HITTS Type 1 and Type 2

Moans, stones, bones, psychic groans

HYPERcalcaemia, eg in primary hyperparathyroidism A helpful mnemonic, "painful bones, renal stones, abdominal groans, and psychic moans," can be used to recall the typical symptoms of hypercalcemia. Painful bones are the result of abnormal bone remodeling due to overproduction of PTH. Nephrolithiasis occurs secondary to hyperparathyroid disease-induced hypercalcemia and resultant hypercalciuria. Abdominal groans refers to hypercalcemia-induced ileus. Psychic moans or depression may occur in the presence of persistently elevated serum calcium levels.

Elimination Half Life Elimination Half Time Context sensitive Half Time

Half Life - The time to eliminate 50% of drug from body after IV injection Half Time taken for plasma drug conc to decrease by 50% during elimination phase 5 half times = time taken for 97% elimination Good for one compartment model only. Gives limited info after an infusion Context sensitive half time - time necc for plasma levels to decrease by 50% (or other percentage) and the time of discontinuing an infusion of a specific duration. The duration of time of infusion will alter the half time. Considres combined effects of resdistribution, metabolism, duration of IV infusion.

Pharmacological hysteresis

Hysteresis is the phenomenon whereby the measurement observed will be different depending on whether the input variable is climbing or falling at that moment in time. The effect site concentration of drug will be diff if plasma level rising or falling.

Most effective treatment for post-sevoflurane agitation following grommets in a 4yo child? A. 1 mg/kg propofol B. 1 mcg/kg fentanyl C. 1 mcg/kg clonidine D. ?dose midazolam E. Sucrose

I had A ?? B Reference from paeds textbook (coats,) says opioids are most effective

During lumbar plexus block placement, which of the following indicates inappropriate needle placement? A: hip flexion B: hip adduction C: knee extension D: knee flexion E: lumbar extension

I have D ?? ANSWER E Method 1. Mark out midline over spinus processes 2. Mark out perpendicular line, line connect iliac spines 3. Needle insertion 4cm from midline along interiliac spine, aim perpendicular, parallel to spine 4. 6-8 cm depth 5. aim is for twitch of quadraceps (femoral nn), but twitch of any of the lumbar nerves possible 6. Nerve stimulation from 1mA to 0.5mA 7. aspirate and inject 20-25ml of solution (0.5-0.75% ropivacaine)

Starling Force's in Brain

IN brain solutes provide the oncotic pressure as BBB not permeable

MTP Complicaitons MTP> 5 units in 1 hr (i thought 4) or > total blood volume in 24 hrs

Immune Acute transfusion reaction (increased due to emergent situation & increased units) - ABO incompatability most common Delayed transfusion reaction - also antibody mediated but less severe Febrile non haemolytic reactions Graft vs host disease Allergy and allergic anaphylaxis TRALI Non-immune Infective - viral - HIV/HCV/HBV/CMV bacterial; prion; parasitic (malaria) TACO Electrolyte abnormalities - hypocalcaemia, hyperkalaemia Citrate toxicity Acidosis (blood pH 6.5 at 2 weeks old) Hypothermia (blood 4C) Dilutional anaemia + coagualopathy (platelets) Altered O2 affinityDecreased 2, 3 DPG Haemosiderosis Coagualopathy multifactorial - Underlying trauma, consumption, dilution, hypoperfusion, acidosis, hypothermia, dilution of clotting factors, dilution of platelets

What is the role of urea in nephron?

In collecting duct urea 1. Maintains high osmolarity of interstitium. High osmolarity promotes movement of water out of tubule into interstitial. Reabsorbed from 1. descending loop of Henle 2. moves out of tubules in deep medulla. Taken up by vasa recta then moves out of vasa recta in areas of low urea but then becomes trapped. 2. Acts as a carrier for ammonia which then can be excreted as ammonium??

EEG during anaesthesia

Induction alpha activity decreases then beta activity INCREASES As depth increases move to that then delta rhythms with burst suppression. Overall -> slow, more synchronous, increase in amplitude. Thio & isofllurane will produce flat isoelectric pattern at high levels. Ketamine - disorganised asynchronous, no electrical silence. N2O fast frontal activity servo gives EEG seizure but not clinical seizures

Furosemide (loop diuretic)

Inhibits Na/K/Cl co-transported in thick ascending LOH Transported usually reabsorbs Na, by preventing this less H2O is reabsorbed in collecting ducts (as interstitial less hypertonic). Side Effects Hypokalaemia Metabolic alkalosis.

Levosemendan

Inotrope which acts by increasing sensitivity of contractile apparatus to calcium in cardiac myocytes. It does not increase Calcium levels so adverse effects of hypecalcaemia not seen.

Cryoprecipitate

Plasma derived concentrate of FVIII, fibrinogen & vWF. Made by rapid thawing FFP then refreezing, stored at -20C. Volume ~ 40mL. Need 10 units to increase fibrinogen by 1g/L TEG/ROTEM - Need larger amounts than previously thought in MTP. ABO match preferred but any will do

Hepatic Extraction Ratio Hepatic Clearance

Low extraction ratio (ER <0.3) means drugs saturate enzyme system and changes in blood flow will not help to get rid of drug. Enzyme induction will improved the metabolism. The higher the bioavailability the more likely the ER will be low as there will be a lot of free drug in plasma that needs to be metabolised. High Extraction Ratio > 0.7 Clearance will depend on hepatic blood flow. Eg propofol Hepatic clearance = Hepatic blood flow x ER

Lung pressure Volume Loop

Lung volume 6L; RV 1L. FRC at 1 to 2.5 L Pressure from 0 to -30cmH2O Bottom part of lungs more compliant than apices. IPPV makes apices more compliant but bases less so ??

What Factors determine what crosses the placenta?

Maternal - Drug dose Degree of protein binding Metabolism & elimination (e.g. sux rapidly metabolised) Maternal pH + pKa Uterine blood flow Hypotension Placental Diffusion (Fick's Law) -> more drug if increase in surface are (i.e. larger chorionic membrane) Placental metabolism (metabolizes some drugs) Late pregnancy - single layer of chorion in connection with endothelial capillaries Foetal - Volume of fetal blood in intervillous space pH - 0.1 lower so ion trapping occurs if fetus becomes acidotic Fetal liver has cytochrome p450 so limited metabolism may occur More acidotic fetus -> more blood flow through ductus venosus so more drug centrally to fetus.

Methyldopa

Methylated dopamine which is used for treatment of HTn in pre-eclampsia. Methylated dopamine taken up & metabolism to false neurotransmitted - methylnoradrenaline & not metabolised by MAO. Has weak effect at alpha 1 R causing vasodilation & prevents release of more NA from alpha 2 R. Decreased SVR with maintenance of uteroplacental blood flow. Hypotensive effect is additive with volatiles (decreases MAC) SEs Orthostatic hypotension Liver toxicity Seadaton Nasal congestion

Magnesium

Mg is cation present in body important for regulation of 1. Movement of Na/K/Ca across membrane 2. Cofactor in enzymatic reaction incl DNA/ATP synthesis 3. Activation of ATP reactions Normal serum level 0.75 - 1 mol/L Dose 5g over 20 minutes As a drug we use Mg Sulphate Uses - Pre-eclampsid + eclampsia with aim > 2 -3.5 mmoL/l ....fxn is unknown. - Hypomagnesia due to malabsorption - Hypertension - Analgesia - Tocolytic in pre-term labour - Torsades...prolonges conduction, -Asthma - bronchodilator - Spasms MOA Often opposes movement of Ca Decreases Ach release from NMJ - reflexes will be lost at > 3mmol/L Effects - Will prolong PR & slow SA node firing. If QRS becomes wide then too much has been given. - CNS depressant -> too much will cause coma & death

What are microshock & macroshock

Microshock - term used to describe phenomenon of very low current causing VF; occurs when the current although small (lecithin 100microvolts) is applied directly to myocardium. e.g. via central line tip Macroschock - Electric shock which occurs at higher current 1mA - tingling 5mA - pain 15mA - local muscle spasm 50mA - muscle spasm 80mA - VF factors which increase risk of electrical shock 1. Current size 2.Resistance --> density 3. AC or DC 4. Path 5. Contact time 6. Freq

Albumin Other Plasma Proteins: Globulin , Fibrinogen, cytokines, complement, coagulation, CRP

Most abundant plasma protein. Synthesised in liver. Weight 69000 daltons. Half life 20 days Crosses into interstitium (returns via lymphatics). Functions - Oncotic pressure NFP > = (Pc -Pi) - reflection coeff (oncotic pressure cap - oncontic pressure interstitium) - Acid-base buffer - Transport - FFA, biliriubin, calcium, cortisol, T3/T4; - barbiturates, benzos, warfarin - Provides building block for other proteins (broken down in cells to aa)

Muscarinic Receptors

Muscarin is poison from mushrooms. Initially used to differentiate the receptors that bound acetylcholine from the nicotinic receptors. Molecular biology now shows that muscarinic receptors are all GPCR. There are several subtypes and the effect of acetylcholine will depend on what system is activate by the GPCR. Found in CNS, PNS, parietal cells, gastric, heart ,glands. Agonists on muscarinic receptors (sometimes called anticholinergics but they block muscarinic effects of Ach) Atropine (nb this may cause brief reflx brady due to Bezold-Jarish reflex before the desired tacky occurs) Hyoscine Ipratropium - bronchodilation & decrease in secretions

Different muscles have diff sensitivity to neuromuscular blockade, what are some reasons?

Muscle blood flow - high to diaphragm + larynx so fast onset despite relative insensitivity. Muscle fibre mix fast/slow twitch Acetyholine R density Acetyclsholinesterase activity No. of NMJ - fine muscles haves more

Pre eclamptic patient post LSCS continue on Mg infusion in ICU. Found to be in respiratory depressed. Next management A. Calcium gluconate B. IV fluid C. Frusemide

My answer A ?? ANSWER C From RWH Request magnesium level and review management if: * respiratory rate < 12 breaths/minute * urine output < 100mLs in 4 hours * loss of patellar reflexes * further seizures occur. Response to magnesium toxicity The following clinical signs of magnesium toxicity must be reviewed by a consultant obstetrician/anaesthetist: * urine output <100mL in 4 hours * absent patellar reflexes * respiratory depression. The antidote for magnesium toxicity is: calcium gluconate (10mL of 10% solution over 10 minutes) by slow intravenous injection. The patient requires ECG monitoring during and after administration because of the potential for cardiac arrhythmias. Resuscitation and ventilator support should be available during and after dose administration of both magnesium sulphate and calcium gluconate.

90. New- Post partum sudden collapse, suspected amniotic fluid embolism. The consistent finding is: A. Low C3, C4 B. Increase complement C. Increase tryptase D. Increase histamine? E. petechial rash

My answer A ?? ANSWER E

ET03 [Repeat] Jehovah's witness refused blood- you have told him you refuse to do the surgery/anaesthesia for his own good. Ethical principle: A. Paternalism B. Maleficience C. Autonomy D. Beneficience

My answer A ANSWER B? Paternalism would be to proceed and give him blood regardless of his autonomy because we know best Non maleficence would be first to do no harm. In this case not proceeding with surgery. Autonomy would be to proceed and not give him blood even if he exsanguinates. Beneficience ?? action that is done for the benefit of others. Examples of beneficent actions: Resuscitating a drowning victim, providing vaccinations for the general population, encouraging a patient to quit smoking and start an exercise program, talking to the community about STD prevention.

63. Repeat- The most effective method for cerebral protection in aortic arch aneurysm repair A. Systemic hypothermia 20degrees B. Antegrade perfusion to carotid arteries C. Retrograde perfusion to jugular veins D. Thiopentone E. Steroid (?)

My answer B ?? ANSWER A

Ageing (adult) causes: A: Decreased FRC B: Decreased Cardiac output C: Diastolic dysfunction D: Increased creatinine

My answer B ?? ANSWER C

Platelets from blood bank

Pooled - 8 donors. - Will increase platelet count by 10 ( x 10 9/L ) Apharesis - single donor Store at 20-24C Shelf life 5 days ->longer increased bacterial contamination Agitate to avoid clumping

When compared to a non-obese patient, in an obese patient (BMI >35) when fasted for an elective procedure the gastric secretion will have: A: more volume, higher pH B: more volume, lower pH C: same volume, same pH D: less volume, lower pH E: less volume, higher pH

My answer B Answer B: according to Obesity in Anaesthesia and Intensive Care. BJA 85(1):91-108 (2000) page 101 CEACCP 2008 8(5) 151-156 but no reference in the article E: according to A&A 1998 pg 147-152 According to A&A 2001 vol 93 no 6 pg 1621-1622 same volume slightly lower pH http://www.anesthesia-analgesia.org/content/93/6/1621.full Gastric content volume was identical in the obese and lean subjects (26 ± 13 mL and 26 ± 8 mL, respectively). The values of pH were 2.3 (1.3-7.1) and 2.8 (1.6-7.1) in obese and lean patients, respectively.

50. New- Most common cause of paediatric post anaesthesia cardiac arrest A. Drug error B. Respiratory cause C. Multifactorial D. Cardiac problem (?)

My answer C ?? ANSWER B

39. NEW. Fontan patient having an open appendicectomy. What do you want? A: long I time and PEEP B: long I time C: short I time D: raised ETCO2 E: spontaneous ventilation

My answer C ?? ANSWER E CEACCP 2008 Fontan procedure is to divert all systemic venous blood into the pulmonary arteries, without the interposition of a ventricle. In patient with a single ventricle the pulmonary and systemic circuit runs in parallel. So change the system into a single pump instead of two pumps in series. Criteria: 1.Sinus rhythm 2.Adequate size pulmonary arteries 3.Good ventricular function Anaesthetic Goal 1.Maintain ventricular function 2.Minimise V/Q mismatch 3.Optimise pulmonary blood flow a.Avoid decrease intravascular volume. i.e. Avoid hypovolaemia. b.Minimise PVR i.Avoid acidosis ii.Avoid hypoxia iii.Avoid hypercarbia iv.Minimal increase in mean intrathoracic pressure v.Avoid vasoactive drugs vi.Adequate analgesia and anaesthesia For relatively short procedures, Fontan patients are probably better off breathing spontaneously, as long as severe hypercarbia is avoided. For major surgery, or when prolonged anaesthesia is required, control of ventilation and active prevention of atelectasis is usually advisable. Potential disadvantages of mechanical ventilation in Fontan patients relate to the inevitable increase in mean intrathoracic pressure. This causes decreased venous return, decreased pulmonary blood flow, and hence, decreased cardiac output. Low respiratory rates, short inspiratory times, low PEEP, and tidal volumes of 5 - 6 ml kg usually allow adequate pulmonary blood flow, normocarbia, and a low PVR. Hyperventilation tends to impair pulmonary blood flow, despite the induced respiratory alkalosis, because of the increased mean intrathoracic pressure.

Hypertension- severe- in pregnancy (or was it PET?) What NOT to use? A: Hydrallazine B: Nifedipine C: Labetalol D: Metoprolol E: SNP

My answer D ?? ANSWER E

Apnoeic oxygenation in obese patient can be increased by A. Sniffing position B. Prone C. Supine D. Lateral E. Head up

My answer E

71 E46 Mar2011 Acromegaly due to excess of growth hormone. Why hard to do direct laryngoscopy? A. Distorted facial anatomy B. Macroglossia C. Glottic stenosis D. Prognathe mandible E. Arthritis of the neck

My answer is D ?? ANSWER B

Fat: blood coefficient- N2O, D, S, I

N2O 2.3 D 27 S 47 I 45

Dopamine

Naturally occurring catecholamine with alpha 1, beta 1 & dopamine R activity. CNS - Basal ganglia & motor pathways - Chemotrigger receptor zone (CTZ) where it stimulaters vomiting - Reward pathways - Causes prolactin release Peripherally - Tachycardia & dysrythmias. - Inotropic Dopamine -> noradrenaline -> adrenaline Caution in phaeocytochromas due to NA release. Bromocriptine is a dopamine agonist; acts on D2 receptors. Used in Parkinson's, neuroleptic malignant syndrome, pituitary tumours.

Vasopressin

Naturally occurring peptide in body release from posterior pituitary, also called ADH. Strong vasoconstrictor (V1) and promotes water reabsorption in distal collecting duct (V2) Synthetic forms are 1. Vasopressin IV solution used as vasopressor e.g. in sepsis where excessive NO is causing vasoplegia 2. DDAVP = desmopressin which is given intranasally to treat central diabetes insipidus. Has greater affinity for V2 receptors. 3. Mx of refractory cardiac arrest 4. Mx of haemorrhage from variceal bleeding - causes marked sphlnchnic vasoconstriction. 5. Post phae resection when catecholamine levels are now low Acts on V1 recept

Starling's Forces

Net fluid movement = K x net driving pressure Net driving pressure = (Pc-Pi) - sigma ( IIc - IIi) K = filtration coeff for water high = leaky for water Determined by permeability & area of capillary P - Hydrostatic pressure in capillaries - interstitium II - capillary oncotic pressure - interstitial oncotic pressure Sigma = reflection coeff effectiveness of capillary wall to to prevent passage of proteins

Pregnancy induced HTN Note values for definition differ from normal stages of HTN

New onset BP > 140 systolic or 90 diastolic 1. Methyl-dopa 1 g tds 2. Calcium channel blockers - nifedipine (class C drugs) Emergency if > 160 systolic, > 110 diastolic. risk of ICH 1 Beta blockers - Labetalol 5 to 10mg increments 2. Hydralazine - infusion 3. Magnesium 4. GTN 5. Sodium nitroprusside Inappropriate are ACE -i (foetal renal dysfxn, death) & diuretics (neonatal thrombocytopenia)

What factors are secreted by endothelial cells?

Nitic oxide - > Dissolved gas, produced from L- arginine Vasodilates. Produciton stimulated by GTN, histamine, substance P Inactivated in seconds. PGI2 (aka prostacyclin) - Vasodilator; produced from COX1/2 Endothelins - Potent vasoconstrictors (related to snake venom). Production increased by ACE, catecholamines, insulin. Imp on closing ductus arteriosus, Inhibited by prostaglandins. Carbon monixide - vasodilator

Etomidate

Non barbiturate imidazole agent used for induction of anaesthesia. Cardio stable in therapeutic doses. Binds to GABA A receptor; Dose - 0.3mg/kg. SE - Pain on injection, adrenocortical suppression. Nausea, vomiting, myoclonic movements

Inflammation

Non specific response to infection or injury. Characterized by swelling, erythema, heat & pain. Advantages - increases blood flow to area (o2, nutrients), pain leads to protection Disadv - pain limits fxn Process involves tissue damage which causes 1. Disruption of cell membrane -> K+ 2. Neutrophils -> bradykinin, stimulate arachidonic acid 3. Nerves release substance P -> vasodilation Arachidonic acid is used by COX to produce prostaglandins and thromboxane A2.

Alpha Adreno Receptors

Post synaptic alpha 1 - Vasoconstrict, bladder sphincter, mydriasis Post synaptic alpha 2 - platelets, CNS Pre-synaptic alpha 2 - inhibit noradrenaline release

Hypercapnia

PaCO2 >44mmHg Co2 rapidly crosses cell membrane and will cause increase in H+ in cells with adverse effects on enzyme systems Effects on organ systems Neurological - vasodialtion -> increased CBF; increase in 1mmHg will increase blood flow by 4%; increase in ICP; stimulte ventilation via chemoR but centrally rest depression; sympahtetic stimulation CVS - vasodilation -> increased coronary blood flow sympathetic stimulation with increase HR & increased risk of arrhythmia. Decreased contractility.

Breast Feeding

Passive diffusion into breast milk. Some transporter systems involved Most invested drugs appear in breast milk at < 1% ingested dose. pH milk 7 Anesthetic drugs are of short duration so amount of transfer is low. Route of administration will affect plasma levels and milk levels

Error with BIS

Patient - Clinical changes - cardiac arrest hypovaolaemia, cerebral ischaemia. Drugs - Use of ketamine, etomidate, halothane Opioids, nitrous Machine - Poor electrode contact (high impedance) - Improper sensor placement - Reusing BIS cable - Electrical interference - Diathermy, forced air warmer - Excessive muscle activity (pt not paralysed)

The incidence and severity of vasospasm post sub arachnoid haemorrhage is greatest at: A. 0 -24 hours B. 2 - 4 days C. 6 - 8 days D. 10 - 12 days E. greater than 2 weeks

Peak incidence of vasospasm is 4-10 days. Surgery previously delayed until this was over but now nimodipine used. (so would choose C here ) Risk of rebleed highest in first 24 hrs; aim f clip/coil is to secure aneurysm & prevent rebleed. Sub Arachnoid Haemorrhage World Federation of Neurosurgeons Grading influeces morbidity/moratlity; used to determine operation or early coiling (1-3) or delay iterventions (4-5) 1 GCS 15 2 GCS 13-14 3 GCS 13-14 4 GCS 7-12 5 - GCS 3-6

Coroanry Perfusion Pressure

Perfusion pressure is arterial = venous pressure however in some organ systems like the heart and brain additional extrinsic factors will affect the pressure - e.g. ICP or intraventricular pressure In cornary circulation the CPP changes in systole & diastole. Need to consider left and right heart. LEFT Systole there is little flow CPP = systolic arterial pressure - LV pressure = 120-120 = o Diastole CPP = aortic diastolic - LVEDP or RA pressure = 80 - 5 = ~ 75-80mmHg RIGHT Systole CPP = 120 - 25 = 95 mmHg Diastole CPP = 80 - 5 = 75mmHg As the pressure on right side less variable the flow is less variable.

Anaesthesia

Pharmacologically induced state of decreased consciousness level which is reversible and controlled. It results in decrease in autonomic & skeletal reflexes, amnesia and analgesia. Not a single state but a continuum.

Ketamine

Phencyldine derivative which has use in 1. Induction & maintenance of GA 2. Prodecural sedation 3. Analgesia 4. Field anaesthetic 5. Severe asthma It causes a dissociative anaesthetic. Superficial sleep; disorganised EEG S ketamine more potent MOA 1. Non competitive antagonist of NMDA receptor 2. Causes decrease release of glutamate 3. Also interacts at opioid receptors (complex mechanism) 0.5 - 2mg/kg IV bolus Oral/rectal/nasally/IM Side effects related to the increase in catecholamines which causes SNS stimualtion,

Diffusion hypoxia

Phenomenon seen with the use of nitrous oxide at the conclusion of GA. Nitrous moves from blood back to alveoli & dilutes oxygen in alveoli lowering PAO2 thus PaO2.

Milrinone

Phosphodiesterase inhibitor which acts as inodilator by increasing cardiac contractility and decreasing pull vascular resistance. Specific for heart and smooth muscle Needs loading dose.

Amiodarone Side Effects

Pulm fibrosis, pnuemonitis Hypo/hyper thyroid (prevents peripheral conversion of T4 to T3) Cirrhosis, jaundice Bradycardia, prolonged QT Corneal deposits Metallic taste Peripheral neuropathy Photosensitivity/slate grey skin

Protamine

Purified mixture of LMW cations from fish sperm. Strongly basic, forms salt with heparin (an acidic anion) SEs - Hypotension, bradycardia, dyspnoea, flushing, anaphylaxis - Decreased myocardial contractility - Anticoagulation if too much due to anti-platelet effects Dose 1mg per 100units heparin. Usually ~300mg in bypass Give slowly.

Symapathetic innervation

T1-L2 nerve roots but chain extends down all spinal cord. Cardiac plexus comes from both cervical and thoracic ganglia. Coeliac plexus comes from T8-L3 CERVICAL region ganglia - 4 of them - Superior -Middle -Inferior - Stellate THORACIC T1-T5 aorta, pulm T5-T12 sphlanchnic T10- T12 lower sphlanchnic renal pelvis LUMBAR Situated in front of vertebral column as part of prevertebral plexus SACRAL Arises from lumbar region but sits in front of sacrum.

What does calcitonin do?

Produced by parafollicular (C) cells of THYROID gland. Oppose PTH and acts to decrease calcium

Atrial Natriuretic Peptide or Factor ANP BNP

Produced from atria when stretched or shear stress. Result in ANP formation which works on kidneys to increase GFR and increase Na & water loss. Dilates afferent arteriole and constricts efferent arteriole so overall hydrostatic pressure increased +more Na excreted Also has effect on adrenals to block aldosterone secretion. BNP released from ventricles and has similar effects. Used as marker of CHF.

Damping

Progressive diminution of amplitude of oscillations in a resonant system by dissipation of energy. In arterial BP may be caused by - kinking, clots, bubble, tubing, viscous drag of fluid

Misoprostal

Prostaglandin (PGI2) used for induction of labour (cervical ripening) & uterine contractions.

Major Histocompatability Complex(MHC) prev HLA

Proteins on surface of cell membrane. Bind antigens and present to cells of immune system MHC I genes MHC II genes All cells in body have MHC I. Peptides from self displayed on these. MHC II only produced by antigen presenting cells (APC) which are macrophages, dendritic cells, some B cells

Cytokines

Proteins released by cells as a means of cell signalling e.g. IL -1, TNF.

Chlorpromazine

Prototypical anti psychotic. Phenothiazine non selective D1/D2 antagonist. The D2 antagonism causes extra-pyramidal side effects. Uses - in schizophrenia, acute intermittent porphyria & serotonin syndrome (due to its serotonin antagonism) Has > 100 metabolites from liver metabolism Increases prolactin release. Can cause tardive dyskinesia and acute dystonias, neuroleptic malignant syndrome (tachycardia, autonomic instability, increased temp, rigidity)

Features of REM Sleep

REM phase has tonic and phasic parts CVS - Phasic part - increase in sympathetic activity Decrease CO. other parameteers will depend on phasic.tonic part. Resp - breathing irregular Further decrease in responsed to hypoxia/hypercapnia MSK - Further loss of muscle tone; atonic except for diaphragm + upper airway. Do get burst of muscle contractions. Met - Brain metabolism increased by 20%; increased glucocorticoid.

Respiratory Quotient (RQ) Resp exchange ratio

Ratio ov volume of CO2 produced per volume of O2 consumed. Will depend on substrate that is being utilised for fuel. COOH - 1 Fat - 0.7 Protein - 0.82 Estimation of respiratory quotient Increased during exercise as increased CO2 production

In patient's with bilateral decompressive craniectomy for raised ICP results showed

Reduced duration of mechanical ventilation Reduced stay in ICU No significant difference in total hospital stay Worse 6 month functional outcome as measured on extended Glasgow Coma Score DECRA Conclusions NEJM 2011 In conclusion, in patients with severe diffuse traumatic brain injury and increased intracranial pressure that was refractory to first-tier therapies, the use of craniectomy, as compared with standard care decreased the mean intracranial pressure and the duration of both ventilatory support and the ICU stay but was associated with a significantly worse outcome at 6 months, as measured by the score on the Extended Glasgow Outcome Scale. Patients in the craniectomy group had a shorter duration of mechanical ventilation and a shorter stay in the ICU than patients in the standard- care group, although there was no significant between-group difference in the total time in the hospital Hydro-cephalus was more common in the craniectomy group (10%) than in the standard-care group (1%). Six months after injury, the primary outcome (functional assessment on the Extended Glasgow Outcome Scale) was worse in the craniectomy group than in the standard-care group

What factors affect renal blood flow?

Renal Blood Flow ~ 1250mL/min, in excess of metabolic requirements so oxygen extraction is low. From Ohm's Law V = IR. Flow - perfusion pressure/resistance Perfusion pressure - Arteriolar - venous pressure Resistance - Arterioles Arteriolar perfusion pressure will depend on - Systemic MAP Venous - Obstruction to venous flow (urinary block) Resistance will depend on vessel diameter which will be affected by local & systemic factors - Sympathetics - Autoregulation - myogenic (decreased strectch - renin, increased stretch calcium) + tubuloglomerular - Humoral - Prostagalandins, ANP - Angiotensin works on mesangial cells Sympathetics - Beta receptor activation of JGA -> renin release -> vasoconstriction -Tubuloglomerular feedback->Increased Na delivery to macula densa -> adenosine -> vasodilation to decrease GFR - Myogenic mechanism - increased stretch -> Ca -> vasoconstriction. Decreased stretch in afferent arteriole -> renin release from granular cells -> angiotensin -> stimulates mesangial cells to contract -> vasoconstriction - Prostaglandins - Vasodilation. ANF- dilate Pressure diuresis

Allergy/anaphylaxis

SKIN TEST - good for beta lactam grip but not for others - overall only 60% of antibiotic reactions are predicted so if high clinical suspicion avoid. - 6/52 post reaction, too soon -> misleading results. Stop antihistamine 5 days before ok to cont steroids - not good for NSAIDS, opioids, dextrans, contrast 1. Skin prick - use positive & neg controls. Wheal > 3mm greater than control is positive. 2. Intradermal - if clinical suspicion but negative skin test go on to this. Good sensitivity - lots of true positive results but poor specificity - lots of false positives. For neuromuscular drugs 95% sensitive. Good for IgE as above but also delayed hypersensitivity. More chance of precipitating systemic rxn. CHALLENGE Only way to exclude clinical allergy. Risky. BLOOD TESTS - RAST superseded by fluorescent detection system CAP which uses radiolabeeled anti IgE antibodies so test detects specific IgE to known allergens. WHO TO TEST - No place for perioperative screening as prevalence is too low. - Previous exposure NOT necc before a reaction occurs - Environmental exposure more imp than genetics so family hx not relevant. Refer: 1. Hx of unexplained reaction during previous GA. 2. Pts with documented allergy to neuromuscular blocking drugs. 3. Those who allege allergy to LA if LA to be used.

Oxycodone

Semi synthetic opioid used to treat severe pain. First made in Germany in 1916 Has mu receptor agonism which produces analgesia, also delta & kappa receptor agonism. A OBA good 85% so 1:1 IV to oral conversion D M - Cytochrome system; those that are poor metabolisers will need lower dose. Oxycodone has primary effect but metabolite oxymorphone still active however there is only a small amount of it so less problematic. E - Liver to oxymorphone & noroxycodone which are renally excreted. Effects CVS - Orthostatic hupotension due to decreased SVR Resp - Resp depression with decreased rate; decreased vent response to hypoxia, hypercarbia. Anti-tussive CNS - Analgesia, sedation, euphoria, anxiolysis, miosis, Gut - decreased motility, N/V, constipation GU - Increased uterine tone, urinary retention

Control of temp

Sensors throughout body including core (thorax, deep abdomen) and dermis Adelta - cold C - warm travel up spinothalamic tract in anterior spinal cord to hypothalamus input compared with set points, if difference noted then thermoregulatory mechanism triggered - sweating, vasodilation, behavioural - shivering, exercise. Set points are altered by pyrogens IL-6, TNF which alter prostaglandins synthesis which means we shiver despite high temp when febrile. Aspirin (COX) prevents synthesis of prostaglandins.

PRBC

Shelf life 35 days. Hct 0.6. RBC shape changes to spherical -> ATPase stops working, this results in increase in extracellular K+ > 25mmol/L. -> RBC also lyse -> Hb -> haptoglobin Leucodeplete to decrease CMV Irradiate to decrease graft vs host disease Don't give with dextrose -> causes lysis Ringer's high Ca -> thrombi risk

Whole Blood

Shelf life 35 days; HCt 0.45 Platelets non fn after 3 days FV + FVII low as labile factors Stored with adenosine, phosphate, glucose, citrate

Methohexitone

Short acting barbiturate; useful in ECT. USe 1% solution (10mg/mL) Faster onset than thio due to higher unionised fraction. Excitatory phenomenon, hypotension, resp depression, hicups. Avoid in porphyria. Dose 1-2mg/kg

Classify inotropes

Site of action 1. Beta agonists 2. Glucagon - glucagon R 3. Cardiac glycosides - digoxin 4. Phosphodiesterase inhibitors - milrinone 5. Other - levosemendan

Sleep

Sleep is a neccssary naturally occurring state of reduced level of consciousness in which brain is less responsive to external stimuli. It is reversible & associated with characterised EEG + EMG patterns along with physiological changes. Restorative but mechanism not well understood. Evidence for selective arousal - e.g. baby crying. 2 different phases Non REM phases of 90 minutes followed by REM sleep. Non REM/slow wave Develops with maturation of nervous system Alert wakefulness to deep sleep gradual change in brain wave pattern Low voltage high frequency to delta waves Asynchronous to synschronous Stage 1 - alpha to theta Stage 2 - spindle waves (bursts of high freq) & K complexes Stage 3 - delta waves Stage 4 - delta waves more synchronous REM Low voltage EEG which resembles awake state Arousal threshold is higher Decrease in muscle tone eg oropharyngeal Dreams likely to be recalled Eye movement Classically thought of as an state of decreased activity research would suggest it is an active process with certain neutrons increasing firing rate. Anaesthesia is phenotypically similar with decreased responsiveness to external stimuli but anaesthesia - Drug induced

Hypnosis

Sleep like state with maintenance of certain behavioural responses. Subject is susceptible to suggestion concerning behaviour. Effects may persist after return on normal consciousness.

Urea

Small water soluble substance which is produced as a by product of protein metabolism, produced and excreted to maintain nitrogen balance in body. H2N - C=O NH2 urea NH3 ammonia NH4+ ammonium Functions - 1. Transport of toxic ammonium to kidneys (ammonium NH4+ crosses blood brain barrier) 2. Formation of meduallary concentration gradient in kidneys. 3. Role in acid secretion in kidney Produce in liver from protein metablolism, 100% filtered in kidneys but has path of reabsorption and secretion. Overall 50% that is filtered is excreted.

Uptake of drugs in lung

Some drugs have high uptake in lungs so peak concentration will be affected & lungs may act as reservoir to release drug back into systemic circulation. Drugs include lipophilic amines - Lidocaine - Propranalol - Pethidine - Fentanyl - Alfentanil

Beta Blockers

Sotalol - Non selective B blocker Also has class III anti arryhtmic properties (blocks K channels & prolongs refractory period) IV or oral. Good OBA of 90%. Need to monitor pt for first days.

What reactions does chlorhex cause?

Spectrum from irritant dermaitits to IgE mediated delayed anaphylaxis.

K+ sparing diuretics - Spironolactone - Amiloride

Spironolactone - inhibits aldosterone receptor in distal tubule, this prevents reuptake of Na. Less K+ & H+ excreted. Weak effect so used with thiazide or loop diuretic. Amiloride - Prevents Na reabsorption in distal tubule.

Most common cause of epidural catheter related infection

Staph aureus - as per Oxford Handbook & google search

Sedation

State of reduced consciousness in which verbal contact with patient is maintained.

Nordarenaline

Stimulation of alpha receptors (a1) with minor beta R effects Catecholamine with - catechol ring, - CH2NH2

What is Frank Starling Law of Heart

Strength of contraction of cardiac myocyte is proportional to initial resting length of fibre. Increased filling pressures (diastolic distension) will lead to an increased stroke volume due to an intrinsic property of cardiac muscle cells whereby stretch increases contractility. This is due to favourable orientation of actin and myocin fibres at optimal sarcomere length (~2microns) Increased contractility will occur up to a point after which an increased stretch will reduce overlap of actin and myosin and force of contraction will be reduced. Diastolic distension approximated by diastolic volume. Along with optimal overlap of myosin/actin there is optimal exposed binding sites for Ca & increased affinity of Ca for troponin C.

If a patient experiences parasthesia in the little finger during supraclavicular brachial plexus blockthe needle is in proximity to what

Supraclavicular block is above clavicle around subclavian artery. Brachial plexus trunks are located here - superior, middle, inferior. The divisions run under the clavicle or might be seen as honeycomb structure. The inferior trunk supplies the ulnar nerve that provides sensation to little finger. Answer is inferior trunk also called lower trunk

Syntometriene

Syntocinon & ergometriene (O.5mg) IM injection Ergometriene - Ergot derivative similar to dopamine & adrenaline. Mechanism of action unclear. Side effects include vasoconstriction, spasm of coronary arteries, HTN, peripheral ischemia. CI in HTn, pre-eclampsis/eclampsia

Catecholamines

The OH groups on ring give max alpha & beta. The longer the end terminnal grip th better better activity - e.g. isopropyl is long and is isoprenaline Synthesis of catecholamines Phenylalanine Tyrosine DOPA Dopamine Noradrenaline In adrenal medulla - Adrenaline

Conciusness

The capacity to perceive & process incoming sensory information therefor be cognoscent of one's environment. Requires cererbral hemisphere interacting with midbrain, brain stem + thalamus & hypothalamus

What is the Bohr Effect?

The decreased affinity of Hb for oxygen with increased H+/increased CO2/increased 2.3 DPG. Due to conformational change in Hb protein by physiological factors. Results in a right shift of OHC. Encourages Hb to offload oxygen in tissues that require it. 1. CO2 binds to Hb N terminal decreases accesibility of hb to O2 2. H+ binds to Hb altering conformation 3. 2, 3 DPG binds to beta chain & causes conformational change. (produced in chronic anaeamia, high altitude, lung dx)

How does lateral position affect anaethesia

The effects of position on ventilation will be altered depending on whether pt is a) awake or anaesthetised b) spont breathing or mechanically ventilated c) chest open or closed If patient awake then spent breathing will favour greater ventilation to dependent lung & gravity will favour greater blood flow to dependent lung so overall V/Q matching maintained. Dependent lung - diaphragm higher up in thorax due to compression from abdomen but this is good as the contractions are more efficient. However anaesthesia will decrease FRC In lateral position the upper lung moves to more compliant position on curve, the dependent lung is less compliant. Ventilation is better in upper lung but blood flow is better to lower lung so overall V/Q mismatch is enhanced. Ventilated Patient - IPPV provides more ventilation to upper lung so V/Q mismatch exacerbated. If paralysis is used the diaphragm will actually impede ventilation in IPPV. Chest Open - Chest wall compliance is high so upper lung gets an even better advantage & V/Q mismatch is further exacerbated

Haldane effect

The increased capacity of deoxygenated blood to transport CO2 compared with oxygenated blood. MOA 1. When O2 is offloaded from Hb the nitrogen grip is no longer ionised so it is free to form carbamino compounds (70% of effect) 2. Deoxy Hb has greater buffering ability and can bind to H+ so more bicarbonate is formed (more carbamino compounds formed)

Ion trapping

The pH of the environment will affect the degree of ionisation of a drug. This may lead to the drug being unable to pass back across a lipid membrane. eg pH of foetus is more acidic in fatal distress so basic drugs e.g. LA get trapped.

Power

The rate at which work is done, measured in Watts I watt = i joule per second Basal metabolic rate = 100 watts or 8300 joules per day

Specificity

The true negative rate The ability of a test to correctly identify those without the disease as not having the disease. The higher the specificity the less chance of a

Sensitivity

The true positive rate The ability of a test to recognise those with the disease as having the disease. The higher the sensitivity the less chance of

Diathermy

The use of an electrical current to produce heat Used in surgery to coagulate or cut tissue. Very high freq current is used unto 1 Megahertz; this is safest as a low freq would cause risk of VF or muscle contraction (burns can still occur at any frequency). AC is used so no matter what current is used it does not flow in one direction long enough to stimulate heart. Monopolar High current density at tip of forceps but patient part of circuit has plate with large surface area so this results in a low current density. Operates at higher power of up to 500 watts so can cut & coagulate. Bipolar Operates at lower power with output applied btw tips of forceps. Low power is suitable for coagulation but poor for cutting, Pt is not part of circuit Suitable for use with pacemaker. Power of 40 watts

Gate Control

Theory developed in 1960s as a way to explain why tissue damage & activation of nociceptors does not always result in perception of pain. Theory suggests that within spinal cord a physiological gate exists which determines signal that reaches brain. So spinal cord dorsal horn is not merely a path for transmission, modulation also occurs. TENs uses this theory. Diff in size & conduction velocity of nervous tissue is exploited. Mechanical vibration stimulates the A beta fibres; this prevents the dorsal horn neurons from responding to Adelta/C input.

Atypical antipsyhotics

These drugs have less SE due to decreased D2 receptor affinity. Uses - Bipolar OCS Schizophrenia Mania/depression Dementia Clozapine (prototype) Risperidone Olanzapine Quitiapine

Current density

This is the factor that will determine risk of electrocution/burns. It considers the resistance to current. Dry skin has a resistance of 50000 ohms Wet decrease it to 500 ohms V = IR so I = voltage/resistance

Repeat- Single lumen intubation after multiple attempts of difficult intubation, you put in a bronchoscope after and the tip is in the trachea. The structure B you seen corresponding to?

Trachealis is posterior, use this to orientate

Phaechromocytoma

Treatment includes tyrosine hydrolase inhibitor metirasine which blocks conversion of tyrosine to dompamine

Dibucaine Number

Used as test for atypical plasma cholinesterases. Plasma cholinesterases are encoded for on char 3. 96% people are homozygous, their dibucaine no is normal. Dibucaine itself is a local anaesthetic. It inhibits plasma cholinesterase but does not inhibit the variants as much. Puts with atypical plasma cholinesterase are at risk of apnoea post sux or mivacurium as they cannot metabolise drug as well.

Determinants of Uterine Blood Flow

Uterine blood flow = arterial - venous pressure /resistance Spiral arteries invade trophoblast with loss of smooth muscle so ability to vassconstrict impaired. Low resistance circuit is set up No autoregulation, heavily dependent on maternal blood flow. Blood flow is greater than fetal requirements - safety buffer Uterine blood flow = 50mL/min -> increase to 800mL/min in pregnancy then back to 120mL/min. 3% of cardiac output -> 12% CO in pregnancy

Ideal properties of depth of anaesthesia monitors

Validated across all population groups Inexpensive Simple to use Independent of agent - Able to be used with all anaethetic agents - Takes into account other medication - opioids, nitrous Anticipatory - predictive of awareness or lightening Good sensitivity/specificity - minimal overlap between value that indicates anaesthetised and awake patients Improves patient safety and outcomes (e.g. decreased post-op cognitive dysfunction, decrease in awareness)

Distribution of blood flow

Vessel rich organs get 75% of blood flow even though they make up only 10% body mass. Thus these tissue equilibrate faster with alveoli partial pressures.

Methoxyflurane

Volatile developed after halothane with aim to decrease dysrythmias. Found to have extensive hepaticc metabolism and produce fluoride with nephrotoxiticity. used now in low concentrations for analgesia Get production of fluoride ions like sevoflurane but much more methoxyflurane is able to be metabolised (70% cf 3% of sevo)

What potentiates neuromuscular blockade?

Volatiles Aminoglycosides Tetracylcins

Humidity benefits (think water not heat)

Water vapour is added to dry gases in the resp tract. The energy required to evaporate water is high so less energy (heat) is expended by the body if the water is already humidified. Humidification is required to - Maintains ciliary funciton - Helps expel mucous - More energy efficient for body as does not need to be heated by resp tract - Minimises heat loss - esp in small children. - Dry gas is more at risk of explsions & i physically uncomfortable Mechanism for hunidification - HME (heat moisture exchanger) single use and often combined with filter on circuits - During expiration the water vapour condenses on a cool matrix, during inspiration the water is added to gas. Hot water baths - Fresh gas passed over water. Risk of burns, drowning, infection, wires Nebulizers - Venturi system, as gas passes through narrow inlet it accelerates, this decreases pressure + draws liquid in. Circle System - Soda lime prodces H2O

Chlorexidine

antispetic present in > 50 products with bacteriocidal & bacteriostatic properties depending on concentration. Binds to negatively charge sites on bacterial wall + upsets the integrity of the cell wall. It does bind to skin proteins but not a problem. Slower than alcohol. Good fro Gram +ve/gram -ve, decreases biofilms, some anti-viral properties

Acute Pain Chronic Pain

pain of recent onset and probable limited duration. Usually has identifiable temporal causal relationship to injury or disease. If pain is a symptom of a disease e.g. pain from osteomyelitis at 1 yr this is still acute pain. Pain that persists beyond time of injury and healing. Frequently there may not be any clearly identifiable cause. A better tern is "persistent pain" as untreated acute pain is one of the determinants of chronic pain development. When pain itself becomes the disease.

EEG

recording if electrical activity on scalp or brain measures voltage fluctuations on post synapatic part of neurone 20 electrodes placed around scalp to pick up characteristic variation in potentials from nerve tracts potentials much smaller than ECG EEG - 0.05mV (50 microvolts) EMG 0.1 ++++ ECG 1-2 MV Need a summation of neuron potentials as one neutron too small. Basic EEG rhythm > 12H beta (small amplitude) 8-12 alpha 4-7 theta < 4 delta (high amplitude)

TAVI (TAVR) vs surgery

●Mortality rates in the TAVR and surgical group were similar at 30 days, 1 yr, 2 yrs. ●Combined strokes and transient ischemic attacks were more frequent after TAVR than after surgical aortic valve replacement at 30 days (borderline diff at 2 yrs) ●More patients undergoing TAVR reported symptom improvement at 30 days, but at one year, symptom improvement was similar in the two groups. •TAVR - more frequent major vascular complications •Surgical aortic valve replacement - more frequent major bleeding & new-onset atrial fibrillation ●Moderate or severe paravalvular aortic regurgitation was more frequent after TAVR than after surgery at 30 days, one, and two years (which increases late mortality) SUMMARY AND RECOMMENDATIONS ●Aortic valve replacement is the mainstay of treatment of symptomatic aortic stenosis (AS). Aortic valve replacement offers substantial improvements in symptoms and life expectancy. ●Transcatheter aortic valve replacement (TAVR) has been developed as a treatment for patients with severe symptomatic AS deemed to be inoperable or at high risk for surgical aortic valve replacement. •In appropriately screened patients with inoperable severe symptomatic AS, TAVR provides better outcomes compared to medical therapy including percutaneous valvotomy. •In appropriately screened patients with inoperable severe symptomatic AS, self-expanding TAVR likely provides better outcomes than medical therapy. •In appropriately screened patients with severe symptomatic AS who are deemed to be at high risk for surgery, mortality rates following balloon-expandable TAVR and surgical aortic valve replacement are similar but self-expanding TAVR is lower than with surgical aortic valve replacement. ●The long-term durability of transcatheter bioprosthetic valves is not yet known. Conventional surgical AVR remains the recommended treatment in patients with AS who do not have a prohibitive or high surgical risk. ●A multidisciplinary team approach is recommended in approaching high-risk patients with symptomatic AS.


Related study sets

Soc 310 CH 10 family and household transition

View Set

Ch 9.2 - Receiving & Storage Equipment

View Set

California Real Estate Terminology

View Set

Google: Week 1. The Bits and Bytes of Computer Networking Introduction to Computer Networking

View Set